Вы находитесь на странице: 1из 757

A-PDF Merger DEMO : Purchase from www.A-PDF.

com to remove the watermark


ANDERSON JUNIOR COLLEGE
Preliminary Examination 2010

CHEMISTRY 8872/01
Higher 1 23 September 2010
Paper 1 Multiple Choice 50 minutes

Additional Materials: Multiple Choice Answer Sheet


Data Booklet

READ THESE INSTRUCTIONS FIRST

Write in soft pencil.


Do not use staples, paper clips, highlighters, glue or correction fluid.

There are thirty questions on this paper. Answer all questions. For each question there are four
possible answers A, B, C and D.
Choose the one you consider correct and record your choice in soft pencil on the Multiple Choice
Answer Sheet.

Each correct answer will score one mark. A mark will not be deducted for a wrong answer.
Any rough working should be done in this booklet.

Multiple Choice Answer Sheet

Write your name, PDG and NRIC/FIN number. DO NOT include the reference letter.

Shade the NRIC / FIN number.

Exam Title: JC2 Prelim

Exam Details: H1 Chem / Paper 1

Date: 23/09/2010

This document consists of 12 printed pages including cover page


2

Section A

For each question there are four possible answers, A, B, C and D. Choose the one you consider to
be correct.

1 Which statement about one mole of a metal is always true?

A It contains the same number of particles as one mole of hydrogen atoms.


B It contains the same number of particles as 1/12 mole of 12C.
C It has the same mass as one mole of hydrogen atoms.
D It is liberated by one mole of electrons.

2 Use of the Data Booklet is relevant to this question.

A garden fertiliser is said to have a phosphorus content of 30.0% by mass P2O5 soluble in
water.

What is the percentage by mass of phosphorus in the fertiliser?

A 3.93% B 6.55% C 13.1 % D 26.2%

3 Use of the Data Booklet is relevant to this question.

The 68Ge isotope of the Group IV element germanium is medically useful because it
undergoes a natural radioactive process to give a gallium isotope, 68Ga, which can be used to
detect tumors. This transformation of germanium occurs when one of its electrons enters the
nucleus, changing a proton into a neutron.

What do the isotopes 68Ga and 68Ge have in common?

A Both isotopes have 37 neutrons in their nuclei.


B Both isotopes have more electrons than protons.
C Both isotopes have an outer electronic configuration 4s2 4p3.
D Both isotopes contain the same number of nucleons in their nuclei.

AJC Prelim 2010 8872/01/H1 [Turn over


3

4 Use of the Data Booklet is relevant to this question.

The graph shows the logarithm, lg, of the first eight ionisation energies of two elements in
Periods 2 and 3.

lg (ionisation energy)

1 2 3 4 5 6 7 8
no. of electrons removed

What is the most likely compound that will be formed between the two elements?

A OF2 B OCl2 C SF2 D SCl2

5 In which of the following pairs does the first species have a larger bond angle than the
second?

A CH4, CH3+
B NCl3, BH3
C XeF4, SF6
D H2O, H2S

6 Which statement concerning the lattice structure of graphite and diamond is incorrect?

A All CC bonds in graphite and diamond are formed from sp3sp3 overlap.
B Graphite is energetically more stable than diamond.
C The bond energy of the CC bonds in graphite is greater than that in diamond.
D The CCC bond angle between nearest neighbours is smaller in diamond than in
graphite.

AJC Prelim 2010 8872/01/H1 [Turn over


4

7 In which of the following do all three compounds have giant lattice structure?

A BaO, AlF3, SiO2


B K2O, BCl3, Cr2O3
C MgO, Fe2O3, P4O10
D Na2O, SiO2, SO3

8 When 25 cm3 of 1.0 mol dm3 sodium hydroxide is completely neutralised by 1.0 mol dm3
hydrochloric acid, the temperature of the mixture rose by 6.8 C. What will be the temperature
change if 50 cm3 of 0.5 mol dm3 sodium hydroxide is completely neutralised by 0.5 mol dm3
hydrochloric acid? (Assume heat loss to be negligible in each case)

A 1.7 C
B 3.4 C
C 6.8 C
D 13.6 C

9 Titanium occurs naturally in the mineral rutile, TiO2. Rutile is a major ore of titanium, a metal
used for high tech alloys because of its light weight, high strength and resistance to corrosion.

One possible method suggested for the extraction of the metal is to reduce the rutile by
heating it with carbon:

TiO2(s) + 2C(s) Ti(s) + 2CO(g)

Given that H f{ [TiO2(s)] = 940 kJ mol1 and H f{ [CO(g)] = 110 kJ mol1, what is the H for
this reaction?

A +720 kJ mol1
B +830 kJ mol1
C 830 kJ mol1
D It cannot be calculated from the information above.

AJC Prelim 2010 8872/01/H1 [Turn over


5

10 The nickel-cadmium rechargeable battery is based upon the following overall reaction.

Cd + 2NiOOH + 4H2O Cd(OH)2 + 2Ni(OH)2.H2O

What is the oxidation number of nickel at the beginning and at the end of the reaction?

beginning end
A +1.5 +2
B +2 +3
C +3 +2
D +3 +4

11 In the Haber-Bosch Process, ammonia gas is synthesised by reacting nitrogen gas with
hydrogen gas.

N2(g) + 3H2(g) 2NH3(g) H{ = 46 kJ mol1

If the temperature of an equilibrium mixture of the three gases is increased at constant


pressure, will the volume of the mixture increase or decrease and why?

A The volume will increase due to only thermal expansion.


B The volume will increase due to a shift of equilibrium to the left and thermal expansion.
C The volume will remain the same as any thermal expansion could be offset by any
shifting of equilibrium to the right.
D The volume will decrease as a shift of equilibrium to the right could more than offset any
thermal expansion.

AJC Prelim 2010 8872/01/H1 [Turn over


6

12 When 0.1 mol of of bismuth chloride is added to 2 dm3 of water, it reacts to form 0.02 mol of
white precipitate of bismuth oxychloride and a solution of hydrochloric acid.

The equation for the reaction is as follows.

BiCl3(aq) + H 2O(l) BiOCl(s) + 2HCl(aq)

What is the correct expression for the equilibrium constant Kc?


2
0
.
0 8
2
2

x
0
.
0

A
0
.
0
2
2 0
0
.
0
2
2

x
.
0 2
8

B

2
0
.
0 8
2

x

2.
0
0

C
2 2
2 0
0
.
0
2
0
.
0 2

x

2
.
0 2
8

13 What is the final pH of the solution formed by mixing equal volumes of two separate solutions
of a strong acid with pH 2.0 and pH 3.0?

A 1.96 B 2.26 C 2.50 D 2.74

14 The reaction of acidified, aqueous potassium iodide with aqueous hydrogen peroxide is shown
below.

2I(aq) + H2O2(aq) + 2H+(aq) I2(aq) + 2H2O(l)

The rate equation is rate = k[H2O2][I]. Which of the following conclusion cannot be drawn
from this information?

A The iodide ion is oxidised by the hydrogen peroxide.


B The colour of the solution turns brownish at the end of reaction.
C The overall order of reaction is two.
D The acid acts as catalyst.

AJC Prelim 2010 8872/01/H1 [Turn over


7

15 The distribution of the number of molecules with energy E is given in the sketch for two
temperatures, T1 and a higher temperature T2. The letters, P, Q, and R refer to the separate
and differently shaded areas. The activation energy is marked on the energy axis.

Activation energy

Which expression gives the fraction of the molecules present which have at least the
activation energy at the higher temperature T2?

Q Q+R Q+R Q+R


A B C D
P P P +Q R P +Q

16 Consecutive elements E, F and G are in Period 3 of the Periodic Table. Element F has the
lowest first ionisation energy and the highest melting point.

What could be the identities of E, F and G?

A magnesium, aluminium, silicon


B aluminium, silicon, phosphorus
C silicon, phosphorus, sulfur
D phosphorus, sulfur, chlorine

17 The oxide and chloride of an element H are separately mixed with water. The two resulting
solutions have the same effect on litmus.

What is element H?

A sodium
B magnesium
C aluminium
D phosphorus

AJC Prelim 2010 8872/01/H1 [Turn over


8

18 Which molecules have isomers that exhibit cistrans isomerism?

I II III IV
C3H6BrI C3H5I C3H4I2 C3H4BrI

A I, II and III only


B II and IV only
C III and IV only
D II, III and IV only

19 A hydrocarbon, on heating with an excess of hot concentrated KMnO4, produces


HO2CCH2CH2CH2COCH3.

Which of the following cannot be the hydrocarbon?

A B

CH3

C D

20 Four drops of 1chlorobutane, 1bromobutane, 1iodobutane were put separately into three
testtubes containing 1.0 cm3 of aqueous silver nitrate at 60 C.

A hydrolysis reaction occurred. (R represents the butane chain C4H9 and X the halogen
atom.

H2O(l) + RX(l) + Ag+(aq) ROH(aq) + AgX(s) + H+(aq)

The rate of formation of precipitate in the tubes was in the order RCl < RBr < RI.

What helps to explain this observation?

A The bond energy of RX decreases from RCl to RI.


B The ionisation energy of the halogen decreases from Cl to I.
C The RX bond polarity decreases from RCl to RI.
D The lattice energy of AgX(s) decreases from AgCl to AgI.

AJC Prelim 2010 8872/01/H1 [Turn over


9

21 Which reagent could detect the presence of added alcohol in petrol consisting mainly of a
mixture of alkanes and alkenes?

A Br2 in CCl4
B anhydrous SOCl2
C aqueous KMnO4
D aqueous NaOH

22 The compound J, CH3COCH2CHO, was reacted in separate experiments with:

(i) alkaline aqueous iodine;


(ii) 2,4-dinitrophenylhydrazine;
(iii) Tollens reagent [Ag(NH3)2]+(aq);
(iv) lithium aluminium hydride in dry ether.

One mole of J could

A form two moles of CHI3 in (i).


B react with two moles of 2,4-dinitrophenylhydrazine in (ii).
C form one mole of CH3COCH2CH2OH and two moles of Ag in (iii).
D form a product in (iv) which further reacts with Na to produce two moles of H2 gas.

23 Testosterone is a steroid hormone found in mammals, reptiles, birds and other vertebrates.

CH3 OH

CH3

O
testosterone

Testosterone is first reacted with hydrogen in the presence of a platinum catalyst, and the
product is then warmed with acidified KMnO4.

Given that no carbon-carbon -bond is broken in this process, how many double bonds will
there be in the structure of the final product?

A 0 B 1 C 2 D 3

AJC Prelim 2010 8872/01/H1 [Turn over


10

24 The ester, CH3CH2CH(CH3)CH2OCOCH3, has an odour of bananas.

Which set of reagents could be used to prepare this ester in the laboratory?

A CH3CH2CH(CH3)CH2CO2H , CH3Cl, H2SO4


B CH3CH2CH(CH3)CH2Cl, CH3CO2H, H2SO4
C CH3CH2CH(CH3)CH2CO2H, CH3OH, H2SO4
D CH3CH2CH(CH3)CH2OH, CH3CO2H , H2SO4

25 Which pairs of reactions could have the same common intermediate?

K CH3CH2CH3 intermediate (CH3)2CHCN


L C6H5CO2CH(CH3)2 intermediate (CH3)2CHBr
M CH3CH=CH2 intermediate (CH3)2CH(NH2)
N CH3CH2CH2Cl intermediate CH3COOH

A K and L
B L and N
C K and M
D M and N

AJC Prelim 2010 8872/01/H1 [Turn over


11

Section B

For each of the questions in this section, one or more of the three numbered statements 1 to 3 may
be correct.

Decide whether each of the statements is or is not correct (you may find it helpful to put a tick
against the statements that you consider to be correct.)

The responses A to D should be selected on the basis of

A B C D
1, 2 and 3 1 and 2 2 and 3 1 only
are correct only are correct only are correct is correct

No other combination of statements is used as a correct response.

26 Which of the following aqueous solutions do not considerably change in pH when relatively
small volumes of strong acid or strong alkali are added?

1 A mixture of HF and NaF


2 A mixture of HCN and NaCN
3 A mixture of H2CO3 and NaHCO3

27 Hydrazine, N2H4, is widely used as a rocket fuel because it reacts with oxygen as shown,
producing environmentally friendly gases.

N2H4(l) + O2(g) N2(g) + 2H2O(g) H = 534 kJ mol1

Despite its use as a rocket fuel, hydrazine does not spontaneously burn in oxygen.

Why does hydrazine not burn spontaneously?

1 The activation energy is too high.


2 The NN bond is very strong.
3 Hydrazine is a liquid.

AJC Prelim 2010 8872/01/H1 [Turn over


12

The responses A to D should be selected on the basis of

A B C D
1, 2 and 3 1 and 2 2 and 3 1 only
are correct only are correct only are correct is correct

No other combination of statements is used as a correct response.

28 Which of the following statements are correct for the sequence of compounds below
considered from left to right?

NaF MgO AlN SiC

1 The formula-units of these compounds are isoelectronic (have the same number of
electrons).
2 The bonding becomes increasingly covalent.
3 The electronegativity difference between the elements in each compound increases.

29 In which way are propane and propene similar?

1 They are both neutral to an indicator solution.


2 They can both produce propan-2-ol under suitable conditions.
3 They both have the same standard enthalpy change of combustion.

30 The alcohol, C5H10O, may be oxidised by acidified aqueous potassium dichromate (VI) to a
compound C5H8O. Which of the following could be the structural formula of the alcohol?

1 CH3CHCHCH2CH2OH
2 CH2C(CH3)CH(OH)CH3

AJC Prelim 2010 8872/01/H1 [Turn over


NAME:______________________________ PDG:__________ Register No: _____

ANDERSON JUNIOR COLLEGE


Preliminary Examination 2010

CHEMISTRY 8872/02
Higher 1 17 Sep 2010
Paper 2 2 hours

Candidates answer Section A on the Question Paper


Additional Materials: Writing paper
Data Booklet

READ THESE INSTRUCTIONS FIRST

Write your name, PDG and register number on all the work you hand in.
Write in dark blue or black pen.
You may use a pencil for any diagrams, graphs or rough working.
Do not use staples, paper clips, highlighters, glue or correction fluid.

Section A
Answer ALL the questions.

Section B
Answer TWO questions on separate writing paper.

At the end of the examination, fasten all your work securely together.
The number of marks is given in brackets [ ] at the end of each question or part question.

For Examiners Use

Paper 1 [30m] Total [110]

1 5

Paper 2 2 Paper 2 6
Section A Section B
[40m] 3 [40m] 7

4 Grade

This document consists of 16 printed pages including cover page.

AJC/H1 Chemistry/Prelim2010 [Turn over


2
Section A

Answer all the questions in the space provided.

1 (a) Draw a detailed diagram to represent the arrangement of protons, neutrons and
electrons in an atom of carbon.

[1]

(b) Carbon forms double bonds with each of the Group VI elements oxygen, sulfur and
selenium. In each case, the double bond is polar. However, the polarities of these
double bonds do not necessarily cancel in the molecules that carbon formed with
the elements.

Overall polarity of molecule


CO2 0
COS 0.71
COSe 0.73

(i) With the aid of an appropriate diagram, explain why carbon dioxide molecule
has no net polarity.

(ii) Why is the COSe molecule more polar than the COS molecule?

[4]

AJC/H1 Chemistry/Prelim 2010 [Turn over


3
(c) Ethanedioate ion (C2O4 ), which is formed on dimerisation of the CO2 anion, can
2

react with many metal ions to produce salts. One example is iron(II) ethanedioate
(FeC2O4) which is used as a yellow dye.

(i) When an aqueous solution of iron(II) ethanedioate reacts with potassium


manganate(VII) under acidic condition, Fe2+(aq) and C2O42(aq) undergo
oxidation as shown.
Fe2+ Fe3+ + e
C2O42 2CO2 + 2e

Construct a halfequation for the oxidation of iron(II) ethanedioate under


acidic condition.

(ii) Hence, calculate the volume of 0.10 mol dm3 potassium manganate(VII)
required to oxidise 25 cm3 of an acidified solution of 0.10 mol dm3 iron(II)
ethanedioate.

[3]
[Total: 8]

AJC/H1 Chemistry/Prelim 2010 [Turn over


4
2 (a) Cinnamaldehyde is the main component in cinnamon oil. It can be converted to acid
B via intermediate A as shown in the reaction pathway below.

II H
I
CH CH CHO A CH CH C CO2H

OH
cinnamaldehyde B

(i) Draw the structural formula for intermediate A.

(ii) State the reagents and conditions for stages I and II and name the type of
reaction that takes place at each stage.

Stage Reagents and Conditions Type of reaction

II

[5]

(b) Cinnamaldehyde can be reduced into 3phenylpropan1ol. Suggest the reagent


and condition for this reaction.

[1]

AJC/H1 Chemistry/Prelim 2010 [Turn over


5
(c) Anhydrous phosphorus pentachloride, PCl5, is an offwhite crystalline solid which is
often used to identify the presence of the OH functional group in alcohol.

(i) With the aid of an equation, explain how anhydrous PCl5 may be used to
identify the OH functional group in 3phenylpropan1ol formed in (b).
(You may represent 3phenylpropan1ol by ROH.)

[1]

(ii) PCl5 is produced when PCl3 reacts with Cl2 gas. In the gaseous phase, the
reaction can be represented as follows:

PCl3(g) + Cl2(g) PCl5(g)

Use the following average bond energies to calculate the enthalpy change of
this reaction.

Bond Average bond energy / kJ mol1


Cl Cl 244
P Cl 320

[2]

AJC/H1 Chemistry/Prelim 2010 [Turn over


6
(iii) The enthalpy change of the reaction in (c)(ii) can be determined indirectly
using the energy cycle shown.

H
4PCl3(g) + 4Cl2(g) 4PCl5(g)

H1 H2

P4(s) + 10Cl2(g)

What does H1 represent?

What is the relationship between H, H1 and H2?

[2]
[Total:11]

AJC/H1 Chemistry/Prelim 2010 [Turn over


7
3 Chlorine is very reactive and will form compounds by direct combination with many
elements or molecules.

(a) Chlorine reacts with magnesium and silicon to form magnesium chloride and silicon
tetrachloride respectively.
Mg + Cl2 MgCl2
Si + 2Cl2 SiCl4
The chlorides each dissolves in or react with water to form solutions with different
pH values.

(i) Suggest the approximate pH values of the solution formed in each case.

MgCl2 :

SiCl4 :

(ii) Explain the difference between the two pH values, giving relevant equations
where appropriate.

[3]

AJC/H1 Chemistry/Prelim 2010 [Turn over


8
(b) Chlorine reacts with methylpropane in the presence of UV light to form a mixture of
products. One of the products, 1chloro-2-methylpropane, undergoes a sequence of
reactions as shown below.

CH3
CH3CHCH2Cl

NaOH in ethanol heat heat NaOH(aq)


under reflux under reflux
C D

KMnO4 / OH- KMnO4 / H+


cold heat

E F

In the appropriate boxes, draw the structures of compounds C, D, E and F.


[4]

(c) 1chloro-2-methylpropane and 2chlorobutane are isomers.

(i) State the type of isomerism exhibited by 1chloro-2-methylpropane and


2chlorobutane.

(ii) Why is 2chlorobutane considered an environmental hazard?

[2]
[Total: 9]

AJC/H1 Chemistry/Prelim 2010 [Turn over


9
4 Bromoalkanes G and H are structural isomers with the molecular formula C5H11Br. Two
experiments were carried out to investigate the kinetics of the reaction of G and H with
aqueous sodium hydroxide separately.

(a) The following initial rates were obtained using different concentrations of G and
aqueous sodium hydroxide.

[G] / [NaOH] / initial rate /


experiment
mol dm3 mol dm3 mol dm3 min1
1 0.10 0.10 5.56 x 104
2 0.20 0.05 5.56 x 104
3 0.30 0.10 1.67 x 103

(i) Use the data to determine the order of reaction with respect to
I bromoalkane G

II aqueous sodium hydroxide

(ii) Use these answers to write a rate equation for the reaction.

AJC/H1 Chemistry/Prelim 2010 [Turn over


10
(iii) Hence calculate the rate constant for the reaction, stating its units.

[5]

(b) The following graphs were obtained in the investigation of the kinetics of the
reaction of H and aqueous sodium hydroxide.
rate of reaction /
mol dm3 min1

rate of reaction /
mol dm3 min1

[H] / mol dm3 [NaOH] / mol dm3

(i) Using the graphs above, deduce the rate equation for the reaction of H with
aqueous sodium hydroxide.

(ii) Hence, explain whether the reaction of aqueous sodium hydroxide with
bromoalkanes G or H proceeds by the same mechanism.

[2]

AJC/H1 Chemistry/Prelim 2010 [Turn over


11
(c) When H reacts with aqueous sodium hydroxide, there is a change in structure from
reactant to product, as shown below.

R R
I II
R'
C Br R C C OH
R
R R
R' R'

H intermediate product
(tetrahedral) (trigonal planar) (tetrahedral)

(i) Identify the change in the hybridisation of the reactive carbon atom in steps I and II.

Step Change in hybridisation of carbon


I
II

(ii) Hence sketch the shape of one hybrid orbital of the reactive carbon atom in reactant
H.

[2]

(d) Bromoalkane is a useful intermediate for synthesising other organic compounds.


The reaction scheme below shows one such example.

III IV
C5H11Br C5H11CN C5H11CH2NH2

(i) Suggest reagents and conditions for steps III and IV.
III :

IV :

(ii) State the type of reaction occurring in step IV.

[3]
[Total: 12]

AJC/H1 Chemistry/Prelim 2010 [Turn over


12
Section B

Answer two of the three questions in this section on separate writing paper.

5 Compounds of sulfur have many uses in everyday life which include disinfectants,
matches and in making of paper.

(a) Phosphorus sulfide, P4S3, is used in small amounts in the tip of a match stick. On
striking a match stick, one mole of this compound burns to form sulfur dioxide and a
solid oxide in the molar ratio of 3:1.

(i) Construct an equation for this reaction.

(ii) Both oxides formed in (a)(i) dissolve in water to give acidic solutions.
Construct an equation for the reaction of each oxide with water and give an
approximate pH for each of the solution formed.

(iii) The melting points of the two oxides formed in (a)(i) differ significantly from
that of silicon (IV) oxide (SiO2).
Briefly relate this observation to their structure and bonding.
[6]

(b) The key reaction during the Contact process for manufacturing sulfuric acid is as
follows.
2SO2(g) + O2(g) 2SO3(g) H < 0

When 34.6 mol of SO2 and 17.3 mol of O2 are passed over a catalyst in a 1 dm3
vessel at 450 oC, the amount of SO3 at equilibrium is found to be 32.8 mol.

(i) Calculate the value and state the units of the equilibrium constant, Kc, for this
reaction.

(ii) Calculate the percentage of SO2 in an SO2SO3 mixture at 450 oC when


[O2] = 10 mol dm3.

AJC/H1 Chemistry/Prelim 2010 [Turn over


13
(iii) The total SO2 and SO3 content in the mixture in (b)(ii) can be found by
dissolving the mixture in water and titrating against aqueous sodium
hydroxide of known concentration.

When a 100 cm3 sample of the mixture was titrated against 0.200 mol dm3
sodium hydroxide, 35.00 cm3 of aqueous sodium hydroxide was required to
neutralise all the oxides of sulfur according to the following equations:

2NaOH + SO2 Na2SO3 + H2O

2NaOH + SO3 Na2SO4 + H2O

Calculate the number of moles of sodium hydroxide required and the total
number of moles of SO2 and SO3 present in the sample. Hence, by using the
answer in (b)(ii), calculate the concentration of SO3, in mol dm3, in the
sample.

(iv) State how the position of equilibrium and rate of achieving equilibrium would
be affected by a decrease in temperature.

[10]

(c) Concentrated sulfuric acid can be used under two different sets of reaction
conditions to synthesise but2ene from but1ene via a twostep reaction scheme
as shown below.

I II
CH3CH2CH=CH2 CH3CH2CH(OH)CH3 CH3CH=CHCH3
but1ene but2ene

(i) Describe the condition necessary for step II.

(ii) But2ene occurs in two isomeric forms but cyclobutene occurs in only one
form. Draw the displayed formulae of the two isomers of but2ene and
explain why such isomerism is not possible with cyclobutene.
[4]
[Total: 20]

AJC/H1 Chemistry/Prelim 2010 [Turn over


14
6 (a) Sodium hydroxide is a common reagent used in laboratory. It can be prepared by
dissolving sodium oxide in water.

(i) With the aid of an appropriate equation, explain why sodium hydroxide can be
used to separate a mixture of magnesium oxide and aluminium oxide.

(ii) The lattice energy of sodium oxide is 742 kJ mol1. Explain how you would
expect the magnitude of the lattice energy of magnesium oxide to compare with
that of sodium oxide.
[4]

(b) Compound J (C8H16O2) is a neutral extract. A chemical analysis of J is carried out,


using sodium hydroxide as one of the reagents. The following results are obtained.

J is insoluble in both water and dilute aqueous sodium hydroxide.


When heated under reflux with dilute aqueous sodium hydroxide followed
by acidification, J gives two compounds, K (C5H12O) and L (C3H6O2).

Reaction of K with aqueous iodine in sodium hydroxide produces a yellow precipitate.


Heating K over Al2O3 produces compound M (C5H10). Strong heating of M with
acidified potassium manganate(VII) produces propanone as one of the products.

Effervescence is observed when aqueous sodium carbonate is added to L.

Use all of the above information to identify the organic compounds J, K, L and M, and
draw a structural formula for each of them.
Write equations where appropriate to show the reactions that are occurring. Clearly
show all of the deductions that you make from the information that you have been
given. Full marks cannot be gained by only giving the structures required.
[13]

AJC/H1 Chemistry/Prelim 2010 [Turn over


15
(c) (i) With reference to compound M (C5H10), explain what is meant by the term
standard enthalpy change of combustion?
(ii) Assuming that air contains onefifth oxygen by volume and that both gas
volumes are measured at the same temperature and pressure, calculate the
volume of air required for complete combustion of 10 cm3 of M.
[3]
[Total:20]

7 (a) (i) What do you understand by the terms strong and weak when applied to
acids?

(ii) It is found that an aqueous solution of benzoic acid of concentration


0.10 mol dm3 has a pH of 2.6. Explain, with reasons, whether benzoic acid is
a strong or a weak acid.
[2]

(b) A sample of 20.0 cm3 aqueous solution of 0.1 mol dm3 benzoic acid was titrated
with 0.15 mol dm3 potassium hydroxide solution.

(i) Calculate the pH of the 0.15 mol dm3 potassium hydroxide solution.

(ii) Calculate the volume of potassium hydroxide required for neutralisation with
the benzoic acid.

(iii) Suggest two reasons why phenolphthalein is a suitable indicator for this
titration.
[4]

(c) A buffer containing potassium benzoate and benzoic acid was formed during the
titration in (b). With the aid of two equations, explain how a solution of potassium
benzoate and benzoic acid can control pH.
[2]

AJC/H1 Chemistry/Prelim 2010 [Turn over


16
(d) The enthalpy changes of neutralisation of potassium hydroxide by two acids are as
follows:
hydrochloric acid 57.3 kJ mol1
nitric acid 57.3 kJ mol1

Explain how you would expect the enthalpy change of neutralisation of potassium
hydroxide by benzoic acid to compare with these values.

[2]

(e) The relative molecular mass of benzoic acid is found to be approximately 244 at
25 oC when it is dissolved in benzene.

(i) Draw a diagram to explain this phenomenon.

(ii) State and explain what will be observed when a piece of dry blue litmus paper
is dropped into this solution.

[2]

(f) Benzoic acid crystals can be synthesized from methylbenzene in the laboratory.

(i) Suggest the reagents and conditions required for this synthesis.

(ii) Write a balanced equation for this reaction.


[2]

(g) When methylbenzene is treated with chlorine under two different conditions, three
isomeric monochlorides, P, Q and R are formed.

(i) Deduce the structures of isomers P, Q and R.

(ii) Describe the two conditions necessary to form the monochlorides.

(iii) Suggest how, if any, methylbenzene and any one of the isomers in (g)(i)
might differ in their boiling points? Explain your answers.
[6]
[Total:20]
End of Paper

AJC/H1 Chemistry/Prelim 2010 [Turn over


A-PDF Merger DEMO : Purchase from www.A-PDF.com to remove the watermark

Part 1

For each question, there are four possible answers, A, B, C and D. Choose the one you
consider to be correct.

1 How many atoms of carbon are present in 18 g of glucose C6H12O6?

A 6.0 x 1022

* B 3.6 x 1023

C 6.0 x 1023

D 3.6 x 1024

2 Nitrogen dioxide reacts with iodide ions under acidic conditions according to
the equation shown.

NO2 + 2I + 2H+ NO + I2 + H2O

How many moles of electrons are gained by 2 moles of nitrogen dioxide?

A 2

B 3

* C 4

D 5

ACJC 2010 8872/01/Aug/10 Preliminary Examination


3

3 Thallium(III) ions can oxidize iodide ions to iodine.

25.0 cm3 of a 0.0480 mol dm-3 solution of Tl3+ ions was added to excess
potassium iodide solution. The liberated iodine is titrated against 0.106 mol dm-3
sodium thiosulfate. 22.60 cm3 of sodium thiosulfate solution was required to
react with iodine in the following reaction.

I2 + 2S2O32- 2I- + S4O62-

What was the oxidation state of Tl after its reaction with potassium iodide?

A -1

B 0

* C +1

D +2

4 10 cm3 of a hydrocarbon CxHy was exploded with an excess of oxygen. There


was a contraction of 30 cm3. When the product was treated with a solution of
sodium hydroxide, there was a further contraction of 40 cm3. What is the
molecular formula of the hydrocarbon?

[All gas volumes are measured at r.t.p.]

A C3 H6

B C3 H8

* C C4 H8

D C4H10

ACJC 2010 8872/01/Aug/10 [Turn over


4

5 To determine the percentage of nitrogen present in a snack, 1.0 g of the snack


was boiled with concentrated sulfuric acid to convert all the nitrogen into
ammonium sulfate. The ammonium salt obtained was then boiled with excess
aqueous sodium hydroxide to liberate the ammonia which was passed into
25.0 cm 3 of 0.20 mol dm-3 HCl. The unreacted HCl required 20.0 cm3 of 0.10
mol dm-3 NaOH for complete neutralisation.

What is the percentage by mass of nitrogen in the snack?

A 2.8%

* B 4,2%

C 6.4%

D 8.4%

6 Which of the following electronic configuration represents an element that forms


a simple ion with a charge of 2?

A 1s2 2s2 2p6 3s2

* B 1s2 2s2 2p6 3s2 3p4

C 1s2 2s2 2p6 3s1

D 1s2 2s2 2p6 3s2 3p5

7 Which of the following particles have more neutrons than protons and more
electrons than neutrons?

A NO2+

13
* B CO32

32
C S2
14
D N2+

ACJC 2010 8872/01/Aug/10 Preliminary Examination


5

8 Which of the following best describes the change in the bond angle in SO2
when the ion SO32- is formed?

A Increase to approximately 120o

* B Increase to approximately 107o

C Decrease to approximately 120o

D Decrease to approximately 107o

9 Which of the following sketches shows the correct trend in the stated property, for the
elements in the third period of the Periodic Table?

A *C First I.E / kJ mol-1


Atomic Radius / nm

Na Mg Al Si P S Cl Na Mg Al Si P S Cl

B D
Melting Point / C Electrical Conductivity

Na Mg Al Si P S Cl Na Mg Al Si P S Cl

ACJC 2010 8872/01/Aug/10 [Turn over


6

10 Which statement does not describe the chemical properties of Period 3


compounds?

A SiO2 is formed from the hydrolysis of SiCl4.

B Al2O3 is soluble in both acid and base.

* C P4O10 can be mixed in water to give a solution of pH 9.

D The acidity of Period 3 chlorides in water increases from NaCl to PCl5.

11 An element Z has second ionisation energy higher than that of the element
preceding it and the element after it in the Periodic Table.

What is the atomic number of Z?

A 10 *B 13 C 14 D 15

12 An unknown solid melts sharply just above 180oC. It does not conduct
electricity in its molten state. It dissolves readily in organic solvents.

What is the most likely structure of the unknown solid?

A An ionic crystal

B A metallic crystal

* C A molecular crystal

D A macromolecular crystal

ACJC 2010 8872/01/Aug/10 Preliminary Examination


7

13 The following is the reaction equation for a compound XY:

XY(g) X(g) + Y(g)

The rate equation for the reaction is rate = k[XY] and the rate constant. k is
ln 2
6.93 s-1. Given that the half-life, t1/2 = , determine the time taken for a
k
sample of 1.80 mol dm-3 to decrease to 0.225 mol dm-3.

A 0.1 s

* B 0.3 s

C 0.4 s

D 0.8 s

14 For which of the following is the lattice energy likely to have the largest
numerical value?

* A Lithium fluoride

B Lithium iodide

C Sodium chloride

D Sodium fluoride

15 Consider the following equilibrium reaction:

3Fe(s) + 4H2O(g) Fe3O4(s) + 4H2(g)

Which of the following gives the unit for the equilibrium constant Kc?

A mol dm-3

B mol-2 dm6

C mol4 dm-12

* D dimensionless

ACJC 2010 8872/01/Aug/10 [Turn over


8

16 How much water must be added to 2.0 dm3 of a solution of a strong acid of pH
2.0 in order to increase the pH to 3.0?

A 1.0 dm3

B 10 dm3

* C 18 dm3

D 20 dm3

17 The bond dissociation energy of HBr is 362 kJ mol-1. In which of the following
processes is 362 kJ released?

* A H(g) + Br(g) HBr(g)

B HBr(g) H(g) + Br(g)

C H2(g) + Br2(g) HBr(g)

D HBr(g) H2(g) + Br2(g)

18 Which one of the following is true about the effect of a catalyst?

A It affects the position of equilibrium in a reversible reaction.

B It affects the enthalpy change of a reaction.

* C It increases the rate of forward reaction.

D It increases the equilibrium constant for the forward reaction.

ACJC 2010 8872/01/Aug/10 Preliminary Examination


9

19 For the equilibrium

HSO4-(aq) + HPO42-(aq) SO42-(aq) + H2PO4-(aq)

Which one of the following is a Bronsted-Lowry acid-base pair?

Base Conjugate Acid

* A HPO42- H2PO4-

B HPO42- SO42-

C HSO4- HPO42-

D HSO4- SO42-

20 How many alcohol structural isomers does molecular formula, C4H10O have?

A 1

B 2

C 3

* D 4

21 Alkanes undergo free radical substitution with Cl2 in the presence of ultraviolet
light.

Which of the following could have been a possible starting material for 2-
chloro-2-methyl-pentane?

A CH3CH2CH2CH2CH3

* B CH3CH(CH3)CH2CH2CH3

C CH3CH2CH(CH3)CH2CH3

D CH3CH2CH2CH2CH2CH3

ACJC 2010 8872/01/Aug/10 [Turn over


10

22 Halogenalkanes have a wide variety of commercial uses. For which group of


compounds is the breaking of the carbon-halogen bond a cause of a major
environmental problem?

A Anaesthetic

* B CFCs

C Flame retardants

D Plastics

23 A compound X, of molecular formula C5H8O3, showed the following behaviour:

It gave orange precipitate with 2,4-dinitrophenylhydrazine

It gave yellow precipitate when warmed with alkaline aqueous iodine

It showed no reaction with Fehlings reagent

It reacted with Na2CO3 and gave off a colourless and odourless gas
which formed white precipitate when passed through limewater

A CHOCH2CH2CH2CH3

B CHOCH2CH2CH2COOH

C CH3CH2COCH2COOH

* D CH3COCH2CH2COOH

24 A sample of ethanal is treated with HCN in the presence of traces of NaCN.


The organic product is then refluxed with LiAlH4 in dry ether.

What will be the final product?

* A CH3CH(OH)CH2NH2

B CH3CH(OH)COOH

C CH3CH(OH)CONH2

D CH3COCOOH

ACJC 2010 8872/01/Aug/10 Preliminary Examination


11

25 Ethanedioic acid can be produced by the following steps:


H H O O
H H
Stage I Stage II
C C H C C H HO C C OH

H H OH OH

Which reagents are used for stages I and II?

Stage I Stage II

A Cold alkaline KMnO4 Acidified K2Cr2O7, distil

* B Cold alkaline KMnO4 Acidified K2Cr2O7, heat under reflux

C Cold alkaline KMnO4 Acidified KMnO4, distil

D Cold alkaline KMnO4 Acidified KMnO4, heat under reflux

ACJC 2010 8872/01/Aug/10 [Turn over


12

Part 2

From questions 26 to 30, one or more of the three numbered statements 1 to 3 may be
correct.

Decide whether each of the statements is or is not correct (you may find it useful to put a
tick against the statements that you consider to be correct).

The responses A to D should be selected on the basis of

A B C D

1, 2 and 3 are 1 and 2 are correct 2 and 3 are correct Only 1 is correct
correct

No other combination of statements is used as a correct response.

26 Which of the following contains two covalent chlorides and two covalent
oxides?

* 1 SiCl4 BeCl2 SiO2 NO2

2 BCl3 AlCl3 Al2O3 Cl2O

3 SCl2 CsCl TiO2 BaO2

27 Which of the following statements correctly describes the characteristics of the


elements within any one particular group of the Periodic Table?

* 1 The first ionisation energy of the elements generally decreases with


increasing atomic number.

2 The melting point of the elements generally increases with increasing


atomic number.

3 The elements are either all metals or all non-metals.

ACJC 2010 8872/01/Aug/10 Preliminary Examination


13

A B C D

1, 2 and 3 are 1 and 2 are correct 2 and 3 are correct Only 1 is correct
correct

No other combination of statements is used as a correct response.

28 Which of the following properties can be attributed to hydrogen bonding?

* 1 Ammonia is very soluble in water.

* 2 The relative molecular mass of ethanoic acid is 120 in benzene.

* 3 Ice is less dense that water.

29 OH

Cl
When CH 3 is added to concentrated H2SO4 at about 170 oC.
What product(s) is/are formed?

1 OH

CH 3

* 2

Cl
CH 3

* 3

Cl
CH 3

ACJC 2010 8872/01/Aug/10 [Turn over


14

A B C D

1, 2 and 3 are 1 and 2 are correct 2 and 3 are correct Only 1 is correct
correct

No other combination of statements is used as a correct response.

30 Compound Y occurs in bananas and compound Z in pears.

CH3 CO2CH 2CH2 CHCH3 CH3 CO2CH 2CH2 CH 2 CH 2CH 3

CH 3
Y Z

Which statements are correct?

* 1 Y and Z are both esters

* 2 Y and Z are structural isomers

B 3 Y and Z have identical physical properties

ACJC 2010 8872/01/Aug/10 Preliminary Examination


2

Section A

Answer all questions in the spaces provided on the question paper

1 Deficiency of iodine, affecting 2 billions people worldwide, is one of the chief causes of
preventable mental retardation which produces a typical reduction in Intelligence Quotient
(IQ) of 10 to 15 IQ points.

Victims of mental retardation may exhibit delay in oral language development, deficiency in
memory skills, difficulty in learning social rules and acquiring problem solving skills.

The best way to get the daily requirement of iodine is to eat a balanced diet that contains a
variety of foods from the food guide pyramid. Most people are able to meet the daily
recommendations by eating seafood, iodized table salt, and plants grown in iodine-rich soil.

Iodized table salt contains potassium iodate(V), KIO3. The amount of potassium iodate(V)
present in the iodized table salt can be determined by completely dissolving it in excess
acidified potassium iodide follow by titrating it with sodium thiosulfate.

The Food and Nutrition Board recommends a minimum dietary intake of 130 mcg of iodine
per day for a 12-month-old infant.
1 mcg = 1 x 10-6 g

(a) Calculate the mass of sodium thiosulfate needed to prepare 1.50 x 10-3 mol dm-3
sodium thiosulfate in 250 cm3 volumetric flask?

[1]
(b) 5.00 g iodized table salt containing potassium iodate(V) is completely dissolved
in 250 cm3 excess acidified potassium iodide.
IO3- + 5I- + 6H+ 3I2 + 3H2O

The iodine produced in the above reaction reacts with sodium thiosulfate,
Na2S2O3 according to the equation shown:

2S2O32- + I2 S4O62- + 2I-

25.0 cm3 of the resultant solution requires 15.00 cm3 of 1.50 x 10-3 mol dm-3
sodium thiosulfate to completely react with the iodine formed. Calculate the
amount of iodine in 25.0 cm3 of the resulting solution.

[1]

ACJC 2010 8872/02/Aug/10 Preliminary Examination


3

(c) Calculate the amount of potassium iodate(V) in 250 cm3 of the resulting solution

[2]
(d) Calculate the mass of potassium iodate(V) in mcg in 1.00 g iodized table salt.

[2]
(e) Potassium iodate(V) is metabolized by an enzyme in the body to form iodine.
Calculate the mass of iodine formed when 1.00 g of iodized salt is being
metabolized. State the assumption made in your calculation.

[3]
(f) Calculate the minimum mass of iodized table salt a 12-month-old infant must take
per day to prevent mental retardation.

[1]

[Total: 10]

ACJC 2010 8872/02/Aug/10 [Turn over


4

2 Electronegativity is a measure of the tendency of an atom to attract a bonding pair of


electrons.
(a) (i) Sketch the general trend of the electronegativity across the third period of
the Periodic Table.

(ii) Suggest an explanation for your sketch.

[4]

ACJC 2010 8872/02/Aug/10 Preliminary Examination


5

(b) Some bond energy values are given in the table below.

Bond Bond energy / kJ mol-1 Bond Bond energy / kJ mol-1

HH 436 HH 436

PP 208 Cl Cl 244

PH H Cl 431

(i) Explain, using H H as an example, what is meant by the term bond


energy.

(ii) The P H bond energy is the mean (average) of the H H and P P


values. Calculate the P H bond energy.

(iii) Explain why the H Cl bond energy is not the mean of the H H and
Cl Cl values, and comment on the value indicated in the table.

..
[6]

[Total: 10]

ACJC 2010 8872/02/Aug/10 [Turn over


6

3 Nylon is a type of synthetic polymer which was first produced on February 28, 1935 by Wallace
Carother at DuPont. It is a thermoplastic silky material and has substituted silk in many different
products after silk became scarce during World War II. It has even replaced silk in military
applications such as parachutes, and was used in many types of vehicle tires. Nylon fibres are
used in applications, including fabrics, carpets and ropes.
Nylons are condensation copolymers formed by reacting equal parts of a nitrogen compound
and a dicarboxylic acid. In the production of nylon, 1,4-dichlorobutane is commonly used as a
precursor.
1,4-dichlorobutane is a product formed from the reaction between butane and chlorine.
(a) (i) Give the conditions for the reaction between butane and chlorine.

..

(ii) What type of reaction is this?

..

(iii) The reaction between butane and chlorine generates a lot of other side
products. Draw the structural formula of two other dichlorobutanes that
could be formed in this reaction.

[4]

(b) 1,4-dichlorobutane can be burned in excess oxygen to generate energy.


However it is not a good source of fuel as a harmful gas, which bleaches red
litmus paper, is given off in the process.

(i) Write an equation for the reaction between between 1,4-dichlorobutane


and oxygen.

ACJC 2010 8872/02/Aug/10 Preliminary Examination


7

(ii) With the aid of a Data Booklet, quote relevant information and calculate
the enthalpy change for the reaction in (b)(i).

(iii) The actual enthalpy change of the reaction in (b)(i) is -2700 kJ mol-1.
Account for the difference between the calculated and actual value.

.. [6]

[Total: 10]

ACJC 2010 8872/02/Aug/10 [Turn over


8

4 A sequence of reactions, starting from 1-chloropropane, is shown below.

H H Cl H H OH
I
H C C C H H C C C H

H H H H H H

II Acidified K2Cr2O7,
distil

A
B
III

IV

C
H H OH
V
H C C C COOH +NaOH (aq)

H H H

Compound X

VI + CH3CH2OH
and conc. H2SO4
heat

ACJC 2010 8872/02/Aug/10 Preliminary Examination


9

(a) In the appropriate boxes draw the structures of compounds A, B, C and D. [4]

(b) For the reactions in the scheme shown above, state the reagents and conditions
for:
(i) reaction III

..

(ii) reaction IV

..
[2]

(c) State the type of reaction in reaction III.

...
[1]

(d) Describe a chemical test to distinguish between compounds B and compound X.


State observations and write relevant equation(s) to show the reaction(s).

[3]

[Total: 10]

ACJC 2010 8872/02/Aug/10 [Turn over


10

Section B

Answer two of the three questions in this section on separate paper

5 (a) An organic compound A has the following composition by mass: C, 35.0%; H,


6.6%; Br, 58.4%. It has a relative molecular mass of 136.9. Determine the
[2]
chemical formula of A.
(b) Heating A under reflux with aqueous NaOH gives a compound B and heating B
with an excess of acidified K2Cr2O7 gives a compound C.
When 2,4-dinitrophenylhydrazine reagent is added to C, an orange precipitate D
was produced. C did not give a silver mirror with Tollens reagent but it
produced a yellow precipitate and an organic compound E when reacted with
aqueous alkaline iodine.
Heating A under reflux with ethanolic NaCN gives compound F.
Heating B under reflux with concentrated H2SO4 gives three isomeric products,
G, H and I, which decolourise aqueous bromine.
[15]
Draw the structures A to I and explain the reactions.
(b) Arrange the following in order of increasing boiling points and explain your
reasoning in terms of bonding and structure.

CH3CH(C2H5)C3H7 CH3CH2C(CH3)2CH3 CH3(CH2)5CH3 [3]

[Total :20]

ACJC 2010 8872/02/Aug/10 Preliminary Examination


11

6 (a) Sodium and phosphorus react with chlorine to form NaCl and PCl3 respectively.
(i) Describe the chemical reaction, if any, of the two chlorides with water.
Suggest the colour of methyl orange when it is added separately to each
of the solutions.

(ii) Explain, with the aid of appropriate equations, the difference in the colour
of the indicator in these solutions.
[7]
(b) Phosphorous trichloride and chlorine react to form phosphorous pentachloride
gas in the equation shown below:

PCl3(g) + Cl2(g) PCl5(g)

(i) Write an expression for the equilibrium constant Kc for this reaction, giving
its units.

(ii) An equilibrium mixture in a 2 dm3 vessel is found to contain 0.0406 mol


PCl3, 0.170 mol Cl2 and 0.302 mol PCl5 at 450 K. Calculate the Kc at this
temperature.

(iii) Describe and explain what would happen to the concentration of PCl5 if

I more chlorine is added to the reaction vessel


II the size of the reaction vessel is reduced to 1 dm3

(iv) Using the Maxwell-Boltzmann Distribution curve, explain what would


happen to the rate of the forward reaction if the reaction is carried out at
500 K. [8]
(c) 2-chlorobutane can react with hydroxide ions under two different sets of reaction
conditions to give two different products.

I II
C4H8 CH3CHClCH2CH3 C4H10O
A B

(i) Describe the conditions necessary for each of the reactions I and II.

(ii) Describe the observations when 2-chlorobutane is heated with ethanolic


AgNO3(aq).

(iii) Using the Data Booklet, describe and explain, the observations when 2-
iodobutane is used for the reaction in (ii).
[5]

ACJC 2010 8872/02/Aug/10 [Turn over


12

[Total: 20]
7
Blood plasma is a biological fluid that plays an important role in maintaining pH in the
body.
In the blood plasma, the equilibrium between carbonic acid, H2CO3(aq), and
hydrogencarbonate ion, HCO3(aq), buffers pH changes.

H2CO3(aq) H+(aq) + HCO3(aq)

The Ka for this equilibrium is 7.90 107 mol dm3.


At body temperature, the pH of the arterial blood plasma is 7.40.
If the pH falls below this normal value, a condition termed acidosis is produced. If the
pH rises above this normal value, the condition is termed alkalosis.
The concentration of H2CO3(aq) is controlled by respiration through the lungs.
H2CO3(aq) is in equilibrium with dissolved CO2.

H2CO3(aq) CO2(aq) + H2O(l)

In the lungs, excess dissolved CO2(aq) is exhaled as CO2(g).

CO2(aq) CO2(g)

During heavy exercise, lactic acid is released into the blood and is buffered by the blood
plasma.
O CH 3

HO OH
Lactic acid
This eventually leads to an increase in CO2(aq) concentration and stimulates increased
breathing.
(a) (i) Explain what is meant by a buffer solution.
(ii) Write an equation to show how blood plasma can buffer the pH change
when lactic acid is released into the blood.
(iii) Write an expression for the acid dissociation constant, Ka, of H2CO3(aq)
and use it to determine the [HCO3(aq)]/[H2CO3(aq)] ratio in the blood
plasma.
Comment on this ratio.

ACJC 2010 8872/02/Aug/10 Preliminary Examination


13

(iv) Using the information given, explain how the production of lactic acid [7]
leads to the increase in rate of breathing.
(b) Lactic acid is converted to glucose via gluconeogenesis in the liver.

OH OH O

HO
H

HO HO

Glucose

(i) Name the functional group(s) present in glucose.


(ii) Describe a simple chemical test that you could use to distinguish between
lactic acid and glucose. You are to include reagents and conditions,
observations and balanced equation(s) in each case. [4]

(c) Sodium, aluminium and phosphorous form the oxides, Na2O, Al2O3 and P4O10.
Na2O is a basic oxide which dissolves in water readily to form a strong alkali.
(i) Define strong alkali.
(ii) 0.93 g of Na2O is completely dissolved in 300 cm3 of water at 298 K.
Calculate the pH of the solution.

(iii) Classify Al2O3 and P4O10 as acidic, basic or amphoteric oxide.

(iv) Illustrate your answers in c(iii), by writing relevant balanced chemical


equations with state symbols when the respective oxide reacts with water, [9]
acid and alkali when applicable.

[Total: 20]

ACJC 2010 8872/02/Aug/10 [Turn over


2

Section A
Answer all questions in the spaces provided on the question paper

1 Deficiency of iodine, affecting 2 billions people worldwide, is one of the chief causes of
preventable mental retardation which produces a typical reduction in Intelligence Quotient
(IQ) of 10 to 15 IQ points.

Victims of mental retardation may exhibit delay in oral language development, deficiency in
memory skills, difficulty in learning social rules and acquiring problem solving skills.

The best way to get the daily requirement of iodine is to eat a balanced diet that contains a
variety of foods from the food guide pyramid. Most people are able to meet the daily
recommendations by eating seafood, iodized table salt, and plants grown in iodine-rich soil.

Iodized table salt contains potassium iodate (V), KIO3. The amount of potassium iodate (V)
present in the iodized table salt can be determined by completely dissolving it in excess
acidified potassium iodide follow by titrating it with sodium thiosulfate.

The Food and Nutrition Board recommends a minimum dietary intake of 130 mcg of iodine
per day for a 12-month-old infant.
1 mcg = 1 x 10-6 g

(a) (i) Calculate the mass of sodium thiosulfate needed to prepare [1]
1.50 x 10-3 mol dm-3 sodium thiosulfate in 250 cm3 volumetric flask?
No of moles of Na2S2O3 = 1.50 x 10-3 x 0.250 = 3.75 x 10-4
Mass of Na2S2O3 = 3.75 x 10-4 x 158.2 = 0.0593 g

(ii) 5.00 g iodized table salt containing potassium iodate (V) is completely [1]
dissolved in 250 cm3 excess acidified potassium iodide.
IO3- + 5I- + 6H+ 3I2 + 3H2O

The iodine produced in the above reaction reacts with sodium thiosulfate,
Na2S2O3.

2S2O32- + I2 S4O62- + 2I-

25.0 cm3 of the resultant solution requires 15.00 cm3 of 1.50 x 10-3 mol dm-3
sodium thiosulfate to completely react with the iodine formed. Calculate the
amount of iodine in 25.0 cm3 of the resulting solution.
No of moles of I2 = 1.50 x 10-3 x 0.015 x = 1.125 x 10-5 = 1.13 x 10-5 [1]
(iii) Calculate the amount of potassium iodate (V) in 250 cm3 of the resulting [2]
solution
No of moles of I2 in 250 cm3 of the resulting solution
= 1.125 x 10-5 x 10 = 1.125 x 10-4
No of moles of IO3- in 250 cm3 of the resulting solution

ACJC 2010 8872/02/Aug/10 Preliminary Examination


3

= 1.125 x 10-4 x 1/3 = 3.75 x 10-5


(iv) Calculate the mass of potassium iodate (V) in mcg in 1.00 g iodized table [2]
salt.
Mass of KIO3 in 5.00 g of iodized table salt
= 3.75 x 10-5 x 214.1 = 8.029 x 10-3 g
Mass of KIO3 in 1.00 g of iodized table salt
= 8.029 x 10-3 / 5 = 1.606 x 10-3 g = 1610 mcg

(v) Potassium iodate (V) is metabolized by an enzyme in the body to form [3]
iodine. Calculate the mass of iodine formed when 1.00 g of iodized salt is
being metabolized. State the assumption made in your calculation.
No. of moles of KIO3 in1.00 g of iodized salt
= 1.606 x 10-3 214.1 = 7.50 x 10-6
No. of moles of I2 formed
= 7.50 x 10-6 x 3 = 2.25 x 10-5
Mass of iodine formed
= 2.25 x 10-5 x 254 5. = 5.72 x 10-3 g
Assumption: There is no loss of iodine during the metabolism.

(vi) Calculate the minimum mass of iodized table salt a 12-month-old infant [1]
must take per day to prevent mental retardation.

Minimum mass = (130 572) X 1.00 = 0.227 g

ACJC 2010 8872/02/Aug/10 [Turn over


4

2 Electronegativity is a measure of the tendency of an atom to attract a bonding pair of electrons.


(a) (i) Sketch the general trend of the electronegativity across the third period of the
Periodic Table.

*values not required*


[2]
(ii) Suggest an explanation for your sketch.
Across the Period, nuclear charge increases, but shielding effect
remains approximately constant. This leads to an increase in attraction
for the electrons.
[2]
(b) Some bond energy values are given in the table below.

bond bond energy / kJ mol-1 bond bond energy / kJ mol-1

HH 436 HH 436

PP 208 Cl Cl 244

PH H Cl 431

(i) Explain, using H H as an example, what is meant by the term bond


energy.

Bond dissociation energy is the heat absorbed when one mole of a


given bond is broken into gaseous molecules/atoms(1). Using H H
as an example, 436 kJ of energy is required for the following equation:

H2(g) 2H(g)

1 mark for using H2 as example


[2]
(ii) The P H bond energy is the mean (average) of the H H and P P
[1]

ACJC 2010 8872/02/Aug/10 Preliminary Examination


5

values. Calculate the P H bond energy.

Bond energy of P H = (436 + 208)


= 322 kJ mol-1

(iii) Explain why the H Cl bond energy is not the mean of the H H and
Cl Cl values, and comment on the value indicated in the table.

Chlorine is more electronegative than hydrogen. The covalent bond


between the atoms is polarized, giving rise to a stronger bond.
[3]

3 Nylon is a type of synthetic polymer which was first produced on February 28, 1935 by Wallace
Carother at DuPont. It is a thermoplastic silky material and has substituted silk in many
different products after silk became scarce during World War II. It has even replaced silk in
military applications such as parachutes, and was used in many types of vehicle tires. Nylon
fibres are used in applications, including fabrics, carpets and ropes.
Nylons are condensation copolymers formed by reacting equal parts of a nitrogen compound
and a dicarboxylic acid. In the production of nylon, 1,4-dichlorobutane is commonly used as a
precursor.
1,4-dichlorobutane is a product formed from the reaction between butane and chlorine.
(a) (i) Give the conditions for the reaction between butane and chlorine. [1]
UV

(ii) What type of reaction is this?


free radical substitution [1]

(iii) The reaction between butane and chlorine generates a lot of other side [2]
products. Draw the structural formula of two other dichlorobutanes that
could be formed in this reaction.

1,2-dichlorobutane
1,3-dichlorobutane
2,3-dichlorobutane

(b) 1,4-dichlorobutane can be burned in excess oxygen to generate energy. However


it is not a good source of fuel as a harmful gas, which bleaches red litmus paper,
is given off in the process.

(i) Write an equation for the reaction between between 1,4-dichlorobutane and
oxygen. [1]

CH2ClCH2CH2CH2Cl + 6O2 4CO2 + 4H2O + Cl2


State symbol is not required.

ACJC 2010 8872/02/Aug/10 [Turn over


6

(ii) With the aid of a Data Booklet, quote relevant information and calculate the [4]
enthalpy change for the reaction in (b)(i).

C Cl: 340 kJ mol-1


C H : 410 kJ mol-1
C C : 350 kJ mol-1
O = O : 496 kJ mol-1
C = O : 740 kJ mol-1
O H : 460 kJ mol-1
Cl Cl: 244 kJ mol-1

Bond breaking = 2 (C Cl) + 8 (C H) + 3 (C C) + 6 (O = O)


= 2(340) + 8(410) + 3(350) + 6(496)
= +7986kJ mol-1
Bond forming = 8 (C = O) + 8 (O H) + 1 (Cl Cl)
= 8(740) + 8(460) + 244
= - 9844 kJ mol-1
Enthalpy change for the reaction = 7986 9844 = -1858 kJ mol-1

(iii) The actual enthalpy change of the reaction in (b)(i) is -2700 kJ mol-1. [1]
Account for the difference between the calculated and actual value.

Water produced is in liquid state and when using bond energy to


calculate the enthalpy change of reaction, it is assumed that all the
products and reactants are in gaseous state.

[Total: 10]

ACJC 2010 8872/02/Aug/10 Preliminary Examination


7

4 A sequence of reactions, starting from 1-chloropropane, is shown below.

H H Cl H H OH [2]
I
H C C C H H C C C H

H H H H H H

II Acidified K2Cr2 O7 ,
distil

A
B
H H O
H H OH III
H C C C
H C C C CN
H
H H
H H H

IV

C
H H OH H H OH
V
H C C C COOH +NaOH H C C C COO- Na +

H H H H H H

Compound X

+ CH 3CH2 OH
VI and conc. H 2 SO 4
heat

D H H OH

H C C C COOCH2 CH3

H H H

ACJC 2010 8872/02/Aug/10 [Turn over


8

(a) In the appropriate boxes draw the structures of compounds A, B, C and D. [4]

(b) For the reactions in the scheme shown above, state the reagents and conditions
for:
(i) reaction III

HCN with traces of NaCN, 10 20 oC

(ii) reaction IV [2]

H2SO4 (aq) or HCl (aq) or H+ (aq) or dil H+, heat under reflux

(c) State the type of reaction in reaction III


[1]

Nucleophilic addition

(d) Describe a chemical test to distinguish between compounds B and compound X. [3]
State observations and write relevant equations to show the reaction.
Reagent / condition Observation Equation
Na2CO3 Effervescence CH3CH2CH(OH)COOH +
observed for X and the Na2CO3
gas produced forms 2CH3CH2CH(OH)COO-
white ppt in limewater. Na+ + CO2 + H2O
No visible reaction for
B

[Total: 10]

ACJC 2010 8872/02/Aug/10 Preliminary Examination


1

Section B
Answer two of the three questions in this section on separate paper

1 (a) An organic compound A has the following composition by mass: C, 35.0%; H,


6.6%; Br, 58.4%. It has a relative molecular mass of 136.9. Determine the
[2]
chemical formula of A.

C H Br
% by mass 35.0 6.6 58.4
No. of moles 35/12 = 2.92 6.6 0.731
Smallest ratio 4 9 1

E.F. = C4H9Br
M.F. = C4H9Br

(b) Heating A under reflux with aqueous NaOH gives a compound B and heating B
with an excess of acidified K2Cr2O7 gives a compound C.
When 2,4-dinitrophenylhydrazine reagent is added to C, an orange precipitate D
was produced. C did not give a silver mirror with Tollens reagent but it produced
a yellow precipitate and an organic compound E when reacted with aqueous
alkaline iodine.
Heating A under reflux with ethanolic NaCN gives compound F.
Heating B under reflux with concentrated H2SO4 gives three isomeric products, G,
H and I, which decolourise aqueous bromine.
[15]
Draw the structures A to I and explain the reactions.

Deductions:
Heating A with NaOH to give B B is an alcohol (or 1o or 2o alcohol)
Heating B with K2Cr2O7 gives C C is either a ketone or an acid
C reacts with 2,4-DNPH to give D C is a ketone or aldehyde
C does not react with Tollens C is a ketone (Or C is not an aldehyde)
C reacts with aqueous alkaline I2 to give yellow crystals and E
C has this structure: CH3-C=O
E is a salt of an acid
(not accepted if students write only this structure: CH3-C-OH as previously
it was deduced that it is a ketone)
Heating A under reflux with ethanolic NaCN gives F nucleophilic
substitution, F is a nitrile

ACJC 2010 8872/02/Aug/10 Preliminary Examination


2

Heating A with concentrated H2SO4 gives 3 products elimination


reaction. H2O is removed. G, H and I are alkenes

Structures:
A: CH3CH(Br)CH2CH3
B: CH3CH(OH)CH2CH3
C: CH3COCH2CH3
H CH3

O2N N N C

CH2 CH3

D: NO2

E: CH3CH2COO-
F: CH3CH(CN)CH2CH3
H CH2 CH 3
C C
G: H H
H3 C CH3
C C
H: H H
H3 C H
C C

I: H CH3

(b) Arrange the following in order of increasing boiling points and explain your
reasoning in terms of bonding and structure.

CH3CH(C2H5)C3H7 CH3CH2C(CH3)2CH3 CH3(CH2)5CH3 [3]

CH3CH2C(CH3)2CH3 CH3CH(C2H5)C3H7 CH3(CH2)5CH3


(1) (2) (3)

(1) has a smaller electron cloud than (2) and (3) and hence has a weaker id-
id intermolecular forces of attraction. OR (1) has a smaller carbon number.

(2) and (3) are isomers, but (2) is a branched structure and (3) is a straight
chain.
(2) has a smaller surface area of contact between molecules than (3), hence
has weaker id-id intermolecular forces of attraction. Therefore, lesser
energy needed to overcome weaker id-id.

ACJC 2010 8872/02/Aug/10 [Turn over


3

2 (a) Sodium and phosphorus react with chlorine to form NaCl and PCl3 respectively.
(i) Describe the chemical reaction, if any, of the two chlorides with water.
Suggest the colour of methyl orange when it is added separately to each of
the solutions.

NaCl undergoes hydration to form ions through ion-dipole interaction.


Methyl orange turns yellow when added to this solution.
PCl3 undergoes hydrolysis to form acidic solution. Methyl orange
[4]
turns red.
(ii) Explain, with the aid of appropriate equations, the difference in the colour of
the indicator in these solutions.

NaCl(s) + aq Na+(aq) + Cl-(aq)


The hydrated Na+ has low charge density, it does not hydrolysed in
water, thus pH remains at 7.

PCl3(s) + 3H2O H3PO3(aq) + 3HCl(aq)


PCl3 undergoes hydrolysis to form acidic solution.

[3]
(b) Phosphorous trichloride and chlorine react to form phosphorous pentachloride
gas in the equation shown below:

PCl3(g) + Cl2(g) PCl5(g)

(i) Write an expression for Kc for this reaction, giving its units.
[ PCl5 ]
Kc = mol 1dm 3
[ PCl3 ][Cl2 ] [2]
(ii) An equilibrium mixture in a 2 dm3 vessel is found to contain 0.0406 mol
PCl3, 0.170 mol Cl2 and 0.302 mol PCl5 at 450 K. Calculate the Kc at this
temperature.
0.302
Kc = 2
00406 0.170
( )( )
2 2
= 87.5 mol 1dm 3
[1]
(iii) Describe and explain what would happen to the concentration of PCl5 if

I more chlorine is added to the reaction vessel


II the size of the reaction vessel is reduced to 1 dm3

I When more chlorine is added, the position of the equilibrium will


shift to the right, increasing the concentration of PCl5
II When the pressure is increased, the position of equilibrium will
shift to the right, increasing the concentration of PCl5. [2]

ACJC 2010 8872/02/Aug/10 Preliminary Examination


4

(iv) Using the Maxwell-Boltzmann Distribution curve, explain what would


happen to the rate of the forward reaction if the reaction is carried out at
500 K.

As temperature increase, the peak moves to the right. There is an


increase in the number of particles with kinetic energy E(1)a. As a
result, the frequency of effective collision increases(1), resulting in an
increase in the rate of the forward reaction.
[3]
(c) 2-chlorobutane can react with hydroxide ions under two different sets of reaction
conditions to give two different products.

I II
C4H8 CH3CHClCH2CH3 C4H10O
A B

(i) Describe the conditions necessary for each of the reactions I and II.

I Heat with alcoholic OH-


II Heat with aqueous OH-
[2]
(ii) Describe the observations when 2-chlorobutane is heated with ethanolic
AgNO3(aq).

White ppt observed.


(iii) Using the Data Booklet, describe and explain, the observations when 2-
iodobutane is used for the reaction in (ii).

Yellow ppt is observed within a shorter time. The substitution requires


the breaking of the C-X bond. C-I having a lower bond energy than C-
Cl, will result in a faster precipitation.

[3]
ACJC 2010 8872/02/Aug/10 [Turn over
5

3
Blood plasma is a biological fluid that plays an important role in maintaining pH in the
body.
In the blood plasma, the equilibrium between carbonic acid, H2CO3(aq), and
hydrogencarbonate ion, HCO3(aq), buffers pH changes.

H2CO3(aq) H+(aq) + HCO3(aq)

The Ka for this equilibrium is 7.90 107 mol dm3.


At body temperature, the pH of the arterial blood plasma is 7.40.
If the pH falls below this normal value, a condition termed acidosis is produced. If the pH
rises above this normal value, the condition is termed alkalosis.
The concentration of H2CO3(aq) is controlled by respiration through the lungs.
H2CO3(aq) is in equilibrium with dissolved CO2.

H2CO3(aq) CO2(aq) + H2O(l)

In the lungs, excess dissolved CO2(aq) is exhaled as CO2(g).

CO2(aq) CO2(g)

During heavy exercise, lactic acid is released into the blood and is buffered by the blood
plasma.
O CH 3

HO OH
Lactic acid
This eventually leads to an increase in CO2(aq) concentration and stimulates increased
breathing.
(a) (i) Explain what is meant by a buffer solution. [1]
Buffer solution is solution which maintains a fairly constant pH when
a small amount of acid or base is added.

(ii) Write an equation to show how blood plasma can buffer the pH change
when lactic acid is released into the blood.
H+(aq) + HCO3(aq) H2CO3(aq) [1]

ACJC 2010 8872/02/Aug/10 Preliminary Examination


6

(iii) Write an expression for the acid dissociation constant, Ka, of H2CO3(aq) and [3]
use it to determine the [HCO3(aq)]/[H2CO3(aq)] ratio in the blood plasma.
Comment on this ratio.


[HCO3 ][H+ ]
Ka =
[H2CO3 ]

pH = 7.4 [H+] = 10 7.4 mol dm3


= 3.98 x 108 mol dm3


[HCO3 ](10 -7.40 )
7.90 10 -7 =
[H2CO3 ]


[HCO3 ] 7.90 10 7
= = 19.8
[H2CO3 ] 3.98 10 8

The ratio is high as HCO3 is needed to remove lactic acid in the blood
during heavy exercises.

(iv) Using the information given, explain how the production of lactic acid leads [2]
to the increase in rate of breathing.
When lactic acid is removed,
H2CO3(aq) CO2(aq) + H2O(l) ----- (1)
When [H2CO3] increases, the equilibrium position in (1) shifts right.
Thus, more CO2(aq) is formed.

CO2(aq) CO2(g) ----- (2)


When [CO2(aq)] increases, the equilibrium position in (2) shifts right,
forming more CO2(g) and thus the rate of breathing increases.

(b) Lactic acid is converted to glucose via gluconeogenesis in the liver.


OH OH O

HO
H

HO HO

Glucose

(i) Name the functional group(s) present in glucose. [1]

Aldehyde, primary alcohol and secondary alcohol.

ACJC 2010 8872/02/Aug/10 [Turn over


7

(ii) Describe a simple chemical test that you could use to distinguish the [3]
following lactic acid and glucose. You are to include reagents and
conditions, observations and balanced equation(s) in each case.

r/c: Na2CO3 (aq)


Observation:
Lactic acid- colourless odourless gas gives white ppt in limewater.
Glucose. No gas evolved. No white ppt formed in limewater.

O CH 3 O CH 3

2 + Na2CO3 2 + CO2 + H O
2
+Na- O
HO OH OH

OR

r/c: 2,4 DNPH


Observation:
Lactic acid: No orange ppt formed
Glucose: Orange ppt formed

HO OH

OH OH O + OH
H 2N NH
HN NO2 + H2O
HO
O 2N N
H O2 N
HO
NO2
HO HO HO H

(c) Sodium, aluminium and phosphorous form the oxides, Na2O, Al2O3 and P4O10.
Na2O is a basic oxide which dissolves in water readily to form a strong alkali.
(i) Define strong akali. [1]

Strong base completely dissociates in water to form OH-.


(ii) 0.93 g of Na2O is completely dissolved in 300 cm3 of water at 298 K. [3]
Calculate the pH of the solution.

Na2O(s) + H2O(l) 2Na+(aq) + 2OH(aq)

[OH-] = 2(0.93 62.0) 0.3 = 0.100 mol dm-3

pH = 14 (-lg 0.100) = 13.0

(i) Classify Al2O3 and P4O10 as acidic, basic or amphoterc oxide. [1]

Al2O3 amphoteric oxide


P4O10 acidic oxide

ACJC 2010 8872/02/Aug/10 Preliminary Examination


8

(ii) Illustrate your answers in c(i), by writing relevant balanced chemical


equations with state symbols when the respective oxide reacts with water, [4]
acid and alkali when applicable.

Al2O3(s) + 6H+(aq) 2Al3+(aq) + 3H2O(l)


Al2O3(s) + 2OH(aq) + 3H2O(l) 2[Al(OH)4](aq)
P4O10(s) + 6H2O(l) 4H3PO4(aq)
P4O10(s) + 12OH(aq) 4PO43(aq) + 6H2O(l)

ACJC 2010 8872/02/Aug/10 [Turn over


CATHOLIC JUNIOR COLLEGE

CHEMISTRY 8872/01
Higher 1

Paper 1 Multiple Choice


JC 2 Preliminary Examinations
17th September 2010

50 minutes

Additional Materials: Multiple Choice Answer Sheet


Soft clean eraser
Soft pencil (type B or HB is recommended)
Data Booklet

READ THESE INSTRUCTIONS FIRST

Write in soft pencil.


Do not use paper clips, highlighters, glue or correction fluid.
Write and shade your name, class and NRIC / FIN on the Answer Sheet.

There are thirty questions in this paper. Answer all questions. For each question there are
four possible answers A, B, C and D.
Choose the one you consider correct and record your choice in soft pencil on the separate
Answer Sheet.

Read the instructions on the Answer Sheet very carefully.

Each correct answer will score one mark. A mark will not be deducted for a wrong answer.
Any rough working should be done in this booklet.
You may use a calculator.

There are a total of 12 printed pages.


2
Section A
For each question there are four possible answers, A, B, C and D. Choose the one you
consider to be correct.

1 Which of the following statements about 1 mol of a metal is correct?

A It is liberated by 1 mol of electrons.


B It has the same mass as 1 mol of hydrogen atoms.
C It has the same number of atoms as 1 mol of 12C.
12
D It contains the same number of atoms as 1 mol of hydrogen atoms.

2 10 cm3 of a gaseous hydrocarbon was completely burnt in 100 cm3 of oxygen. The
volume of gas obtained after reaction was 80 cm3. When the gas was shaken with
aqueous sodium hydroxide, the volume decreased to 20 cm3.
Which one of the following represents the structure of the hydrocarbon? (All volumes
are measured at the same temperature and pressure, and water is produced.)

A B

C D

3 Gaseous particle X has a proton number n and a charge of +1.


Gaseous particle Y has a proton number of (n+1) and is isoelectronic with X.
X and Y have the same number of neutrons.
Which one of the following statements that describes X and Y is incorrect?
A The ionic charge on X is half that on Y.
B X has a larger ionic radius than Y.
C X requires less energy than Y when an electron is removed from each
particle.
D X will be deflected by a larger magnitude than Y when each particle is passed
through an electric field.

4 Which of the following groups contains species with linear, trigonal pyramidal and
tetrahedral shapes?
A CO2 SO2 NH3
B HCN H3O+ SiCl4
C SiCl4 NH3 H3O+
D CO2 HCN H3O+

8872/01/CJC JC 2 Prelims Exam 10


3
5 The following graph is obtained when a certain property was plotted against atomic
number.

Atomic number

Which property when plotted against atomic number gives the shape of the above
graph?
A Boiling points of HF, HCl, HBr and HI.
B Melting points of Na, Mg, Al and Si.
C Electronegativity of N, O, F and Ne.
D Ionic radius of Cl, Na+, Mg2+ and Al3+.

6 Which of the following features will contribute to a significant degree of covalent


character in an ionic compound?
A Large cationic radius.
B Small anionic radius.
C Large cationic charge.
D Small anionic charge.

7 Element X is a solid with a very low electrical conductivity at room temperature. It


forms only one chloride, which is a liquid at room temperature and is a non-conductor
of electricity. The chloride hydrolyses in water forming a white solid and a strongly
acidic solution.
Which of the following could be X?
A Aluminium B Phosphorus
C Silicon D Sulfur

8872/01/CJC JC 2 Prelims Exam 10


4
8 The standard enthalpy change of the following reaction is -896.4 kJ mol-1.
2HN3(l) + 2NO(g) H2O2(l) + 4N2(g)

NO (g) H2O2 (g)


-1
H f/kJ mol + 90.3 - 187.8

Using the given information in the above table, what is the standard enthalpy change
of formation, in kJ mol-1, of HN3(l)?
A +264 B +528 C +618 D +632

9 Which equation correctly describes the reaction whose H is the standard enthalpy
change of formation of water at 298 K?
A H+(aq) + OH-(aq) H2O(l)
1
B H2(g) + O2(g) H2O(g)
2
1
C H2(g) + O2(g) H2O(l)
2
D 2H(g) + O(g) H2O(l)

10 Using a colorimeter, the following reaction is studied by finding the time taken for a
coloured reactant, A, to decolourise. The reaction is catalysed by Y.

Y
A+B C+D
The following results are obtained:
Expt Vol. of A Vol. of B Vol. of Y Vol. of H2O Time taken/
no. added/ cm3 added/ cm3 added/ cm3 added/ cm3 s
1 10 20 10 10 20
2 10 10 10 20 40
3 10 20 5 15 40
4 5 20 10 15 40

What is the rate equation for the reaction?


A rate = k[B][Y] B rate = k[A][B]
C rate = k[A][Y] D rate = k[A][B][Y]

11 Lead is the final product formed by a series of changes in which the slowest step is
the radioactive decay of uranium-238. This radioactive decay is a first order reaction
with rate constant of 1.54 x 10-10 year-1.
What would be the age of a rock sample, originally lead-free, in which the molar
proportion of uranium to lead is now 1:15?
A 2.25 x 109 years B 9.00 x 109 years
C 1.35 x 1010 years D 1.80 x 1010 years

8872/01/CJC JC 2 Prelims Exam 10


5
12 The following reaction is one of the steps involved in the manufacture of sulfuric acid.
2 SO2(g) + O2(g) 2 SO3(g) H = -197 kJ mol-1
The operating conditions are:
Temperature: 450 550 oC
Pressure: 10 atm
Catalyst: V2O5 catalyst

Which factor influences the choice of these conditions?


A The V2O5 catalyst increases the equilibrium yield of SO3.
B At lower temperature, the rate of formation of SO3 increases.
C At lower temperature, the equilibrium yield of SO3 increases.
D At lower pressures, the rate of formation of SO3 increases.

13 When calcium carbonate is heated in a closed vessel, it dissociates to calcium oxide


and carbon dioxide according to the following equation.
CaCO3(s) CaO(s) + CO2(g)
Which of the following gives the correct expression for the equilibrium constant, Kc?

[
C a
a O
C ]
O C
] 2
[
C [
a C
O a
]
[
C O
O 3
]

3O
2]
C

[
C

]
A Kc = B Kc =
[
C
a
O
]
[
C
a
C
O
Kc = ]
C Kc = [CO2] D 3

14 NH4+(aq) + NaOH(aq) NH3(g) + H2O(l) + Na+(aq)


Which one of the following is true for the above reaction?
A Nitrogen in NH4+ is oxidised.
B NH4+ is behaving as a base.
C Phenolphthalein can be used as an indicator for the titration.
D The H-N-H bond angle in NH4+ is smaller than that in NH3.

15
Temperature / K Kw / mol2 dm-6
289 5 x 10-15
298 1 x10-14

Which of the following can be used to explain this difference in Kw values?


A H+(aq) and OH-(aq) are formed through endothermic reaction.
B Formation of H+(aq) and OH-(aq) is favoured at lower temperatures.
C The pH of water is independent of the temperature.
D The concentration of water is lower at higher temperature.

8872/01/CJC JC 2 Prelims Exam 10


6
16 Vitamin C has the following structure.

Which of the following shows the correct hybridisation of the carbon atoms?
sp2 sp3
A 6 0
B 4 2
C 3 3
D 2 4

17 An alkene can exist as a pair of geometric isomers. When refluxed with hot,
concentrated acidified potassium manganate(VII), no effervescence was observed.
Which of the following correctly shows the structure of the alkene?

A B

C D

8872/01/CJC JC 2 Prelims Exam 10


7
18 4-propylbenzoic acid has the following structure.

Which of the following pairs of structures correctly shows the major products formed
when 4-propylbenzoic acid is reacted with bromine with iron powder as a catalyst in
the dark?

8872/01/CJC JC 2 Prelims Exam 10


8
19 2-bromo-2-chloro-1-fluoroethylbenzene has the structure below.

Which of the following shows the correct structure when it is refluxed with aqueous
potassium hydroxide?

A B

C D

20 2-hydroxypropanoic acid can be synthesised from ethanal via an intermediate Y:


CH3CHO Y CH3CH(OH)CO2H
ethanal 2-hydroxypropanoic acid
Which of the following shows the correct set of reagents that can be used for this
conversion?
A with cold dilute HCN in trace amount of NaOH, followed by H+ hydrolysis
B with cold dilute NaCN in trace amount of NaOH, followed by H+ hydrolysis
C reflux with aqueous HCN, followed by H+ hydrolysis
D reflux with alcoholic NaCN, followed by H+ hydrolysis

8872/01/CJC JC 2 Prelims Exam 10


9
21 The aromatic compound

was made to react with excess alkaline potassium manganate(VII) solution.


Which of the following shows the correct structure of the product formed?
A B

C D
CH2CO2H

22 1 mol of PCl5 reacts with 1 mol of compound X.


Which of the following could be X?
A HOCC6H4CO2CH3
B HO(CH2)2OH
C HOCH2C6H4OH
D HCO2CH2C6H4CO2H

23 What are the products formed upon warming J with Tollens Reagent?

CH3 CHO

OH CH3

A Silver mirror and a carboxylic acid


B Silver mirror and a carboxylate salt
C Silver mirror, a ketone and a carboxylic acid
D Silver mirror, a ketone and a carboxylate salt

8872/01/CJC JC 2 Prelims Exam 10


10
24 Compounds T and U are isomers with the following structures:

Compound T Compound U

Which of the following sets of reagents can distinguish T from U?


A 2,4-dinitrophenylhydrazine
B Alkaline aqueous iodine
C Alkaline aqueous copper(II) ions
D Phosphorus pentachloride

25 2,4-D and MCPA are two common selective weed killers, with the following
structures.

Which of the following reagents can be used to distinguish between them?


A Sodium metal
B Sodium carbonate
C Aqueous iodine in aqueous sodium hydroxide
D Hot concentrated potassium manganate(VII)

For questions 26 to 30, one or more of the three numbered statements 1 to 3 may be
correct. The responses A to D should be selected on the basis of

A B C D
1, 2 and 3 1 and 2 only 2 and 3 only 1 only is
are correct are correct are correct correct

No other combination of statements is used as a correct response.

26 Which of the following statements are true?


1 H2S is a stronger acid than H2O.
2 The bond angle in H2S is larger than that in H2O.
3 The boiling point of H2S is higher than that of H2O.

8872/01/CJC JC 2 Prelims Exam 10


11
27 The two steps involved in the conversion of compound X to compound Z are as
follows:
XY --- Step I
YZ --- Step II

Y is an intermediate that could be isolated. The energy profile of the reaction is as


shown below.

Energy

X Y

Z
Reaction
Pathway

Which statement correctly describes the reaction?


1 Step I occurs faster than step II.
2 Activation energy for step I is smaller than that for step II.
3 Z is thermodynamically less stable than X.

28 Which sequence below shows the compounds in decreasing order of Ka?


1 CH3CF2CO2H > CH2FCHFCO2H > CHF2CH2CO2H
2 CH2BrCO2H > CH2ClCO2H > CH2FCO2H
3 CH3CH2CO2H > CH2ClCH2CO2H > CH3CH(OH)CO2H

29 Chlorofluorocarbons (CFCs), such as CCl3F, are a class of chemically stable


compounds and were once used widely used in daily in aerosol sprays. However in
recent years, CFCs have been replaced by hydrofluorocarbons (HFCs), such as
CH3CHF2, as CFCs contribute to ozone depletion,
Which of the following properties explain why HFCs are used?
1 They are less reactive than the corresponding chloroalkanes.
2 They may be used as aerosol propellants.
3 They are non-flammable.

8872/01/CJC JC 2 Prelims Exam 10


12
30 Malic acid, which occurs in apples, has the following structure.

O O
CCHCH2C
HO OH
OH
Which of the following substances will react with all the three -OH groups present in
one malic acid molecule?
1 Sodium
2 Phosphorus pentachloride
3 Sodium carbonate

8872/01/CJC JC 2 Prelims Exam 10


1

Name Class: 2T

CATHOLIC JUNIOR COLLEGE

CHEMISTRY 8872/02
Higher 1

Paper 2
JC 2 Preliminary Examinations
13th September 2010

2 hours

Candidates answer Section A on Question Paper.


Additional Materials: Writing Paper
Graph Paper
Data Booklet

READ THESE INSTRUCTIONS FIRST

Write your name and class on all the work you hand in.
Write in dark blue or black pen on both sides of the paper.
You may use a soft pencil for any diagrams, graphs or rough working.
Do not use paper clips, highlighters, glue or correction fluid.

Section A
Answer all questions.

Section B
Answer two questions on separate answer paper.
Circle their numbers in the box provided below.

At the end of the examinations, fasten all your work securely together.
The number of marks is given in brackets [ ] at the end of each question or part question.
For Examiners Use
Paper 1
Paper 2 Section A
Q5
Q6
Q7
Total

There are a total of 11 printed pages and 1 black page.


8872/02/CJC JC 2 Prelims Exam 10
2
Section A
Answer all questions in this section.

1. (a) When arsenic was reacted with nitric acid, HNO3, arsenic(V) acid, H3AsO4 and NO2 were
produced. Potassium iodide, KI was added to the resulting mixture and a brown solution, X
was observed to form and arsenic(V) acid was also converted to H2AsO3.
Given that the half equations for the first and second reactions arsenic are:

As + 4 H2O AsO43- + 8 H+ + 5 e- --- 1st reaction

AsO43- + 2 H+ + e- AsO32- + H2O --- 2nd reaction

(i) State the identity of the brown solution, X that was formed.

(ii) With the use of the equations given and the Data Booklet, write down the two
balanced ionic equations for the reactions above.

[3]

(b) The amount of brown solution, X formed can be found by titrating it against a standard
solution of sodium thiosulfate, Na2S2O3 solution.
When 0.1058 g of a sample containing arsenic was subjected to the above reactions, the X
liberated required 28.70 cm3 of 0.0198 mol dm-3 of solution of sodium thiosulfate in the final
titration.
(i) Calculate the number of moles of sodium thiosulfate required for the titration.

(ii) Given that 1 mol of X can react with 2 mol of sodium thiosulfate, calculate the
number of moles of arsenic in the sample.

8872/02/CJC JC 2 Prelims Exam 10


3
(iii) Hence, calculate the percentage of arsenic in the sample.

[5]
[Total: 8]

2. (a) Using the diagram below, sketch how S+ and S2+ ions will behave in an electric field and
state clearly the relative magnitudes of deflection of the two beams.

[2]
(b) (i) Draw the dot-and-cross diagrams of SO2 and SO3 molecules.

(ii) With respect to their intermolecular forces, explain why SO3 has a higher boiling
point than SO2.

(iii) With the use of chemical equations, show the reactions between SO2 and SO3 with
an alkali.


[6]

8872/02/CJC JC 2 Prelims Exam 10


4
(c) Sulfur and chlorine can react together to form disulfur dichloride, S2Cl2. When 1.00 g of
S2Cl2 was reacted with water, 0.36 g of sulfur, S was formed, together with a solution
containing a mixture of sulfurous acid, H2SO3, and hydrochloric acid, HCl.
(i) Calculate the mole ratio between S2Cl2 and S in the above reaction.

(ii) Hence, deduce the equation for the reaction between S2Cl2 and water.


[4]
[Total: 12]

3. In June 2009, the World Health Organization (WHO) has declared the new strain of flu virus,
H1N1 as a pandemic. This strain is known as swine flu as it has its origin from the swine. This
novel virus spread worldwide and had caused about 17,000 deaths by the start of 2010.
To reduce the extent of spread of this virus, one of the common drugs that has been used to
prevent or treat flu is Oseltamivir or better known as Tamiflu.
The starting reagent to synthesise Tamiflu is shikimic acid, which is extracted from star anise,
the seed pod of an evergreen tree.

HO CO2H

HO
OH
Shikimic acid

Shikimic acid molecule can exhibit geometric isomerism. A large amount of star anise has to be
used before enough shikimic acid is obtained to synthesize Tamiflu.

(a) (i) Explain what is Geometric isomerism.


...

(ii) Explain why shikimic acid molecule can exhibit geometric isomerism.

...
[2]

8872/02/CJC JC 2 Prelims Exam 10


5
A student suggested the following reaction scheme to synthesize shikimic acid from 5-chloro
Cl CO2H

cyclohex-3-ene-carboxylic acid, .

(b) State the types of reaction in steps I, II and IV by filling in the table below.

Step Type of reaction

II

IV

[3]

(c) Suggest two possible structures of intermediate W and state, with brief explanation, which
structure fits the description of the reaction scheme above.

[3]
8872/02/CJC JC 2 Prelims Exam 10
6
(d) There is a mistake to one of the steps of the reaction scheme that the student suggested.
Identify the wrong step and suggest how it can be modified.
[Hint: Could either be wrong reagents or conditions used.]

. [2]

[Total: 10]

4. The Singapore 2010 Youth Olympic Games or YOG is an international


multi-sport event for youths that has taken place, making Singapore the first
to host this event in the XXIX Olympiad. The Games consisted of 3,600
athletes between 14 and 18 years of age from 204 National Olympic
Committees (NOCs) competing in 184 events in 26 sports.
To ensure that the young athletes can perform to their best ability,
designing appropriate training for them becomes very important. This will
help them build strong foundation so as to enable them to perform to their best during such
competitions.
During a normal training, when an athletes body receives enough oxygen, most cells in the
body will burn glucose in a two-step process to form water and carbon dioxide. Glucose is first
broken down to pyruvate (salt of pyruvic acid) which will then undergo oxidation in the
presence of oxygen to form water and carbon dioxide.
Step 1:

O
O -
2 O

Pyruvate
Glucose
Step 2:

O
4 O - + 5 O2 8 CO2 + 6 H2O
O
Pyruvate

However, when training becomes more intense, the body will receive inadequate supply of
oxygen. As such, the excess pyruvate produced from the first step will in turn be converted into
lactate (salt of lactic acid), which allows glucose to break down to produce energy. To maintain
electroneutrality, equal amount of protons will also be released to the blood. In the presence of
an enzyme, lactate dehydrogenase, it helps to catalyse the conversion at high rates. This will
result in large amounts of lactate to accumulate in the blood and in turn allows more protons to
be released causing the pH to drop. (Note: If a youth athlete weighs 50 kg, then approximately
49 % of his body weight is made up of blood.)

The normal pH of blood plasma is approximately 7.4. If it falls below 7.35, a state of acidosis
will exists. At this stage, athletes will show symptoms of shortness of breath, increased heart
rate, and muscles soreness. This will affect the performance of the athletes.
8872/02/CJC JC 2 Prelims Exam 10
7
(a) (i) Define what is pH.

(ii) Calculate the volume of blood in a 50 kg young athlete, assuming that the volume of
blood is equal to the mass of blood. (1 kg = 1000 cm3)

(iii) Calculate the excess amount of protons in the blood to cause the soreness of
muscles to occur. Given that lactic acid is a monobasic acid, hence state the
amount of lactate that has accumulated.

[5]
3
(b) (i) If an athlete took in 2500 cm of oxygen during a normal workout and only 16 % of
the oxygen was used up, calculate the amount of CO2 produced in the blood,
assuming that it is at room temperature and pressure.

(ii) As CO2 is produced, it will dissolve in water present in the blood to form a buffer
mixture made up of carbonic acid with bicarbonate salt, H2CO3/HCO3-.

Explain, with the help of appropriate equations, how this buffer works to maintain
constant pH in blood under normal conditions.

[5]

[Total: 10]
8872/02/CJC JC 2 Prelims Exam 10
8
Section B
Answer two of the three questions in this section on separate answer paper.

5. Retinol is one of the animal forms of Vitamin A. It is a fat-soluble (or water-insoluble)


vitamin and can be used to treat many conditions, especially skin conditions such as acne
and wrinkles.
Retinol is produced in the body from the hydrolysis of retinyl esters such as retinyl
ethanoate (C22H32O2) and from the reduction of retinal. It can also be obtained through
consumption of animal produce such as liver and eggs or vegetables such as carrots and
spinach.
CH3 CH3
CH3 CH3
OH

CH3
Retinol
(a) With reference to the structure of the molecule, suggest a reason why retinol is insoluble in
water.
[1]
(b) (i) Explain why for the synthetic reaction to form retinol from retinyl ethanoate (ester of
retinol), alkaline hydrolysis was preferred to acidic hydrolysis.
Hence draw the organic products that were produced from the alkaline hydrolysis,
with the aid of the simplified diagram provided below.
O

R O CH3
Retinyl ethanoate
(ii) The graph below shows the variation of the concentration of retinyl ethanoate in the
hydrolysis reaction with time.
Using the graph below, deduce the order of reaction with respect to retinyl
ethanoate.

110
-3
[Retinyl ethanoate] (x 10 ) / mol dm

100
90
80 [NaOH] = 0.1 mol dm-3
-4

70
Initial rate = 0.4 mol dm-3 min-1
60
50
40
30
20
10
0
0 20 40 60 80 100
Time / min

8872/02/CJC JC 2 Prelims Exam 10


9
(iii) When the concentration of sodium hydroxide, NaOH is doubled, the rate of reaction
for this reaction is also doubled.
State the order of reaction with respect to NaOH and hence write down the rate
equation of this reaction.
(iv) Calculate the value of the rate constant, k, of this reaction.
[8]

(c) Pure retinol is very sensitive to oxidation. Thus, when retinol is prepared synthetically as a
dietary supplement, it has to be stabilized by forming into its ester derivatives.
25.0 cm3 of 0.1 mol dm-3 of retinol is added to 50.0 cm3 of ethanoic acid of equal
concentration to react in a reversible reaction to form retinyl ethanoate (C22H32O2).

conc. H2SO4
Retinol(l) + CH3CO2H(l) Retinyl ethanoate(l)
+ H2O(l)
(C22H32O2)
heat

(i) State the Le Chateliers Principle.


(ii) The equilibrium constant of the backward reaction, Kd is 5.2. Calculate the
value of equilibrium constant, Kf of the forward reaction.
(iii) Calculate the initial amounts of retinol and ethanoic acid.
(iv) Hence calculate the percentage conversion of retinol when equilibrium is attained.
[7]
(d) The presence of concentrated sulfuric acid, H2SO4 can help to hasten the rate of reaction
and to increase the yield of the ester.
Explain how concentrated sulfuric acid, H2SO4 can help to increase:
(i) Yield of products
(ii) Rate of reaction.
[4]
[Total: 20]

6. (a) Pure anhydrous nitric acid, HNO3 is a colorless liquid which solidifies at 42 C to form
white crystals and boils at 83 C.
(i) Given that the H atom is bonded to one of the O atoms, draw the structural formula
of HNO3 and state the type(s) of bonding it contains.
(ii) With reference to the structure of HNO3, suggest a reason why it acts as a strong
acid in water.
[4]

8872/02/CJC JC 2 Prelims Exam 10


10
(b) In an experiment, 100 cm3 of a 0.500 mol dm-3 of dilute nitric acid was placed in a plastic
beaker.
100 cm3 of 0.500 mol dm-3 sodium hydroxide was then added and the mixture stirred.
Both solutions were initially at the same temperature. After mixing, the temperature of the
mixture was found to have risen by 3.2 oC.
(i) Define the standard enthalpy of neutralisation with respect to the above reaction.
(ii) Using the given information and relevant data in the Data Booklet, calculate the
enthalpy change of neutralisation.
(iii) The standard enthalpy change of neutralisation of this reaction should be -57.2 kJ
mol-1. Suggest an explanation for the difference between this and the value you
have calculated in (ii).
[6]
(c) The salt formed in (b) is an ionic compound.
How would you expect, giving your reasoning, the lattice energy of the salt formed in the
above reaction to differ from that of sodium chloride? [2]

(d) Toluene is a clear water-insoluble liquid with the typical smell of paint thinners. It is widely
used as an industrial feedstock and as a solvent. Similar to other solvents, toluene is
sometimes also used as an inhalant drug for its intoxicating properties; however, this can
potentially cause severe neurological harm.
Toluene has the following structure.

Compound Z can be synthesized from toluene. Outline the three-step synthesis of the Z
from the intermediate I, indicating clearly the reagents and conditions, and intermediates
formed.

[5]
(e) Suggest a chemical test which can be used to distinguish between the following two
compounds. In your answer, include the reagents and conditions used, and any
observations made.

[3]
[Total: 20]

8872/02/CJC JC 2 Prelims Exam 10


11
7. (a) An organic compound A with molecular formula C3H6O, gives a orange precipitate with 2,4-
dinitrophenylhydrazine, but gives no precipitate when warmed with Tollens reagent.

A is then reacted with dilute HCN in the presence of trace amount of NaCN, followed by
refluxing with excess dilute H2SO4 to form an organic liquid B. Liquid B turns moist blue
litmus paper red.

Upon prolonged hydrolysis of B, a sweet-smelling liquid C, with molecular formula C8H14O5,


is formed.

Use all of the above information to identify the organic compounds A, B and C, and draw a
structural formula for each of them, showing clearly all the deductions that you make from
the information that have been given. Include equations for the reactions that have
occurred.
[10]

(b) Some physical properties of the oxides of elements D, E, F and G are given below.

Formula of oxide Melting point/ oC Appearance at room Conductivity in


temperature and molten state
pressure

DO2 1700 White solid None

E2O 0 Colourless liquid None

FO 2850 White solid Good

GO2 -73 Colourless gas None

Describe the types of structures and all the bonds in the above four oxides.
[10]
[Total: 20]

8872/02/CJC JC 2 Prelims Exam 10


12
- BLANK PAGE -

8872/02/CJC JC 2 Prelims Exam 10


CANDIDATE NAME ____________________________ CLASS________

DUNMAN HIGH SCHOOL (SENIOR HIGH)


YEAR 6 PRELIMINARY EXAMINATION 2010

H1 CHEMISTRY 8872/01
Paper 1 Multiple Choice 22 September 2010
50 minutes
Additional materials:
Optical Mark Sheet
Data Booklet

INSTRUCTIONS TO CANDIDATES
1 Write your name and class on this question paper.
2 There are thirty questions on this paper. Answer all questions. For each question there are
four possible answers A, B, C and D.
Choose the one you consider correct and record your choice in soft pencil on the separate
Optical Mark Sheet.
3 Each correct answer will score one mark. A mark will not be deducted for wrong answer.
4 Any rough working should be done in this booklet.
5 You may use a calculator.

This question paper consists of 13 printed pages and 1 blank page.


[Turn over
2

Section A

For each question, there are four possible answers A, B, C, and D. Choose the one you
consider to be correct.

1 Heating phenylethanone, C6H5COCH3 with iodine and an aqueous alkali yields 12.0 g of
triiodomethane. What is the mass of phenylethanone that has reacted?

A 0.36 g B 1.80 g C 3.65 g D 7.20 g

2 Acidified aqueous sodium dichromate (VI) and acidified aqueous potassium


manganate (VII) each liberates iodine from aqueous iodide.

Cr2O72 (aq) + 14H+ (aq) + 6e 2Cr3+ (aq) + 7H2O (l)


MnO4 (aq) + 8H+ (aq) + 5e Mn2+ (aq) + 4H2O (l)
2I (aq) I2 (s) + 2e

25.0 cm3 of acidified Na2Cr2O7 liberate the same quantity of iodine from a solution of
potassium iodide as 25.0 cm3 of 0.030 mol dm3 KMnO4. What is the concentration of
Na2Cr2O7 in mol dm3?

A 0.015 B 0.025 C 0.030 D 0.050

3 10 cm3 of a gaseous hydrocarbon are mixed with 70 cm3 of oxygen and the mixture
explodes. After cooling to room temperature, the residual gas occupies a volume of
50 cm3. Upon shaking these gases with aqueous potassium hydroxide, the final volume
of gas remaining is 20 cm3. What is the molecular formula of the hydrocarbon?

A C2H4 B C2H6 C C3H6 D C3H8

4 The use of Data Booklet is relevant to this question.

The successive ionisation energies, in kJ mol1, of an element X are given below.

870 1800 3000 3600 5800 7000 13200


What is X?

A 33As

B 53I

C 8O

D 52Te

DHS 2010 8872/01/Prelim 2010 [Turn over


3

5 What is the electronic configuration of vanadium atom, with proton number 23?

A 1s22s22p63s23p63d5
B 1s22s22p63s23p63d44s1
C 1s22s22p63s23p63d34s2
D 1s22s22p63s23p64s24p3

6 Compound A is a coloured compound of iron which has the formula [Fe(NH3)4][FeCl4].


The compound is ionic and the oxidation number of the iron is the same in each ion.

What are the formulae of the two ions present?

A [Fe(NH3)4]+ and [FeCl4]


B [Fe(NH3)4]2+ and [FeCl4]2
C [Fe(NH3)4]3+ and [FeCl4]3
D [Fe(NH3)4]4+ and [FeCl4]4

7 Hydrogen trioxide, H2O3, is an unstable compound but can be isolated in small quantities.
Carbon suboxide, C3O2, is a foulsmelling gas obtained by fully dehydrating
propan1,3dioic acid. Which of the following shows reasonable Lewis structures for
these two molecules?

Hydrogen trioxide Carbon suboxide


A O C C C O
H O O O H

B
O H O H O O C C C O

C
H O O O H O C C C O

D
O H O H O O C C C O

DHS 2010 8872/01/Prelim 2010 [Turn over


4

8 A solid Z has the following physical properties.


It is insoluble in hydrocarbon solvents.
It melts at 1290 C.
It conducts electricity in both aqueous and molten states.

What is the likely structure of Z?

A an ionic crystal
B an atomic crystal
C a molecular crystal
D a giant molecular lattice

9 Trimethoprim (TMP) is used for the treatment of urinary tract infections. It has the
following structure:
OCH3
CH3O N NH2
H y H
x
H C O C N
w
H Z
H
H N H

Trimethyoprim (TMP)

In which sequence is the bond angles quoted in decreasing order?


A y>w>z>x

B x>y>z>w

C y>z>w>x

D x>y>w>z

10 The bond dissociation energy of HF is 565 kJ mol1. Which equation correctly


describes the reaction whereby 565 kJ of energy is released?

A HF(l) HF(g)

B H(g) + F(g) HF(g)

C HF(g) H(g) + F(g)

D H2(g) + F2(g) HF(g)

DHS 2010 8872/01/Prelim 2010 [Turn over


5

11 When 13.08 g of zinc dust is added to 250 cm3 of 1.0 mol dm3 aqueous copper(II)
sulfate, the temperature rises by 15 C. The specific heat capacity of the final solution is
4.20 J g1 K1.

Cu2+ + Zn Zn2+ + Cu

What is the enthalpy change for the above reaction?

A 4.12 kJ mol1 B 66.3 kJ mol1 C 78.8 kJ mol1 D 82.9 kJ mol1

12 Given the following data and energy cycle:

N2O4(g) 2NO2(g) H = +57.9 kJ mol1

1
NO(g) + O2(g) NO2(g) H = 56.6 kJ mol1
2

Determine the enthalpy change of the following reaction:

2NO(g) + O2(g) N2O4(g)

A +114.5 kJ mol1 B 114.5 kJ mol1 C +171 kJ mol1 D 171 kJ mol1

13 Which one of the following statements is correct about a reaction for which the
equilibrium constant is independent of temperature?

A Its rate constant does not vary with temperature.


B The activation energies for both the forward and reverse reactions are zero.
C The enthalpy change of reaction is zero.
D There are equal number of moles of reactants and products.

DHS 2010 8872/01/Prelim 2010 [Turn over


6

14 The energy diagram represents a reaction that occurs in the absence, and in the
presence of a catalyst.
Energy / kJ mol1

E1 E4

E2 E3

Reactants

Products
Reaction
pathway

Which one of the following statement is correct?

A E4 is the activation energy for the reverse catalysed reaction.

B The forward catalysed reaction is endothermic.

C The enthalpy change of the forward reaction is (E2E3).

D The enthalpy change of reaction is decreased by using a catalyst.

15 The graphs below show the variation of the percentage of gaseous products present at
equilibrium, with temperature and pressure.

% products
at equilibrium

T2 C
T2 < T1
T1 C

Pressure

Which one of the following systems could the graphs represent?

A N2(g) + 3H2(g) 2NH3(g) H = 92 kJ mol1


B 3O2(g) + 4NH3(g) 2N2(g) + 6H2O(g) H = 1248 kJ mol1

C 2N2(g) + O2(g) 2N2O (g) H = +82 kJ mol1

D CO2(g) + C(s) 2CO (g) H = +173 kJ mol1

DHS 2010 8872/01/Prelim 2010 [Turn over


7

16 One enzyme operates at maximum efficiency when in an aqueous solution buffered at


pH 5. Which combination of substances, when dissolved in 12 dm3 of water, would give
the necessary buffer solution?

A 1 mol of HCl and 1 mol of CH3CO2Na

B 2 mol of CH3CO2H and 1 mol of NaOH

C 2 mol of NH3 and 1 mol of CH3CO2NH4

D 2 mol of NaOH and 1 mol of CH3CO2H

17 The graph below shows the variation in the boiling points for eight consecutive elements
in the Periodic Table, all with atomic number between 10 and 20.

Which of the following can be deduced from the above information?

A Element H and beryllium are in the same group.

B When chloride of element D reacts with water, the solution turns red litmus paper
blue.

C Element B conducts electricity in aqueous solution but not in solid state.

D Element A forms a basic oxide only.

18 An element Z produces a brilliant white flame when burns in air to form oxide of Z. The
oxide of Z turns Universal Indicator blue when it is added to water.
What is the element Z?

A Magnesium

B Phosphorus

C Silicon

D Sodium

DHS 2010 8872/01/Prelim 2010 [Turn over


8

19 A compound X exhibits structural isomerism, the isomers being members of different


homologous series.

To which pair of isomers could X belong?

A carboxylic acids and esters.


B acylchlorides and carboxylic acids.
C amino acids and ammonium salts.
D amides and amino acids.

20 A compound used in optical brightener in detergents has the following formula.


CN

CH CH CH CH

NC
Which of the following is likely to be a property of this compound?

A It is readily soluble in water.


B The molecules exist as cistrans isomers.
C It is hydrolysed by hot dilute sulfuric acid to give an amine.
D It requires 4 mol of hydrogen gas for reduction.

21 The first stage of the cumene process for the industrial production of phenol is as
follows.

H3C CH3
CH
H2SO4
+ CH3CH=CH2

Which one of the following would be the product of reaction, under similar conditions,
between benzene and cyclohexene?

A B C D
CH2(CH2)4CH3 CH=CH2(CH2)3CH3

DHS 2010 8872/01/Prelim 2010 [Turn over


9

22 Cl

COOH
Methylbenzene gives the compound as the product under suitable
reacting conditions.

What are the types of reactions taking place?

A electrophilic substitution and oxidation

B electrophilic substitution and reduction

C nucleophilic substitution and oxidation

D nucleophilic substitution and reduction

23 Deuterium, D, is a heavy isotope of hydrogen. Deuteriobenzene is reacted with a


mixture of chlorine gas under suitable conditions such that only monosubstitution takes
place.

Assuming that the carbondeuterium bond is broken as easily as a carbonhydrogen


bond, which proportion of the chlorinated products will be 3chlorodeuteriobenzene?

D D

Cl

Deuteriobenzene 3-chlorodeuteriobenzene

A 16% B 20% C 33% D 45%

DHS 2010 8872/01/Prelim 2010 [Turn over


10

24 Hot ethanolic sodium hydroxide is added separately to two compounds C6H5CH2Br and
C6H5CH2CH2Br to form compound P and Q respectively. What are the structural
formulae of compounds P and Q?

P Q
A
CH2 OH CH=CH2

B
CH2 OH
CH2 CH2 OH

C OH

CH=CH2
CH2 Br

D
CH2 Br CH2 CH2 OH

25 One mole of a compound T reacts with two moles of Na and one mole of Na2CO3.
Which of the following could be compound T?

A HOCH2CH2CO2H

B HCO2CH2CH2OH

C HOCH2CH2COCH3

D HOCH2CH(OH)CO2H

DHS 2010 8872/01/Prelim 2010 [Turn over


11

Section B

For each question, one or more of the three numbered statements 1 to 3 may be correct.

Decide whether each of the statements is or is not correct (you may find it helpful to put a
tick against the statements which you consider to be correct).

The responses A to D should be selected on the basis of


A B C D
1, 2 and 3 1 and 2 only 2 and 3 only 1 only
are correct are correct are correct is correct

No other combination of statements is used as a correct response.

26 Which of the following species can be represented by the electronic configuration


1s22s22p63s23p3?

1 S+ ion.

2 P atom.

3 Ar ion.

27 The graph shows the results of an investigation of the initial rate of hydrolysis of
maltose by the enzyme amylase. In the experiments, the initial concentration of maltose
is varied but that of amylase is kept constant.
Initial
reaction rate

[maltose]

Which conclusions can be deduced from the results?

1 When [maltose] is low, the rate is first order with respect to [maltose].

2 When [maltose] is high, the rate is independent of [maltose].

3 When [maltose] is high, the rate is independent of [amylase].

DHS 2010 8872/01/Prelim 2010 [Turn over


12

The responses A to D should be selected on the basis of


A B C D
1, 2 and 3 1 and 2 only 2 and 3 only 1 only
are correct are correct are correct is correct

No other combination of statements is used as a correct response.

28 The diagram shows some laboratory apparatus:

Which preparations could this apparatus be used for?

1 Bromoethane, from ethanol, sodium bromide and concentrated sulfuric acid.

2 Ethanal, from ethanol, sodium dichromate(VI) and sulfuric acid.

3 1,2dibromoethane, from bromine and ethene.

29 Lactic acid builds up in muscles when oxygen is in short supply. It can cause muscular
pain. Part of the reaction sequence is shown.

CH2OHCH(OH)CHO CH3COCO2H CH3CH(OH)CO2H


glyceraldehyde pyruvic acid lactic acid

Which statements about the reaction sequence are correct?

1 A secondary alcohol is oxidised to a ketone.

2 A ketone is reduced to a secondary alcohol.

3 An aldehyde is oxidised to a carboxylic acid.

DHS 2010 8872/01/Prelim 2010 [Turn over


13

The responses A to D should be selected on the basis of


A B C D
1, 2 and 3 1 and 2 only 2 and 3 only 1 only
are correct are correct are correct is correct

No other combination of statements is used as a correct response.

30 Deuterium, D, is a heavy isotope of hydrogen. Which of the following reactions will


incorporate D in the final organic product?

1 CH3COOCH2CH3 + hot aqueous NaOD

2 CH2=CH2 + Br2 (in D2O)

3 CH3COCH3 + DCN

DHS 2010 8872/01/Prelim 2010


14

DHS 2010 8872/01/Prelim 2010


CANDIDATE NAME ____________________________ CG________

DUNMAN HIGH SCHOOL (SENIOR HIGH)


YEAR 6 PRELIMINARY EXAMINATION 2010

H1 CHEMISTRY 8872/02
Paper 2 20 September 2010
2 hours
Additional materials:
Answer Paper
Data Booklet

INSTRUCTIONS TO CANDIDATES
1 Write your name and class on all the work you hand in.
2 Write in dark blue or black pen on both sides of the paper.
3 You may use a soft pencil for any diagrams, graph or rough working.
4 The number of marks is given in brackets [ ] at the end of each question or part
question.
5 You may use a calculator.
6 At the end of the examination, fasten all your work securely together.

Section A FOR EXAMINERS USE


Answer all questions. Question Marks
1 /10
Section B
2 /10
Answer two questions on separate answer paper. 3 /10
4 /10
Paper 2 Section A /40
5 /20
6 /20
7 /20
Paper 2 Section B /40
Paper 1 /40
Total /110
Grade

This question paper consists of 14 printed pages and 0 blank page.


[Turn over
2

Section A

Answer all the questions in this section in the spaces provided.

1(a) It has always been stated that the elements of the Periodic Table show general trend from Comment:
metallic to nonmetallic behavior, coupled with an increase of oxidation state of their
compounds.

Oxidation State

+8
+7
+6
+5
+4
+3
+2
+1
Na Mg Al Si P S Cl

= oxidation state of oxides of Period 3 elements

(i) Mark with a cross (X) on the diagram the highest oxidation state of the oxide of
chlorine.

(ii) Magnesium and aluminium are both metals. Explain, with the aid of appropriate
equations, why magnesium oxide reacts with only acids but aluminium oxide reacts
with both acids and bases.

[4]

DHS 2010 8872/02/Prelim 2010 [Turn over


3

1(b) Magnesium oxide, phosphorus pentoxide and phosphorus pentachloride all exist as white
solids. Suggest reactions you could carry out to distinguish between the three compounds,
writing equations where appropriate.

[3]

(c) Phosphorus pentachloride can undergo decomposition at high temperature to form


phosphorus trichloride and chlorine. This reaction is known to be a firstorder reaction.
PCl5(g) PCl3(g) + Cl2(g)

(i) Explain what is meant by firstorder reaction.

(ii) Given that the rate constant for the reaction is 1.55 x 103 s1, calculate the time taken
for the concentration of PCl5(g) to decrease from 1 mol dm3 to 0.125 mol dm3.

[3]

[Total: 10]

DHS 2010 8872/02/Prelim 2010 [Turn over


4

2 Ethanoic acid, an important industrial commodity in the world, finds a variety of uses in
vinegar, polymer formation and ester formation.

(a) In an experiment, 20.0 cm3 of 0.250 mol dm3 barium hydroxide, a strong base, is titrated
against an aqueous solution of ethanoic acid. The following titration graph is obtained.

pH

Vacid added/ cm3


0 15.0

(i) Write a balanced chemical equation for the reaction between ethanoic acid and
barium hydroxide.

(ii) Calculate the pH of the barium hydroxide solution.

(iii) Calculate the initial concentration (in mol dm3) of the ethanoic acid.

DHS 2010 8872/02/Prelim 2010 [Turn over


5

(iv) Suggest a suitable indicator for this titration.

(v) After 30 cm3 of ethanoic acid is added to the mixture, the solution is able to resist pH
changes when a small amount of acid or alkali is added. Write two equations to
explain why this is so.

[6]

(b) Aspirin, also known as acetylsalicylic acid, is a salicylate drug, often used as an analgesic
to relieve minor aches and pains, as an antipyretic to reduce fever, and as an
antiinflammatory medication. The structure of aspirin is as shown below.

O OH

O CH3

Aspirin

(i) Give the reagents and conditions to produce ethanoic acid from aspirin.
Reagents:
Conditions:

(ii) Explain why aspirin is insoluble in water.

DHS 2010 8872/02/Prelim 2010 [Turn over


6

(iii) However, aspirin is soluble in aqueous sodium hydroxide at room temperature.


Explain, with the aid of an equation, why this is so.

[4]
[Total: 10]

3(a) Nitrogen trichloride is a yellow, oily, pungentsmelling liquid is most commonly


encountered as a byproduct of chemical reactions between ammoniaderivatives and
chlorine. It is also used as a bleaching agent for flour in the Agene process. An energy
level diagram involving nitrogen trichloride is shown below.

Enthalpy / kJ mol-1

N (g) + 3Cl (g)

3
N (g) + Cl2 (g)
2
Hat(N2) NCl3 (l)
1 3
N (g) + Cl2 (g)
2 2 2

(i) Define the term standard enthalpy change of formation.

(ii) Using Hf(NCl3), bond energy (NCl) and bond energy (ClCl), label the enthalpy
changes of the processes involved on the energy level diagram above.

(iii) Hence, use the information given below and relevant data from the Data Booklet to
calculate the bond energy of NCl bond. Comment: Something wrong with
Hf( NCl3) = +232 kJ mol1 diagram. The arrow should point up!

[6]

DHS 2010 8872/02/Prelim 2010 [Turn over


7

(b) A student used the apparatus below to determine the enthalpy change of combustion of
propan1ol. It was found that 0.60 g of propan1ol was used to raise the temperature of
200 g of the water. The initial temperature of the water was 21.0C.
The specific heat capacity of water is 4.18 J g1 K1.

thermometer

copper can containing


200 g of water

burner containing
propan-1-ol

(i) Calculate the number of moles of propan1ol burnt.

(ii) Given that the enthalpy of combustion of propan1ol is 2021 kJ mol1, calculate the
final temperature of water at the end of the experiment.

(iii) The actual temperature of the water measured at the end of experiment is 38.0C.
Give an explanation for the difference.

[4]

[Total: 10]

DHS 2010 8872/02/Prelim 2010 [Turn over


8

4 Acrolein is formed when fats or vegetable oils are cooked at high temperatures. In fried
foods, it may upset the stomach.

H H
H2C C C O

(a) (i) Name the functional group(s) found in acrolein.

(ii) Give the IUPAC name for acrolein.

[2]

(b) The C=O and C=C bonds have the same electronic structure and they undergo typically
addition reactions.

(i) What do you understand by the term addition?

(ii) Suggest suitable reagent(s) and condition(s) to bring about an addition reaction of the
C=O bond.
Reagent for C=O:
[2]

(c) Draw the structural formula(e) of the product(s) formed when acrolein reacts with each of
the following reagents. Include the formula of other products formed (if any).

(i) hydrogen in platinum catalyst

(ii) acidified potassium


dichromate(VI)

(iii) Fehlings solution

[3]

DHS 2010 8872/02/Prelim 2010 [Turn over


9

(d) Methacrolein is a clear colourless flammable liquid that has a similar structure to acrolein.
CH3 H
H2C C C O

There are two isomers of methacrolein, S and T, both of which react differently when
subjected to alkaline aqueous iodine and Tollens reagent.

Like methacolein, S forms a precipitate with Tollens reagent but not with alkaline aqueous
iodine.

On the other hand, T forms a precipitate with alkaline aqueous iodine but not with Tollens
reagent.

(i) What deduction can you make about the isomers when a precipitate is formed upon
reacting with

alkaline aqueous iodine, and

Tollens reagent

(ii) Use your deductions to propose the structures of S and T.

S: T:

[3]

[Total: 10]

DHS 2010 8872/02/Prelim 2010 [Turn over


10

Section B

Answer two questions from this section on separate answer paper.

1(a) Chlorine is a common element which can be found in many inorganic as well as organic
compounds. The most common isotope of chlorine is 35chlorine.

(i) Write the symbol (include mass number) of the other isotope of chlorine and calculate
the relative abundance of this isotope.

(ii) Write an equation to represent the first ionisation energy for chlorine.

(iii) Using the Data Booklet, explain the difference in first ionisation energy between
chlorine and bromine.
[7]

(b) Chlorine can form ionic compounds with Group I and II metals. The table below shows the
lattice energies of two chlorine-based ionic compounds.

Ionic compound Lattice energy/ kJ mol1


potassium chloride 701
calcium chloride 2237

(i) Define the term lattice energy.

(ii) Explain the difference in the lattice energy between potassium chloride and calcium
chloride.

(iii) Calcium ion and chloride ion are separately subjected to the effect of an electric field
as shown below. Copy this diagram, and draw and label clearly the path of the ions
under the influence of the electric field.

+
2+
Ca

Cl_
_

[6]

DHS 2010 8872/02/Prelim 2010 [Turn over


11

(c) The commercial insecticide DDT is an organochlorine first discovered by Swiss chemist
Paul Hermann Mller. It is nearly insoluble in water but very soluble in organic solvents
and fats.
Cl
Cl Cl

Cl Cl

DDT
Consider the reaction scheme given below.

Cl Cl
Cl Cl Cl Cl
CH3

Step I Step II

Cl Cl
A

(i) Provide the reagents and conditions required for Step I and Step II.

(ii) Predict and explain if Compound A or DDT will have a higher melting point.

Cl
Cl Cl

Cl Cl

(iii) Why is the compound Cl Cl not formed in Step II?

(iv) Draw the structural formula of the product formed when DDT reacts with excess
aqueous sodium hydroxide assuming chlorobenzene will not react.
[7]
[Total: 20]

DHS 2010 8872/02/Prelim 2010 [Turn over


12

2. Methane is used to produce synthesis gas (syngas), a mixture that includes carbon
monoxide and hydrogen, by reacting with steam on a nickel catalyst. Syngas is then used
to produce liquid hydrocarbons and methanol.
CH4 (g) + H2O (g) CO (g) + 3H2 (g)
A total of 5 mol gas mixture comprising of methane and steam is mixed in a 1:1 ratio in a
2 dm3 vessel. After some time, 1 mol of methane is found to be remaining at equilibrium.

(i) How many moles of methane have reacted? Calculate the number of moles of steam
reacted as well as those of carbon monoxide and hydrogen gas produced.
Comment: Increase space between
(ii) Write an expression for the equilibrium constant, Kc and determine its value, including question parts?

units.

(iii) It is known that this reaction is an endothermic process. What is meant by the term
in italics?

(iv) Predict and explain the effect of separately increasing pressure and decreasing
temperature on the above equilibrium as well as on the yield.

(v) In practice, a moderately high pressure is used in the production of syngas. Suggest a
reason for this.

(vi) Sketch a graph showing how the rates of the forward and reverse reactions change
with time, from the point of mixing to the point after equilibrium is established. Label
your graphs clearly, showing the time at which equilibrium is established. Comment: showing the time at which
[13] equilibrium is established

(b) To manufacture methanol, the carbon monoxide and hydrogen mixture is passed over a
mixture of copper, zinc oxide and alumina catalysts. Besides being a common laboratory
solvent, the largest use of methanol (CH3OH) is to produce methanal (HCHO).

(i) Industrially, oxygen is used to react with methanol to produce methanal. Write an
equation for this reaction.

(ii) Alternatively, what reagent(s) and condition(s) can be used in the laboratory to
convert methanol to methanal?

(iii) Sketch the shape of the hybrid orbitals around the carbon in methanal molecule.
[3]

(c) A sample of methane reacts with excess bromine to form a mixture of brominated
products.

(i) Identify the type of reaction that has occurred.


(ii) One of the products is found to have the composition by mass: C, 6.9%; H, 1.2% and
Br, 91.9%. Draw the displayed formula of this product.
(iii) Write the structural formulae of two other brominated products.
[4]
[Total: 20]

DHS 2010 8872/02/Prelim 2010 [Turn over


13

3(a) The oxo reaction is an important industrial process in which an alkene combines directly
with carbon monoxide and hydrogen under suitable conditions. The reaction with ethene is
shown below.
catalyst
CH2=CH2 + CO + H2 CH3CH2CHO
high T, p

(i) What is the simple relationship between the time taken for the reaction and the initial
rate of the reaction? Comment: More space between
question parts

(ii) A series of experiments are carried out to investigate the kinetics of the oxo reaction
with ethene. The following results are obtained.

[CH2=CH2]
Experiment [CO] / mol dm3 [H2] / mol dm3 Time / s
/ mol dm3

1 0.200 0.100 0.500 500

2 0.400 0.200 0.250 250

3 0.400 0.200 0.500 125

It is determined that the order of reaction with respect to carbon monoxide is 1.


(I) Calculate the initial rate for each of the three experiments, and use the results to
determine the order of reaction with respect to CH2=CH2 and H2.

(II) Hence write a rate equation for the reaction and calculate a value for the rate
constant, giving its units.

(iii) Draw the structural formula of the product of the oxo reaction if the starting alkene is
but2ene instead. [8]

(b) Propanal can be prepared from chloroethane under laboratory conditions, using propanoic
acid as an intermediate. Propose a reaction scheme, in not more than 4 steps, for this
conversion. Your answer should include the reagents and conditions, as well as the
structures of the intermediate compounds formed. [3]

(c) Both ethanol and chloroethane can undergo a similar reaction to form ethene but using
different reagents and conditions.
(i) Suggest the name of this reaction.

(ii) Give the reagents and conditions to form ethene from ethanol and chloroethane
respectively. [3]

DHS 2010 8872/02/Prelim 2010 [Turn over


14

(d) While Sean is doing his experiment in the laboratory, the labels of 3 testtubes
each containing 1phenylethanol, 1bromoethylbenzene and 1
chloroethylbenzene fell off. The structures of the 3 compounds are shown below:

CHCH3 CHCH3 CHCH 3

OH Br Cl

1phenylethanol 1bromoethylbenzene 1chloroethylbenzene

(i) Suggest how Sean can carry out 2 different tests to distinguish the 3
compounds from one another. For each test, state the reagents and
conditions, and give the observations you would make with each compound.

(ii) Both 1bromoethylbenzene and 1chloroethylbenzene can react with


ethanolic ammonia to form amines. Predict and explain which of the two
compounds will have a faster reaction with ethanolic ammonia. [6]

[Total: 20]

DHS 2010 8872/02/Prelim 2010


HWA CHONG INSTITUTION

C2 PRELIMINARY EXAMINATION
H1 CHEMISTRY 8872/01
PAPER 1
Multiple Choice
24 September 2010 1 hour

Do not open this booklet until you are told to do so.

READ THESE INSTRUCTIONS FIRST


Write in soft pencil.
Do not use staples, paper clips, highlighters, glue or correction fluid.
Complete the information on the optical mark sheet (OMS) as shown below.

Write your name

Write your CT group

Write and
shade your
NRIC
or FIN number

There are thirty questions on this paper. Answer all questions. For each question, there are four
possible answers A, B, C and D.
Choose the one you consider correct and record your choice in soft pencil on the OMS.

Each correct answer will score one mark. A mark will not be deducted for a wrong answer.
Any rough working should be done in this booklet.

This document consists of 12 printed pages.


2

Section A
For each question, there are four possible answers, A, B, C and D. Choose the one you consider
to be correct.

1 Which of the following statements is correct?

A 24 dm3 of hydrogen gas at 25 C and 1 atm contains 6.0 1023 hydrogen atoms.

B 500 cm3 of 1 mol dm-3 aqueous copper(II) nitrate, Cu(NO3)2, contains 3.0 1023
nitrate ions.

C 256 g of sulfur contains 6.0 1023 S8 molecules.

D 1 mole of sodium hydrogensulfate, NaHSO4, dissolves in water to form 1.8 1024


ions.

2 When 10 cm3 of a gaseous hydrocarbon was completely burnt in excess oxygen, 30 cm3
of carbon dioxide and 40 cm3 of water vapour were formed.
Given that all volumes were measured at the same temperature and pressure, what is the
formula of the hydrocarbon?

A C2H2 B C2H6 C C3H4 D C3H8

3 Which of the following ions has more electrons than protons and more neutrons than
protons? [T = 31H ]

A T B He+ C OH D H3O+

4 An element X forms an ion Xn+.


Which of the following statements about X and Xn+ is correct?

A The radius of Xn+ is greater than that of X.

B The number of protons in Xn+ is greater than that of X.

C Less energy is released when an electron is added to Xn+ than to X.

D More energy is required to remove the outermost electrons from Xn+ than from
X.

8872/01/HCI/C2 Prelim/2010
3

5 An ion IF4n- has a square planar structure as shown below.


n-
F F
I

F F

What is the value of n?

A 1 B 2 C 3 D 4

6 Ice is the crystalline form of water. The diagram below shows part of the structure of ice.

Which of the following statements is not true about ice?

A Ice has a lower density than water at 0 oC due to its open structure.

B The bond angle about oxygen in ice is 109.5o.

C Ice has a giant covalent structure.

D Ice does not conduct electricity.

7 When 13.08 g of zinc was added to 250 cm3 of 1.0 mol dm3 aqueous copper(II) sulfate, the
temperature of the solution rose by 15 oC.
[Specific capacity of the final solution is 4.20 J g1 K1.]
Cu2+(aq) + Zn(s) Zn2+(aq) + Cu(s)
What is the enthalpy change (in kJ mol1) for the above reaction?

A 82.9 B 78.8

C 66.3 D 4.12

8872/01/HCI/C2 Prelim/2010
4

8 Given: KOH(aq) + HBr(aq) KBr(aq) + H2O(l) Hr = 57 kJ mol1


Which of the following will have the same value for the enthalpy change of reaction, Hr,
as the above reaction?

A NH3(aq) + HBr(aq) NH4Br(aq) + H2O(l)

B NaOH(aq) + H2SO4(aq) Na2SO4(aq) + H2O(l)

C Ba(OH)2(aq) + CH3CO2H(aq) Ba(CH3CO2)2(aq) + H2O(l)

D Ba(OH)2(aq) + 2HNO3(aq) Ba(NO3)2(aq) + 2H2O(l)

9 Potassium-argon dating is used to determine the age of igneous rocks. Potassium in these
rocks gradually breaks down into argon and the gas is trapped in the rock. This radioactive
decay is first order reaction with a half-life of 1.3 x 109 years.
A sample of igneous rock, containing some potassium, is examined. If the amount of argon
gas measured is 7 times as much as the amount of potassium, what is the age (in years)
of the rock?

A 1.3 x 109 B 2.6 x 109

C 3.9 x 109 D 9.1 x 109

10 An experiment was carried out to investigate the initial rate of reaction between K2S2O8(aq),
an oxidising agent, and KI(aq). The initial concentrations of K2S2O8(aq) and KI(aq) in the
mixture together with the time taken for the mixture to darken for the various experimental
runs are given below.

Initial [K2S2O8] / mol dm3 Initial [KI] / mol dm3 Time taken to darken / seconds

0.10 0.20 35

0.05 0.20 70

0.10 0.067 105

0.02 0.75 Y

What is the value of Y in the table?

A 72 B 60 C 47 D 40

8872/01/HCI/C2 Prelim/2010
5

11 An experiment is set up to measure the rate of hydrolysis of ethyl ethanoate.


CH3CO2CH2CH3 + H2O CH3CO2H + CH3CH2OH
The hydrolysis is found to be slow in neutral aqueous solution, but proceeds at a
measurable rate when the solution is acidified with hydrochloric acid.
What is the function of hydrochloric acid in the reaction mixture?

A to maintain a constant pH during the reaction

B to ensure that the reaction reaches equilibrium

C to increase the reaction rate by catalytic action

D to suppress ionisation of the ethanoic acid formed

12 Given: A2(g) + 4B(g) 2AB2(g) Kc = 4.8 (numerical value).


What is the value of Kc for the following reaction?
AB2(g) A2(g) + 2B(g)

1
A 2.4 B
2.4

1 1
C D
4.8 4.8

13 The table below shows the values of the ionic product of water, Kw, at two different
temperatures.

Temperature / oC Kw / mol2 dm6

25 1.00 x 1014
62 1.00 x 1013

Which of the following is correct for pure water at 62 oC?

A [H+] > [OH] B pH = 7

C pOH < 7 D pH = 14 - pOH

8872/01/HCI/C2 Prelim/2010
6

14 Ga, Ge, As and Se are successive elements in Period 4 of the Periodic Table.
Which of the following statements is true?

A Ga2+ and Ni are isoelectronic.

B Both Ge and Se have two unpaired electrons at ground state.

C First ionisation energy increases from Ga to Se.

D As has only three unpaired electrons, thus the highest chloride formed is AsCl3.

15 One mole of magnesium, aluminium and sulfur are each completely burnt in an excess of
oxygen gas.
Which graph shows the volume of oxygen used in each case?

A volume of B volume of
oxygen used oxygen used
x x x

x x

0 0
Mg Al S Mg Al S

C volume of D volume of
oxygen used oxygen used
x x

x x
x
x

0 0
Mg Al S Mg Al S

8872/01/HCI/C2 Prelim/2010
7

16 The following reaction scheme shows the synthesis of compound C from methylbenzene
through a series of reactions.

O
CH3 CH3 CH3
C N
CH2Cl CH2NH2 C
CH3Cl NH3 O

II II
II

compound C
Which of the following shows the correct reaction types for steps I and II?

I II

A addition addition

B addition condensation

C substitution substitution

D substitution elimination

17 Compound E, C4H10, reacts with chlorine gas in the presence of light to form two
monochlorinated alkanes, F and G, in an approximate molar ratio of 9 : 1.
What are the structures of E and F?

E F

A CH3CH(CH3)CH3 CH3CH(CH3)CH2Cl

B CH3CH(CH3)CH3 CH3CCl(CH3)CH3

C CH3CH2CH2CH3 CH3CH2CHClCH3

D CH3CH2CH2CH3 CH3CH2CH2CH2Cl

18 Which reagent, in its reactions with compound H, shows both addition and substitution?

CO2H

CH3 CH CH compound H

CO2H

A bromine B ethanol

C hydrogen bromide D steam

8872/01/HCI/C2 Prelim/2010
8

19 The following compound is tested with two reagents separately.

COCH3

CHO

What are the expected observations?

Warm with Fehlings reagent Warm with Tollens reagent

A brick-red precipitate silver mirror

B no precipitate silver mirror

C brick-red precipitate no silver mirror

D no precipitate no silver mirror

20 Compound J has the following structure.

1 2 3 4
CH3CH(OH)CH=CHBr

Which statement is not correct for compound J?

A None of its structural isomers reacts with 2,4-dinitrophenylhydrazine.

B The terminal carbon atom (4) has a trigonal planar geometry.

C J reacts with acidified potassium dichromate(VI).

D J exhibits geometrical isomerism.

21 Which sequence shows the compounds in order of decreasing pKa values?

A CH3CO2H > CF3CO2H > C2H5OH > C6H5OH

B CF3CO2H > CH3CO2H > C2H5OH > C6H5OH

C C2H5OH > C6H5OH > CH3CO2H > CF3CO2H

D C6H5OH > C2H5OH > CH3CO2H > CF3CO2H

8872/01/HCI/C2 Prelim/2010
9

22 1 mole of an organic compound K reacts with 2 moles of NaOH(aq) when heated. 1 mole of
K also reacts with PCl5 to produce 1 mole of HCl(g).
Which compound could K be?

CH2OH CH2Br

A B
Br CH2Br Br CO2H

CHO

C D Br CO2H
Br CO2H

23 Aspirin is a common analgesic used in the treatment of headaches and muscle aches.

C OH

O CH3
aspirin
C

Which of the following is true about aspirin?

A It is soluble in water.

B Its aqueous solution is acidic.

C It gives a yellow precipitate when warmed with OH/I2(aq).

D It gives an orange precipitate with 2,4-dinitrophenylhydrazine.

8872/01/HCI/C2 Prelim/2010
10

24 The reaction scheme below shows the synthesis of compound S.

KMnO4 / H2SO4 (aq)

Which of the following shows the structural formula of S?

A B

C D

25 Compound T has the following properties:


(i) it does not decolourise aqueous bromine;
(ii) it produces an effervescence when reacted with Na2CO3(aq), and
(iii) it decolourises hot acidified KMnO4(aq).
Which of the following can be T?

CH3 CH2 CH3 CH2OH


CH

CH3
A B

CO2H CO2H

CH2OH

CH3
C D

CN

8872/01/HCI/C2 Prelim/2010
11

Section B

For each of the questions in this section, one or more of the three numbered statements
1 to 3 may be correct.
Decide whether each of the statements is or is not correct. (You may find it helpful to put a tick
against the statements that you consider to be correct).

The responses A to D should be selected on the basis of

A B C D

1, 2 and 3 are 1 and 2 only 2 and 3 only 1 only is


correct are correct are correct correct
No other combination of statements is used as a correct response.

26 The conversion of CrO42(aq) into Cr2O72(aq) is represented by the following equation:

2CrO42(aq) + 2H+(aq) Cr2O72(aq) + H2O(l)


yellow orange
Which statements are true of this reaction?

1 Addition of OH(aq) to Cr2O72(aq) causes a change of colour.

2 CrO42(aq) acts as a Brnsted base.

3 The conversion of CrO42(aq) into Cr2O72(aq) involves a change of oxidation state.

27 Magnesium reacts readily with dilute hydrochloric acid to form magnesium chloride and
hydrogen.
Mg(s) + 2HCl(aq) MgCl2(aq) + H2(g)
The standard enthalpy change for this reaction can be measured in the laboratory.
What further information is needed in order to calculate the standard enthalpy change of
formation of magnesium chloride?

1 Hfo for HCl(aq)

2 Hfo for H2(g)

3 first and second ionisation energies of Mg

8872/01/HCI/C2 Prelim/2010
12
The responses A to D should be selected on the basis of
A B C D

1, 2 and 3 are 1 and 2 only 2 and 3 only 1 only is


correct are correct are correct correct
No other combination of statements is used as a correct response.

28 A theoretical reaction involves A + B product.


The rate equation is: Rate = k[A]p[B]q and the units of the rate constant, k, are (mol dm3)r
min1.
Which sets of the values of p, q and r fit the above information?

p Q r

1 0 1 0

2 1 2 2

3 2 0 1

29 A mixture of two oxides of Period 3 elements is added to water. The solution formed is
almost neutral.
What could the constituents of the mixture be?

1 Na2O and P4O10

2 MgO and SiO2

3 P4O10 and SO3

30 Which of the following statements are true regarding compound Z?

1 A molecule of Z contains four bonds.

2 Z can exist as four cis-trans isomers.

3 Z can be oxidised by acidified potassium dichromate(VI).

~ End of Paper 1~

8872/01/HCI/C2 Prelim/2010
NAME: _______________________________________ CT: ____________

Centre Number: _______________________ Index Number: _____________

HWA CHONG INSTITUTION

C2 PRELIMINARY EXAMINATION
H1 CHEMISTRY 8872
PAPER 2

21 September 2010 2 hr

Candidates answer Section A on the Question Paper.


Additional Materials: Answer Paper
Data Booklet

READ THESE INSTRUCTIONS FIRST


Write your Centre number, index number and name on all the work you hand in.
Write in dark blue or black pen.
You may use a pencil for any diagrams, graphs or rough working.
Do not use staples, paper clips, highlighters, glue or correction fluid.

Section A
Answer all the questions.

Section B
Answer two questions on separate answer paper.
At the end of the examination, fasten all your work securely together.
The number of marks is given in brackets [ ] at the end of each question or part question.

FOR EXAMINERS USE ONLY


TOTAL
Section A Section B
Multiple Choice
(Structured) (Free Response)

Q1 / 9 Q5 / 20

Q2 / 9 Q6 / 20

Q3 / 12 Q7 / 20

Q4 / 10

/ 30 Subtotal / 40 Subtotal / 40 110

This question booklet consists of 13 printed and 1 blank pages.


2

Section A
Answer all the questions in this section in the spaces provided.

1 Copper and iodine are both shiny crystalline solids. They have the same face-centered cubic
crystal structures.
The diagram shows the arrangement of the particles in this type of crystal lattice.

In the diagram above, the particles present are represented by .

(a) Complete the table below about the type of particles and their interactions within the
crystals.

solid type of particles interactions within the crystals

copper
crystals

iodine
crystals

[2]
(b) Explain briefly why copper is malleable and ductile.


[1]
(c) Give another physical property present in one of the above solids but absent in the other.
Explain your answer.

Physical property:

Reason:


[2]

8872/02/HCI/C2 Prelim/2010 [Turn over


3

(d) A sample of copper contains the two isotopes 63Cu and 65Cu only. An experiment is
conducted to find the relative atomic mass, Ar, of this sample and is found to be 63.9.
(i) Explain why the value of Ar found is not a whole number.

(ii) Suggest why the value of Ar stated above differs from the value given in the
Periodic Table.


[2]
(e) When separate beams of 63Cu2+ and 127I are passed through an electric field in the
apparatus below, they behave differently.

(i) Sketch to show the paths of the beams of 63Cu2+ and 127
I in the electric field on the
above diagram.

(ii) Given that the angle of deflection of the Cu2+ beam is 7.0o, calculate the angle of
deflection of the I beam, correct to 1 decimal place.

Angle of deflection: _______________


[2]
[Total: 9]

8872/02/HCI/C2 Prelim/2010 [Turn over


4

2 Bromoethane, C2H5Br, is hydrolysed by NaOH(aq). The rate of reaction can be determined by


withdrawing samples of the solution containing C2H5Br and NaOH(aq) at known times, and
titrating them with HNO3(aq).
Two sets of experiments were performed where the initial concentration of C2H5Br was varied
while the initial concentration of NaOH(aq) was kept the same at 0.1 mol dm3. The results
obtained are shown in the graphs below.
Volume of HNO3 / cm3
Volume of nitric acid / cm

[C2H5Br] = 1.0 mol dm3

[C2H5Br] = 2.0 mol dm3

Time / min

(a) (i) Using the graph where [C2H5Br] = 1.0 mol dm3, find the order of reaction with
respect to NaOH(aq).

(ii) Using the graphs above, deduce the order of reaction with respect to C2H5Br.

(iii) Hence, write down the rate equation for this reaction.

[5]

8872/02/HCI/C2 Prelim/2010 [Turn over


5

(b) With the aid of a sketch of the Boltzmann distribution in each case, explain why an
increase in
(i) temperature,
(ii) concentration of C2H5Br,
increases the initial rate of the hydrolysis reaction.

(i)

...

...

...

(ii)

...

...

...
[4]
[Total: 9]

8872/02/HCI/C2 Prelim/2010 [Turn over


6

3 (a) (i) Complete the following reaction scheme starting from compound B,
CH3COCH(OH)CH(CH3)2.
Write the structural formulae of all the organic compounds, C, E, F and G.

B
CH3COCH(OH)CH(CH3)2

excess conc. F
H2SO4, 170 oC

D C
CH3COCH2CCl(CH3)2 HCl(g) Step II

steam,
conc. H3PO4
high T and P CH3COCO2H

E
Step I

conc. H2SO4,
heat

(ii) State the reagents and conditions used in Steps I and II.

Reagents and conditions

Step I

Step II

[5]

8872/02/HCI/C2 Prelim/2010 [Turn over


7

(b) Name the two functional groups in B and show how you would identify each
functional group using a simple chemical test.

functional group chemical test observations

[4]

(c) (i) H, CH3COC4H7, is a structural isomer of C which exhibits cis-trans


isomerism. Draw and label the structures of this pair of isomers.

(ii) Account for the existence of cis-trans isomers in H.

...

...
[3]
[Total: 12]

8872/02/HCI/C2 Prelim/2010 [Turn over


8

4 (a) Ibuprofen is a non-steroidal ant-inflammatory drug for treating chronic arthritic pain of
the joints. It is a white powder with a melting point of 74 77 C. It is only slightly
soluble in water but readily soluble in organic solvents such as ethanol.
To determine the amount in each tablet, the ibuprofen tablets are weighed, crushed
and a known amount is dissolved in a solvent to prepare a standard 100 cm3 solution.
The solution is then filtered to remove the additives. A known aliquot of ibuprofen
solution is then titrated against a standard solution of sodium hydroxide.

CH3
CH3 O
CH CH2 CH C
CH3 OH

Ibuprofen

(i) Write down the molecular formula of ibuprofen.

(ii) Calculate the relative molecular mass, Mr, of ibuprofen and hence the mass of
ibuprofen needed to make 100 cm3 of a 0.15 mol dm-3 solution.

(iii) Oxidation of ibuprofen produces a dibasic acid J.


A solution containing 0.10 g of J required 12.0 cm3 of 0.10 mol dm-3 NaOH(aq)
for neutralisation. Suggest a structure for J, showing your working.

[6]

8872/02/HCI/C2 Prelim/2010 [Turn over


9

b (i) Write an expression for the Ka for ibuprofen.


[You may represent ibuprofen using RCO2H.]

(ii) Calculate the pH of 0.15 mol dm-3 solution of ibuprofen.


[Given Ka for ibuprofen = 6.3 x 10-6 mol dm-3.]

(iii) Use the data in the following table to suggest a suitable indicator for the titration
of ibuprofen solution with aqueous sodium hydroxide. Justify your answer.
indicator pH at which colour changes
methyl violet 01
methyl orange 34
bromothymol blue 67
phenolphthalein 9 10

...

...
[4]
[Total: 10]

8872/02/HCI/C2 Prelim/2010 [Turn over


10

SECTION B
Answer two of the three questions in this section on separate answer paper.

5 (a) 5-hydroxypentanoic acid can undergo intramolecular esterification to form a cyclic


ester, P, as shown.
O
HOCH2CH2CH2CH2CO2H(l) O (l) + H2O(l)

5-hydroxypentanoic acid P

This esterification system exists in a dynamic equilibrium.


(i) Explain what is meant by the term dynamic equilibrium.
(ii) Write an expression for the equilibrium constant, Kc, for this reaction.
(iii) In an experiment, 2.0 mol of 5-hydroxypentanoic acid was allowed to
undergo esterification.
Given that 5% of 5-hydroxypentanoic acid remained at equilibrium and the
total volume of the mixture remained constant at 200 cm3, calculate the Kc
for this reaction, giving its units.
[4]
(b) The decomposition of hydrogen peroxide is a first order reaction.
H2O2 H2O + O2 Ho = 98 kJ mol1
The uncatalysed reaction has activation energy of 79 kJ mol1. Both the enzyme
catalase and platinum act as catalysts and speed up the reaction.
(i) Explain what is meant by each of the following terms.
I order of reaction
II activation energy
(ii) Use the data provided to construct a reaction pathway diagram for the
uncatalysed reaction.
(iii) Describe how the activation energy is affected by the presence of enzyme
catalase, and explain how this affects the rate of the decomposition of
hydrogen peroxide.
(iv) Suggest a simple method as how the rate of decomposition of hydrogen
peroxide can be measured in the laboratory.
[7]
(c) Compound Q, C11H14, exhibits cis-trans isomerism. Q reacts with hot acidified
potassium manganate(VII) to give R, C9H8O3. R reacts with aqueous alkaline iodine
to give a yellow precipitate, S and a residual solution. Acidification of the residual
solution yields benzene-1,2-dicarboxylic acid as shown below.

Q reacts with steam under suitable conditions to give T, C11H16O. T does not react
with acidified potassium dichromate(VI).
Give the structural formulae of Q, R, S and T. Explain the reactions described and
write equations for the reaction where appropriate.
[9]
[Total: 20]

8872/02/HCI/C2 Prelim/2010 [Turn over


11

6 (a) The chlorides of elements sodium to phosphorus all dissolve in or react with water.
Two of the chlorides, magnesium chloride and silicon chloride, differ in their
reactions with water.
Describe and explain these differences in terms of their structure and bonding,
writing equations for any reactions that occur.
[4]
(b) The table below gives the data on some oxides of elements in Period 3 of the
Periodic Table.
Oxide Na2O MgO Al2O3 SiO2 P4O6 SO2
Melting point / K 1193 3125 2345 1883 297 200
Boiling point / K 1548 3873 3253 2503 448 263

(i) Drawing diagrams where appropriate, suggest, in terms of structure and


bonding, explanations for the following.
I SO2 has a low boiling point.
II The melting point of MgO is higher than that of Na2O.
III The melting point of SiO2 is much higher than that of P4O6.
[6]
(c) Sulfur and chlorine react to give a compound, A, containing 31% by mass of S.
(i) Calculate the empirical formula of A.
(ii) Draw a dot-and-cross diagram and state the shape of A, assuming its
molecular formula is the same as its empirical formula.
[4]
(d) The tertiary alcohol 3-methylpentan-3-ol is used in the manufacture of flavourings.
A possible synthetic route, starting from chloromethane, is given below.

(i) State the reagents and conditions required for reactions I, II and III.
(ii) Use the bond energy values from the Data Booklet to explain how the rate of
reaction I would change if iodomethane, CH3I, is used as the starting
material.
[6]
[Total: 20]

8872/02/HCI/C2 Prelim/2010 [Turn over


12

7 (a) Phosphorus is a non-metal discovered by the German physician Hennig Brand in 1669.
In his experiments, Brand found that the white material (white phosphorus) glowed in
the dark and spontaneously burst into a bright white flame when burned in air.
(i) Write equations for the reactions that occur when a sample of phosphorus is
burned in excess oxygen, and water is added to the resulting oxide.
(ii) Suggest the pH of the aqueous solution formed.
[3]

(b) The equation below shows how phosphorus pentachloride, PCl5, reacts with a limited
amount of water.
PCl5(s) + H2O(l) POCl3(l) + 2HCl(g)
Given the following data.
Standard enthalpy change of formation of PCl5(s) = 444 kJ mol1
Standard enthalpy change of formation of POCl3(l) = 1186 kJ mol1
Standard enthalpy change of formation of HCl(g) = 92 kJ mol1
Standard enthalpy change of combustion of H2(g) = 286 kJ mol1
(i) What do you understand by the term standard enthalpy change of formation?
(ii) Use the energy cycle below and the given data to calculate Hro.
Hro
PCl5(s) + H2O(l) POCl3(l) + 2HCl(g)

5
P(s) + H2(g) + O2(g) + Cl2(g)
2
[3]
(c) Phosphorus pentachloride, PCl 5, is involved in the following reaction scheme.

PCl5(s) V conc. NH3 W


CH2CH2OH heat
C8H11N

U
I
II CH3O Na+
heat
CH2CO2H

X Y

(i) Draw the structural formulae of compounds V, W and Y.


(ii) State the reagents and conditions needed for reaction I.
(iii) State the reagents and conditions needed to convert X to U.
(iv) Name the type of reaction involved in II.
[6]

8872/02/HCI/C2 Prelim/2010 [Turn over


13

(d) Phosphates have an amazing array of applications in home and industry. For example,
sodium hydrogenphosphate, Na2HPO4, is a laxative ingredient and is used to adjust the
acidity of boiler water.
(i) A buffer solution of H2PO4(aq) and HPO42(aq) can be made by adding a small amount
of solid NaOH to an aqueous solution of NaH2PO4.
Given: H2PO4(aq) H+(aq) + HPO42(aq) Ka = 6.30 107 mol dm3
I Explain what is meant by the term buffer solution.
II Write equations to explain how the addition of a small amount of solid sodium
hydroxide to an aqueous solution of NaH2PO4 enables it to maintain a fairly constant
pH.
[4]
2
(ii) The structure of hydrogenphosphate ion, HPO4 , is as shown.
O
O P O H
O
When two hydrogenphosphate ions undergo a condensation reaction, they form a
diphosphate ion, P2O74 and a water molecule.
I Write an equation for the above reaction.
II Draw the structure of P2O74.
III State the shapes and bond angles of the ion about each central atom.
[4]
[Total: 20]
- End of paper -

8872/02/HCI/C2 Prelim/2010 [Turn over


Mark Scheme for 2010 H1 Chemistry Prelim

Paper 1

1 C 2 D 3 A 4 D 5 A

6 C 7 B 8 B 9 C 10 C

11 C 12 C 13 C 14 B 15 D

16 C 17 A 18 A 19 B 20 A

21 C 22 B 23 B 24 D 25 B

26 B 27 D 28 A 29 B 30 D

Paper 2 Section A

1 (a)

type of particles interactions within the crystals

Copper
cations / Cu2+ ions metallic bonding
crystals

Iodine van der Waals forces / dispersion forces


I2 molecule
crystals / id-id interactions

(b) The sea of delocalised electrons is able to accommodate any deformation to the
cationic lattice

OR The layers of cations can slide past each other while electrons hold them
together.

(c) Physical property: electrical conductivity / solubility in organic solvent

Reason:

Conductivity: mobile electrons available to conduct electricity in Cu but no


mobile electrons or ions in I2.

Solubility: similar intermolecular forces of attraction for iodine & organic


solvent, thus I2 can form favourable interactions with the organic solvent , but
Cu cannot form favourable interactions with the organic solvent

(d)(i) Relative atomic mass is the weighted average of the masses of the isotopes (or
words to this effect).

(ii) % abundance of the isotopes in this sample differs from what is normally
obtained.
2

OR There are more than 2 types of isotopes of copper.

(e)(i) correct direction & angle of deflection.

e
(ii) angle of deflection
m

1
angle of deflection of I- = 7 x 127 = 1.8o
2
63.5

2 (a)(i)

Use graph to show constant t1/2 ~3 min.

Reaction is first order wrt aqueous sodium hydroxide.


3

(ii) Calculation (using initial rates) or other points on the graphs to show that when
[C2H5Br] doubled, rate (or gradient of the graph) doubled.

Reaction is thus first order wrt to C2H5Br

(iii) Rate = k [C2H5Br] [OH]

(b)(i) T1 < T2 (label of axes can be different)

increasing T increases the proportion (number) of molecules with minimum Ea


for reaction & thus ..

(ii) increase in [C2H5Br]

When [C2H5Br] increases, collision frequency / number of effective collisions


increases as number of molecules with the minimum activation energy
increases

3(a)(i) C E

CH3COCH=C(CH3)2 CH3COCH2C(OH)(CH3)2

F G
4
O

CH3 C CH3 CH3COCO2C(CH3)2CH2COCH3

(ii) Step I: NaOH(aq), heat

Step II: KMnO4(aq), dil H2SO4, heat

(b)

Functional group Chemical test + Observations

K2Cr2O7(aq), dil H2SO4, heat Orange solution turns green


Secondary alcohol
PCl5(s) White fumes HCl

Ketone 2,4-dinitrophenylhydrazine Orange ppt

(c) (i) H

CH3CO H CH3CO CH3

C C C C

CH3 CH3 CH3 H

cis trans

OR

CH3CO H CH3CO CH2CH3

C C C C

H CH2CH3 H H

cis trans

OR

CH3COCH2 H CH3COCH2 CH3

C C C C

H CH3 H H

cis trans

(ii) The C =C bond cannot undergo free rotation as the bond in C=C bond will be
broken.

OR There is a restriction in rotation of C=C bond.

4(a)(i) C13H18O2
5

(ii) Mr = 206

Mass = 0.15 x (100/1000) x 206 = 3.10 g

(iii) No. of moles of NaOH = 0.1 x 12/1000 = 1.2 x 10-3

No. of moles of acid, J = 0.6 x 10-3

Mr = 0.1/ 0.6 x 10-3 = 167

(Mr of 2-CO2H = 90, thus Mr of remaining part = 97, the benzene ring)

HO2C CO2H

Structure of J:

[H + ] [ RCO -2 ]
b(i) Ka =
[RCO 2H]

(ii) [H+] = (Ka x C) = (6.3 x 10-6 x 0.15) = 9.72 x 10-4 mol dm-3

pH = 3.0

(iii) phenolphthalein

pH at equivalence point of a strong base & weak acid titration is about 9, which
corresponds to its working range

SECTION B

5(a)(i) A system is said to be in dynamic equilibrium when the rate of forward reaction
is equal to the rate of backward reaction.
H2O
O O

[ester][H 2 O] O

(ii) Kc = or
[acid] HO OH

1.9 2
( )
(iii) Kc = 0.2 = 180.5 mol dm3
0.1
0.2
6

(b)(i) I Order of Reaction with respect to a reactant is the power to which the
concentration of that reactant is raised in the rate equation.

II activation energy: energy that must be overcome in order for a chemical


reaction to occur / the minimum energy required to start a chemical reaction.

(ii)

(iii) activation energy is lowered in the presence of enzyme catalase, more


molecules have the minimum Ea & hence higher collision frequency of collision
& higher rate of the decomposition of hydrogen peroxide

(iv) Measure vol of O2 evolved at various time intervals & plot graph of the vol vs.
time to obtain gradient at various t

(c) Q R T S: CHI3
CO2H

C O

CH3

Q R: Oxidative cleavage of alkene

Q R: Side-chain oxidation of benzene ring

R undergoes oxidation with alkaline iodine as it is a methyl ketone

Q T: Hydration of alkene to alcohol

S is a tertiary alcohol as it has no reaction with acidified dichromate(VI)

Q exhibits cis-trans isomerism due to 2 different groups on each C of double


bond
7

6(a) MgCl2(s) - giant ionic lattice structure with strong electrostatic forces of
attraction between Mg2+ & Cl ions.

It dissolves in water to form a slightly acidic solution as Mg2+ has a high charge
density & undergoes hydrolysis in water.
MgCl2(s) + 6H2O(l) [Mg(H2O)6]2+(aq) + 2Cl(aq)

[Mg(H2O)6]2+(aq) + H2O(l) [Mg(H2O)5(OH)]+(aq) + H3O+(aq)

SiCl4(l) - simple covalent molecules with inter-molecular dispersion forces.

It undergoes complete hydrolysis in water to give a highly acidic solution of pH


1 - 2.

SiCl4(l) + 2H2O(l) SiO2(s) + 4HCl (aq)

OR SiCl4(l) + 4H2O(l) SiO2.2H2O(s) + 4HCl(aq)

(b)(i) I SO2 is simple molecular

& has weak intermolecular van der Waals forces / pd-pd forces & thus little E is
needed to break them & hence low bp

(reject dispersion forces as SO2 is polar)

q+ x q
II LE ; q+ of Mg2+ > q+ of Na+ while r+ of Mg2+ < r+ of Na+,
r+ + r

stronger electrostatic forces of attraction between Mg2+ & O2 (ionic bonding) in


MgO than in Na2O & hence MgO has a higher mp

III SiO2 is macromolecular with large no of covalent bonds to be broken for melting
to occur;

P4O6 is simple molecular with much weaker intermolecular van der Waals
forces...

(c)(i)

S Cl

% 31 69

Rel mol 0.97 1.94

ratio 1 2

empirical formula is SCl2

(ii) Dot-&-cross diagram:


8

Shape: bent (S has 2 bp & 2 lone pair of electrons)

(d)(i) I: ethanolic KCN, heat

II: dilute H2SO4, heat OR dilute HCl, heat

III: PBr3, heat OR red phosphorus with Br2, heat

(ii) BE(C-I) = 240 kJ mol1; BE(C-Br) = 280 kJ mol1

BE(C-I) is lower, i.e., C-I bond is weaker & is easier to be broken


Iodomethane has a higher rate of hydrolysis

7(a)(i) Phosphorus is burned in excess oxygen:

4P(s) + 5O2(g) P4O10(s)

Or P4(s) + 5O2(g) P4O10(s)

Or 2P(s) + 5/2O2(g) P2O5(s)

OR P4(s) + 5/2O2(g) P2O5(s)

Water is added to the resulting oxide:

P4O10(s) + 6H2O(l) 4H3PO4(aq)

Or 2P2O5(s) + 6H2O(l) 4H3PO4(aq)

(ii) pH: 1 - 3

(b)(i) Standard enthalpy change of formation energy change when 1 mol of a


compound is formed from its constituent elements at 298 K & 1 atm pressure.

Hr
(ii) PCl5(s) + H2O(l) POCl3(l) + 2HCl(g)

-444 -286 -1186 2(-92)

5
P(s) + H2(g) + O2(g) + Cl2(g)
2


H r = 2(-92) + (-1186) {(-444) + (-286)}

= -640 kJ mol1

c(i) V W Y
9

CH2CH2Cl CH2CH2NH2 CH2CH2OCH3

(ii) Acidified K2Cr2O7(aq), heat

(iii) LiAlH4 in dry ether

(iv) Substitution

(d)(i) I buffer solution: a solution that is able to resist pH changes upon


addition of a small amount of an acid or a base.

II Equation for reaction when a small amount of NaOH(s) is added:

H2PO4(aq) + NaOH(s) HPO42(aq) + H2O(l) + Na+(aq)

Comaprable maounts of H2PO4(aq) & HPO42(aq) would be present in solution


to enable it to act as a buffer.

Equation for reaction when small amount of an acid is added:

HPO42(aq) + H+(aq) H2PO4(aq)

Equation for reaction when small amount of a base is added:

H2PO4(aq) + OH(aq) HPO42(aq) + H2O(l)

(ii) I 2HPO42(aq) P2O74(aq) + + H2O(l)

O O
O
P P
-O O-
O- -O
II

III sahpe:tetrahedral wrt P; V-shaped wrt O

Bond angles: 109.5o wrt P; 105o wrt O


1
INNOVA JUNIOR COLLEGE
JC 2 PRELIMINARY EXAMINATION 2
in preparation for General Certificate of Education Advanced Level
Higher 1

CANDIDATE
NAME

CLASS INDEX NUMBER

CHEMISTRY 8872/01
Paper 1 Multiple Choice 20 September 2010
50 minutes
Additional Materials: Data Booklet
Multiple Choice Answer Sheet

READ THESE INSTRUCTIONS FIRST

Write your name and class on all the work you hand in.
Write in soft pencil.
Do not use staples, paper clips, highlighters, glue or correction fluid.

There are thirty questions on this paper. Answer all questions. For each question there are four
possible answers A, B, C and D.
Choose the one you consider correct and record your choice in soft pencil on the separate
Answer Sheet.

Read the instructions on the Answer Sheet very carefully.

Each correct answer will score one mark. A mark will not be deducted for a wrong answer.
Any rough working should be done in this booklet.

This document consists of 12 printed pages.

Innova Junior College [Turn over


2
Section A

For each question there are four possible answers, A, B, C, and D. Choose the one you consider
to be correct.

1 When 20 cm3 of a gaseous hydrocarbon was completely burnt in an excess of oxygen,


60 cm3 of carbon dioxide and 40 cm3 of water vapour were formed. All volumes are
measured at the same temperature and pressure.

What is the formula of the hydrocarbon?

A C3H4 C C3H8
B C3H6 D C6H8

2 In an experiment, 50 cm3 of a 0.1 mol dm-3 solution of a metallic salt reacted exactly with
25 cm3 of a 0.1 mol dm-3 sodium sulfite solution. The half-equation for oxidation of sulfite
ion is:

SO3 2-(aq) + H2O(l) SO4 2-(aq) + 2H+(aq) + 2e

If the original oxidation number of the metal in the salt was +3, what would be the final
oxidation number of the metal?

A 0 C +2
B +1 D +4

3 Which one of the following corresponds to the configuration of the three electrons of the
highest energy for a Group III element?

A 1s2 2s1
B 2s1 2p2
C 3p3
D 4s2 4p1

4 The use of Data Booklet is relevant to this question.

In research on the atomic nucleus, scientists have been comparing the stability of isotopes
with the same neutron : proton ratio.

Which isotope has the same neutron : proton ratio as 10B?


32 32
A P C S
40 40
B Ar D K

PRELIM 2 INNOVA 8872/01/2010 [Turn over


3
5 The use of Data Booklet is relevant to this question.

The successive ionisation energies, in kJ mol-1, of an element A are given below.

870 1800 3000 3600 5800 7000 13200

What could A be?

A 51Sb C 49In

B 40Zr D 34Se

6 Three substances, D, E and F have physical properties as shown.

substance melting point/ oC boiling point/ oC electrical conductivity


of solid of liquid
D 801 1413 poor good
E 2852 3600 poor good
F 4827 4827 good not known

What would be the most likely substances of D, E and F?

D E F

A NaF BeCl2 Cu
B NaBr CaO SiO2
C NaCl MgO graphite
D NaI Al2O3 diamond

7 Which of the following molecules exhibits the strongest intermolecular forces?

A HF
B H2O
C H2
D HCl

PRELIM 2 INNOVA 8872/01/2010 [Turn over


4
8 The overall equation for the formation of the red coloured compound G3H is given below.

3G2 + H2 2G3H
colourless colourless red

Two experiments were carried out and it was found that the rate of reaction is given by the
following relationship:
1
Rate
time

If the rate is found to be Rate = k[G2][H2], which of the following statements is not true?

A An increase in the concentration of G2 will increase the rate of formation of G3H.


B The rate of consumption of G2 is three times higher than the rate of consumption of
H2.
C A catalyst added will increase the rate of consumption of G2.
D The unit of the rate constant is mol-1 dm3 min-1.

9 If the rate of decay of a radioactive isotope decreases from 1200 counts per minute to
75 counts per minute in 48 hours, what is its half life?

A 6 hours
B 8 hours
C 12 hours
D 16 hours

10 The decomposition of dinitrogen pentoxide, N2O5, was found to be first order with respect
to N2O5.

Which of the following graphs confirm the result?

A Rate / mol dm3 s1 C Rate / mol dm3 s1

[N2O5] / mol dm3 [N2O5] / mol dm3

B D Rate / mol dm3 s1


Rate / mol dm3 s1

[N2O5] / mol dm3 [N2O5] / mol dm3

PRELIM 2 INNOVA 8872/01/2010 [Turn over


5

11 The equilibrium constants, at 823 K, for two reactions are given below.

CoO(s) + H2(g) Co(s) + H2O(g) K1 = 67

CoO(s) + CO(g) Co(s) + CO2 (g) K2 = 490

What is the numerical value of Kc for the following equilibrium at 823 K?

CO2(g) + H2(g) CO(g) + H2O(g)

A 0.137 C 423
B 7.313 D 557

12 The pH of a solution mixture containing equal amounts of aqueous ammonia and


ammonium chloride is 9.26.

Which of the following happens when 2 cm3 of 0.10 mol dm-3 nitric acid is added to the
mixture?

A The pH of the resulting solution increases.


B The number of moles of NH4+ ions in the resulting solution increases.
C The number of moles of NH3 molecules in the resulting solution remains unchanged.
D The ratio of moles of NH4+ ions to NH3 molecules in the resulting solution decreases.

13 The table shows some data on two acid-base indicators.

Colour change
Indicator Approximate pH range Acid Alkali
Bromocresol green 3.8 5.5 Yellow Blue
Phenol red 6.8 8.5 Yellow red

Which conclusion can be drawn about a solution in which bromocresol green is blue and
phenol red is yellow?

A It is weakly acidic.
B It is neutral.
C It is weakly alkaline.
D It is strongly alkaline.

PRELIM 2 INNOVA 8872/01/2010 [Turn over


6
14 Titanium(IV) oxide, which is widely used in the paint industry as a white pigment dissolves
in hot concentrated sulfuric acid according to the equation.

TiO2 + H2SO4 TiO2+ + SO42 + H2O

Which term best describes the above reaction?

A acid-base
B redox
C dehydration
D displacement

15 The radius and charge of six cations and anions are shown below.

Ion L+ X2+ W2+ M K2 I2


Radius / pm 220 170 140 180 150 200

If the ionic solids LM, XK and WI are of the same lattice type, what is the correct order of
their lattice energies, placing the one with the highest numerical value first?

A WI, XK, LM C LM, XK, WI


B WI, LM, XK D XK, WI, LM

16 The enthalpy changes for two reactions are given by the equations below.

1
2Ag(s) + O2(g) Ag2O(s) H = 31.0 kJ mol-1
2
C(s) + O2(g) CO2(g) H = 393.5 kJ mol-1

What is the enthalpy change for the following reaction where silver is obtained from the
reduction of silver(I) oxide?

2Ag2O(s) + C(s) 4Ag(s) + CO2(g)

A 331.5 kJ mol-1 C 850.4 kJ mol-1


B 767.4 kJ mol-1 D 1054.4 kJ mol-1

17 The use of Data Booklet is relevant to this question.

Cyclohexene can be reacted with hydrogen, using a nickel catalyst, to form cyclohexane.

C6H10(g) + H2(g) C6H12(g)

What is the enthalpy change for the reaction?

A 124 kJ mol-1 C 944 kJ mol-1


B 560 kJ mol-1 D 1380 kJ mol-1

PRELIM 2 INNOVA 8872/01/2010 [Turn over


7
18 The diagram below shows the energy cycle for the formation of CH4.

Hf
C(graphite) + 2H2(g) CH4(g)

+ O2 + 2O2
+ O2

CO2(g) + 2H2O(l)

The appropriate enthalpy changes of combustion are given in the table.

H / kJ mol-1
Enthalpy change of combustion of C(graphite) 393.5
Enthalpy change of combustion of hydrogen 285.8
Enthalpy change of combustion of CH4 890.4

What is the enthalpy change of formation of CH4?

A 74.7 kJ mol-1 C 74.7 kJ mol-1


B 211 kJ mol-1 D 211 kJ mol-1

19 Both sodium and magnesium burn in oxygen.

What would be observed when the two metals were burnt separately?

sodium magnesium
A yellow flame yellow flame
B yellow flame white flame
C white flame yellow flame
D white flame white flame

20 Phosphorus, sulfur and chlorine are Period 3 elements of the Periodic Table. The ionic
radii of the three anions are given in the table below.

Ion P3 S2 Cl
Radius / nm 0.212 0.185 0.181

Which of the following statements correctly explains the trend from P3 to Cl ?

A The total number of electrons and protons increases.


B The number of electrons decreases and the number of neutrons remain constant.
C The number of electrons remains constant and the number of protons increases.
D The number of electrons increases and the number of protons decreases.

PRELIM 2 INNOVA 8872/01/2010 [Turn over


8
21 Warfarin is an anticoagulant which is often used as a pesticide against rats and mice.

O O
C
H H
C
C C C C CH3

OH H O

warfarin

Which statement about warfarin molecule is incorrect?

A It decolourises purple potassium permanganate (VII) under heating.


B It produces HBr as a side product when reacted with aqueous bromine under room
temperature.
C It has no geometric isomers.
D It produces a yellow crystal on warming with aqueous, alkaline iodine.

22 Tetracyanoethylene, C6N4 has the molecular formula shown below.

NC CN
C C
NC CN

How many sigma and pi bonds are there in one molecule of tetracyanoethylene?

A 5 sigma and 1 pi C 9 sigma and 1 pi


B 5 sigma and 9 pi D 9 sigma and 9 pi

PRELIM 2 INNOVA 8872/01/2010 [Turn over


9

23 Which reaction yields a organic product that has more hydrogen atoms than the reactant?
A H H O
I2, NaOH
H C C C CH3
warm

H H

B CH3
conc H2SO4
H3C C OH

CH3

D
H H O
acidified KMnO4
H C C C OH
heat

H OH

24 What are the functional groups present in the molecule shown below?

CH3

Br

A arene, ketone and secondary alkyl bromide


B arene, ketone and tertiary alkyl bromide
C arene, aldehyde and tertiary alkyl bromide
D arene, aldehyde and secondary alkyl bromide

PRELIM 2 INNOVA 8872/01/2010 [Turn over


10
25 Compound J has all the following properties.

It is neutral.
It gives a red precipitate with Fehlings solution.
It decolourises a hot aqueous solution of acidified KMnO4 with evolution of CO2
gas.

Which of the following is compound J?

A COCH3 C

CH(OH)CH3

B D
CH2CHO

CH2OH

PRELIM 2 INNOVA 8872/01/2010 [Turn over


11
Section B

For each of the questions in this section, one or more of the three numbered statements 1 to 3
may be correct.

Decide whether each of the statements is or is not correct (you may find it helpful to put a tick
against the statements that you consider to be correct).

The responses A to D should be selected on the basis of

A B C D
1, 2 and 3 are correct 1 and 2 only are 2 and 3 only are 1 only is correct
correct correct

No other combination of statements is used as a correct response.

26 The reaction between bromine and ethane under ultraviolet light is represented by the
equation below:
light
C2H6 + Br2 C2H5Br + HBr

Experiment Initial concentration/ mol Light intensity Initial rate of


C2H6 Br2 HBr formation
1 2 1 4 2
2 2 1 1 1
3 1 1 4 1
4 1 2 4 1

From the information above, what can be concluded regarding the rate of formation of
hydrogen bromide?

1 It is directly proportional to the square root of light intensity.


2 It is not dependent on the initial concentration of bromine.
3 It is directly proportional to the initial concentration of ethane.

27 Water dissociates as follows.

H2O H+ + OH H = +57.2 kJ mol-1 at 298 K

The ionic product of water, Kw, is defined as [H+][ OH] = 1 x 10-14 mol2 dm-6 at 298 K.

What deductions can be made from these information?

1 When water is heated, the concentration of H+ increases.


2 When water is heated, the concentration of OH increases.
3 The pH of pure water at temperatures greater than 25 C is equal to 7.

PRELIM 2 INNOVA 8872/01/2010 [Turn over


12
The responses A, B, C, and D should be selected on the basis of
A B C D
1, 2 and 3 1 and 2 2 and 3 1 only
are correct only are correct only are correct is correct

No other combination of statements is used as a correct response.

28 The elements sodium to chlorine are found in Period 3 of the Periodic Table. Which of the
following statements explain the general increase in the first ionisation energy of the
elements?

1 The atomic radius decreases across the period.


2 The nuclear charge increases across the period.
3 The shielding effect decreases across the period.

29 Carbonyl compounds undergo condensation reactions with 2,4 dinitrophenylhydrazine as


shown in the equation below.

Which pair of compounds could be used to form the compound shown below?

1 C6H5COCH3 and H2NNH2


2 C6H5(CH3)C=NH and C6H5(CH3)C=NH
3 C6H5CHO and C6H5(CH3)C=NNH2

30 Which of the following reagents and conditions will form CH3CH2NH2?

reagents conditions
1 CH3CH2Br excess NH3(g) in sealed tube, heat
2 CH3Br KCN in ethanol, heat followed by LiAlH4 in dry ether
3 CH3CH2Br KCN in ethanol, heat followed by H2SO4(aq), heat

PRELIM 2 INNOVA 8872/01/2010


INNOVA JUNIOR COLLEGE
JC 2 PRELIMINARY EXAMINATION 2
in preparation for General Certificate of Education Advanced Level
Higher 1

CANDIDATE
NAME

CLASS INDEX NUMBER

CHEMISTRY 8872/02
Paper 2 Structured and Free Response Questions 20 September 2010
2 hour
Section A: Structured Questions
Candidates answer Section A on the Question Paper

Section B: Free Response Questions


Additional Materials: Writing Papers
Data Booklet

READ THESE INSTRUCTIONS FIRST

Write your index number, name and civics group on all the work you hand in.
Write in dark blue or black pen.
You may use pencil for any diagrams, graphs or rough working.
Do not use staples, paper clips, highlighters, glue or correction fluid. For Examiners Use
Section A: Structured Questions (40m) Section A
Answer all questions in the space provided.
1
13
Section B: Free Response Questions (40m) 2
Answer two questions on separate writing papers. 8
3
You are advised to show all working in calculations. 9
You are reminded of the need for good English and
4
clear presentation in your answers. 10
You are reminded of the need for good handwriting.
Your final answers should be in 3 significant figures. Total
40
You may use a calculator.

The number of marks is given in brackets [ ]


at the end of each question or part question.

At the end of the examination, fasten all your work


securely together.

This document consists of 16 printed pages.

Innova Junior College [Turn over


2
For
Examiners
Section A Use
Answer ALL questions in this section on the space provided.

1 Iron(II) sulfate and ammonium sulfate combine to form the double salt iron(II) ammonium
sulfate, FeSO4.(NH4)2SO4.xH2O.

11.0 g of the double salt was dissolved in dilute sulfuric acid and made up to 250 cm3
mark in a standard flask. 25.0 cm3 of the solution required 28.00 cm3 of 0.02 mol dm-3
KMnO4 for titration.

(a) (i) Write a balanced ionic equation for the reaction between Fe2+ and acidified
MnO4.

....

(ii) Calculate the number of moles of MnO4 used in the titration.

(iii) Calculate the number of moles of Fe2+ in 250 cm3 solution.

PRELIM 2 INNOVA 8872/02/10 [Turn over


3 For
Examiners
Use
(iv) Calculate the Mr of the double salt.

(v) Calculate the value of x.

[6]

(b) Draw the Lewis structure of the NH4+ and state its shape.

Shape of NH4+ : ..
[2]

PRELIM 2 INNOVA 8872/02/10 [Turn over


4 For
Examiners
Use

(c) The reaction between zinc and copper(II) sulfate solution is exothermic and is
represented by the following equation:

Zn(s) + CuSO4(aq) ZnSO4(aq) + Cu(s)

(i) Apart from a temperature rise, state two observations which could be made
from the above reaction.

....

....

....

(ii) When excess zinc powder was added to a polystyrene cup containing 75.0 cm3
of aqueous copper(II) sulfate at 20.5 C, the temperature rose to 33.7 C. The
copper metal formed was collected, washed, dried and found to weigh 0.42 g.
Calculate the enthalpy change for this reaction in kJ per mole of copper
formed.

[5]

[Total: 13 ]

PRELIM 2 INNOVA 8872/02/10 [Turn over


5 For
Examiners
Use
2 Acetic acid, CH3COOH, also known as ethanoic acid, is an organic acid that gives vinegar
its sour taste and pungent smell. It is a weak acid as it is only partially dissociated in an
aqueous solution.

(a) Write the Ka expression for ethanoic acid.

[1]

(b) (i) When a weak acid dissociates in aqueous solution, the concentration of H+ is
given by the following expression.

[H+] = [weak acid ]K a

Given that the concentration of ethanoic acid is 0.720 mol dm-3 and its Ka is
1.8 x 10-5 mol dm-3. Calculate the concentration of H+ and pH of ethanoic acid.

(ii) Suggest and explain the choice of an indicator for the titration of ethanoic acid
and aqueous NaOH.

....

....

....
[4]

PRELIM 2 INNOVA 8872/02/10 [Turn over


6 For
Examiners
Use
(c) It is known that the relative molecular mass of ethanoic acid is 60. However, when
ethanoic acid is dissolved in benzene, it is found that the relative molecular mass of
the acid is 120. Explain this phenomenon using a clearly labelled diagram.

......

..............

..............

Labelled diagram:

[3]

[Total: 8]

PRELIM 2 INNOVA 8872/02/10 [Turn over


7 For
Examiners
Use
3 Succinic acid, C4H6O4 is a colourless crystalline solid with a melting point of 185 187 oC.
It occurs naturally in plant and animal tissues.

(a) The scheme below shows how succinic acid, can be prepared from ethene.

OH

Ethene P CN O
NC O

Q OH
succinic acid

(i) Suggest the structure of P.

(ii) Suggest how P can be prepared from ethene and name the type of reaction
involved.

Reagent(s) : .

Conditions : .

Type of reaction : .

(iii) Suggest how succinic acid can be synthesised from Q. State the type of
reaction involved.

Reagent(s) : .

Conditions : .

Type of reaction : .

(iv) Draw the structural formula of the product formed when ethene reacts with
steam and H3PO4 catalyst. State the functional group of the product formed.

Functional group : .
[7]

PRELIM 2 INNOVA 8872/02/10 [Turn over


8 For
Examiners
Use
(b) Describe a chemical test to distinguish between succinic acid and 2-hydoxy-3-
methylbutanal.

OH
OH
O C CH3
O C C
O H H
and
H CH3
OH
succinic acid 2-hydroxy-3-methylbutanal

Reagent(s) : .

Conditions : .

Observations for succinic acid:

Observations for 2-hydroxy-3-methylbutanal:

[2]

[Total: 9]

PRELIM 2 INNOVA 8872/02/10 [Turn over


9 For
Examiners
Use

4 (a) Phenolphthalein is one of the most common indicators used to determine the end
point in a titration. When excess base is present at the end of an acid-base titration,
the pink colour fades after the solution is left to stand in air for a while.

The structure of phenolphthalein is abbreviated as Y.

pH Form of phenolphthalein Colour


Below 8 H2Y Colourless
2
8 to 10 H2Y + OH Y + H2O Pink
Above 10 Y2 + OH YOH3 Colourless

The fading of phenolphthalein in aqueous NaOH (above pH 10) can be studied to


determine its rate equation. The concentration of phenolphthalein in solution can be
followed by a spectrophotometer, which measures the concentration of Y2 in terms
of Absorbance. [Y2] is taken to be directly proportional to the absorbance of
the solution.

Fig. 4.1 shows the absorbance versus time graphs obtained for two different
concentrations of NaOH, 0.200 mol dm-3 and 0.400 mol dm-3. In each case, only one
drop of phenolphthalein was used.

450

400

350

300
Absorbance/ A

250

200

150
[NaOH] = 0.400 mol dm-3
100
[NaOH] = 0.200 mol dm-3
50

0
0 10 20 30 40 50 60 70 80 90 100 110 120 130 140 150
Time/ sec

Fig. 4.1

PRELIM 2 INNOVA 8872/02/10 [Turn over


10 For
Examiners
Use
Use the graphs in Fig. 4.1 to find the order of reaction with respect to

(i) NaOH;

(ii) Y2.

(iii) Hence state the rate equation based on a(i) and a(ii).

....
[5]

PRELIM 2 INNOVA 8872/02/10 [Turn over


11 For
Examiners
Use
(b) Finland currently generates 55 % of its heat and energy from burning of fossil fuels
which produces CO2. This gives rise to increased CO2 emission and poses
environmental problems. However, CO2 can be removed by reacting with MgO to
form MgCO3.

(i) MgO is formed from the thermal decomposition of MgSiO3, a major component
in mineral rocks. In this process, SiO2 is also produced. Write a balanced
equation for the thermal decomposition of MgSiO3.

...

(ii) During the thermal decomposition of mineral rocks, impurities like SO2 are also
produced. Arrange MgO, SO2 and SiO2 in order of increasing acidity.

...

(iii) Suggest a method how MgO can be separated from a mixture of MgO, SO2
and SiO2. Write balanced equations for the reactions involved.

...

...

...

...

...

...
[5]

[Total: 10]

PRELIM 2 INNOVA 8872/02/10 [Turn over


12

Section B
Answer two of the three questions in this section on separate writing papers.

Begin each answer on a fresh sheet of paper.

5 (a) Ethyl acetate or ethyl ethanoate is the organic compound with the formula
CH3COOCH2CH3. Ethyl acetate is the ester of ethanol and acetic acid; it is
manufactured on a large scale for use as a solvent. It is formed according to the
equation below.

CH3COOH(l) + CH3CH2OH (l) CH3COOCH2CH3(l) + H2O(l)

In an experiment, 1 mol of ethanoic acid and 2 mol of ethanol were shaken for a
long time to reach equilibrium. The whole mixture was titrated quickly with 80 cm3 of
2.00 mol dm3 of sodium hydroxide.

(i) Calculate the number of moles of ethanoic acid present at equilibrium.

(ii) Calculate the number of moles of ethanoic acid that had reacted with ethanol.

(iii) Calculate the number of moles of ethanol, ester and water present at
equilibrium.

(iv) Write an expression of the equilibrium constant, Kc for this system.

(v) Calculate the value of the equilibrium constant, Kc.

(vi) State the effect on Kc if 0.1 mol of ethanol liquid was introduced into the
container after equilibrium has been established.
[7]

(b) Ethanoic acid, CH3COOH is a corrosive acid. Its vapour irritates the eyes, produces
a burning sensation in the nose, and can lead to a sore throat and lung congestion.

(i) Suggest the reagents and conditions for the conversion of ethanoic acid into
an alcohol.

(ii) Name the type of reaction that occurs and write a balanced equation for the
conversion of ethanoic acid into an alcohol.

(iii) Describe what you would observe when ethanoic acid was reacted with
sodium metal.
[4]

(c) Period 3 chlorides such as PCl5, POCl3, PSCl3 and SiCl4 have many uses.

(i) Describe and explain what you would observe when water is gradually added
to PCl3, until in excess. Suggest the pH of the solution thus formed.

(ii) Suggest a balanced equation with state symbols for the reaction of liquid
silicon tetrachloride with heavy water, D2O [deuterium, D= 2 H ].
[4]

PRELIM 2 INNOVA 8872/02/10 [Turn over


13

(d) The graphs below show the variation in first ionisation energy of some elements.
Fig. 5.1 refers to successive elements in the same period, and Fig. 5.2 refers to
successive elements in the same group as element J.

Fig. 5.1 Fig. 5.2

First M First
Ionisation J
Ionisation
Energy Energy
J N
L

K Atomic No. Atomic No.

x x+1 x+2 x+3 x+4 x-8 x x+8

(i) Given that element J is a Group II element. Copy the diagram below into your
answer paper. Label clearly how the beams of J+ and K+ ions are deflected
when subjected to an electric field.

You should relate clearly the magnitude and the direction of deflection of
each beam to the other.

Source

(ii) State and explain which period element J is in and hence identify J.
[5]

[Total: 20]

PRELIM 2 INNOVA 8872/02/10 [Turn over


14

6 Compound A, a derivative of Decalin is a colourless liquid. It can undergo the following


3-step reaction scheme.

(a) (i) What reagents and conditions are needed for reactions I, II and III?

(ii) Name the type of reaction for reaction III.

(iii) Reaction II is an oxidation reaction. Write a balanced equation for reaction II


(the symbol [O] is acceptable in this equation).
[5]

(b) Draw the organic product formed when B is reacted separately with each of the
reagents and conditions stated below.

(i) Al2O3, 350C;

(ii) PCl5.
[2]

(c) (i) Compound A, C7H13Br has a structural isomer with a straight chain and
displays geometric isomerism. Suggest a possible structure of the isomer.

(ii) Suggest a chemical test to distinguish A and the isomer identified in (c)(i).
State the observations for the test.
[3]

(d) Compound D with molecular formula C9H8O is used as a drug carrier. D is insoluble
in sodium hydroxide but reacts with 1 mole of bromine gas in the dark to form E,
C9H8OBr2.

When E is heated with aqueous potassium hydroxide, F, C9H10O3, a neutral


compound is formed.

Treatment of D with Tollens reagent will yield silver mirror but when treated with
Fehlings solution, blue colour solution remains. When D is heated with acidified
potassium manganate(VII), G, C8H6O4, is formed. G will produce effervescence
when Na2CO3 is added to it.

Use all of the above information to identify the organic compounds D, E, F, and G.
Include the following in your answers.

(i) Structural formula for each of the organic compounds.

(ii) Clearly show all of the deductions that you make from the information that you
have been given.
[10]

[Total: 20]

PRELIM 2 INNOVA 8872/02/10 [Turn over


15

7 Chromium is a very shiny, silvery metal whose Greek name, Chroma, refers to its many
colourful compounds.

(a) In air, chromium readily forms a thin and transparent coating of Cr2O3. Like Al2O3,
Cr2O3 is an amphoteric oxide and can react with both acids and bases.

(i) Suggest a reason why chromium is used extensively as a coating for other
metals like iron.

(ii) Write 2 balanced equations with state symbols that shows the amphoteric nature
of Cr2O3.
[3]

(b) Chromium can also exist in the form of chromate, CrO42, which can be prepared from
chromium(III) chloride. This conversion takes place in the presence of hydrogen
peroxide, H2O2.

(i) Write the half equation for the conversion of H2O2 to OH in alkaline medium.

(ii) Given that the half equation for the conversion of Cr3+ to CrO42- in alkaline
medium is as shown below.

Cr3+(aq) + 8OH(aq) CrO42(aq) + 4H2O(l) + 3e

Using your answer to b(i), write the overall balanced equation of the conversion
of Cr3+ to CrO42 in the presence of H2O2 in alkaline medium.

(iii) Hence calculate the minimum volume of 0.200 mol dm3 H2O2 required to
convert a sample of 0.500 g of CrCl3 completely to CrO42.
[5]

(c) Draw the dot-and-cross diagram of H2O2 and state its shape.
[2]

PRELIM 2 INNOVA 8872/02/10 [Turn over


16

(d) Besides chromium(III) chloride, chlorides of Period 3 elements are also well-studied.
State and explain the electrical conductivity of the chlorides of magnesium and
phosphorous respectively in both molten and aqueous states.
[3]

(e) The following diagram shows how the NH3 molecules are arranged around the central
Cr3+ ion in a [Cr(NH3)6]3+ complex.

The arrow denotes a dative bond from the N atom to Cr3+, as shown below:

Using the Valence Shell Electron Pair Repulsion (VSEPR) Theory, explain the
phenomenon that the H-N-H bond angle in ammonia when it is bound to Cr3+ is about
110 o, while that in an isolated ammonia molecule is about 107 o.
[2]

(f) In another compound containing boron chloride and phosphine, an equilbrium exists
between the compound, PH3BCl3, and its adducts PH3 and BCl3 as shown.

PH3BCl3(s) PH3(g) + BCl3(g) H > 0

(i) Write an expression for the equilibrium constant, Kc, for the decomposition of
solid PH3BCl3.

(ii) A solid sample of PH3BCl3 is introduced in a closed vessel and decomposes


until equilibrium is established. Explain how the equilibrium position will be
affected when the

I temperature is increased to 373 K.

II solid PH3BCl3 is added to the system at equilibrium.


[5]

[Total: 20]

PRELIM 2 INNOVA 8872/02/10


INNOVA JUNIOR COLLEGE
JC 2 PRELIMINARY EXAMINATION (2010)
Higher 1 CHEMISTRY

Paper 1 Answers

1 A 11 A 21 B
2 C 12 B 22 D
3 D 13 A 23 C
4 C 14 A 24 D
5 D 15 D 25 B

6 C 16 A 26 A
7 B 17 A 27 B
8 D 18 A 28 B
9 C 19 B 29 D
10 A 20 C 30 B

Paper 2 Section A Answers

1 (a) (i) 5Fe2+ + MnO4- + 8H+ 5Fe3+ + Mn2+ + 4H2O [1], without s.s

(ii)
No. of moles of MnO4 used in the titration =
28.00
1000 0.02 = 5.60 10 4

(iii) No. of moles of Fe2+ in 25.0 cm3 solution


= 5.60 x 10-4 x 5
= 2.80 x 10-3

No. of moles of Fe2+ in 250 cm3 solution


= 2.80 x 10-3 x 10
= 2.80 x 10-2

(iv) 2.80 x 10-2 Mr = 11.0


Mr = 392.8 OR
393

(v) [55.8 + 32.0 + 4(16.0) + 2(14 + 4) + 32 + 4(16) + x(18.0) = 393


x=6

(b) Shape of NH4+:

Shape is tetrahedral.
PRELIM 2 INNOVA 8872/02/2010 [Turn over
(c) (i) Red / brown / pink solid (Cu) forms
Blue solution fades/ become colourless
Zinc dissolves

(ii) Assume that the specific heat capacity and density of the solution is similar to
that of water.

Energy released = mcT


= (75.0)(4.18)(33.7 20.5)
= 4138.2 J = 4.14 kJ

No. of moles of Cu formed = 0.42 / 63.5


= 6.61 x 10-3

H per mole of copper = - 4.14 / 6.61 x 10-3


= -626 kJ

2 (a) [ H + ][CH 3COO ]


Ka =
[CH 3COOH ]

(b) (i)
[H+] = (1.8 105 )(0.720)
= 3.60 x 10-3
pH = - log (3.60 x 10-3)
= 2.44

(ii) Phenolphthalein. The equivalence point of a weak acid-strong base titration is


more than 7, which falls in the working range of the indicator (8 - 10).

(c) In benzene, two molecules of ethanoic acid combine together to form a dimer.

+
O H O

H3C C C CH3
+
O H O

Hydrogen bond

3 a (i)

(ii) Reagent : Br2 in CCl4 OR Br2(l) ,


Condition : room temp (in the dark)
Type of reaction : Addition

(iii) Suggest how succinic acid can be synthesised from compound Q. State the
type of reaction involved.
(iii) Reagent : Dilute HCl (or H2SO4)
PRELIM 2 INNOVA 8872/02/2010 [Turn over
Condition : Heat with reflux

Type of reaction : Acidic hydrolysis OR hydrolysis

(iv) H H

H C C H

H OH

Functional group : primary alcohol

(b) Reagent : Na2CO3

Observation :
Succinic acid : Effervescence seen, gas forms white ppt with Ca(OH)2 (aq)
2-hydroxy-3-methylbutanal : No effervescence seen
OR

Reagent : 2,4-dinitrophenylhydrazine

Observation :
Succinic acid : No orange ppt
2-hydroxy-3-methylbutanal : Orange ppt seen

OR

Reagent : Tollens reagent, warm

Observation :
Succinic acid : No silver mirror
2-hydroxy-3-methylbutanal : Silver mirror seen

OR

Reagent : Fehlings solution, warm

Observation :
Succinic acid : No brick red ppt
2-hydroxy-3-methylbutanal : Brick red ppt and blue Cu2+ solution decolourised

OR

Reagent : KMnO4, dilute H2SO4, heat with reflux

Observation :
Succinic acid : Purple colour remains
2-hydroxy-3-methylbutanal: Purple colour of KMnO4 decolourised

4 (a) (i) Initial rate when [NaOH] = 0.200 mol dm-3


400 300
= = 10 A / s
10
Initial rate when [NaOH] = 0.400 mol dm-3

PRELIM 2 INNOVA 8872/02/2010 [Turn over


400 300
= = 20 A / s
5
For initial rates values that are outside the range, marks will not be given.
However order of reaction can still be awarded.

When [NaOH] doubles, keeping [Y2-] constant, the rate also doubles. This
implies that the order with respect to NaOH is 1.

(ii) From Graph when [NaOH] = 0.200 mol dm-3


(show 2 sets of dotted lines to find half life)
The half-life of reaction is approximately constant at 25 sec, hence order of
reaction with respect to Y2- is 1

OR

From Graph when [NaOH] = 0.400 mol dm-3


(show 2 sets of dotted lines to find half life)
The half-life of reaction is approximately constant at 12.5 sec, hence order of
reaction with respect to Y2- is 1.

(iii) Rate = k [Y2-] [NaOH]

(b) (i) MgSiO3 MgO + SiO2

(ii) MgO, SiO2 and SO2

(iii) Use concentrated NaOH, both the other acidic oxides will be dissolved, leaving
behind MgO/ filter the solution to obtain MgO as residue.

SiO2 + 2OH- SiO32- + H2O


SO2 + 2OH- SO32- + H2O

PRELIM 2 INNOVA 8872/02/2010 [Turn over


Paper 2 Section B Answers

5 (a) (i) No. of moles of ethanoic acid present at equilibrium


= No. of moles of NaOH required
80
= x 2 = 0.16
1000

(ii) No. of moles of ethanoic acid reacted


= 1 0.16 = 0.84

(iii) Number of moles of ethanol = 2-0.84 = 1.16


Number of moles of ester formed = 0.84
Number of moles of water formed = 0.84

(iv) [CH 3COOCH 2 CH 3 ][ H 2 O]


Kc =
[CH 3COOH ][CH 3CH 2 OH ]

(v) 0.84 0.84


[
][ ]
[CH 3COOCH 2 CH 3 ][ H 2 O] V V
Kc = =
[CH 3COOH ][CH 3CH 2 OH ] 0.16 1.16
[ ][ ]
V V
= 3.80

(vi) No effect on Kc as it is only affected by temperature.

(b) (i) Reagents: Lithium aluminium hydride, LiAlH4


Conditions: in dry ether

(ii) Type of reaction: Reduction

H H
O
H
H C C OH + H2O
H C C OH + 4 [H]

H H
H

(iii) Effervescence observed.

(c) (i) Phosphorus (III) chloride undergoes hydrolysis with water, to produce steamy
HCl fumes and phosphonic acid, H3PO3. The solution has a pH = 2

(ii) SiCl4(l) + 2D2O(l)


SiO2(s) + 4DCl(aq)
Or SiCl4(l) + 4D2O(l)
Si(OD)4(s) + 4DCl (aq)

PRELIM 2 INNOVA 8872/02/2010 [Turn over


(d) (i) J: Group II element
K: Group III element

charge
Deflection
mass
Mass of K+ > mass of J+
Angle of deflection for of J+ > Angle of deflection for of K+
NOTE: mark given for working or shown in diagram

Source

K+

-
J+

(ii) State and explain which period element J is in and hence identify J.
(ii) Based on Figure 5.2, J is in period 3.

Only in period 2-4, atomic number have a difference of 8 down the group.

From period 5, the increase in atomic number down the group is 18.

J is Magnesium

6 (a) (i) Reaction I: NaOH/KOH (aq) reflux

Reaction II: K2Cr2O7 dilute H2SO4, distill

Reaction II: HCN trace amount of NaOH/NaCN, 10-20C

(ii) Addition

(iii)

PRELIM 2 INNOVA 8872/02/2010 [Turn over


(b) (i)

(ii) CH2Cl

(c) (i)

any variation of C7H13Br and must have cis-trans

(ii) Reagent: Br2 (aq)


Condition: Room temperature
Observation:
Structural isomer: Orange Br2(aq) will decolourise
Compound A: Orange Br2(aq) will not decolourise

OR

Reagent: Liquid Br2


Condition: Room temperature
Observation:
Structural isomer: Reddish brown Br2 will decolourise
Compound A: Reddish brown Br2 will not decolourise

OR

Reagent: KMnO4
Condition: dilute H2SO4, reflux
Observation:
Structural isomer: Purple KMnO4 will decolourise
Compound A: Purple KMnO4 will not decolourise

(d) There is comparable number of C and H in compound D, hence, Compound D


contains a benzene ring.
Compound D is most likely an alcohol since it is insoluble in sodium hydroxide
It reacts with bromine gas in the dark, hence an alkene is present. It reacts in
addition reaction.
Compound E reacts with aqueous potassium hydroxide in substitution reaction.
It contains alcohol group as the compound is neutral.
It reacts with Tollens reagent but not Fehlings solution, hence it contains aromatic
aldehyde.
When reacted with potassium manganate (VII), oxidation takes place.
PRELIM 2 INNOVA 8872/02/2010 [Turn over
Compound G reacts with Na2CO3 via acid-carbonate reaction as it contains
carboxylic acid. The effervescence is due to CO2.
H

O C H

C C

H H Compound D
H Br

O C H

C C

H Br H Compound E
H OH

O C H

C C

H HO H Compound F
O

C COOH

HO Compound G

Accept 1, 2 position and 1,3 position.

7 (a) (i) Chromium forms an inert oxide layer , will protect the metal from corrosion.

(ii) Cr2O3 (s) + 6H+ (aq) 2Cr3+ (aq) + 3H2O (l)


Cr2O3 (s) + 3H2O (l) + 2OH- (aq) 2 Cr(OH)4- (aq)

(b) (i) H2O2 (l) + 2e- 2OH- (aq)

(ii) 2Cr3+ (aq) + 10 OH- (aq) + 3H2O2 (l) 2CrO42- (aq) + 8H2O (l)

(iii) 0.500
No. of moles of CrCl3 = = 0.00315 mol
52.0 + 3(35.5)
No. of moles of H2O2 required = 3/2 x 3.15 x 10-3 = 4.73 x 10-3 mol
4.73 10 3
Volume = =0.0237 dm3
0.200

(c)

Shape: Bent

(d) Chlorides of magnesium can conduct electricity in molten and aqueous states, while
chlorides of P cannot.
MgCl2 is has giant ionic structure. In the molten and aqueous states, there are mobile
charge carriers (ions), hence able to conduct electricity.
PCl3 or PCl5 has a simple molecular structure. In both molten and aqueous states,
there no mobile charge carriers, hence unable to conduct electricity.

PRELIM 2 INNOVA 8872/02/2010 [Turn over


(e) In the complex [Cr(NH3)6]3+, the N atom in NH3 is dative-bond to Co3+ ion.
Hence, there are 4 bond pairs and zero lone pairs on N atom.

To minimize repulsion, electron pairs are directed to the 4 corners of a tetrahedron,


hence bond angle is about 109.5 o (about 110 o). In an isolated ammonia molecule,
there are 3 bond pairs and 1 lone pair. Since lone pair-bond pair repulsion is larger
than bond pair-bond pair repulsion, the bond angle is compressed to 107 o.

(f) (i) Kc = [PH3][BCl3]

(ii) (I) When temperature is increased, by Le Chateliers Principle,


equilibrium position shifts to the right to favour the endothermic
reaction to absorb additional heat.

(II) Equilibrium position is unaffected as the addition of solid does not


affect the concentration of PH3 and BCl3 OR the concentration of
solid is constant OR total pressure remains unchanged .

PRELIM 2 INNOVA 8872/02/2010 [Turn over


H1 Prelim 2
Structured Questions

1 Iron(II) sulfate and ammonium sulfate combine to form the double salt iron(II) ammonium
sulfate, FeSO4.(NH4)2SO4.xH2O.

11.0 g of the double salt was dissolved in dilute sulfuric acid and made up to 250 cm3
mark in a standard flask. 25.0 cm3 of the solution required 28.00 cm3 of 0.02 mol dm-3
KMnO4 for titration.

(a) (i) Write a balanced ionic equation for the reaction between Fe2+ and acidified
MnO4.

5Fe2+ + MnO4- + 8H+ 5Fe3+ + Mn2+ + 4H2O [1], without s.s

(ii) Calculate the number of moles of MnO4 used in the titration.

[1]
No. of moles of MnO4 used in the titration =
28.00
1000 0.02 = 5.60 10 4
1
[ ]for working
2
1
[ ]for answer
2

(iii) Calculate the number of moles of Fe2+ in 250 cm3 solution.


[2]
2+ 3
No. of moles of Fe in 25.0 cm solution
1
= 5.60 x 10-4 x 5 [[ ]for ratio 5, ecf from (a)(i)]
2
1
= 2.80 x 10-3 [ ]
2

No. of moles of Fe2+ in 250 cm3 solution


1
= 2.80 x 10-3 x 10 [[ ] for dilution factor 10]
2
1
= 2.80 x 10-2 [[ ], ecf]
2

(iv) Calculate the Mr of the double salt.

1
2.80 x 10-2 Mr = 11.0 [[ ], ecf]
2
Mr = 392.8 OR
1
393[[ ], ecf part (iii)]
2

(v) Calculate the value of x.

1
[55.8 + 32.0 + 4(16.0) + 2(14 + 4) + 32 + 4(16) + x(18.0) = 393 [[ ], ecf]
2
1
x = 6 [[ ], ecf part (iii)]
2

[6]

(b) Draw the Lewis structure of the NH4+ and state its shape.
[2]
Shape of NH4+:

[1] - include square bracket and ve charge,


-do not penalise if dative bond is not shown / arrow is absent/pointing
in wrong direction
- must be tetrahedral shape
Shape is tetrahedral. [1]

(c) The reaction between zinc and copper(II) sulfate solution is exothermic and is
represented by the following equation:
Zn(s) + CuSO4(aq) ZnSO4(aq) + Cu(s)

(i) Apart from a temperature rise, state two observations which could be made
from the above reaction.

1 1
Red / brown / pink solid (Cu) forms [[ ] for colour; [ ] for Cu (solid)]
2 2
Blue solution fades/ become colourless [1]
Zinc dissolves [1]

Any two of the three above

(ii) When excess zinc powder was added to a polystyrene cup containing 75.0 cm3
of aqueous copper(II) sulfate at 20.5 C, the temperature rose to 33.7 C. The
copper metal formed was collected, washed, dried and found to weigh 0.42 g.
Calculate the enthalpy change for this reaction in kJ per mole of copper
formed.
Assume that the specific heat capacity and density of the solution is similar to
that of water.

Energy released = mcT


= (75.0)(4.18)(33.7 20.5)
1 1
= 4138.2 J = 4.14 kJ [[ ] for working; [ ] ans]
2 2

No. of moles of Cu formed = 0.42 / 63.5


1 1
= 6.61 x 10-3 [[ ] for working; [ ] ans]
2 2

H per mole of copper = - 4.14 / 6.61 x 10-3


1 1
= -626 kJ [[ ] for working; [ ] ans, ecf from mcT]
2 2

[5]

[Total: 13]

2 Acetic acid, CH3COOH, also known as ethanoic acid, is an organic acid that gives vinegar
its sour taste and pungent smell. It is a weak acid, in that it is only partially dissociated in
an aqueous solution.

(a) Write down the Ka expression for ethanoic acid.

[1]
+
[ H ][CH 3COO ]
Ka = [1]
[CH 3COOH ]

(b) (i) When a weak acid dissociates in aqueous solution, the concentration of H+ is
given by the following expression.

[H+] = [weak acid ]K a

Given that the concentration of ethanoic acid is 0.720 mol dm-3 and its Ka is
1.8 x 10-5 mol dm-3. Calculate the concentration of H+ and pH of ethanoic acid.

1
[H+] = (1.8 105 )(0.720) [ ]correct substitution
2
1
= 3.60 x 10-3 [
]
2
1
pH = - log (3.60 x 10-3) [ ]
2
1
= 2.44 [ ]
2

(ii) Suggest and explain the choice of an indicator for the titration of ethanoic acid
and aqueous NaOH.

Phenolphthalein [1]. The equivalence point of a weak acid-strong base titration


1
is more than 7 [ ], which falls in the working range of the indicator (8 - 10)
2
1
[ ].
2
[5]
(c) It is known that the relative molecular mass of ethanoic acid is 60. However, when
ethanoic acid is dissolved in benzene, it is found that the relative molecular mass of
the acid is 120. Explain this phenomenon using a clearly labeled diagram.
[3]
In benzene, two molecules of ethanoic acid combine together to form a dimer [award
[1] if std write out the word or show the dimer in the diagram] through hydrogen
bonding [award [1] if std write out the word or show the hydrogen bonding in the
diagram].

both partial charges 0.5m

lone pair 0.5m

O H O

H3C C C CH3

O H O

Hydrogen bond
If didn't label H-bond, deduct 0.5m

[Total: 8]

3 Succinic acid, C4H6O4 is a colourless crystalline solid with a melting point of 185 187 oC.
It occurs naturally in plant and animal tissues.

(a) The scheme below shows how succinic acid, can be prepared from ethene.

(i) Draw the structure of compound P.

[1]
(ii) Suggest how compound P can be prepared from ethene and name the type of
reaction involved.
Reagent : Br2 in CCl4 OR Br2(l) [1],
1
Condition : room temp (in the dark) [deduct [ ]for heat]
2
1
(If std write Br2(g) or Br2(s), deduct [ ])
2

Type of reaction : Addition [1]

(iii) Suggest how succinic acid can be synthesised from compound Q. State the
type of reaction involved.
1
Reagent : Dilute HCl (or H2SO4) [ ]
2
1
Condition : Heat with reflux [ ]
2

Type of reaction : Acidic hydrolysis OR hydrolysis [1]


1
(if std write alkaline hydrolysis, deduct [ ])
2

(iv) Draw the structural formula of the product formed when ethene reacts with
steam and H3PO4 catalyst. State the functional group of the product formed.

H H

H C C H

H OH [1]

1 1
Functional group : primary [ ] alcohol [ ]
2 2

[7]
(b) Describe a chemical test to distinguish between succinic acid and 2-hydoxy-3-
methylbutanal.

Reagent : Na2CO3 [1], room temp

Observation :
1
Succinic acid : Effervescence seen, gas forms white ppt with Ca(OH)2 (aq) [ ]
2
1
2-hydroxy-3-methylbutanal : No effervescence seen [ ]
2

OR

1
Reagent : 2,4-dinitrophenylhydrazine [0.5], room temp [ ]
2

Observation :
1
Succinic acid : No orange ppt [ ]
2
1
2-hydroxy-3-methylbutanal : Orange ppt seen [ ]
2

OR

1 1
Reagent : Tollens reagent [ ], warm [ ]
2 2

Observation :
1
Succinic acid : No silver mirror [ ]
2
1
2-hydroxy-3-methylbutanal : Silver mirror seen [ ]
2

OR

1 1
Reagent : Fehlings solution [ ], warm [ ]
2 2

Observation :
1
Succinic acid : No brick red ppt [ ]
2
1
2-hydroxy-3-methylbutanal : Brick red ppt [ ] and blue Cu2+ solution decolourised
2

OR

1 1
Reagent : KMnO4 [ ], dilute H2SO4, heat with reflux [ ]
2 2

Observation :
1
Succinic acid : Purple colour remains [ ]
2
1
2-hydroxy-3-methylbutanal: Purple colour of KMnO4 decolourised [ ]
2
[2]

[Total: 9]

4 (a) Phenolphthalein is one of the most common indicators used to determine the end
point in a titration. If excess base is present at the end of an acid-base titration, the
pink colour fades if the solution is left to stand in air for a while.

The structure of phenolphthalein is abbreviated as Y.

pH Form of phenolphthalein Colour


Below 8 H2Y Colourless
8 to 10 H2Y + OH- Y2- + H2O Pink
Above 10 Y2- + OH- YOH3- Colourless

The fading of phenolphthalein in aqueous NaOH (above pH 10) can be studied to


determine its rate equation. The concentration of phenolphthalein in solution can be
followed by a spectrophotometer, which measures the concentration of Y2- in terms
of Absorbance. [Y2-] is taken to be directly proportional to the absorbance of the
solution.

Figure 4.1 shows the absorbance versus time graphs obtained for two different
concentrations of NaOH, 0.200 mol dm-3 and 0.400 mol dm-3. In each case, only one
drop of phenolphthalein was used.

450

400

350

300
Absorbance/ A

250

200

150
[NaOH] = 0.400 mol dm-3
100
[NaOH] = 0.200 mol dm-3
50

0
0 10 20 30 40 50 60 70 80 90 100 110 120 130 140 150
Time/ sec

Figure 4.1
Use the graphs to find the order of reaction with respect to
(i) NaOH
Initial rate when [NaOH] = 0.200 mol dm-3
400 300 1
= = 10 A / s [ ] (units not marked for)
10 2
2.5 value is accepted
Initial rate when [NaOH] = 0.400 mol dm-3
400 300 1
= = 20 A / s [ ] (units not marked for)
5 2
5 value is accepted

For initial rates values that are outside the range, marks will not be given.
However order of reaction can still be awarded.

When [NaOH] doubles, keeping [Y2-] constant, the rate also doubles. This
implies that the order with respect to NaOH is 1. [1]
1
(if guess order correctly, [ ])
2

(ii) Y2-

From Graph when [NaOH] = 0.200 mol dm-3


1
(show 2 sets of dotted lines to find half life - [ ])
2
1
The half-life of reaction is approximately constant at 25 sec [ ], hence
2
order of reaction with respect to Y2- is 1 [1]

OR

From Graph when [NaOH] = 0.400 mol dm-3


1
(show 2 sets of dotted lines to find half life - [ ])
2
1
The half-life of reaction is approximately constant at 12.5 sec [ ], hence
2
order of reaction with respect to Y2- is 1 [1]

(iii) Hence, state the rate equation that describes the fading of phenolphthalein in
aqueous NaOH above pH 10.

Rate = k [Y2-] [NaOH] [1]


[5]

(b) Finland currently generates 55 % of its heat and energy from burning of fossils fuel
which produces CO2. This gives rise to increased CO2 emission and poses
environmental problems. However CO2 can be removed by reacting with MgO to form
MgCO3.

(i) MgO is formed from the thermal decomposition of MgSiO3, a major component in
mineral rocks. In this process, SiO2 is also produced. Write a balanced equation
for thermal decomposition of MgSiO3.

MgSiO3 MgO + SiO2 [1], dont mark for state symbols

(ii) During the thermal decomposition of mineral rocks, impurities like SO2 are also
produced. Arrange MgO, SO2 and SiO2 in the order of increasing acidity.

MgO, SiO2 and SO2 [1]

(iii) Suggest a method how MgO can be separated from a mixture of MgO, SO2, and
SiO2. Write balanced equations for the reactions involved.

1
Use concentrated NaOH [ ], both the other acidic oxides will be dissolved,
2
1
leaving behind MgO/ filter the solution to obtain MgO as residue. [ ]
2

SiO2 + 2OH- SiO32- + H2O [1] , dont mark for state symbols
SO2 + 2OH- SO32- + H2O [1], dont mark for state symbols

[5]

[Total: 10]
H1 Prelim 2
Essay Questions
5 (a) Ethyl acetate or ethyl ethanoate is the organic compound with the formula
CH3COOCH2CH3. This colorless liquid has a characteristic sweet smell (similar to
pear drops) like certain glues or nail polish removers, in which it is used. Ethyl
acetate is the ester of ethanol and acetic acid; it is manufactured on a large scale for
use as a solvent. It is formed according to the equation below.

CH3COOH (l) + CH3CH2OH (l) CH3COOCH2CH3 (l) + H2O (l)

In an experiment, 1 mol of ethanoic acid and 2 mol of ethanol were shaken for a
long time to reach equilibrium. The whole mixture was titrated quickly with
2.00 mol dm3 of sodium hydroxide and 80cm3 of alkali was required.

(i) Calculate the number of moles of ethanoic acid present at equilibrium.


No. of moles of ethanoic acid present at equilibrium
= No. of moles of NaOH required

80 1 1
= x 2 [ 2 ] = 0.16 [ 2 ]
1000

(ii) Calculate the number of moles of ethanoic acid that had reacted with ethanol.
No. of moles of ethanoic acid reacted
1 1
= 1 0.16 [ 2 ] = 0.84 [ 2 ]

(iii) Calculate the number of moles of ethanol, ester and water present at
equilibrium.
1 1
Number of moles of ethanol = 2-0.84 [ 2 ] = 1.16 [ 2 ]
1
Number of moles of ester formed = 0.84 [ 2 ]
1
Number of moles of water formed = 0.84 2 ]
[

OR combining answers (i) to (iii)

CH3COOH (l) + CH3CH2OH (l) CH3COOCH2CH3 (l) + H2O (l)

Initial 1 2 0 0
moles
Change 1 1 +0.84 +0.84
in moles 1 0.16 2 ]
[ -0.84 2 ]
[
1
= 0.84 [ 2 ]

Change
= -0.84

Eqm 80 1 1 1 1
moles x 2 [2] 1.16 [ 2 ] 0.84 [ 2 ] 0.84 [ 2 ]
1000
1
= 0.16 [ 2 ]

(iv) Write an expression of the equilibrium constant, Kc for this system.


[CH 3COOCH 2 CH 3 ][ H 2 O]
Kc = [1]
[CH 3COOH ][CH 3CH 2 OH ]

(v) Calculate the value of the equilibrium constant, Kc.

0.84 0.84
[ ][ ] 1
[CH 3COOCH 2 CH 3 ][ H 2 O]
Kc = = V V [ ]
[CH 3COOH ][CH 3CH 2 OH ] 0.16 1.16 2
[ ][ ]
V V
1
= 3.80 [ 2 ] wrong units not penalised

(vi) State the effect on Kc if 0.1 mol of ethanol liquid was introduced into the
container after equilibrium has been established.

No effect on Kc [1] as it is only affected by temperature.


[7]

(b) Acetic acid, CH3COOH, also known as ethanoic acid, is an organic acid that gives
vinegar its sour taste and pungent smell. It is a weak acid, in that it is only a partially
dissociated acid in an aqueous solution

(i) Suggest the reagents and conditions for the conversion of ethanoic acid into
an alcohol.
1
Reagents: Lithium aluminium hydride, LiAlH4 [ 2 ]
1
Conditions: in dry ether [ 2 ]

(ii) Name the type of reaction that occurs and write a balanced equation for the
conversion of ethanoic acid into an alcohol.

Type of reaction: Reduction [1]

H H
O
H
H C C OH + H2O
H C C OH + 4 [H]

H H
H
[1]

(iii) Describe what you would observe when ethanoic acid was reacted with
sodium metal.
Effervescence observed. [1]

Gas evolved will extinguish lighted splint with a pop sound. [0.5] [Award this
mark only if std didnt mention effervescence.]

[4]

(c) Period 3 chlorides have many uses.

Phosphorus trichloride, PCl3 is an essential precursor to PCl5, POCl3 and PSCl3.


which in turn have many applications in herbicides, insecticides, plasticisers, oil
additives and flame retardants.

Silicon tetrachloride, SiCl4 is sometimes used as an intermediate in the manufacture


of extremely pure silicon.

(i) Describe and explain what you would observe when water is gradually added
to PCl3, until in excess. Suggest the pH of the solution.

Phosphorus (III) chloride undergoes hydrolysis [1] with water, to produce


1 1
steamy HCl fumes [ 2 ] and phosphonic acid, H3PO3. [ 2 ] The solution has a

pH = 2 [1]

(ii) Suggest a balanced equation with state symbols for the reaction of liquid
silicon tetrachloride with heavy water, D2O. [deuterium, D= 2 H ]

SiCl4(l) + 2D2O(l)
SiO2(s) + 4DCl(aq) [1]
Or SiCl4(l) + 4D2O(l)
Si(OD)4(s) + 4DCl (aq)
[4]
(d) The graphs below show the variation in first ionisation energy of some elements.
Figure 5.1 refers to elements in the same period, and Figure 5.2 refers to elements
in the same group as element J.

Figure 5.1 Figure 5.2

First M First
Ionisation J
Ionisation
Energy Energy
J N
L

K Atomic No. Atomic No.

x x+1 x+2 x+3 x+4 x-8 x x+8

(i) Element J is a Group II element. Recopy the diagram below and label clearly
how the beams of J+ and K+ ions are deflected when subjected to an electric
field.

You should relate clearly the magnitude and the direction of deflection
of each beam to the other.

-
J: Group II element
K: Group III element

charge
Deflection
mass
1
Mass of K+ > mass of J+ [ 2 ]
1
Angle of deflection for of J+ > Angle of deflection for of K+ [ 2 ]

NOTE: mark given for working or shown in diagram

Source

1
+ [ ]
K 2 or [1]
- 1
J+ [ 2 ] or [1]
If no working is given, [1] can be awarded for J+ and K+

(ii) State and explain which period element J is in and hence identify J.
Based on Figure 5.2, J is in period 3. [1]

Only in period 2-4, atomic number have a difference of 8 down the


1
group. [ 2 ]

1
From period 5, the increase in atomic number down the group is 18. [ 2 ]

J is Magnesium [1]

[5]

[Total: 20]
6 Compound A, a derivative of Decalin is a colorless liquid. It can undergo the following 3-
step reaction scheme.

(a) (i) What reagents and conditions are needed for reactions I, II and III?
Reaction I: NaOH/KOH (aq) [1/2] reflux [1/2]

Reaction II: K2Cr2O7 [1/2] dilute H2SO4, distill [1/2]

Reaction II: HCN [1/2] trace amount of NaOH/NaCN, 10-20C [1/2]

(ii) Name the type of reaction for reaction III.


Addition [1]

(iii) Reaction II is an oxidation reaction. Write a balanced equation for reaction II.
(The symbol [O] is acceptable in this equation.)

[1]
[5]

(b) Draw the organic product formed when B is reacted separately with each of the
reagents and conditions stated below.
(i) Al2O3, 350C

[1]

(ii) PCl5

CH2Cl

[1]
[2]
(c) (i) Compound A, C7H13Br has a structural isomer with a straight chain and
displays geometric isomerism. Suggest a possible structure of the isomer.

[1]

any variation of C7H13Br and must have cis-trans

(ii) Suggest a chemical test to distinguish A and the isomer identified in (c)(i). You
should include suitable observations for the test.
Reagent: Br2 (aq) [1]
Condition: Room temperature (penalise [1/2] for heat)
Observation:
Structural isomer: Orange Br2(aq) will decolourise [1/2]
Compound A: Orange Br2(aq) will not decolourise [1/2]

OR

Reagent: Liquid Br2 [1]


Condition: Room temperature (penalise [1/2] for heat)
Observation:
Structural isomer: Reddish brown Br2 will decolourise [1/2]
Compound A: Reddish brown Br2 will not decolourise [1/2]

OR

Reagent: KMnO4 [1/2]


Condition: dilute H2SO4, reflux [1/2]
Observation:
Structural isomer: Purple KMnO4 will decolourise [1/2]
Compound A: Purple KMnO4 will not decolourise [1/2]

Ecf for test if compound for c(i) drawn wrongly but c(i) must have double
bond.
[3]

(d) D with molecular formula C9H8O is used as a drug carrier. D is insoluble in sodium
hydroxide but reacts with 1 mole of bromine gas in the dark to form E, C9H8OBr2.

When E is heated with aqueous potassium hydroxide, F, C9H10O3, a neutral


compound is formed.

Treatment of D with Tollens reagent will yield silver mirror but when treated with
Fehlings solution, blue colour solution remains. When D is heated with acidified
potassium manganate (VII), G, C8H6O4 is formed. G will produce effervescence
when Na2CO3 is added to it.

Use all of the above information to identify the organic compounds D, E, F, and G.
Include the following in your answers

(i) Structural formula for each of the organic compounds.


(ii) Clearly show all of the deductions that you make from the information that you
have been given.

There is comparable number of C and H in compound D, hence, Compound D


contains a benzene ring. [1]
Compound D is most likely an alcohol. [1] since it is insoluble in sodium
hydroxide
It reacts with bromine gas in the dark, hence an alkene is present. [1]. It reacts in
addition reaction. [1]
Compound E reacts with aqueous potassium hydroxide in substitution reaction. [1]
It contains alcohol group as the compound is neutral. [1]
It reacts with Tollens reagent but not Fehlings solution, hence it contains aromatic
aldehyde. [1]
When reacted with potassium manganate (VII), oxidation takes place. [1]
Compound G reacts with Na2CO3 via acid-carbonate [1] reaction as it contains
carboxylic acid. [1]The effervescence is due to CO2. [1]
H

O C H

C C

H H Compound D [1],
H Br

O C H

C C

H Br H Compound E [1],
H OH

O C H

C C

H HO H Compound F [1],
O

C COOH

HO Compound G [1]
10 max out of 15
Accept 1, 2 position and 1,3 position.
[10]
[Total:20]

7 Chromium is a very shiny, silvery metal whose Greek name, Chroma, refers to its many
colourful compounds.

(a) In air, chromium readily forms a thin and transparent coating of Cr2O3. Like Al2O3,
Cr2O3 is an amphoteric oxide and can react with both acids and bases.

(i) Suggest a reason why chromium is used extensively as a coating for other
metals like iron.
Chromium forms an inert oxide layer [ 1 ], will protect the metal from
2
corrosion [ 1 ].
2
(ii) Write 2 balanced equations with state symbols that shows the amphoteric nature
of Cr2O3.
Cr2O3 (s) + 6H+ (aq) 2Cr3+ (aq) + 3H2O (l) [1]
Cr2O3 (s) + 3H2O (l) + 2OH- (aq) 2 Cr(OH)4- (aq) [1]
(b) Chromium can also exist in the form of chromate, CrO42, which can be prepared from
chromium(III) chloride. This conversion takes place in the presence of hydrogen
peroxide, H2O2.

(i) Write the half equation for the conversion of H2O2 to OH in alkaline medium.
H2O2 (l) + 2e- 2OH- (aq) [1] (state symbols not marked for)
(ii) Given that the half equation for the conversion of Cr3+ to CrO42- in alkaline
medium is as shown below.

Cr3+(aq) + 8OH(aq) CrO42(aq) + 4H2O(l) + 3e

Using your answer to b(i), write the overall balanced equation of the conversion
of Cr3+ to CrO42 in the presence of H2O2 in alkaline medium.
2Cr3+ (aq) + 10 OH- (aq) + 3H2O2 (l) 2CrO42- (aq) + 8H2O (l) [1]
(state symbols not marked for)
(iii) Hence calculate the minimum volume of 0.200 mol dm3 H2O2 required to
convert a sample of 0.500 g of CrCl3 completely to CrO42.
0.500
No. of moles of CrCl3 = [ 1 ] = 0.00315 mol [ 1 ]
52.0 + 3(35.5) 2 2
No. of moles of H2O2 required = 3/2 x 3.15 x 10-3 [ 1 ] = 4.73 x 10-3 mol [ 1 ]
2 2
3/2 ecf from equation in part iv)

4.73 10 3
Volume = [ 1 ] =0.0237 dm3 [ 1 ]
0.200 2 2
[8]

(c) Draw the dot-and-cross diagram of H2O2 and state its shape.

[1]
Shape: Bent [1]
[2]
(d) Besides chromium (III) chloride, chlorides of Period 3 elements are also well-studied.
State and explain the electrical conductivity of the chlorides of magnesium and
phosphorous in both molten and aqueous states.
1
Chlorides of magnesium can conduct electricity [ ] in molten and aqueous states,
2
1
while chlorides of P cannot [ ].
2
MgCl2 is has giant ionic structure. In the molten and aqueous states, there are mobile
charge carriers (ions) [1], hence able to conduct electricity.
PCl3 or PCl5 has a simple molecular structure. In both molten and aqueous states,
there no mobile charge carriers [1] (if std write no mobile ions [0.5]), hence
unable to conduct electricity.
[3]
(e) When chromium (III) chloride is dissolved in water and aqueous ammonia is
subsequently added in excess, the compound [Cr(NH3)6]3+ is formed. The following
diagram shows how the NH3 molecules are arranged around the central Cr3+ ion.

The arrow denotes a dative bond from the N atom to Cr3+, as shown below:

Using the Valence Shell Electron Pair Repulsion (VSEPR) Theory, explain the
phenomenon that the H-N-H bond angle in ammonia when it is bound to Cr3+ is about
110 o, while that in an isolated ammonia molecule is about 107 o.

In the complex [Cr(NH3)6]3+, the N atom in NH3 is dative-bond to Co3+ ion.


Hence, there are 4 bond pairs and zero lone pairs on N atom [1]. [If std did not
indicate N atom, deduct 0.5m. If std write Cr atom, 0m]

To minimize repulsion, electron pairs are directed to the 4 corners of a


tetrahedron [ 1 ], hence bond angle is about 109.5 o (about 110 o). In an isolated
2
ammonia molecule, there are 3 bond pairs and 1 lone pair [1]. Since lone pair-bond
pair repulsion is larger than bond pair-bond pair repulsion [ 1 ], the bond angle is
2
compressed to 107 o.

Max 2 out of 3 marks

[2]

(f) In another compound containing boron chloride and phosphine, an equilbrium exists
between the compound, PH3BCl3, and its adducts PH3 and BCl3 as shown.

PH3BCl3 (s) PH3 (g) + BCl3 (g) H > 0

(i) Write an expression for the equilibrium constant, Kc, for the decomposition of
PH3BCl3 (s).

Kc = [PH3][BCl3] [1]

(ii) A solid sample of PH3BCl3 is introduced in a closed vessel and decomposes


until equilibrium is established. Explain how the equilibrium position will be
affected when the

I temperature is increased to 373 K.


II solid PH3BCl3 is added to the system at equilibrium
(I) When temperature is increased, by Le Chateliers Principle,
equilibrium position shifts to the right [1] to favour the endothermic
reaction [0.5] to absorb additional heat [0.5].

(II) Equilibrium position is unaffected [1]as the addition of solid does


not affect the concentration of PH3 and BCl3 OR the
concentration of solid is constant OR total pressure remains
unchanged [1].

[5]

[Total: 20]
JURONG JUNIOR COLLEGE
JC 2 PRELIMINARY EXAMINATION
Higher 1

CHEMISTRY 8872/01
Paper 1 Multiple Choice 3 September 2010
50 minutes
Additional Materials: Multiple Choice Answer Sheet
Data Booklet

READ THESE INSTRUCTIONS FIRST

Write in soft pencil.


Do not use staples, paper clips, highlighters, glue or correction fluid.
Write your name, class and shade your exam number on the Answer Sheet in the spaces provided.

There are thirty questions on this paper. Answer all questions. For each question there are four
possible answers A, B, C and D.
Choose the one you consider correct and record you choice in soft pencil on the separate Answer
Sheet.

Read the instructions on the Answer Sheet very carefully.

Each correct answer will score one mark. A mark will not be deducted for a wrong answer.
Any rough working should be done in this booklet.

A Data Booklet is provided. Do not write anything on the Data Booklet.

This document consists of 13 printed pages.


[Turn over
SECTION A

For each question there are four possible answers, A, B, C and D. Choose the one you
consider to be correct.

1. Mass spectrometry (MS) is an analytical technique for the determination of the elemental
composition of a sample. The MS principle consists of ionising chemical compounds to
generate charged fragments and measure the masstocharge ratios (m/e) of isotopes.
The m/e value corresponds to the relative atomic mass of the elemental isotopes.
Element X has two isotopes, 63X and 65X, and the graph below shows the mass spectrum
of element X obtained by mass spectrometry.

% abundance
70
60
50
40
30
20
10
m/e
61 62 63 64 65 66

What is the relative atomic mass of element X?

A 63.3 B 63.4 C 63.5 D 63.6

2. Calcium sulfate occurs naturally as gypsum, CaSO42H2O. When heated to 160C,


gypsum loses some of its water of crystallisation to give plaster of Paris which is used for
making fine plaster casts and for setting broken limbs.
During the heating process, 100 g of gypsum loses 15.7 g of water according the
equation below:
CaSO42H2O CaSO4(2 n)H2O + nH2O
heat
gypsum plaster of Paris

What is the formula of plaster of Paris?

A CaSO4 C CaSO4 H2O


1 3
B CaSO4 H2O D CaSO4 H2O
2 2

JJC/8872/Prelim P1/2010 Page 2 of 13 [Turn over


3. TNT is used as an explosive and it has the structure as shown below.

It can decompose according to the following equation :

2 TNT 7CO(g) + 7C(s) + 5H2O(g) + 3N2(g)

What volume of gas will be produced when 10 g of TNT is completely decomposed at


400C and 1 atm?
It is given that the molar volume of gas at 400C and 1 atm is 55 dm3.

A 9.1 dm3 B 9.7 dm3 C 18.2 dm3 D 36.4 dm3

4. The successive ionisation energies of two elements, W and Y, are given below:

ionisation energies in kJ mol1


element 1st 2nd 3rd 4th 5th 6th 7th 8th
W 578 1817 2745 11577 14842 18379 23326 27465
Y 1314 3388 5301 7469 10990 13327 71330 84078
What is the formula of the compound that W and Y are likely to form?

A WY4 B W2Y4 C W3Y2 D W2Y3

5. Which of the following statements describes a phenomenon which cannot be explained


by intermolecular hydrogen bonding?

A Ice has a lower density than water at 0C.


B Propanone is more volatile than propan2ol.
C Ethanoic acid vapour has a relative molecular mass of 120 instead of 60.
D The boiling point of alcohols decreases with increasing branching of carbon chain.

6. What is the shape of SF4 molecule?

A square C seesaw
B trigonal pyramidal D tetrahedral

JJC/8872/Prelim P1/2010 Page 3 of 13 [Turn over


7. Which of the following statements explain why calcium sulfide has a higher melting point
than sodium fluoride?

A Charge density of Ca2+ is greater than that of Na+.


B Size of electron cloud of S2 is larger than that of F.
C More energy is required to form Ca2+ from Ca than to form Na+ from Na atom.
D The electrostatic force of attraction between doubly charged ions is stronger.

8. Which of the following statements about ammonia is true?

A Ammonia forms ionic bond with H+ to produce ammonium ion.


B Covalent bonds between nitrogen and hydrogen atoms in ammonia are overcome
during boiling.
C Ammonia reacts with hydrochloric acid to give a salt solution which turns Universal
Indicator yellow.
D In aqueous ammonia, the only species present are simple molecules with hydrogen
bonding between them.

9. The reduction of ethanal to ethanol using H2(g) is shown in the equation below:
CH3CHO(l) + H2(g) CH3CH2OH(l)
ethanal ethanol

1
substance Hc /kJ mol
ethanal 1170
hydrogen 286
ethanol 1370
Given the values for the enthalpy change of combustion of ethanal, hydrogen and ethanol,
what is the enthalpy change of reaction?

A +86 kJ mol1 B 86 kJ mol1 C 2826 kJ mol1 D +2826 kJ mol1

10. The following equilibria is subjected to an increase in volume at constant temperature.


For which equilibrium will the change result in an increase in the proportion of products?

A H2(g) + I2(g) 2HI(g)


B N2(g) + 3H2(g) 2NH3(g)
C 4NH3(g) + 5O2(g) 4NO(g) + 6H2O(g)
D 2HBr(g) + H2SO4(l) Br2(g) + SO2(g) + 2H2O(l)

JJC/8872/Prelim P1/2010 Page 4 of 13 [Turn over


11. A 1 mol sample of a monobasic weak acid, CH3COOH, is diluted at constant temperature
to a volume V.
CH3COOH(aq) + H2O(l) CH3COO(aq) + H3O+(aq)
Which of the following graphs correctly illustrates the relationship between V and the acid
dissociation constant, Ka, of CH3COOH?

Ka Ka

A C

0 V 0 V

Ka Ka

B D

0 V 0 V

12. Ethyl ethanoate is synthesised industrially via the classic Fischer esterification reaction of
ethanol and ethanoic acid as shown in the equation below. This mixture converts to the
ester in about 65 % yield at room temperature.
CH3COOH(l) + CH3CH2OH(l) CH3COOCH2CH3 (l) + H2O(l)
4 mol each of ethanoic acid and ethanol were put into a 1 dm3 container. The mixture was
heated and cooled to reach equilibrium at room temperature.
What is the numerical value of the equilibrium constant, Kc, at room temperature?

0.35 4 0.65 4 0.35 4 2 0.65 4 2


A B C D
0.65 4 2 0.35 4 2 0.65 4 2 0.35 4 2

JJC/8872/Prelim P1/2010 Page 5 of 13 [Turn over


13. Consider the hypothetical reaction : 2A B C .
Given that the rate constant, k, of the reaction is 0.156 s1, which of the following graphs
correctly reflects the reaction kinetics of the reaction?

Rate Rate

A C

0 [A] 0 [A]

Rate Rate

B D

0 [A] 0 [A]

14. Iodine131 is a radioactive isotope with a halflife of 8 days. Following the nuclear power
plant disaster at Chernobyl in 1986, it was stated that the situation would become safe
1
only when of the iodine131 isotopes remained in the cloud of vapour formed.
1024
Given that the radioactive decay is a firstorder reaction, how long would it take for the
situation to become safe?

A 1024 days B 80 days C 72 days D 40 days

JJC/8872/Prelim P1/2010 Page 6 of 13 [Turn over


15. Which of the following graphs correctly explains an increase in rate of reaction when the
temperature of reaction increases from T1 to T2?

T2
fraction of particles

fraction of particles
T1

T2 T1
A C

0 Ea energy 0 Ea energy
fraction of particles

fraction of particles
T1 T2

B D
T2 T1

0 Ea energy 0 Ea energy

16. The table gives data for the reaction between D and E at constant temperature.

Experiment [D] /mol dm3 [E] /mol dm3 Initial rate /mol dm3 s1
3
1 0.055 0.33 1.5 10
2 0.110 0.33 6.0 103
3 0.110 0.16 6.0 103
What is the rate equation for the reaction?

A rate k [D]2 C rate k [D]2 [E]

B rate k [E]2 D rate k [D][E]2

JJC/8872/Prelim P1/2010 Page 7 of 13 [Turn over


17. The graph below shows the ionic radii of seven elements found in Period 3.

ionic radius

element
Na Mg Al Si P S Cl

Which of the following statements correctly explains the trends shown in the graph?

3 4+
A The ionic radius of P ion is greater than that of Si ion due to greater mutual
repulsion between electrons.
B The ionic radius decreases from Na+ ion to Si4+ ion due to decreasing shielding
effect by inner shell electrons.
C The ionic radius decreases from Na+ ion to Si4+ ion due to decreasing nuclear
charge.
3
D The ionic radius decreases from P ion to Cl ion due to increasing nuclear charge.

18. P, Q and R are main group elements in the same period of the Periodic Table. The oxide
of P is basic, the oxide of Q is acidic and the oxide of R is amphoteric.
What is the order of increasing atomic number for these elements, starting from the
lowest?

A P<Q<R
B P<R<Q
C Q<R<P
D R<P<Q

19. Limonene is an oil formed in the peel of citrus fruits. It has the structure as shown below.

How many geometric isomer(s) does limonene possess?

A 0 B 2 C 4 D 6

JJC/8872/Prelim P1/2010 Page 8 of 13 [Turn over


20. The structures of four organic compounds S, T, U and V are given below.

Which of the following correctly gives the Ka of the four compounds in decreasing order?

highest lowest
A S T U V
B U T V S
C S V U T
D V U T S

21. The structures of three halogencontaining organic compounds, E, F and G, are shown
below.

In which order will the precipitate of respective silver halides be formed when hot sodium
hydroxide is added to E, F and G, followed by acidified silver nitrate solution?

fastest slowest
A G E F
B E F G
C G F E
D F G E

JJC/8872/Prelim P1/2010 Page 9 of 13 [Turn over


22. Ibuprofen is used for the relief of symptoms of arthritis and fever, and also as an
analgesic. It has the chemical structure as shown below.

Which of the following proposed route would be the most appropriate for the synthesis of
Ibuprofen in laboratory?

(The alkyl group can be simplified as R.)

23. An ester, C4H8O2, undergoes the following reactions :


reacts with hot H2SO4(aq) to give carboxylic acid H and alcohol J
alcohol J reacts with hot alkaline I2(aq) to give pale yellow precipitate
carboxylic acid H reacts with LiAlH4 in dry ether to give alcohol J

Which of the following could be the structure of the ester?

A CH3COOCH2CH3 C HCOOCH(CH3)2
B CH3CH2COOCH3 D HCOOCH2CH2CH3

JJC/8872/Prelim P1/2010 Page 10 of 13 [Turn over


24. Phenol is a class of hydroxycontaining compound with the OH group attached to a
benzene ring. It is a stronger acid than ethanol but a weaker acid than ethanoic acid.
An organic compound X has the following structure as shown below.

How many moles of hydrogen gas will be formed when 2 mol of X is reacted with sodium
metal?

A 3 mol B 2 mol C 1 mol D 0 mol

25. Which of the following reactions would produce ?

JJC/8872/Prelim P1/2010 Page 11 of 13 [Turn over


SECTION B

For each of the questions in this section, one or more of the three numbered statements 1 to 3
may be correct.

Decide whether each of the statements is or is not correct.

The responses A to D should be selected on the basis of

A B C D
1, 2 and 3 are 1 and 2 only are 2 and 3 only are 1 only is
correct correct correct correct

No other combination of statements is used as a correct response.

26. An element L, found in Period 3, is a good conductor of electricity in solid and liquid state.
It also reacts violently with water and oxygen.
Which of the following statements are true about element L?

1 L has a giant metallic structure.


2 The oxide of L reacts with alkali but not acid.
3 The mobile charged carriers of L in solid state are ions and electrons.

27. The chloride of an element G, where G is found in Period 3, is a liquid which has a boiling
point of 76C and fumes in air.
3 3
100 cm of 0.30 mol dm silver nitrate is required for the complete precipitation of the
chloride ion in the resulting solution when 0.010 mol of the chloride is mixed with water.
Which of the following statements are true about element G?

1 Element G belongs to Group III of the Periodic Table.


2 The oxide of G is a solid under standard conditions.
3 The chloride of G turns Universal Indicator red when dissolved in water.

28. Which of the following reagents and conditions can distinguish ethanol (CH3CH2OH) and
ethanal (CH3CHO)?

2+
1 alkaline Cu complex solution, heat
2 hot [Ag(NH3)2]+ solution
3 2,4dinitrophenylhydrazine

JJC/8872/Prelim P1/2010 Page 12 of 13 [Turn over


A B C D
1, 2 and 3 are 1 and 2 only are 2 and 3 only are 1 only is
correct correct correct correct

29. Benzil, which has the structure shown below, is commonly employed as a photoinitiator
for freeradical curing of polymer network. Ultraviolet radiation decomposes benzil,
creating freeradical species which propagate throughout polymer material.

Which of the following statements are true about benzil?

1 It is a nonpolar molecule.

2 It can be synthesised by reacting with cold, alkaline KMnO4

followed by hot acidified K2Cr2O7.

3 It does not give orangeyellow crystals upon reaction with 2,4DNPH.

30. Smoke from a bonfire contains a compound that causes irritation to the eyes. It has the
following properties:
gives white fumes with phosphorous pentachloride;
decolourises hot acidified purple KMnO4;
decolourises aqueous bromine readily.
What are the possible structures of the compound?

JJC/8872/Prelim P1/2010 Page 13 of 13 [Turn over


JURONG JUNIOR COLLEGE
JC 2 PRELIMINARY EXAMINATION
Higher 1

CANDIDATE
NAME

EXAM INDEX
CLASS NUMBER

CHEMISTRY 8872/02
Paper 2 Structured Questions 27 August 2010
2 hours
Candidates answer Section A on the Question Paper.
Additional Materials: Answer Paper
Data Booklet

READ THESE INSTRUCTIONS FIRST

Write your name and class on all the work you hand in.
Write in dark blue or black pen on both sides of the paper.
You may use a soft pencil for any diagrams, graphs or rough working.
Do not use staples, paper clips, highlighters, glue or correction fluid.

A Data Booklet is provided. Do not write anything on the Data Booklet.

You are reminded of the need for good English and clear presentation in your answers.

Section A
Answer all questions.

Section B
Answer two questions on separate answer paper.

At the end of the examination, fasten all your work securely together.
The number of marks is given in brackets [ ] at the end of each question or part question.

For Examiners Use

Section A

B5

B6

B7

Total

This document consists of 15 printed pages and 1 blank page.

[Turn over
JJC/8872/Prelim P2/2010 Page 1 of 15
Section A

Answer all questions in this section in the spaces provided.

1. (a) A gaseous mixture of 16O, 16O+ and 16O2 is passed through an electric field
which is at right angles to their direction of travel. Sketch and label, on the
diagram below how each beam of the 3 particles is affected by the electric field.

+
16
O,
16
O +,
16 2
O
[3]

(b) Oxygen reacts with non-metals of Period 3 to form oxides of different physical
and chemical properties. Suggest the pH of the following two Period 3 oxides in
water.

Compound pH in water

SiO2

P4O10

Write chemical equations to describe the reactions, if any, of these oxides with
water to give the different pH values.

..

..

.. [2]

(c) The graph below shows the variation in ionisation energies for the outermost 8
electrons in an atom of element X.
log10 IE

1 2 3 4 5 6 7 8
no. of electrons removed

JJC/8872/Prelim P2/2010 Page 2 of 15 [Turn over


1. (c) (i) State, giving reasons, the group that element X belongs to.

..

..

..

..

..

(ii) Hence, draw a dot-and-cross diagram for the compound formed between
chlorine and element X.

[4]

[Total: 9]

2. Solid aluminium chloride sublimes at 180oC. In the vapour phase, an equilibrium is


established between aluminium chloride and its dimer.

2AlCl3(g) Al2Cl6(g)

(a) Draw a dot-and-cross diagram to show the bonding in the dimer, Al2Cl6.

[1]

(b) With the aid of a diagram, state and explain whether AlCl3 is a polar or
non-polar molecule.

[2]

JJC/8872/Prelim P2/2010 Page 3 of 15 [Turn over


2. (c) Solid AlCl3 dissolves readily in water to give aqueous Al3+ and Cl ions. The
aqueous Al3+ ion hydrolyses in water to produce an acidic solution, as shown
by the following equation:

[Al(H2O)6]3+ + H2O [Al(OH)(H2O)5]2+ + H3O+

(i) What are acids and bases according to Bronsted-Lowry theory?

..

..

(ii) With reference to the equation given above, identify the acid and base,
and their respective conjugate base and acid.

Acid : ..

Conjugate base: ..

Base : ..

Conjugate acid: ..
[3]

(d) The oxide of aluminium, Al2O3, however, is not soluble in water but reacts
with acids and alkalis to form salt and water.

Give equations for the reactions that occur between:

(i) Al2O3 and HCl(aq),

(ii) Al2O3 and NaOH(aq)

[2]

[Total: 8]

JJC/8872/Prelim P2/2010 Page 4 of 15 [Turn over


3. Compound A exhibits geometrical isomerism.

CH=CHCH 2OH

CH3

Compound A

(a) Draw the geometrical isomers of compound A.

[2]

(b) Draw the products formed when compound A reacts with the reagents
indicated. Describe any observations, if any, made.

(i) H2, Pt catalyst, heat

(ii) PCl5, room temperature

(iii) Cold, alkaline KMnO4

[5]

JJC/8872/Prelim P2/2010 Page 5 of 15 [Turn over


3. (c) A reaction scheme involving compound A is shown below.

CH=CHCH 2OH CH=CHCH 2OH


step I step II
Compound
CH3 CH2Cl Hot acidified B
K2Cr2O7(aq)
A

(i) Suggest reagents and conditions for step I.

..

(ii) State the type of reaction for step II.

..

(iii) Draw the structure of compound B.

[3]

[Total: 10]

4. The Olympic flame arrived in Singapore on August 6 2010 for a traditional six-day
torch relay around the island before it was used to ignite the Singapore 2010 YOG
cauldron to mark the start of the Game.

The fuel used to light the Olympic flame is propane gas which is also used in blow
torches to melt bitumen needed to apply felt material to flat roofs and in outdoor
flames such as patio heaters.

Most patio heaters are powered by the combustion of propane gas stored in small
cylinders. A typical patio heater designed to produce 15 kW of energy runs from a
cylinder containing 13 kg of propane. The standard enthalpy change of combustion
of propane is 2220 kJ mol 1.

As pure propane gas is odourless, small amounts of another compound are usually
added so that gas leaks from propane cylinders can be detected. An example of
such an odorant is ethanethiol, C2H5SH; which has an odour that resembles that of
onion. Ethanethiol is chosen since the human nose can detect its presence at
levels of only about 0.02 moles of it per billion (109) moles of propane.

(a) A propane blow torch was used to heat 200 cm3 of water. Calculate the mass
of propane needed to raise the temperature of water from 18.0C to 68.5C.

[2]

JJC/8872/Prelim P2/2010 Page 6 of 15 [Turn over


4. (b) Another value of the standard enthalpy change of combustion of propane,
Hc, can be calculated using bond energies and the energy cycle below.
Hc
C3H8(g) + 5O2(g) 3CO2(g) + 4H2O(g)

+6460 kJ mol1 H1

3 C(g) + 8 H(g) + 10 O(g)

(i) Use suitable bond energies given in the Data Booklet to calculate H1.

(ii) Hence, calculate Hc.

(iii) Give one reason why the value you calculated in (b)(ii) differs from the
value for the enthalpy change of combustion of propane given in the
passage.

..

.. [3]

(c) (i) Calculate the total amount of heat energy released by the complete
combustion of all the propane in a cylinder used in a patio heater.

(ii) Calculate the rate at which propane must leave the cylinder (in cm3 s1)
to produce 15 kW (i.e. 15 kJ s1). Assume that one mole of gas
occupies 24 dm3.

[3]

JJC/8872/Prelim P2/2010 Page 7 of 15 [Turn over


4. (d) (i) Draw a displayed formula to show the structure of ethanethiol and
predict the bond angle around the sulfur atom.

Bond angle around sulfur atom:

(ii) Predict and explain the difference in boiling points between ethanethiol
and ethanol.

..

..

..

..

..

..

(iii) Calculate the minimum mass of ethanethiol which must be added to


13 kg of propane so that it can be detected by the human nose in the
event of a propane gas leak.

[5]

[Total:13]

JJC/8872/Prelim P2/2010 Page 8 of 15 [Turn over


BLANK PAGE

JJC/8872/Prelim P2/2010 Page 9 of 15 [Turn over


Section B

Answer two of the three questions in this section on separate answer paper.

5. Elemental sodium, found in Period 3 of the Periodic Table, was first isolated by
Sir Humphry Davy in 1807 by passing an electric current through molten
sodium hydroxide. Elemental sodium does not occur naturally on Earth due to
its high reactivity with air and water.

(a) Write balanced equations to explain the words in italics. [2]

(b) (i) State and explain the differences between sodium and chlorine in the
following two physical properties:

melting point

electrical conductivity

(ii) Describe, with an equation, the reaction between sodium metal and
chlorine gas. Include state symbols in your answer.

(iii) Predict, with reasoning, the pH of the resulting solution when your
product in (b)(ii) is dissolved in water. [6]

(c) A student did a titration involving 0.100 mol dm3 sodium hydroxide and
15 cm3 of CH3COOH at 298K. The titration curve was obtained as follows.

pH

7
Y
x

0 Volume of NaOH added / cm3


28

(i) With the aid of the titration curve, determine the initial concentration of
CH3COOH.

(ii) State and explain the choice of indicator for the above titration.

(iii) At point Y, the solution contains a mixture of CH3COOH and CH3COONa


which resists pH changes when small amount of acid or base is added
to it.

Using balanced equations, illustrate how the solution at point Y resists


pH changes when small amount of acid or base is separately added to it. [6]

JJC/8872/Prelim P2/2010 Page 10 of 15 [Turn over


5. (d) Sodium hydroxide is commonly used in organic synthesis for functional group
conversions and acidbase reactions. The role of sodium hydroxide in organic
reaction differs under different conditions.

Consider the following reaction scheme:

H I2 in NaOH(aq),
hot NaOH(aq) heat
H3C C CH3 P Q
I II
Br
KMnO4 in
NaOH in NaOH(aq), heat
ethanol, heat R
III IV

(i) Suggest the structures of compounds P, Q and R.

(ii) Suggest the type of reaction for reactions I to IV. [6]

[Total: 20]

JJC/8872/Prelim P2/2010 Page 11 of 15 [Turn over


6. Ethanol is an important industrial chemical and is usually made by the direct
hydration of ethene.

C2H4(g) + H2O(g) CH3CH2OH(g) H = 46 kJ mol1

(a) (i) Define the term standard enthalpy change of formation.

(ii) Use the standard enthalpy change for the above reaction and the
following enthalpy change of formation values, Hf, given in the table
below to calculate the enthalpy change of formation of gaseous ethanol.

Compound Hf/kJ mol1

C2H4(g) 52.3

H2O(g) 242

(iii) Given that the standard enthalpy change of formation of liquid ethanol is
248 kJ mol1, calculate the enthalpy change for the conversion of one
mole of liquid ethanol to one mole of gaseous ethanol at 298 K. [4]

(b) A 1.8 dm3 sealed vessel initially contained 1 mol of gaseous ethene and
3 mol of steam at 400 K and 1 atm. Upon reaching equilibrium, it was found
that 27% of the ethene was converted to ethanol at 400 K.

(i) Write an expression for the equilibrium constant, Kc, for the reaction.

(ii) Using the data provided, calculate a value for Kc at 400 K.

(iii) The percentage conversion of ethene to ethanol at equilibrium at


various temperatures and pressures is shown below.

% of ethene
converted

9 atm
5 atm
3 atm
Temperature
With reference to the graph above, explain the trend in the percentage
of ethene converted with changes in

temperature

pressure [7]

JJC/8872/Prelim P2/2010 Page 12 of 15 [Turn over


6. (c) A, B and C are isomeric alcohols with the molecular formula of C4H10O. On
warming with acidified KMnO4, the reagent does not change colour with A but
decolourises with B and C, producing compounds D and E respectively.
D gives an orange precipitate with 2,4-dinitrophenylhydrazine while E does
not. When heated over Al2O3, A and C give the same alkene, F.

Suggest a structure for each lettered compound, and explain the reactions
involved. [9]

[Total: 20]

JJC/8872/Prelim P2/2010 Page 13 of 15 [Turn over


7. This question is about food preservatives.

(a) Cinnamaldehyde is a common carbonyl compound derivative used in the food industry.
It can be used to synthesise compound H as shown in the following reaction scheme:

OH
step I step II CH CHCH
Compound G
CH CHCHO
COOH
cinnamaldehyde Compound H

(i) Draw the structure of the intermediate organic compound, G.

(ii) State the reagents and conditions required for steps I and II.

(iii) Describe a simple chemical test to distinguish between cinnamaldehyde


and compound H. [5]

(b) Sulfur dioxide is sometimes used as a preservative for dried apricots and other
dried fruits. As a preservative, it maintains the appearance of the fruit and
prevents rotting.

In the laboratory, the presence of sulfur dioxide gas can be confirmed with an
acidified solution of potassium dichromate(VI). Sulfur dioxide reacts with the
Cr2O72 ions of potassium dichromate(VI) in the following way.

3SO2(g) + Cr2O72(aq) + 2H+(aq) 3SO42(aq) + 2Cr3+(aq) + H2O(l)

(i) State the oxidation numbers of sulfur in SO2 and SO42.

(ii) A gaseous sample of SO2 was dissolved in 50 cm3 of


potassium dichromate(VI) solution and was completely reacted. The
resulting solution was found to contain 0.120 mol dm3 of Cr3+(aq).
Calculate the amount of sulfur dioxide dissolved.

(iii) State what will be observed when the above reaction takes place. [5]

(c) Vanillin is used widely as a flavoring, usually in sweet foods such as ice cream
and chocolate. It exists as a yellow solid and dissolves readily in water to form a
yellow solution.

CHO

OCH3
OCH 3
OH
vanillin
vanillin

JJC/8872/Prelim P2/2010 Page 14 of 15 [Turn over


7. (c) The kinetics of the reaction between vanillin and HCN were investigated using
two different initial concentration of HCN. Both experiments were conducted
at 25oC in a pH 9.0 buffer.

0.035

[vanillin]/ 0.03
[HCN] = 0.100 mol dm3
mol dm3

0.025

0.02

0.015

0.01

0.005
[HCN] = 5.00 mol dm3

0
0 0.1 0.2 0.3 0.4 0.5 0.6 0.7
Time/ min

(i) Assuming that the reaction produces a colourless product, suggest one
plausible method by which the concentration of vanillin may be
followed during the course of the reaction.

(ii) From the graph, deduce the order of the reaction with respect to
vanillin. Give your reasoning.

(iii) In the experiment using 0.100 mol dm3 HCN, the half-life of the
reaction is 4.75 min. Hence determine, with explanation, the order of
reaction with respect to HCN.

(iv) Hence, write down the rate equation for the reaction between vanillin
and HCN. State the units for the rate constant.

(v) With the help of a Boltzmann distribution diagram, explain how the
addition of a catalyst speeds up the rate of the reaction. [10]

[Total: 20]

JJC/8872/Prelim P2/2010 Page 15 of 15


Answer Key to JJC 2010 8872 Preliminary Examination Paper 1

1 D 11 D 21 C
2 B 12 D 22 B
3 C 13 A 23 A
4 D 14 B 24 A
5 D 15 A 25 C
6 C 16 A 26 D
7 D 17 D 27 C
8 C 18 B 28 A
9 B 19 A 29 B
10 C 20 D 30 B
JJC 2010 8872 Preliminary Examination Paper 2 (Worked Solutions)

16
1. (a) A gaseous mixture of O, 16O+ and 16O2 is passed through an electric [3]
field which is at right angles to their direction of travel. Sketch and label,
on the diagram below how each beam of the 3 particles is affected by
the electric field.

16
O,

16
O+,
16 2
O

(b) P4O10(s) + 6H2O(l) 4H3PO4(aq) [2]

SiO2 has no reaction with water.

(c) (i) There is a sharp increase from 2nd to 3rd IE. The 3rd electron to be [4]
removed belongs to the inner quantum shell and requires a much
greater energy to be removed.
X has 2 valence electrons X is in Group II.
(ii)2+ _ _ 2+ _ _
x x X
Ca x x
Ca Cl Cl Cl Cl

[Total: 9]

Page 1 of 8
2. (a) [1]
Cl Dative bond

Cl Al Cl
Cl Al Cl
Dative bond
Cl

(b) Non-polar [2]

-
Cl

Al
+ -
- Cl Cl

(c) (i) Acids are proton donors and bases are proton acceptors. [3]

(ii) Acid: [Al(H2O)6]3+


Conjugate base: [Al(OH)(H2O)5]2+
Base: H2O
Conjugate acid: H3O+
(d) (i) Al2O3 and HCl(aq), [2]

Al2O3(s) + 6H+(aq) 2Al3+(aq) + 3H2O(l)


(or Al2O3(s) + 6HCl(aq) 2AlCl3(aq) + 3H2O(l) )

(ii) Al2O3 and NaOH(aq)

Al2O3(s) + 2OH(aq) + 3H2O(l) 2[Al(OH)4](aq)


(or Al2O3(s) + 2NaOH(aq) + 3H2O(l) 2Na[Al(OH)4](aq))

[Total: 8]

Page 2 of 8
3. (a) H CH2OH H H [2]
C=C C=C
H CH2OH

CH3 CH3

trans-isomer cis-isomer

(b) (i) CH2CH2CH2OH [5]

CH3

(ii) H CH2Cl
C=C
H

CH3

White fumes of HCl observed.

(iii) OHOH

C-C CH2OH
HH
CH3

Purple KMnO4 turns green, brown precipitate MnO2 formed.

(c) (i) UV light, limited Cl2 [3]

(ii) Oxidation

(iii) CH=CHCOOH

CH2Cl
[Total: 10]

Page 3 of 8
4. (a) Amount of heat gained by 200 cm3 of water [2]

= 200 4.18 (68.5 18.0) = 42218 J = 42.2 kJ

Mass of propane required = 42.2 44.0 = 0.836 g


2220

(b) (i) [3]


Hc
C3H8(g) + 5O2(g) 3CO2(g) + 4H2O(g)

+6460 kJ mol1 H1

3 C(g) + 8 H(g) + 10 O(g)


H1 = 6E(C=O) + 8E(OH) = 6 740 + 8 460 = + 8120 (kJ mol1)

(ii) Hc = +6460 (+8120) = 1660 (kJ mol1)

(iii) The bond energy values obtained from the Data Booklet are average
(mean) bond energy values derived from a full range of molecules that
contain the particular bond and would differ from the experimental
values.

(c) (i) 13000 2220 = 6.55 105 kJ [3]


Total amount of heat energy released =
44.0

(ii) Total amount of time needed to empty one cylinder


5
= 6.5510 = 4.37 104 s
15

Rate at which propane must leave the container

13000
24000
= 44.0 = 162 cm3 s1
4
4.3710
H H
(d) (i) [5]
H C C S
Bond angle around sulfur atom = 104.5 or 104
H
H H
(ii) Ethanol has a higher boiling point than ethanethiol.
More energy is needed to overcome the stronger hydrogen bonding
between ethanol molecules than the weaker van der Waals forces
between ethanethiol molecules.

(iii) x
Minimum mass of ethanethiol required = 62.1 109 = 0.02
13000
44.0

x = 3.67 107 g

[Total: 13]

Page 4 of 8
5. (a) High reactivity with air : Na(s) + O2(g) Na2O(s) [2]

High reactivity with water : Na(s) + H2O(l) NaOH(aq) + H2(g)

(b) (i) Sodium has giant metallic structure while chlorine has simple [6]
covalent/molecular structure.

Large amount of energy is required to overcome the strong metallic


bond/electrostatic force of attraction between Na+ and delocalized
electrons as compared to the weak van der Waals/intermolecular force
of attraction between chlorine molecules.

Hence, sodium has a higher melting point than chlorine.

Sodium has free mobile electrons as charged carrier to conduct


electricity but chlorine does not have any mobile charged carrier.

Hence, sodium can conduct electricity in solid and liquid state but
chlorine cannot conduct electricity in all physical state.

(ii) 2Na(s) + Cl2(g) 2NaCl(s)

Sodium reacts very vigorously with chlorine gas.

(iii) Sodium chloride is an ionic chloride which dissolves in water to give a


solution of pH 7.

(c) (i) 28 [6]


amount of NaOH used 0.100
1000
0.00280mol
Since 1 NaOH 1 CH3COOH,
1000
CH3COOH 0.00280 15 0.187mol

(ii) Phenolphthalein can be used as the indicator.

the pH of the solution at the equivalence point falls in the pH working range
of the indicator.

(iii) With acid : CH3COO + H+ CH3COOH

With base : CH3COOH + OH CH3COO + H2O

(d) (i) P : CH3CH(OH)CH3 [6]

Q : CH3COONa+

R : CH2=CHCH3

(ii) I : nucleophilic substitution or alkaline hydrolysis

II : oxidation or triidomethane/iodoform test

III : elimination

IV : oxidation

Page 5 of 8
6. (a) (i) The standard enthalpy change of formation of a substance refers to the [4]
enthalpy change which occurs when one mole of substance is formed
from its elements in their standard states under standard conditions.

(ii) Hf(CH3CH2OH(g)) 52.3 + 242 = 46

Hf(CH3CH2OH(g)) = 236 kJ mol1

(iii) Enthalpy change = 236 (248) = +12 kJ mol1

(b) (i) [CH3CH2OH] [7]


Kc =
[C2H4 ][H2O]

(ii) C2H4(g) + H2O(g) CH3CH2OH(g)

Initial amount/mol 1 3

Equilibrium amount/mol ( 100-27 1) 3 0.27 0.27


100 = 2.73
= 0.73

0.27
Kc =
[CH3CH2OH] = = 0.243
1.8
[C2H4 ][H2O] 0.73 2.73

1.8 1.8

(iii) As temperature increases, the percentage of ethene converted


decreases.

This shows that equilibrium position has shifted to the left to favour the
endothermic backward reaction so as to remove some heat.

As pressure increases, the percentage of ethene converted increases.

This shows that equilibrium position has shifted to the right to produce
less gaseous molecules so as to decrease the pressure.

Page 6 of 8
6. (c) A does not undergo oxidation with acidified KMnO4. [9]

A is a tertiary alcohol.

B and C undergo oxidation with acidified KMnO4.

B and C are either primary or secondary alcohols.

D undergoes condensation with 2,4-dinitrophenylhydrazine while E does not

D is a ketone.

B is a secondary alcohol.

C is a primary alcohol.

E is a carboxylic acid.

A and B undergoes elimination/ dehydration to give the same alkene.

OH
H
H3C CH3 CH3CH(OH)CH2CH3
C
H3C C CH2OH
CH3 B
CH3
A
C

H
H3C C CH2
CH3COCH2CH3 H3C C COOH
D CH3
CH3
F
E

Page 7 of 8
7 (a) (i) OH [5]
CH CHCH
G. CN
(ii) Step I: HCN, trace amount of base
Step II: dilute HCl, heat under reflux
(iii) Add PCl5.
Compound H will give white fumes of HCl, but cinnamadehyde will not.
(b) (i) oxidation number of S in SO2 : +4 [5]
oxidation number of S in SO42 : +6
(ii) 50
Amount of Cr3+(aq) = 0.120 x = 6 x 103 mol
1000
Since 3SO2 2Cr3+,
3
Amount of SO2 dissolved = x 6 x 103 = 9 x 103 mol
2
(iii) Orange dichromate(VI) solution turns green.

(c) (i) Concentration of vanillin may be followed by measuring the colour [10]
intensity of vanillin.

(ii) Using the graph [HCN] = 5.00 mol dm-3, (t1/2)1 = (t1/2)2 = 0.095 min

Since half-lives is constant, it is 1st order with respect to vanillin.

(iii) When the concentration of HCN increased by 50 times (from 0.1 mol
dm-3 to 5.00 mol dm-3), half life decreased by 50 times/ rate of reaction
increased by 50 times. Order of reaction with respect to HCN is one.

(iii) Rate = k[vanillin][HCN] Units for k = mol1dm3min1

(iv) A catalyst provides an alternative reaction path of lower activation


energy than that of the uncatalysed reaction.

In the presence of a catalyst, the number of molecules with energy />


Ea, increases.
Therefore, the frequency of effective collisions between these reactant
molecules increases and hence, the reaction rate increases.

No. of particles with energy /> Ea


Fraction of
molecules
No. of particles with energy /> Ea

Ea: activation energy for uncatalysed rxn


Ea : activation energy for catalysed rxn

0
Ea Ea Energy

Page 8 of 8
Class Adm No

Candidate Name:

Promotional Examination 2 2010


Pre-university 2

H1 CHEMISTRY 8872
PAPER 1 8872 / 1
Thursday 23 September 2010 50 m

Additional materials:
OMR
Data Booklet

INSTRUCTIONS TO CANDIDATES

1. Do not turn over this question paper until you are told to do so.

2. Write your name, class and index number in the spaces provided at the top of this
page and on the OMR provided.

3. Answer ALL questions and shade the correct answers on the OMR provided.

4. Hand in the question paper and the OMR separately.

FOR EXAMINERS USE

Section Total

Marks
30

INFORMATION FOR CANDIDATES

Marks will not be deducted for wrong answers; your total score on this test will be the
number of correct answers given.

This question paper consists of 12 printed pages.

[Turn over
2

Section A

For each question there are four possible answers, A, B, C and D. Choose the one you
consider to be correct.

1 Which of the following contains one mole of the stated particles?


A Hydrogen atoms in 2 g of hydrogen gas
B Hydrogen atoms in 22.4 dm3 of methane gas at s.t.p.
C Chlorine molecules in 22.4 dm3 of chlorine gas at r.t.p.
# D Aqueous chloride ions in a 2 dm3 of 0.50 mol dm-3 NaCl solution.

2 Carbon disulphide vapour was burnt in oxygen according to the equation:

CS2(g) + 3O2(g) CO2(g) + 2SO2(g)

20 cm3 of carbon disulphide was burnt in 80 cm3 of oxygen. Upon cooling, the mixture of
gases was treated with an excess of aqueous sodium hydroxide and the volume of gas
was again measured. All measurements were made at the same temperature and
pressure.

What was the final measurement of the volume of gas that was left behind?
A 50 cm3
B 40 cm3
# C 20 cm3
D 10 cm3

3 The standard enthalpy of formation of carbon dioxide and water are -394 kJ mol-1 and
-286 kJ mol-1respectively. If the standard enthalpy of combustion of propyne, C3H4 is
-1938 kJ mol-1, calculate the standard enthalpy of formation for propyne.
A +1258 kJ mol-1
# B +184 kJ mol-1
C -184 kJ mol-1
D -680 kJ mol-1

[Turn over
3

4 A chemist discovered that an element N forms N2+ ions with the electronic configuration
1s2 2s2 2p6 3s2 3p6 3d8. Element N has a relative atomic mass of 2y+5, with y
representing the proton number. How many neutrons are present in an atom N.
A 28
B 30
C 32
# D 33

5 What pair of compound has the same molecular shape?


# A AlCl3 and BCl3
B AlCl3 and PCl3
C BF3 and NH3
D BeCl2 and H2O

6 The rate of removal of the pain-killing drug paracetamol from the body is a first order
reaction with a rate constant, k = 0.26 h-1. How long will it take to remove 75% of the
paracetamol that a patient consumes?
A 2.3 h
B 2.7 h
# C 5.3 h
D 8.0 h

7 In an aqueous solution at 25oC, the [OH-] is 6.0 x 10-9 mol dm-3. The given Kw is
1.0 x 10-14 mol2 dm-14. What is the pH of the solution?
A 6.00
B 7.00
# C 5.77
D 4.22

[Turn over
4

8 Ammonia is produced from hydrogen and nitrogen, according to the equation:


N2 + 3H2 2NH3 H = -ve
The graph shows the yield of ammonia produced at 200C and 100 kPa.

Which of the following graphs show the correct yield of ammonia at 400C and 100 kPa?

[Turn over
5

9 The energy profile for a reversible reaction is shown below.

Energy

Reactant A
y

Product B
Reaction pathway

A The backward reaction is exothermic.


B The activation energy of the forward reaction is y x.
# C The activation energy of the backward reaction is x + y
D The enthalpy change of the forward reaction is x+y.

10 Which of following oxides will produce a solution with the lowest pH when treated with
water?

A SiO2

#
B SO2

C MgO

D Na2O

11 In which of the following pairs is the difference in melting point the largest?
A Sodium and magnesium
B Magnesium and silicon
C Chlorine and argon
# D Silicon and argon

[Turn over
6

12 The following graph shows how a property of the elements Na to C/ varies with proton
number.

What is the property?

A Electronegativity

B First ionisation energy

#
C Ionic radius

D Melting point

13 As the number of carbon atoms in a homologous series of alkane molecules increases,


for which property of the alkanes does the numerical value decrease?
A Density
B Enthalpy change of vaporization
C Number of isomers
# D Vapour pressure.

14 Which of the following is the main product when 2-methylbutan-2-ol reacts with
concentrated H2SO4?
A pent-2-ene
# B 2-methylbut-2-ene
C 2,2-dimethylpropene
D Propanone

[Turn over
7

15 When chlorine is passed through ethanoic acid in the presence of UV light, the organic
compound formed is likely to be
A CH3COCl
B ClCH2CHO
C ClCH2CH2OH
# D ClCH2COOH

16 When propanone is heated with iodine and aqueous alkali, 3.94g of triiodomethane is
formed. What is the mass of propanone heated?
A 0.16g
# B 0.58g
C 3.98g
D 4.12g

17 The oxidation of an alkene X produces a type of diol. A continuous oxidation process will
produce a diketone. What could X be?
A (CH3)2C=C(CH3)2
B CH3CH=C(CH3)2
C (CH3)2CHCH=CH2
# D CH3CH=CHCH3

18 When a student tested samples of insecticide extracts with 2,4-dinitrophenylhydrazine,


colourful crystals with high melting point were obtained. Which compound most probably
reacted in the extract to give such a result?
A CH3CH2OH
# B CH3CHO
C CH3COOH
D CH3CH3

[Turn over
8

19 The graph shows the mass and amount of carbon, fluorine and chlorine atoms in one
mole of a compound.

What is the molecular formula for this compound?


A CF2Cl
B CF2Cl2
C C2F3Cl3
# D C2F4Cl2

20 The rate of reaction between propanone and iodine using acid as a catalyst is determined
using spectrophotometer in the laboratories. In this reaction, why is the concentration of
propanone kept high?
A It acts as a buffer solution.
B It increases the rate of the reaction.
C It maintains the consistency of the rate of the reaction.
# D It ensures that the concentration of propanone is kept a constant.

[Turn over
9

Section B

For each of the questions in this section, one or more of the three numbered statements
1 to 3 may be correct.

Decide whether each of the statements is or is not correct.

The responses A to D should be selected on the basis of

A B C D
1, 2 and 3 are 1 and 2 only 2 and 3 only 1 only is
correct are correct are correct correct

No other combination of statements is used as a correct response.

21 Boron nitride has a structure that is similar to graphite.

Which of the following characteristic/s is/are shown by this compound?


# 1 It is a lubricant.
2 It is very hard.
3 It is transparent at standard conditions.

22 Consider the reaction PCI5(g) PCI3(g) + CI2(g)


For such a system, of a total volume 1.0 dm3 and a total pressure of 202.6 kPa, at
equilibrium, the percentage dissociation of PCI5(g) at 100C is 28 % and that at 220C is
78 %.
# 1 The dissociation of PCI5(g) is an endothermic reaction.
# 2 The presence of a catalyst will allow dynamic equilibrium to be achieved much
faster.
# 3 An increase in the concentration of PCI5(g) will cause the rate of the forward
reaction to increase.

[Turn over
10

23 The following reaction is believed to be of first order.

X products

In an experiment, [X], the concentration of X, varied with time as shown in the graph.

What can the graph be used to for?

[X]

time
# 1 To determine the rate of reaction at any given instant.
# 2 To determine if the reaction is of first order throughout.
# 3 To determine half life of the reaction.

24 In which of the following process/es is/are energy being absorbed?


1 F(g) + e F-(g)
# 2 Na(g) Na+(g) + e
# 3 NaCl(s) Na+(g) + Cl-(g)

25 Which of the statement/s is/are true concerning a metallic bond?


1 Strong electrostatic forces of attraction exist between oppositely charged ions.

# 2 Lattice of positive ions is present in a metallic bond.


# 3 The strength of the metallic bond increases when the number of delocalized
electrons increases.

[Turn over
11

Which of the following solids is an example of a substance with a macromolecular


26
structure?
# 1 Diamond

# 2 Silicon(IV) oxide

3 Silicon (IV) chloride

27 Bromine that is dissolved in an inert solvent is added to hexane, benzene, hexene and
methylbenzene in the absence of light. Which of the following pairs of compound will
produce the same observation?
# 1 Hexane and benzene
2 Hexane and Hexene
3 Hexene and methylbenzene

28 After an oil spill in the ocean, liquid hydrocarbon was found floating on the water surface.
Which statements explain why liquid hydrocarbon is less dense than water causing it to
float on the water?
# 1 There are only van der waal forces of attraction between the hydrocarbon
molecules.
2 The hydrogen bonds between water molecules cause the water molecules to be
closer.
3 Hydrocarbon molecules do not undergo hydration by the water molecules.

29 The queen bee secretes a type of chemical substance that encourages the building of
colony cells by the other ordinary bees. That chemical substance has the following
molecular formula:

CH3CO(CH2)5CH=CHCO2H

Which statement/s is/are true about the compound above?


# 1 It can exist as a cis-trans isomer.

# 2 1 mole of the compound reacts with Na to give 1 mole of H2.

3 It provides a positive test with Fehlings solution.

[Turn over
12

30 Which of the following reagents, when added to the compound below, would show a
colour change?

CH3 O
O CH C
C CH H
H2C CH2
CH2
1 Hot aqueous alkaline iodine

# 2 Tollens reagent

# 3 Hot acidified potassium manganate (VII)

End of Paper

[Turn over
Class Adm No

Candidate Name:

Promotional Examination 2 2010


Pre-university 2

H1 CHEMISTRY 8872
PAPER 2 8872 / 2
16 September 2010 (Thur) 2h

Additional materials:
Answer booklet
Cover page
Data Booklet
5 pieces of writing paper

READ THESE INSTRUCTIONS FIRST

Do not open this booklet until you are told to do so.


Write your name, index number and class on all the work you hand in.
Write in dark blue or black pen on both sides of the writing paper.
You may use a soft pencil for any diagrams, graphs or rough working.
Do not use staples, paper clips, highlighters, glue or correction fluids.

Section A
Answer all questions.

Section B
Answer any two questions on the writing papers. Start each new question on a fresh sheet
of writing paper.

At the end of the examination, hand in your answers to Section A and Section B separately.
Attach the cover page to your answer scripts for Section B.

The number of marks is given in brackets [ ] at the end of each question or part question.

FOR EXAMINERS USE


A B (2 questions) Total
Section
Q1 Q2 Q3 Q4 Q5 Q6 Q7

Marks
13 7 9 11 20 20 20 80

This question paper consists of XX printed pages.

[Turn over
2

SECTION A (40 MARKS)

Answer all questions in the space provided.

1 Oxides of nitrogen is a generic term for the mono-nitrogen oxides NO and NO2. They
are produced during combustion, especially at high temperatures.

At ambient temperatures, the oxygen and nitrogen gases in air will not react with
each other. In an internal combustion engine, combustion of a mixture of air and fuel
produces combustion temperatures high enough to drive endothermic reactions
between atmospheric nitrogen and oxygen in the flame, yielding various oxides of
nitrogen.

In a car engine, it was found that a mixture of nitrogen dioxide and carbon monoxide
which was initially in the ratio of 1: 2 will reach dynamic equilibrium when 25% of
the carbon monoxide has reacted.

NO2(g) + CO(g) NO(g) + CO2(g)

In areas of high motor vehicle traffic, such as in large cities, the amount of nitrogen
oxides emitted into the atmosphere as air pollution can be quite significant.

To help reduce the amount of oxides of nitrogen in the environment, scientists came
up with several methods as named in the graph below.

(i) Draw the dot and cross diagram for the compound NO. [1]

[Turn over
3

(ii) Draw a well labeled energy profile diagram to illustrate the reaction between [2]
nitrogen and oxygen in order to produce nitrogen oxide.
Correct shape with axes 1 mark.
Labeled diagram( N2, O2, NO, CO2) 1 mark.

(iii) What do you understand by the term dynamic equilibrium? [1]


Dynamic equilibrium refers to a state of balance in a reversible reaction or
process in which the rates of the forward and reverse processes are equal.

(iv) State and explain the effect of decreasing the volume of the vessel on the [3]
yield of nitrogen monoxide when NO2 reacts with CO(g).
When the volume of the vessel is decreased, the pressure of the system will
increase. ;

By LCP, the system will try to decrease the pressure by shifting the
equilibrium to the side that has less number of moles of gaseous particles. ;

Since there is no change in the number of moles of gaseous particles on


both side, there will not be a change in the equilibrium position.;

(v) Calculate equilibrium constant for the reaction between NO2 and CO. [3]

NO2(g) + CO(g) NO(g) + CO2(g)


Initial 1 2 0 0
Change -x -x +x +x
Eqm 1-x 2-x x x
; 0.5 1.5 0.5 0.5

Since 25% of the carbon monoxide has reacted, x = 0.25 x 2 = 0.5 ;

( PNO )( PCO2 )
Kp =
( PNO2 )( PCO )
(0.5) 2
=
(0.5)(1.5)
1
= (no units) ;
3

(vi) From the information provided above, propose the most effective method for [1]
removing oxides of nitrogen produced by the combustion of fuel?
Catalytic scrubber/ catalytic converter.

[Turn over
4

(vii) Write equations to show how CO and NO are removed by the car engines. [2]
2NO + 2CO 2CO2 + N2;
2CO + O2 2 CO2 ;
[Total: 13 marks]

2 A 1.00g sample of steel containing manganese was dissolved in nitric acid to give a
solution of manganese with an oxidation state of +2. All the manganese present was
then oxidized to manganate (VII) by adding sodium bismuthate, NaBiO3. After
destruction of any excess bismuthate ion, the resulting purple solution required 36.0
cm3 of a 0.100 mol dm-3 iron(II) sulphate solution for complete reaction.

[5 mol. of Fe2+ required 1 mol. of MnO4- for complete reaction]

(i) What is the oxidation number of bismuth in NaBiO3? [1]


+5

(ii) Given that the bismuthate ion is reduced to Bi3+, write a reducing half [1]
equation for the reaction in which it functions like an oxidizing agent.
BiO3-(aq) + 6H+(aq) + 2e Bi3+(aq) + 3 H2O(l)

(iii) Hence, write a balanced equation for the oxidation of manganese (II) to [1]
manganese (VII) in an acidic solution.
2Mn2+(aq) + 5 BiO3-(aq) + 14H+(aq) 2MnO4-(aq) + 5 Bi3+(aq) + 7H2O(l)

(iv) Calculate the % by mass of manganese in the steel sample. [4]

36.0
Amount of Fe2+ = 0.100 x
1000
= 3.600 x 10-3 ;(4 sf)

Since 5Fe2+ MnO4-

1
Amt of MnO4- produced = 3.60 x 10-3 x
5
= 7.200 x 10-4 mol; (4 sf)

Amt of Mn2+ present in steel sample = Amt of MnO4- produced


=7.200 x 10-4 mol; (4 sf)

Mass of Mn2+ present in sample = 7.20 x 10-4 x 54.9


= 0.03950g ; (4 sf)

0.0395
% by mass = 100 = 3.95% ; (3 sf)
1.00
[Total: 7 marks]

[Turn over
5

3 The use of the Data Booklet is relevant to this question.


The elements of Period 2 and 3 form some of the most common compounds that we
come across.
(i) Define the term ionisation energy. [1]
Ionisation energy is the minimum amount of energy required to remove 1
mole of electrons from 1 mole of gaseous atom to form 1 mole of gaseous
ion.;

(ii) Describe and explain the difference in second ionisation energy of aluminium [2]
and silicon.
Electronic configuration of AI+: 1s2 2s2 2p6 3s2
Electronic configuration of Si+ :1s2 2s2 2p6 3s2 3p1 ;

The second ionisation energy of Si is lower than that of Al as the electron


removed is in the higher energy 3p orbital. Hence less energy is needed.;

(iii) Between Li+ and Be+, state and explain which ion would be deflected the [2]
most when they are passed through an electric field?

Li+ will be deflected more. ; (mark is not awarded if the explanation is


incorrect.)

This is because the angle of deflection is proportional to charge/ionic radius.


Since Li+ has a smaller ionic radius, the angle of deflection will be larger.;

(iv) Write balanced equations to show the reaction of chlorides of Na, Al and Si [4]
with water. State the pH of the solution formed.
NaCl(s) + aq Na+(aq) + Cl- (aq); pH = 7

AlCl3 + aq [Al(H2O)6]3+ (aq) + 3Cl- (aq)


[Al(H2O)6]3+ (aq) [Al(H2O)5(OH)]2+ (aq) + H+ ; pH = 3

SiCl4 + 2 H2O SiO2 + 4HCl; pH=2

; if all the pH are correct.


[Total: 9 marks]

4a Compound A, 2-bromobutane reacts with sodium hydroxide under two different sets
of reacts to give 2 different product mixtures Compound C (C4H10O) and Compound
D(C4H8).

(i) Describe the conditions necessary to form Compound C and Compound D [2]
respectively.
Compound C:
Aqueous NaOH, heat under reflux. ;

Compound D:
Alcoholic NaOH, heat under reflux. ;

[Turn over
6

(ii) Using your answers from (i), describe a further test to verify that the [2]
Compound A is 2-bromobutane.
After reacting the compound with aqueous NaOH to give compound C.
acidify the solution with HNO3., Add AgNO3(aq) to the compound. ;

The presence of a cream ppt indicates the presence of 2-bromobutane. ;

(iii) Compound D is a mixture of 3 isomers. Draw the displayed formulae of the [3]
3 isomers.
But-1-ene;
Cis-but-2-ene ;
Trans-but-2-ene ;

(All bonds must be fully shown)

(iv) The use of the Data Booklet is relevant to this question. [2]

2-chlorobutane can also react with NaOH to give compound C. Compare the
rate of reaction of 2-bromobutane and 2-chlorobutane with NaOH to give
Compound C.

Explain the difference in reactivity using information from the data booklet.
2-chlorobutane will react more slowly than 2-bromobutane. (mark is not
awarded if the explanation is incorrect.)

The bond energy of C-Cl in 2-chlorobutane is much higher than C-Br in 2-


bromobutane. Hence, it is much harder to break the bond.

(b) Archaeologists treat wood artifacts with ethane-1,2-diol to prevent drying and
preserve their quality.

(i) Why is it that wood artifacts in museum must not be left to dry out? [1]

Wood artifacts will crack and spoil if left to become too dry.;

(ii) Suggest the reagents and conditions required to convert ethene to ethane- [1]
1,2-diol.

Cold dilute alkaline KMnO4;


[Total: 11 marks]

[Turn over
7

SECTION B (40 MARKS)

Answer two of the three questions in this section on separate paper.

5(a) A solution containing propanoic acid and its salt, sodium propanoate acts as a buffer [3]
solution.
(i) What do you understand by the term buffer solution?
A buffer is a solution whose pH remains almost unchanged when a small
amount of acids or alkalis are added to it.;
(ii) Write ionic equations to show how this solution reacts with
I added H+ (aq) ions,
II added OH- (aq) ions.

I: H+ + CH3CH2COO- CH3CH2COOH ;
II: OH- + CH3CH2COOH CH3CH2COO- + H2O ;

(b) Propanoic acid is a monoprotic acid. A solution of 0.100 mol dm-3 propanoic acid has
a pH of 2.94. [5]
(i) Explain, with the aid of appropriate calculations, whether propanoic acid is a
strong or weak acid.

pH = 2.94; [H+] = 10-2.94 = 1.15 x 10-3 mol dm-3


Propanoic acid is a weak acid as [H+] is much lower than the concentration of
the Propanoic acid.

OR

[H+] [Propanoic acid].

(ii) Use the data given to calculate the value of Ka of propanoic acid.

Ka = [H+]2 / [HA] = (1.15 x 10-3)2 / 0.1 = 1.32 x 10-5 mol dm-3

(iii) Suggest a suitable indicator for the titration of propanoic acid with aqueous
sodium hydroxide. Explain your choice.
WA-SB titration: phenolphthalein.;
The working range of the indicator corresponds to the sharp pH change of
the solution at end point. ;

[Turn over
8

(c) [5]
H3C CH 2CH 2CH 2OH
H3C CH 2CH 2CH 2CN
CH Step I
CH

OH
CN
Compound A
+
KMnO 4/H
heat under reflux

Step II
H3C CH 2CH 2COCl
Compound B C

CH 3OH/ conc H 2SO 4


CH 3OH

Compound C

[Turn over
9

H3C CH 2CH 2CH 2OH


H3C CH 2CH 2CH 2CN
CH Step I
CH
alcoholic KCN
OH
Compound A CN
+
KMnO 4/H
heat under reflux

Step II
H3C CH 2CH 2COOH H3C CH 2CH 2COCl
C PCl 5 C

O O

CH 3OH/ conc H 2SO 4


CH 3OH

H3C CH 2CH 2COOCH 3


C

(i) State the IUPAC name for compound A.


Penta-1,4-diol.

(ii) State the reagents and conditions for Step I and Step II.
Step I: alcoholic KCN/ heat under reflux;
Step II: PCl5, PCl3, SOCl2;

(iii) Draw the structural formula of Compound B and Compound C.


H3C CH 2CH 2COOH
C

Compound B: O

H3C CH 2CH 2COOCH 3


C

Compound C: O

[Turn over
10

(iv) State the hybridisation, shape and number of sigma and pi bonds about Cx [2]
and Cy for the following molecule.
H
H3C CH 2CH 2CH 2Cy N
Cx

CN
Cx: sp3, 4 sigma ;
Cy; sp, 2 sigma, 2 pi ;

(d) A student wrote the following summary in his notebook. [5]

Atomic radius of period 2 elements increases across the period because of the
increase in nuclear charge. The increase in nuclear charge is due to an increase in
the number of electrons. There is negligible shielding effect across the period.
Hence, the outermost electrons are more strongly attracted towards the nucleus.

Ionic radius of elements increases across the period because of the greater effective
nuclear charge experienced by the outermost electron. The ionic radius of cation is
greater than the atomic radius of its element because of the isotopic nature of the
ion.

Identify the incorrect chemistry in his summary and substantiate your answer with
correct chemistry.
Atomic radius of period 2 elements decreases across the period because of the
increase in nuclear charge. ;

The increase in nuclear charge is due to an increase in the number of protons.;

There is negligible increase shielding effect across the period. Hence, the
outermost electrons are more strongly attracted towards the nucleus.;

Ionic radius of elements decreases across the period because of the greater
effective nuclear charge experienced by the outermost electron.

The ionic radius of cation is smaller than the atomic radius of its element because of
the decrease in number of electron shells for the cation.

6(a) The theoretical enthalpy of combustion values are given as follow:


Fuel Main Sources Enthalpy of Boiling
combustion/ point / oC
kJ g-1
Methane Petrochemical 55.6 -161.5
industry
Propane Petrochemical 50.3 -42.1
industry
Natural Gas
Octane Refined from crude 47.9 125.7

[Turn over
11

oil
Ethanol Hydration of ethane 29.7 78.3
Fermentation [10]

(i) Define the term enthalpy of combustion.


Enthalpy of combustion is the amount of energy released when 1 mole of a
compound is burnt in excess oxygen.

(ii)

The following results were obtained.

Initial mass of burner/ g 133.20


Final mass of burner/ g 132.05
Initial temperature of 25.0
water/oC
Final temperature of 45.5
water/oC
Determine in the enthalpy of combustion of ethanol in kJ g-1 based on the
above experimental conditions. (Heat capacity of water is 4.2 JK-1)

mcT = Hc x nethanol
133.2 132.05
(300)(4.2)(45.4-25.0) ; = Hc x ( );
12.0x 2 6x1.0 16.0

1.05
25704 = Hc x ( );
46.0

Hc= 1126.08 kJ mol-1;

= 1126.08/ (2x 12.0+5.0+ 17.0) kJ g-1;


= 24.5 kJ g-1;

(iii) State 2 assumptions that you have made in your calculations for (ii)
The density of water in the can has a density of 1g cm-3.;

The heat efficiency of the system is 100%/ There is no heat loss to the

[Turn over
12

environment/ All the heat that is produced by ethanol is transferred to the can
of water.

(iv) Explain the difference in boiling point between methane, octane and ethanol.

Ethanol has the highest boiling point because a lot of heat energy is required
to overcome the stronger hydrogen bonding between the ethanol molecules.;

Both octane and methane are non-polar molecules.


Octane has a higher boiling point compared to methane because it has a
larger electron cloud size due to its higher Mr. ;

More energy is required to overcome the stronger id-id interaction between


octane molecules than methane molecules.; Since hydrogen bonding is
stronger than id-id interaction, ethanol will have a higher boiling point than
octane.

(b) Ethanol is a popular starting reagent for many organic reactions.

(i) In 2 steps, suggest the reagents and conditions needed to convert ethanol to [3]
propanoic acid. Write equations to show the formation of the intermediate.
CH 3CH 2OH + KCN(alcoholic) CH 3CH 2CN ; + KOH

Reagents: Alcoholic KCN


Conditions: Heat under reflux ;

CH 3CH 2CN + 2H O + H+ CH 3CH 2COOH + NH +


2 4

Reagents: H2SO4
Conditions: Heat under reflux ;

(ii) Compare the difference in acidity between ethanol and ethanoic acid. [3]
Ethanoic acid is more acidic than ethanol. ;
The conjugate base CH3COO- for ethanoic acid is resonance stablised
where the negative charge on the ion is dispersed across the 2 oxygen.
Hence, ethanoic acid undergoes dissociation more easily to give H+ ion.;
The CH3CH2O- ion is destablised by the electron donating CH3CH2 group.
Hence, ethanol undergoes dissociation less easily to give H+ ion.;

(c) Describe a simple chemical test which will distinguish the following pairs of [4]
compounds from each other. You should include the reagents and conditions needed
and the expected observations for each compound in your answer.

(i) CH3CH(OH)CH2CH2CH3 and CH3CH2CH(OH)CH2CH3

Reagents/ conditions: aqueous alkaline iodine/ heat under reflux.;

Observations: CH3CH(OH)CH2CH2CH3 will give a yellow ppt of CHI3 and an


antiseptic smell while CH3CH2CH(OH)CH2CH3 will not.;

[Turn over
13

(ii) CH3CH2CH=CH2 and CH3CH=CHCH3


Reagents: acidified KMnO4, heat under reflux.;

Pass the gas produced from the reaction through Ca(OH)2.


Products from the oxidation of CH3CH2CH=CH2 will give a white ppt with
Ca(OH)2 while CH3CH=CHCH3 will not.;

7(a) Trichloromethane is hydrolysed by sodium hydroxide solution according to the


equation shown below.

2CHC/3 + 7OH- CO + HCO2- + 6Cl- + 4H2O

Some data was recorded in order to study the kinetics of the reaction.

Initial [CHCl3] Initial [OH-] Initial rate


(mol dm-3) (mol dm-3) (mol dm-3 s-1)
I 1.0 x 10-1 1.2 x 10-1 1.0 x 10-4
II 0.5 x 10-1 2.4 x 10-1 1.0 x 10-4
III 0.5 x10-1 4.8 x 10-1 2.0 x 10-4 [10]

(i) Explain the terms order of reaction and half life.


The order of reaction (or overall order of reaction) is the sum of the orders
of reaction with respect to the reactants. ;
OR
Assume Rate = k[A]m[B]n.
For the above rate equation, overall order of reaction = m+n.;

Half life is the time taken for the concentration of the reactants to drop to half
of its original concentration.;

(ii) Deduce the rate equation for the reaction between trichloromethane and
hydroxide ions.
1st order wrt to trichloromethane. ;

1st order wrt to hydroxide ions; (Workings must be shown.)

(iii) Calculate the initial rate of the reaction in experiment (I) at the time when half
of the hydroxide ions have reacted.
2
[CHCl3] left = 1.0 x 10-1 ( )(0.5x1.2 x 10-1) = 0.0829 mol dm-3 ;
7

In experiment (I)

[Turn over
14

1.0 10 4
k= 1 1
= 8.33 x 10-3 ;
(1.0 10 )(1.2 10 )

Rate = k (0.6 x 10-1) ( 0.0829)


= (8.33 x 10-3) (0.6 x 10-1) ( 0.0829) = 4.15 x 10-5 moldm-3s-1 ;

(v) Define catalyst and explain how the rate of the reaction will be affected when
a catalyst is used. Illustrate your answer with a Boltzmann distribution
diagram where appropriate.
A catalyst is a substance which increases the rate of a reaction by providing
an alternative reaction path of lower activation energy, without itself
undergoing any permanent chemical change.;

With the catalyst, the number of reactants with energy greater than this
lowered activation energy increases as observed by the larger area under
the curve. The rate of reaction increases as the frequency of collisions
between these molecules increases.;

Fraction of
molecules
Fraction of molecules with kinetic
energy Ea for catalysed reaction.

Fraction of molecules with kinetic


energy Ea for uncatalysed reaction.

Kinetic
E'a E
(Catalysed) energy
a
Must include legend for diagram;

(b) Compound X has the molecular formula C4H6O2. X does not dissolve in acid or alkali [10]
at room temperature. X reacts with LiAlH4 in the presence of dry ether to give V. X
reacts with hydrogen cyanide to form Y, C6H8O2N2. X also reacts with hot acidified
KMnO4 to produce Z, C4H6O3.

2.50g of Z is dissolved in 250cm3 of water. 25.0cm3 of this solution is neutralized by


12.25 cm3 of an aqueous solution of 0.100 mol dm-3 sodium carbonate giving W,
which has a high melting point.

Suggest the structural formula of V, W, X, Y and Z. Explain the chemistry involved.

[Turn over
15

X does not dissolve in X is neutral. X is a carbonyl compound or ester.;


acid or alkali at room
temperature.
X reacts with LiAlH4 in X is a carbonyl compound which has undergone
the presence of dry reduction.;
ether to give V.
X reacts with hydrogen X undergoes nucleophilic addition reaction with HCN to
cyanide to form Y, form Y.;
C6H8O2N2.
X also reacts with hot X contains aldehyde groups which can be further
acidified KMnO4 to oxidized to carboxylic acid.;
produce Z, C4H6O3.
2.50g of Z is dissolved Amount of Z that reacted with Na2CO3
in 250cm3 of water. 2.5 25
25.0cm3 of this solution = = 2.45 x 10-3
102.0 250
is neutralized by 12.25
cm3 of an aqueous
12.25
solution of 0.100 mol Amt of Na2CO3 that reacted = 0.1 x = 1.225 x 10-3
dm-3 sodium carbonate 1000
giving W. Z: Na2CO3 = 2: 1
Z is a monobasic acid.;

X contains 1 aldehyde group only.;


X contains another ketone group.
W, which has a high W is an ionic salt.;
melting point.

Max 5 for explanations.

[Turn over
16

X: Y:
O H H H OH H OH

C C C H C C C C CN
H
C H O H CN H H
H
H
Z: V:
H H OH H OH
H
C O H C C C C H
C
H
H H H H
H C H

C
O OH
H
H
C O
C
H
H C H

C
- +
W: O O Na .
* Displayed formula is not expected.

[Turn over
Class Reg Number

Candidate Name _____________________________

MERIDIAN JUNIOR COLLEGE


JC2 Preliminary Examination
Higher 1

_________________________________________________________________________

H1 Chemistry 8872/01
Paper 1 Multiple Choice 23 September 2010

50 min

Additional Materials: Data Booklet

_________________________________________________________________________

READ THIS INSTRUCTIONS FIRST


Write your name, class and register number in the spaces provided at the top of this page.

There are thirty questions in this section. Answer all questions. For each question, there are four
possible answers labeled A, B, C and D.

Choose the one you consider correct and record your choice in soft pencil on the OMR answer sheet.

Read the instructions on the OMR answer sheet very carefully.

You are advised to fill in the OMR Answer Sheet as you go along; no additional time will be given for
the transfer of answers once the examination has ended.

Use of OMR Answer Sheet


Ensure you have written your name, class register number and class on the OMR Answer
Sheet.
Use a 2B pencil to shade your answers on the OMR sheet; erase any mistakes cleanly.

Multiple shaded answers to a question will not be accepted.

For shading of class register number on the OMR sheet, please follow the given examples:
DIDATES
If your register number is 1, then shade 01 in the index number column.
If your register number is 21, then shade 21 in the index number column.

This document consists of 13 printed pages.

CopyrightMJCChemistryDepartment
Section A

For each question there are four possible answers, A, B, C, and D. Choose the one you
consider to be correct.

1 In a titration, a 30.0 cm3 sample of 0.05 mol dm-3 of the phosphoric acid was
found to require 15.00 cm3 of 11.22 g dm-3 solution of potassium hydroxide to
reach the endpoint.

Which of the following is the salt formed from the reaction?

A KH2PO4
B K2HPO4
C K3PO4
D KPO3

2 Which species are oxidized and reduced in the following reaction?

IO3 + 2I + 6H+ + 6Cl 3ICl2 + 3H2O

species oxidized species reduced


A IO3 I
B I, IO3 Cl
C I IO3
D H+, Cl IO3

3 Which one of the following ions has more protons than neutrons, and more
electrons than protons?

A He+
B OH
C H3O+
D 14
CO32

4 Which is the most likely shape of a molecule of dinitrogen pentoxide, N2O5?

A O B O O
C O O
D O
120o 90o

120o 120o 104.5o
O N N O O N N N N O N O N O
90 o 109.5 o O O O 107o
O O O O 180o O

CopyrightMJCChemistryDepartment
5 Which of the following molecule is the least soluble in tetrachloromethane,
CCl4?

A Br2
B NH3
C H2O
D CH3CH3

6 The following shows the energy profile diagram for the forward reaction and
the backward reaction of a particular reaction:

A + B 2C

The activation energy, Ea for the forward reaction is +57 kJ mol-1 while that for
the backward reaction, Ea is +85 kJ mol-1.

Ea

Ea


What is the enthalpy change of reaction for the forward reaction?

A +28 kJ mol-1
B - 28 kJ mol-1
C +56 kJ mol-1
D - 56 kJ mol-1

CopyrightMJCChemistryDepartment
7 Oxygen reacts with nitrogen monoxide in the equation shown.

O2(g) + 2NO(g) 2NO2(g)

In an experiment to investigate the effects of concentrations on the rate of the


reaction, the following results were obtained.

Expt [O2] / moldm-3 [NO] / moldm-3 Rate / moldm-3s-1


1 1.0 1.0 0.0007
2 2.0 1.0 0.0014
3 0.5 2.0 0.0007
4 1.5 2.0 x

The value of x is
A 0.0007
B 0.0014
C 0.0011
D 0.0021

8 The rate of a typical reaction is roughly doubled by raising the temperature by


10 C. Which of the following statements explains this observation?

A The activation energy is halved when temperature is raised by 10C.


B The kinetic energy of the molecules is doubled when temperature is
raised by 10C.
C The number of effective collision doubled when temperature is raised
by 10C.
D The number of particles is doubled when temperature is raised by
10C.

9 For which of the following reactions does Kc have the unit mol dm-3?
A CH3OH(l) + CH3COOH(l) CH3COOCH3(l) + H2O(l)
B 2SO3(g) 2SO2(g) + O2(g)
C CO(g) + H2O(g) CO2(g) + H2(g)
D 2NO2(g) N2O4(g)

CopyrightMJCChemistryDepartment
10 Deuterium is a heavier isotope of hydrogen. When pure H2O and pure D2O
are mixed, the following equilibrium is established,

D2O(l) + H2O(l) 2HDO(l) H = +55.7 kJ mol-1


Which of the following statement about the reaction is correct?

[ HDO]
A The Kc expression for the reaction is
[D2O][ H2O]
B Increase in temperature will shift the equilibrium position to the left.
C Adding more D2O will cause equilibrium position to shift right to
increase Kc.
D The O-D in D2O and O-H bonds in H2O have different bond energies
from the O-D and O-H bonds in HDO.

11 Which of the following mixtures is not an acid/conjugate base pair?

A H2O/OH-
B H2PO4-/HPO42-
C NaH/Na
D NH3/NH2-

12 Which of the following statements regarding enthalpy change is true?

A Hrxn is the same regardless of the reaction pathway taken as long as


the initial and final conditions are the same.
B Hc is the energy change when one mole of the substance is burnt in
oxygen at 1 atm and 298 K.
C Hn is the amount of energy released when 1 mole of acid reacts with
1 mole of base at 1 atm and 298 K.
D Hf is the energy change when 1 mole of substance is formed from its
constituent elements at 1 atm and 293 K.

13 Which of the following elements is expected to show the least tendency to


form covalent compounds?

A Barium
B Calcium
C Caesium
D Potassium

CopyrightMJCChemistryDepartment
14 Which element has the same oxidation number in all of its known
compounds?

A Fluorine
B Hydrogen
C Nitrogen
D Phosphorus

15 I, J and K are elements in Period 3. J has a smaller ionic radius than I, and K
has a more endothermic first ionisation energy than J.

What are elements I, J and K?

I J K
A P S Cl
B P Mg Al
C Al Mg Si
D Al S P

16 Compound X has the following structure:

CH3CH=CHCH(Br)CH=CHCH3

What is the total number of geometric isomers possible for this molecule?

A 1
B 2
C 3
D 4

17 Alkanes can be prepared from bromoalkanes by heating with sodium (in


ether) according to the equation:

2RBr + 2Na R-R + 2NaBr

Which alkanes will not be produced if a mixture containing equal amounts of


CH3CH2Br (bromoethane) and CH3CHBrCH3 (2-bromopropane) is used?

A CH3CH(CH3)CH(CH3)2
B CH3CH2CH(CH3)2
C CH3CH2CH2CH3
D CH3C(CH3)3

CopyrightMJCChemistryDepartment
18 Which of the following compounds form two organic products when it is
heated with acidified potassium manganate (VII) solution?

A CH2=CHCH2CH=CH2

CH3
CH3

CopyrightMJCChemistryDepartment
19 When 3-methyltoulene is treated with bromine in the presence of FeBr3, a
mixture of two mono-brominated isomers is formed.

What is the structure of these two isomers?


CH3

Br2
2 isomers
CH3 FeBr3
3-methyltoulene

CH3 Br CH2
Br

A +
CH3 CH3

CH3 CH3
Br

B +
CH3 CH3

Br Br

CH3 CH3
Br

C +
CH3 CH3

Br
CH3 CH3
Br

D +
CH3 Br CH3

20 Which of the following pairs react to form more than one organic product
under suitable conditions?

A C(CH3)4 + limited Cl2


B CH3CHBrCH2CH3 + KOH in ethanol
C CH3CH2CH2CH2Br+ KOH in ethanol
D CH2BrCH2CH2CH3 + aqueous NaOH

CopyrightMJCChemistryDepartment
21 Compound W is heated in a sealed tube with excess ammonia while
compound X is reacted with LiAlH4 in dry ether.

Which of the following pairs of W and X give the same product?

W X
A BrCH2CH2CH2OH HOOCCH2CN
B CH3CH2CH2OH CH3CH2CN
C (CH3)2CHCl (CH3)2CHCN
D C6H5CHBrCH2CH3 C6H5CH(CN)CH2CH3

22 Ethanol reacts explosively with sodium metal to form a salt, sodium ethoxide,
CH3CH2O-Na+.

Given that sodium ethoxide reacts in a similar manner as aqueous sodium


hydroxide when added to halogenoalkane, what should be the product formed
from the reaction between 2-bromopropane and aqueous sodium ethoxide?

A CH3CH=CH2
B (CH3)2CHOCH2CH3
C CH3CH2CH2OCH2CH3
D CH3CH2OCH2CH3

23 Arrange the following organic compounds in the increasing order of pKa value.

CH3CH2OH, CH3COOH, FCH2COOH, ClCH2COOH

A CH3COOH, FCH2COOH, ClCH2COOH, CH3CH2OH


B FCH2COOH, ClCH2COOH, CH3CH2OH, CH3COOH
C CH3CH2OH, CH3COOH, ClCH2COOH, FCH2COOH
D FCH2COOH, ClCH2COOH, CH3COOH, CH3CH2OH

CopyrightMJCChemistryDepartment
24 Which of the following reactions would not produce 1,4-dicarboxylic acid?

A O

+ Fehlings solution and heat followed by


acidification

CH3
O O

B CH2

+ hot acidified potassium manganate (VII)

CN

C CN

+ aqueous H2SO4, heat

CN

D H3C OH
CH

+ alkaline aqueous iodine and heat followed by


acidification
C CH3
O O

CopyrightMJCChemistryDepartment
25 Compound E is subjected to the following tests and the results were recorded
below.

Reagents and conditions Observations


Test 1: Acidified KMnO4 and heat Purple solution turns colourless
Effervescence of colourless gas that
turns limewater chalky

Test 2: Tollens reagent and warm Silver mirror formed

Which of the following could be compound E?

A CHO B COCH3 C CH(CH3)OH D CH2CHO

CH3 CH2CH3
CH2CH3 CHCH2

CopyrightMJCChemistryDepartment
For questions 26 to 30, one or more of the three numbered statements 1 to 3 may be
correct.

Decide whether each of the statements is or is not correct (you may find it helpful to
put a tick against the statements which you consider to be correct.)

The responses A to D should be selected on the basis of

A B C D
1, 2 and 3 are 1 and 2 only are 2 and 3 only are 1 only is
correct correct correct correct

26 Which of the particles has a half-filled p-subshell when an electron is lost?

1 C+
2 N
3 O

27 Water can undergo auto-ionization as follow:

H2O H+ + OH-

Kw at 25C Kw at 60C
1.00 x 10-14 9.5 x 10-14

Which of the following statements about this reaction is true?

1 The auto-ionization of water is endothermic


2 The pOH of water is 7.49 at 60C
3 Water is acidic at 60C

28 The diagram illustrates the energy changes for a set of reactions


H2 < 0
R S

H1 > 0 H3 > 0

T U
H4 > 0
Which of the following statement(s) is/are correct?

1 The enthalpy change from S to T is equal to H3 - H4 which is also


equal to H1 H2
2 The enthalpy change from U to R is exothermic
3 IH3I > IH2I
CopyrightMJCChemistryDepartment
The responses A to D should be selected on the basis of

A B C D
1, 2 and 3 are 1 and 2 only are 2 and 3 only are 1 only is
correct correct correct correct

29 Which of the following reactions will give similar side products?

1 Bromine and cyclohexene


2 Bromine and cyclohexane in the presence of uv light
3 Bromine and benzene in the presence of Fe

30 The pinacol rearrangement involves the reaction of a diol in acidic conditions


to form a carbonyl compound as shown below.

H3C CH3 O CH3


H+
HO C C OH C C CH3 + H2O

H3C CH3 H3C CH3

2,3-dimethylbutan-2,3-diol 3,3-dimethylbutan-2-one

Which one of the following structural formula is a product of the pinacol


rearrangement of 3,4-dimethylhexa-3,4-diol with the structure as shown
below?
H3C OH OH CH3

CH3 CH3

3,4-dimethylhexa-3,4-diol

O CH3
1
H3C CH3 CH3

O CH3
2 CH3

H3C CH3

O CH3 CH3

3
H3C CH3 CH3

CopyrightMJCChemistryDepartment
Paper 1 Answers

1 B 2 C 3 B 4 C 5 C
6 B 7 D 8 C 9 B 10 D
11 C 12 A 13 C 14 A 15 A
16 C 17 D 18 D 19 C 20 B
21 A 22 B 23 D 24 A 25 D
26 C 27 B 28 A 29 C 30 B

CopyrightMJCChemistryDepartment
Class Reg Number

Candidate Name _____________________________

MERIDIAN JUNIOR COLLEGE


JC2 Preliminary Examination
Higher 1

___________________________________________________________________
H1 Chemistry 8872/02
Paper 2 20 September 2010

2 hours
Candidates answer Section A on the Question Paper
Additional Materials: Data Booklet
___________________________________________________________________
READ THIS INSTRUCTIONS FIRST

Write your name, class and register number in the spaces at the top of this page.
This booklet contains Section A and B of your paper.

Section A

Answer all the questions.

Section B

Answer two questions on separate answer paper

Fasten your answers for Section B behind the


given Cover Page on Pg 17.

At the end of the examination, fasten all your work securely together.
The number of marks is given in brackets [ ] at the end of each question or part
question.
Examiners Use
Section A Q1 /8

Q2 / 11

Q3 /6

Q4 / 15

Section B / 40
Total / 80

This document consists of 16 printed pages.

Copyright MJC Chemistry Department


Section A

Answer all the questions in this section.

1(a) 0.08 mol of solid sulfur is burned in excess oxygen in a calorimeter to form
SO2 gas. The heat produced is used to heat up 815 cm3 of water at 28 C.
Given that 30% of the heat is lost to the surroundings and the enthalpy
change of combustion of sulfur, HC(S) is -297 kJ mol-1.

Calculate the final temperature reached for the water.


[2]

(b) SO2 is also an important precursor in making sulfuric acid in which the SO2 is
first converted to sulfur trioxide, SO3. It is possible to obtain SO3 by reacting
SO2 with oxygen but this reaction does not occur readily. Instead, the SO2 is
reacted with NO2 to form SO3 according to the equation:

SO2(g) + NO2(g) NO(g) + SO3(g)

Using the given enthalpy change values:

SO2(g) + O2(g) SO3(g) H1 = -98.9 kJ mol-1


S(s) + O2(g) SO2(g) H2 = -296.9 kJ mol-1

(i) Determine the enthalpy change of formation of SO3.

(ii) Use the Data Booklet and H1, calculate the bond energy of the S=O
bond, giving your answer to one decimal place and state the
assumption made in performing the calculation.
[3]

Copyright MJC Chemistry Department


(c) SO3 can undergo reaction with benzene to form benzenesulfonic acid via the
reaction below with concentrated sulfuric acid as the catalyst.

HSO3

Conc. H2SO4

Benezenesulphonic acid is used in the pharmaceutical field to produce a


variety of drugs. In an attempt to improve the efficiency of the drug production,
the relationship between the rate and the concentration of the reactants is
investigated in laboratory. The following two graphs were obtained.
Rate Rate

0 0
[Benzene]/mol dm-3 [SO3]2/mol2 dm-6

(i) From the graph, deduce the order of reaction with respect to benzene
and SO3 and hence write the rate equation for the reaction.

(ii) When the same concentration was used for both benzene and sulfur
trioxide, the initial rate of the reaction was measured to be
4.29 x 10-5 mol dm-3 s-1.

Given that the rate constant was 3.54 x 10-4 mol-2 dm6 s-1, calculate the
concentration for benzene used.
[3]

[Total: 8 marks]

Copyright MJC Chemistry Department


2 This question is on the properties of elements and its compounds in Period 3.

Magnesium is an element while silicon dioxide and phosphorus(V) chloride


are compounds of their respective elements. The three differ in their chemical
structures and bonding but all exist as crystalline solids.

(a) Describe these differences in their structure and bonding.


[3]

(b) Explain why phosphorus(III) chloride exists as a liquid but not phosphorus(V)
chloride.
[2]

(c) (i) Describe the reaction when magnesium is burnt in oxygen.

(ii) Indicate whether or not the product formed in (c)(i) reacts with
aq NaOH and aq HCl. Include all relevant equations in your answer.
[3]

Copyright MJC Chemistry Department


(d) Briefly describe the action of water on silicon dioxide and phosphorus(V)
chloride, writing balanced equations where appropriate.
[2]

(e) With reference to your answer in (c)(ii), suggest why the following reaction is
possible.
BeO + 2NaOH Na2Be(OH)4
[1]

[Total: 11 marks]

Copyright MJC Chemistry Department


3(a) Ozone layer depletes at a slow, steady rate per decade since 1969 due to the
use of aerosol and refrigerant containing chlorofluorocarbons (CFCs) such as
CF3CCl3. A healthy ozone layer is one which ozone, O3, is in dynamic
equilibrium with oxygen, O2, molecules in the upper atmosphere:

2O3 (g) 3O2 (g).

The equilibrium concentration of oxygen in the upper atmosphere over


America has been approximately constant at 650 DU (Dobson units).

In 1966, a scientist started to measure the concentration of ozone over


America for a period of 28 years and realized that a new equilibrium
concentration is reached.

[1 Dobson unit = 27 x 109 mol dm-3]

(i) Write the Kc expression for the equilibrium between O3 and O2 . Hence
calculate the Kc for the reaction in 1966 in Dobson unit.

(ii) From the graph, determine the new equilibrium concentration of ozone
after the use of CFCs started.

Copyright MJC Chemistry Department


(iii) Calculate the new Kc in 1994 in Dobson unit.

(iv) Suggest a possible reason for the change in value of Kc from 1970 to
1994.
[5]

(b) The rapid breaking down of the ozone layer was due to the production of the
reactive Cl radicals from ClO radicals . One of the intermediate reactions
involved is shown in the equation below:

ClO (g) + O (g) Cl (g) + O2 (g)

If this reaction is happening in the lower atmosphere instead of upper


atmosphere, will there be any changes in the equilibrium position for the
reaction? Explain your answer
[1]

[Total: 6 marks]

Copyright MJC Chemistry Department


4(a) Compound F can be converted to G via a series of reactions.

CH3 CH3

CH C NH2

CH2OH CH3
F G

(i) Suggest a synthesis route for the conversion of F to G.

H is a positional isomer of G that is also formed from the above process.

(ii) Suggest the percentage of H formed.


[5]

Copyright MJC Chemistry Department


(b) Compound P can be synthesised in the following manner.
CH3

H3C
C5H10
OH

Cold alkaline KMnO4


Step 3

N C4H8O

Step 1 Step 4
CH3 O

H3C OH Q

O
HCl (aq), heat
Step 2
CH3 O

H3C OH

OH
P

Give the structural formula for compounds N and Q. Suggest reagents and
conditions for steps 1 to 4:
[6]

Copyright MJC Chemistry Department


(c) Describe a simple chemical test to distinguish between the following pairs of
compounds.

(i) O O

O
A: and B:
O

(ii) O O

C: and D:

[4]

[Total: 15 marks]

Copyright MJC Chemistry Department


Section B

Answer two of the three questions in this section on separate answer paper.

5(a) The table below gives the bond energies and boiling points of the various
hydrogen halide and halogen bonds.

Bond Energy/kJ mol-1 Boiling Point/oC


H-F 562 ?
H-Cl 431 -85.1
H-Br 366 -66.4
H-I 299 -50.8
F-F 158 -188.1
Cl- Cl 244 -34.0
Br-Br 193 58.8
I- I 151 184.3

(i) Draw the dot-cross diagrams of HF and F2 molecules.

(ii) Suggest a reason for the decreasing bond energies from H-F to H-I.

(iii) With reference to the answer you have given in (i), suggest a reason
why the F-F bond is weaker than expected.

(iv) Suggest, with reasoning, a value for the boiling point of H-F.
[6]

(b) 0.3 dm3 of HF dissolved in 1 dm3 of water at r.t.p, produces a solution of pH 3.

(i) Define the term pH.

(ii) Explain whether HF is a weak or strong acid.


[3]

Copyright MJC Chemistry Department


(c) 0.1M NaOH solution is titrated against equal volumes of HCl and HF. The pH
values of the solutions were recorded and the following graphs were obtained.

pH
Graph 1 Graph 2

Vol of 0.1M NaOH added

(i) State with explanation which graph represents the titration of HF?

(ii) Explain qualitatively which hydrogen halide is of a higher concentration.

(iii) State with reason, a suitable indicator for the titration of HF.
[5]

(d) Given that the dissociation of CH3COOH is an endothermic process.

CH3COOH (aq) CH3COO- (aq) + H+ (aq) H > 0

Give an explanation for the following observations.

(i) The temperature rise recorded for the neutralization reaction between
HCl and NaOH is higher than between CH3COOH and NaOH.

(ii) When the experiment in (d)(i) is repeated at a higher temperature. The


temperature rise recorded for the neutralization reaction between
CH3COOH and NaOH is higher.
[4]

(e) A few drops of sulfuric acid were added to a solution containing equal
amounts of CH3COOH and CH3COO-Na+. With the aid of an equation, explain
the change in pH of the solution.
[2]

[Total: 20 marks]

Copyright MJC Chemistry Department


6(a) Beams of X2+, X3+ and S2- ions, travelling at the same speed from different
sources are subjected to an electric field as shown in the diagram below.

+++++++ P1 .
P2 .
.
.
P3

.
P4
_____ P5

(i) State the positions at which the X2+ and X3+ and S2- ions will hit the
detector.

(ii) Given that the angle of deflection for S2- and X2+ ions is 2o and
1.33o respectively and X also contains 25 neutrons.

Find the mass number of X and identify X.


[6]
(b) Sulfur can have molecules in various oxidation states, examples of which are
sulfur difluoride, SF2, and sulfur hexafluoride, SF6.

(i) Using the electron pair repulsion theory, predict the shape of SF4 and
SF6.

(ii) State whether the molecules are polar or non-polar.

(iii) The boiling point of SF4 is -38oC and SF6 is -64oC. Give an explanation
for the difference in boiling points.

(iv) SF2 reacts readily with F2 to give SF4 but OF2 is unable to react with F2
to form OF4. Explain why.
[8]

Copyright MJC Chemistry Department


(c) The rates of chemical reactions determine everything from explosions to how
efficiently food is digested. There are four basic factors that can affect the rate
of a chemical reaction. They are temperature, concentration/pressure,
physical state and catalyst.

(i) Explain what is meant by the term catalyst.

(ii) By using the concept of activation energy, explain why a catalyst


affects the rate of a chemical equation.

(iii) For the reaction of A + B D, it was discovered that the reaction can
proceed via the following mechanism:

Step 1: A+DC
Step 2: C + B 2D

State which substances are acting as the intermediate and as the


catalyst.

(iv) Given that the graphs representing the catalysed and uncatalysed of a
generic reaction are as followed:

[reactant]

uncatalysed

catalysed

[time]
Plot a [A] versus time graph for the reaction A + B D from (c)(iii).
[6]

[Total: 20 marks]

Copyright MJC Chemistry Department


7(a) On complete combustion, 1 g of an unknown organic compound E produces
2.955 g of CO2 and 0.671 g of H2O only.

(i) Calculate the mass of carbon present in 1 g of E.

(ii) Calculate the mass of hydrogen present in 1 g of E.

(iii) Calculate the mass of oxygen present in 2.955 g of CO2 and


0.671 g of H2O.

The mass of oxygen gas used for the complete combustion of E was
however, found to be 2.63g.

(iv) Account the difference from your answer in (a)(iii).

(v) Hence or otherwise determine the molecular formula of E.

(vi) Construct an equation for the complete combustion of E.


[7]

(b) A neutral compound T with molecular formula C5H10O3 reacts with


phosphorous pentachloride to form compound U, C5H9O2Cl. Compound U
gives compound V and CH3COONa when heated with aqueous sodium
hydroxide.

Compound V gives compound W, C4H7ON, on heating with ethanolic


potassium cyanide.

When W is heated with dilute sulfuric acid, a liquid X, C4H8O3, is obtained. On


heating X alone, a sweet smelling liquid Y, C4H6O2, is formed. Oxidation of X
gives a solid Z, C4H6O4, which reacts with one mole of sodium carbonate to
give one mole of carbon dioxide.

Deduce the structural formula of compounds T, U, V, W, X, Y and Z giving


your reasoning.
[13]

[Total: 20 marks]

Copyright MJC Chemistry Department


1

2010 Meridian Junior College JC2 H1 Chemistry Prelim Paper 2 Answers

Section A

1(a) Quantity of heat released from reaction = -297 x 0.08 = -23.76 kJ

Amount of heat absorbed by water = 0.7 x 23.76 = 16.632 kJ

Temperature changed = 16.632 x 1000/(815 x 4.18) = 4.88 K

Final temperature of water = 28 + 4.88 = 32.9 C

(b)(i) Hrxn = Hf (product) Hf (reactants)

Hrxn = Hf (SO3) [Hf (SO2) + Hf (O2)]


-98.9 = Hf (SO3) (-296.9) 0
Hf (SO3) = -396 kJ mol-1

(ii) Hrxn = BE (reactants) - BE (products)


-98.9 = [2 x BE(S=O) + BE(O=O)] 3 x BE(S=O)
-98.9 = BE(O=O) BE(S=O)
BE(S=O) = +98.9 + (496) = 346.9 kJ mol-1

The assumption is that the S=O bond has the same bond energy in both SO2
and SO3.

(c)(i) Order wrt benzene = 1


Order wrt SO3 = 2

Rate = k[Benzene][SO3]2

(ii) 4.29 x 10-5 = 3.54 x 10-4 x [Benzene]3 (since [Benzene] = [SO3])


[Benzene] = (4.29 x 10-5 / 3.54 x 10-4)1/3 = 0.495 mol dm-3

Copyright MJC Chemistry Department


2

2(a) Mg has a giant metallic structure. Held together by strong electrostatic forces
of attraction between Mg2+ cations and sea of delocalized electrons.

SiO2 has a giant molecular structure. Held together by strong and extensive
covalent bonding between the atoms in a giant 3-dimensional network.

PCl5 has a simple molecular structure. Held together by van der Waals forces
of attraction between PCl5 molecules.

(b) Size of molecule: PCl3 < PCl5


Size of electron cloud: PCl3 < PCl5
Degree of distortion of electron cloud : PCl3 < PCl5
Extent of intermolecular van der Waals forces: PCl3 < PCl5
Energy required to overcome: PCl3 < PCl5
Melting point: PCl3 < PCl5
Hence, PCl3 liquid, PCl5 solid.

(c)(i) Mg burns in oxygen very vigorously with a bright white flame to produce white
solid.

(ii) MgO + 2HCl MgCl2 + H2O

MgO + NaOH no reaction

(d) SiO2
Does not react with water due to its very stable giant molecular structure.

PCl5
Undergoes complete hydrolysis to form pH=2.0 / strongly acidic solution.

PCl5 + H2O POCl3 + 2HCl (cold)


or
PCl5 + 4H2O H3PO4 + 5HCl (hot)

(e) Be has a diagonal relationship / similar charge density with Al instead, due to
similar charge densities of Be2+ and Al3+, hence is amphoteric and will also
react with a base unlike MgO in part(c)(ii).

Copyright MJC Chemistry Department


3

3(a)(i) Kc = [O2]3/[O3]2

Kc = 6503/[300]2
= 3.05 x 103 DU

(ii) 120 DU

(iii) New Kc = (650)3/(120)2 = 1.91 x 104 DU



(iv) Global warming/There is a change in global temperature.

(b) There is no change in the equilibrium of the reaction because the number of
moles of gaseous reactants is the same as the number of moles of gaseous
product hence the equilibrium position will not change with the change in
pressure.

4(a)(i)
CH3 CH3

CH C NH2

CH2OH CH3

conc. H3PO4 / Excess NH3 (g), heat in


conc H2SO4 / Al2O3 sealed tube
and heat

CH3 CH3

C C Br
HBr (g)/in CCl4
CH2 rtp CH3

(ii) Any value < 50%

(b)
CH3 CH3
CN
H3C OH H3C

OH N OH Q

Step 1: KMnO4 in dilute H2SO4; heat


Step 2: NaBH4 in ethanol, rtp
Step 3: K2Cr2O7 in dilute H2SO4 heat with distillation
Step 4: HCN, trace aq NaOH, cold

Copyright MJC Chemistry Department


4

(c)(i) Test : Add 2,4-DNPH with heating to the two unknown in separate test tubes.

Observation: Orange crystals of hydrazone is formed for the test-tube


containing A. No orange ppt will be formed for the test-tube containing B.

(ii) Test: Add aqueous alkaline iodine with heating to the two unknown in
separate test-tubes.

Observation & deduction: Yellow ppt of CHI3 formed for the test-tube
containing C. No yellow ppt of CHI3 formed for the test-tube containing D.

Copyright MJC Chemistry Department


5

Section B

5(a)(i)
H F F F

(ii) The increasing H-X bond length from H-F to H-I.

(iii) The repulsion between the lone pairs of the two F atoms weakens the bond.

(iv) Any value > -50.8 oC (but must be able to exist as gas at r.t.p)

HF has strong intermolecular hydrogen bonding which requires more energy


to overcome as compared to the weaker intermolecular Van Der Waals forces
of attraction for HCl, HBr and HI. Hence higher boiling point.

(b)(i) pH = log10[H+]

0.3
(ii) [HF]= = 0.0125 mol dm-3
24

[H+] = 1 x 10-3 mol dm-3

Since [H+] << [HF] or [H+] partially dissociated


HF is a weak acid.

(c)(i) Graph 2. The salt formed, NaF, contains a conjugate base, F-. Therefore the
equivalent point of the titration will be > 7.

(ii) Vol of NaOH added No. of moles of HX

Since Vol of NaOH added to titrate HF > Vol of NaOH to titrate HCl
No. of moles of HF > No. of moles of HCl
As volume of HF and HCl is the same
[HF] > [HCl]
(iii) Phenolphthalein

The pH transition range of phenolphthalein lies within the rapid pH change


over the equivalence point.

(d)(i) Some of the heat released by the neutralization between the weak acid,
CH3COOH and NaOH is absorbed to further dissociate CH3COOH. Thus the
temperature rise is higher for the reaction between HCl and NaOH as no such
heat is absorbed.

(ii) By Le Chateliers Principle, at higher temperature, endothermic reaction is


favoured. Therefore the equilibrium position shifts to the right to absorb heat.

Copyright MJC Chemistry Department


6

The dissociation of CH3COOH is greater thus less heat is absorbed resulting


in a larger temperature rise recorded.

(e) When a small amount of H2SO4 is added,

CH3COO- + H+ CH3COOH

H+ is removed as CH3COOH.
[H+] is slightly changed and pH of the solution remains fairly constant.

6(a)(i) X2+ - P4 X3+ - P5 S2- - P1/P2


q 2
(ii) ratio of S2- = = 0.0625
m 32
q
let the ratio of X2+ be y
m
since
q
m
angle of deflection
q
of S2-
0.0625 2
mq = =
of X2+ y 1.33
m
2
y = 0.0416 =
Mass no. of X
Mass no. of X = 48.0
Proton no. of X = Mass no. no. of neutrons
= 48 25
= 23

X is Vanadium.

(b)(i) SF4 has 4 bond pairs and 1 lone pair around the central S atom .
To minimize repulsion and maximize stability , the 5 electron pairs are
directed to the corners of a trigonal bipyramidal
But lone pair bond pair repulsion > bond pair bond pair repulsion
Shape is distorted tetrahedral

SF4 has 6 bond pairs and 0 lone pair around the central S atom
To minimize repulsion and maximize stability, the 6 electrons pairs are
directed to the corners of a octahedron
Shape is octahedral

(ii) SF4 is polar and SF6 is non-polar

(iii) SF4 has stronger permanent dipole permanent dipole intermolecular forces
of attraction which requires higher amount of energy to overcome as
compared to the weaker induced dipole induced dipole intermolecular forces
of attraction of SF6.

Copyright MJC Chemistry Department


7

(iv) O is in Period 2 and does not have energetically available and accessible d
orbitals to expand beyond the octet structure

(c)(i) A catalyst is a substance that increases the rate of a chemical reaction


without itself being consumed.

(ii) The catalyst provides an alternative reaction pathway with a lower activation
energy. Thus there are more reactant particles with energy > Ea and the
frequency of effective collisions increases. Since rate is proportional to
frequency of effective collisions, rate increases.

(iii) Intermediate C
Catalyst D

(iv) [A]

[time]

Copyright MJC Chemistry Department


8

2.955
7(a)(i) 12 = 0.806 g
12+2(16)

(ii) H2O 2H

0.671
21 = 0.0746 g
2+16

(iii) Mass of O = (2.955 + 0.671) (0.806 + 0.0746)


= 2.75 g

(iv) E contains oxygen atoms.

(v) Mass of O in 1 g E = 2.75 2.63 = 0.120 g

C H O
No. of mol 0.0672 0.0746 0.00750
Mole ratio 9 10 1

Molecular formula: C9H10O

(vi) C9H10O + 11 O2 9 CO2 + 5 H2O

(b) T is neutral
T is not a carboxylic acid

T undergoes substitution with PCl5 to form U,


T is an alcohol

U undergoes basic hydrolysis with hot NaOH(aq) to form CH3COONa and V,


U is an ester
V is an alcohol

V undergoes substitution with KCN to form W


W contains a nitrile group
V is a halogenoalkane group

W undergoes acid hydrolysis with dilute H2SO4 to form X.


X is a carboxylic acid

X undergoes self-esterification on heating to form Y.


Y is an ester
X is an alcohol

X undergoes oxidation to form Z


Z undergoes neutralization/acid-carbonate reaction with sodium carbonate.
X is a primary alcohol.
Z is a dicarboxylic acid.

Copyright MJC Chemistry Department


9

Structure of T is HOCH2CH2CH2OCOCH3

Structure of U is ClCH2CH2CH2OCOCH3

Structure of V is ClCH2CH2CH2OH

Structure of W is HOCH2CH2CH2CN

Structure of X is HOCH2CH2CH2COOH

O
Structure of Y is

Structure of Z is HOOCCH2CH2COOH

Copyright MJC Chemistry Department


NATIONAL JUNIOR COLLEGE
SH2 PRELIMINARY EXAMINATIONS
Higher 1

Class Registration No
Candidate Name:

CHEMISTRY 8872/01
Paper 1 Multiple Choice Fri 17 September 2010
50 minutes

INSTRUCTIONS TO CANDIDATES
Do not open this booklet until you are told to do so.
Write your name, class and registration number in the spaces provided at the top of
this page as well as on the optical mark sheet provided.

Shade the index number in a 5 digit format (12345) on the optical mark sheet.
Row 1: Year of admission
9 for SH2 7 for IP
Row 2: Level on admission
5 for SH2 3 for IP

Rows 3, 4 and 5: Last 3 digits of registration number

Eg: Reg No 0905648 = 95648 Reg No 0703093 = 73093

There are 30 questions in this paper. Answer ALL questions.


For each question, there are four possible answers, A, B, C and D. Choose the one you
consider correct and record your choice in 2B pencil in the optical mark sheet.
Submit both the optical mark sheet as well as the question paper at the end of the
examination.
Section A

This question paper consists of 11 printed pages (including this page)

1
For each question in this section, there are four possible answers A, B, C and D.
Choose the one you consider to be correct.

1. Which of the following does not contain 4 moles of hydrogen atoms at


standard temperature and pressure?

A 22.4 dm3 of methane


B 36 g of water
C 2 moles of hydrogen gas
D 28 g of ethane

2. 1.08 g of an oxide MO of a metal M was dissolved in excess sulfuric acid.


25.0 cm3 of 0.10 mol dm3 potassium dichromate solution was required to
oxidize M2+ to M3+.

What is the relative atomic mass of M?

A 52
B 56
C 64
D 72

3. An element Y forms an ion Yn+. The atom Y and the ion Yn+ have the same

A size
B number of protons
C number of electrons
D electronic configuration

4. Which one of the following shows the electronic arrangement of the strongest
reducing agent?

A 1s2 2s2 2p5


B 1s2 2s2 2p6 3s2
C 1s2 2s2 2p6 3s2 3p5
D 1s2 2s2 2p6 3s2 3p6 4s2

2
5. The graph below shows the ionic radii of five consecutive elements, P, Q, R,
S and T in Period 3.

Ionic radius
T

P
Q
R
S

Element

All of the following statements are true except

A Ions of P to S are isoelectronic.


B Elements P to S exist as giant structures.
C There are 3 unpaired electrons in Element T.
D Element P has higher melting point than Element Q.

6. Which of the following pairs of compounds have the same shape?


A AlCl3 and PCl3
B NH3 and BCl3
C H2O and H2Se
D CO2 and SO2

7. Which of the following statements about the properties associated with ionic
and covalent bonds is correct?

A Ionic bonds and covalent bonds may both occur in the same
compound.
B All covalent compounds cannot conduct electricity in their aqueous
state.
C The only covalent compounds with high boiling points are those in
which hydrogen bonds exist.
D Any covalent compound that contains both oxygen and hydrogen in its
molecule forms hydrogen bonds.

3
8. Tetrafluoroethene, CF2=CF2, is produced by the reaction

2CHClF2(g) CF2=CF2(g) + 2HCl(g) Hreaction

Bond energies (in kJ mol-1) involved in the reaction are given below:

C-H p
C-Cl q
C-F r
C=C s
H-Cl t

Which of the following calculations will give the value of Hreaction?


A q s 2r p
B s + 2t 2p 2q
C 2p + 2q s 2t
D 2p + 2q s 2t 2r

9. Which of the following definitions is not correct?


A First ionization energy is the energy taken in when one mole of
valence electron is removed from one mole of its gaseous atoms to
form one mole gaseous singly-charged ions.
B Lattice energy is the heat taken in when one mole of ionic solid is
formed from its gaseous ions under standard conditions.
C The standard enthalpy change of neutralization is the heat evolved
when one mole of water is formed from the reaction between an acid
and an alkali under standard conditions.
D The standard enthalpy change of combustion is the heat evolved
when one mole of a substance is completely burnt in excess oxygen
under standard conditions.

10. Two first order reactions were started at the same time. Reaction A had an
initial concentration of 1 mol dm3 and a half-life of 20 minutes. Reaction B
had an initial concentration of 4 mol dm3 and a half-life of 10 minutes. After
how long would the concentration of the reactants in the two reactions
become equal?

A 15
B 35
C 40
D 50

4
11. A researcher carried out a series of experiments to determine the order of
reaction with respect to C and D. The results are summarised in the table
below.

C+D E

Experiment [C] / mol dm3 [D] / mol dm3 Rate / mol dm3 s1
1 0.5 0.5 0.02
2 1.0 0.5 0.04
3 0.5 1.0 0.02

What is the unit for the rate constant, k in the overall rate law?

A s1
B mol1 dm3
C mol1 dm3 s1
D mol2 dm6 s1

12. The equation below gives the decomposition of HI.

2HI(g) H2(g) + I2(g)


At time t0, 0.500 moles of HI was placed in a 250 cm3 vessel at 445C and
equilibrium was established at time t1 as shown in the diagram below. At time
t2, the system was subjected to a change and equilibrium was re-established
at time t3. Which of the following would give rise to the changes observed at
time t2?

No of moles

0.500
H2
H2, I2
I2

HI
HI

t0 t1 t2 t3 time

A Increasing the temperature of the system.


B Removal of H2 gas as soon as it is formed.
C Addition of more HI at the same temperature.
D Addition of sodium thiosulfate solution at the same temperature.

5
13. A major source of carbon monoxide comes from the reaction between steam
and carbon.
H2O(g) + C(s) H2(g) + CO(g) H = +ve

Which of the following conditions favour the highest proportion of CO in the


equilibrium mixture?

Temperature Pressure
A Low High
B Low Low
C High Low
D High High

14. Which one of the following pairs will give a buffer solution when the two
solutions are mixed?

A 50 cm3 of 0.10 mol dm3 NaHCO3 and 100 cm3 of 0.10 mol dm3 HCl.
B 50 cm3 of 0.10 mol dm3 NaHCO3 and 100 cm3 of 0.10 mol dm3
NaOH.
C 50 cm3 of 0.10 mol dm3 NaHCO3 and 100 cm3 of 0.10 mol dm3
NaOH.
D 100 cm3 of 0.10 mol dm3 NaHCO3 and 100 cm3 of 0.10 mol dm3
H2CO3.

15. The curve below shows a titration reaction between 10 cm3 of a weak acid
HClO of concentration 0.5 mol dm-3 and NaOH of concentration 1.0 mol dm-3.
pH

Vol of NaOH added / cm3

Which of the following statements is true about the titration reaction?


A The chemical species present at point W are ClO, NaOH and H2O.
B The pH value at equivalence point is 7.
C The volume of NaOH required for complete reaction is 10 cm3.
D A suitable indicator for this titration is phenolphthalein, which changes
from colourless to pink.

6
16. Which of the following statements about Period 3 elements is true?

A The melting points decrease in the order P > S > Cl > Ar.
B On heating in air, sulfur burns with a blue flame to give colourless
SO2 gas.
C The electrical conductivity increases from sodium to silicon.
D Aluminium burns in oxygen to form a covalent oxide Al2O3.

17. A mixture of 10 cm3 of methane and 10 cm3 of ethane was sparked with an
excess of oxygen. After cooling to room temperature, the residual gas was
passed through aqueous potassium hydroxide of concentration
0.100 mol dm3. What is the volume of aqueous potassium hydroxide required
to react with the residual gas?

A 12.5
B 13.4
C 25.0
D 30.0

18. Which of the following substances has the highest boiling point?

A CH3(CH2)2CH3
B (CH3)2CHCH3
C CH3(CH2)3CH3
D (CH3)4C

19. How many different alkenes are formed when 3-methylbutan-2-ol reacts with
excess concentrated sulfuric acid at 170oC?

A 2
B 3
C 4
D 5

20. Oxidation of an alkene X gives a diol which cannot be further oxidized. Which
of the following could be X?

A CH3CH=CH2
B CH3CH=CHCH3
C CH3CH=C(CH3)2
D (CH3)2C=C(CH3)2

7
21. Which one of the following statements explains best why fluoroalkanes are
the least reactive halogenoalkanes?

A Fluorine is much more electronegative than carbon.


B The F ion is the most stable halide ion.
C The CF bond is the strongest carbonhalogen bond.
D The CF bond is the most polar carbonhalogen bond.

22. Which one of the following is formed when an excess of CH3Br reacts with
(CH3CH2)2NH?

A
CH3CH2 N CH2CH3

CH2CH3

B
CH3CH2 N CH3

CH3

C
CH3

CH3CH2 N CH3 Br

CH3

D
CH3

CH3CH2 N CH2CH3 Br

CH3

23. The Williamsons ether synthesis involves a nucleophilic substitution of an


organic halogen compound. Which of the following the reagents can be used
for the synthesis of methoxyethane, CH3CH2OCH3 starting from CH3CH2Br?

A NaOH in CH3OH
B CH3OH with Na
C CH3CH2OK+
D CH3OH

8
24. Which of the following statements about the reaction scheme is incorrect?

Step (i) Step (ii) Step (iii)


CH3COOH ClCH2COOH NCCH2COOH HOOCCH2COOH
J K L M

Step (iv)

H2NCH2CH2COOH
N

A Step (i) could be achieved using chlorine in the presence of heat.


B Step (ii) could be achieved using aqueous potassium cyanide.
C A reduction reaction is involved in the reaction scheme.
D An acid hydrolysis reaction is involved in the reaction scheme.

25. The scheme shows the formation of phenylmethanol from the starting
compound, 1-phenylpropan-1-ol

CH(OH)CH2CH3 CH CHCH3 COOH CH2OH

Stage 1 Stage 2 Stage 3

What are the reagents required for each stage?

Stage 1 Stage 2 Stage 3


A Excess concentrated Acidified K2Cr2O7 ; heat H2(g) / Pt(s)
H2SO4; heat
B Excess concentrated Acidified K2Cr2O7 ; heat LiAlH4 in dry ether
H2SO4; heat
C Excess concentrated Acidified KMnO4 ; heat H2(g) / Ni(s) ; heat
H2SO4; heat
D Al2O3 ; heat Acidified KMnO4 ; heat LiAlH4 in dry ether

9
Section B

For each of the question in this section one or more of the three numbered
statements 1 to 3 may be correct.

Decide whether each of the statements is or is not correct (you may find it helpful to
put a tick against the statements which you consider to be correct).

The response A to D should be selected on the basis of

A B C D
1, 2 and 3 are 1 and 2 only 2 and 3 only 1 only is
correct are correct are correct correct

26. In which pair is the bond angle in the first species larger than that in the
second species?

1 H2O, H2S
2 CO2, CS2
3 CH4, CCl4
(D)

27. The equilibrium constant Kc for the reaction

Q(s) + R(g) U(g) + 2V(g)

is found to vary with temperature as shown in the diagram below.

Which of the following conclusion(s) can be drawn from this information?


1 The production of U and V is an endothermic process.
2 The equilibrium mixture contains a higher proportion of R at higher
pressures.
3 The addition of a catalyst will increase the value of the equilibrium
constant Kc.
(B)

10
28. Which of the following shows the correct trend across Period 3 elements?

1 Electronegativity of the elements increases.


2 Ionization energy of the elements increases.
3 Melting point of the oxides increases.
(B)

29. Ionone, shown below, is a compound responsible for the smell of raspberries.

CH3 CH3
CH CHCOCH3

CH3

The type of reactions that ionone can undergo include

1 Electrophilic substitution
2 Nucleophilic addition
3 Reduction
(C)

30. What are the functional groups present in the naturally occurring compound,
lovastation?

CH3
CH2
OH
CH O
CH3

CH3 O O O
CH2
CH2

lovastatin

1 alkene
2 ketone
3 tertiary alcohol
(D)

End

11
NATIONAL JUNIOR COLLEGE
PRELIMINARY EXAMINATIONS
Higher 1

CANDIDATE
NAME

SUBJECT REGISTRATION
CLASS NUMBER

CHEMISTRY 8872/02
Paper 2 Structured Questions 13 Sept 2010

2 hours

READ THE INSTRUCTION FIRST For Examiners


Write your subject class, registration number and name on all Use
the work you hand in.
Section A
Write in dark blue or black pen on both sides of the paper.
You may use a soft pencil for any diagrams, graphs or rough A1
working.
Do not use paper clips, highlighters, glue or correction fluid. A2

Section A A3
Answer all questions.
A4
Section B
Answer any two questions. A5

At the end of the examination, fasten all your work securely Section B
together. B1
The number of marks is given in brackets [ ] at the end of
each question or part question. B2

B3

Total

This question paper consists of 14 printed pages (including this page).


2

Section A
Answer all the questions in this section in the spaces provided.

1 Butanoic acid, CH3CH2CH2COOH (Mr = 88.0), also known as butyric acid have
important uses in the industry, such as perfume and food additives.

In an experiment, a 10.0 cm3 sample of butanoic acid of concentration 0.025


mol dm3 was titrated against 0.010 mol dm-3 barium hydroxide, Ba(OH)2.

(a) Calculate the volume of barium hydroxide, Ba(OH)2 required for the
neutralization of butanoic acid in the experiment.

[2]

(b) Write an expression for the acid dissociation constant, Ka for butanoic acid.
Hence, determine its Ka value given that the initial pH of butanoic acid is 3.2.

[2]

[Turn Over
3

(c) Another experiment was conducted to determine the enthalpy change of


reaction between butanoic acid and barium hydroxide.

Given the following data, determine the enthalpy change of reaction for the
above reaction.

Volume of 3.00 mol dm-3 barium hydroxide = 30.0 cm3


Volume of 2.00 mol dm-3 butanoic acid = 40 cm3
Temperature rise = 14.9 C
Specific heat capacity of water = 4.18 J g-1 K-1

[2]

(d) Predict and explain how the magnitude of the temperature rise will change
when the reaction in (c) was repeated using 2-chlorobutanoic acid with the
same volume and concentration as butanoic acid.

[2]

[Total: 8]

[Turn Over
4

2 The first seven ionisation energies of Element A from Period 2 are as follows:

No of electrons lost 1st 2nd 3rd 4th 5th 6th 7th

Ionisation energy/ 1410 2866 4576 7473 9442 53250 64340


1
kJ mol

(a) Deduce the group that Element A belongs to in the Periodic Table. Explain
your answer.

[2]

(b) State the identity of Element A and hence write its electronic configuration.

[1]

(c) Magnesium reacts with A to form an ionic compound of the formula Mg3A2.
State and explain which one of the ions in the compound would have a larger
ionic radius.

[2]

[Total: 5]

[Turn Over
5

3 (a) The boiling points of some compounds are given in the table below.

Compounds Mr Boiling Points / C


Lithium chloride 42.5 1382
Lithium iodide 134 462
Ammonia 17 33
Hydrogen fluoride 20 19.5
Iodine 254 185

(ii) Explain the difference in the boiling point between

I. lithium chloride and lithium iodide

[2]
II. ammonia and hydrogen fluoride

[2]
III. iodine and hydrogen fluoride.

[2]

[Total: 6]

[Turn Over
6

4 (a) An organic compound B of the molecular formula C4H6O, undergoes a series of


reactions as shown by the table below. State the deductions for each of the following
reactions.

Test Deductions
(I) Brown precipitate is observed when
cold dilute alkaline KMnO4 is added
to 1 cm3 of a solution containing B in
a test-tube.

(II) Silver mirror is observed when


Tollens reagent is added to 1 cm3 of
a solution containing B in a test-
tube.

(III) 0.1 mole of compound B will react


with 4.8 dm3 of hydrogen gas in the
presence of a platinum catalyst at
room temperature and pressure.

[4]

(b) Compound B exhibits geometric isomerism. Together with the information obtained in
(a), give the structural formula of B.

[1]

[Turn Over
7

(c) State the structural features present in compound B that enables it to exhibit
geometric isomerism. Hence draw the geometric isomers of B.

[2]

(d) Compounds C and D have the same molecular formula C4H6O. Both gives the same
positive test as shown in (a). C and D are structural isomers of one another but they
do not exhibit geometric isomerism. Draw the structures for compounds C and D.

[2]

(e) Suggest a test-tube reaction to differentiate compound B from either C or D. State the
reagents and conditions used and the observations for each of the compounds.

[2]

[Total: 11]

[Turn Over
8

5 Methane, one of the main components in natural gas is an important fuel used
worldwide. It has an enthalpy change of combustion of 890 kJ mol1.

Methane is being depleted at an alarming rate. The current worldwide consumption of


methane is about 4.9 x 109 m3 per year. Thus, alternative sources of methane need to
be sought to sustain the worlds consumption.

One of the alternative sources of methane comes in the form of methane hydrate. It is
also known as methane ice or fire ice in which a large amount of methane is trapped
within the crystal structure of ice. Part of the crystal structure of ice is shown below.

Larger hydrocarbon molecules such as ethane and propane can also form hydrates,
although as the molecule length increases (butanes, pentanes), the tendency to form
hydrates decreases.

Significant deposits of methane hydrate are found under sediments on the ocean floors
and the arctic permafrost of the Earth, about 500 to 2000 m below the ocean surface. It
is estimated that there are 11.3 x 1018 m3 of methane hydrate available worldwide.

Thus, international research and exploration by governmental and industrial entities


evaluated that methane hydrates could be important primary energy sources in the
future due to the depletion of fossil fuels. To utilize the methane in methane ice,
excavators would likely melt the ice underground first, and then extract the methane gas.
When brought to the surface, it is estimated that 1 m3 of methane hydrate contains about
180 m3 of methane, measured at room temperature pressure.

Although methane hydrate is plentiful throughout the world, there are many key
challenges being faced in the utilizing of methane hydrate as a source of methane. With
these challenges in mind, China's Ministry of Land and Resources estimated that the
country could begin using its combustible ice within 10 to 15 years, joining other
countries in methane hydrate exploration.

Water gas, which consists of an equimolar mixture of hydrogen and carbon monoxide is
another type of fuel used in the industries. The enthalpy of combustion of hydrogen and
carbon monoxide are 242 kJ mol1 and 283 kJ mol1 respectively.

[Turn Over
9

Water gas could be produced according to the following equation.

H2O (g) + C(s) H2 (g) + CO (g)

It is a useful product but requires careful handling because of the risk of carbon
monoxide poisoning.

(a) Calculate the amount of energy that can be produced from the methane extracted
from 1 m3 of methane hydrate.

[2]

(b) Calculate the volume of methane that can be obtained at room temperature and
pressure from the methane hydrate available worldwide.

[1]

(c) Based on the current worldwide consumption of methane, calculate how many
years the methane obtained from methane hydrate could sustain the world.

[1]

(d) Suggest a possible challenge faced by the world in methane hydrate exploration

[1]

[Turn Over
10

(e) Suggest a reason why hydrocarbons form hydrates less readily as the molecule
length increases.

[1]

(f) Calculate the amount of energy that can be produced from 1 m3 of water gas.

[2]

(g) Suggest two reasons why methane is used as a preferred fuel compared to water
gas in industries.

[2]

[Total:10]

[Turn Over
11

Section B

Answer two of the following three questions. Answer these questions on separate answer paper.

1 Butane can react with Cl2(g) to form monochlorinated butane.

(a) (i) Name the type of reaction and state the condition required for the reaction to occur.

(ii) Draw the structures of all the possible monochlorinated butanes formed.

(iii) State and explain the ratio in which they are formed. [6]

(b) The rate for the reaction between chlorobutane and aqueous sodium hydroxide was
studied using two solutions of different initial concentrations of sodium hydroxide and the
same initial concentration of chlorobutane. The results are as shown below:

Time / min Experiment 1 Experiment 2


[Chlorobutane] / mol dm-3 when [Chlorobutane] / mol dm-3
[NaOH] = 0.20 mol dm-3 when [NaOH] = 0.30 mol
dm-3
0 0.0100 0.0100
40 0.0079 0.0070
80 0.0062 0.0049
120 0.0049 0.0034
160 0.0038 0.0024
200 0.0030 0.0017

(i) Plot these data on suitable axes.

(ii) From the graphs, deduce the order of reaction with respect to chlorobutane and
aqueous sodium hydroxide.

(iii) Hence, deduce the rate equation and calculate a value for the rate constant, stating
its units.
[6]
(c) With the aid of a relevant diagram, explain how an increase in temperature will increase
the rate of the above reaction.
[3]

(d) The table below shows the pH values of Period 3 chlorides in water.

Period 3 chloride pH of a 1.0 mol dm-3 solution in


water at 25oC
NaCl 7.0
MgCl2 6.5
AlCl3 3.0
SiCl4 2.0
PCl5 1.0 - 2.0

[Turn Over
12

(i) By writing relevant balanced equations, explain the difference in pH of aqueous


solutions of NaCl, AlCl3 and PCl5.

(ii) Using the information given in the above table and the Data booklet, predict with
explanations, the pH value of a 1 mol dm-3 solution of BeCl2.
[5]

[Total: 20]

[Turn Over
13

2 At room temperature, dinitrogen tetraoxide dissociates and forms an equilibrium with nitrogen
dioxide in a closed vessel, as shown in the equation below.

N2O4(g) 2NO2(g) )

Pale yellow Dark brown

(a) (i) Draw the dot-cross diagram of N2O4 & NO2.

(ii) Hence state and explain if the forward reaction is endothermic or exothermic. [3]

(b) (i) At 1400C, 3 moles of N2O4 was placed in a 2 dm3 vessel and allowed to reach
equilibrium. At equilibrium, 60% of the N2O4 dissociated into NO2. Calculate the
equilibrium constant, Kc at this temperature, stating its units.

(ii) State and explain the expected colour of the equilibrium mixture in the vessel when
(I) the volume is increased to 4 dm3
(II) the vessel is immersed in a beaker containing hot water.

[7]

(c) (i) At 1400C, the activation energy for the forward reaction is 150 kJ mol-1 and that for
the reverse reaction is 91.3 kJ mol-1. Sketch a reaction pathway diagram for this
reaction.

(ii) Hence determine the value of the enthalpy change of dissociation of N2O4.

(iii) In the presence of a catalyst, the activation energy for the reverse reaction is
60.3 kJ mol-1. On your sketch in c(i), draw the energy profile of the catalysed
reaction. Suggest a value for the activation energy for the dissociation of N2O4 under
these conditions.
[5]

(d) The oxides of Period 3 elements differ considerably in their physical and chemical
properties.

(i) State the formulae of the oxides formed by sodium, aluminium and phosphorus.

(ii) Explain the acid-base nature of these oxides in terms of their structures and
bonding. Hence write equations for the reactions, if any, of oxides of these elements
with aqueous alkali and acid.
[5]

[Total: 20]

[Turn Over
14

3 (a) A compound E has the following composition by mass:

C: 29.3% H: 5.70% Br: 65.0%

Heating E in the presence of ethanolic KOH forms F.

F can decolourise Br2 in CCl4. F can also react with gaseous HBr to form E and G.

G forms H when heated with aqueous NaOH. When compound H is reflux with acidified
K2Cr2O7, J is formed. Both compounds H and J give a yellow precipitate when warmed
with alkaline I2(aq).

White fumes are observed and compound K is formed when SOCl2 is added to H.

K reacts with ethanolic KCN to form compound L which when heated in the presence of
dilute HCl forms M.

M and H are heated in the presence of excess concentrated sulfuric acid forms N.

Deduce the structures of compounds E to N, explaining the chemistry for the reactions
involved. Write equations where appropriate.

[15]

(b) Suggest the reagents and conditions used for the following synthesis and draw the
structures of the intermediates formed in the reaction.

(i) H
I II III CH3 CH2NH2
CH3CH2OH P Q C

OH
(ii) CH3 COOH
Br
I II
R

[5]

[Total: 20]

[Turn Over
NATIONAL JUNIOR COLLEGE
PRELIMINARY EXAMINATIONS
Higher 1

CANDIDATE
NAME

SUBJECT REGISTRATION
CLASS NUMBER

CHEMISTRY 8872/02
Paper 2 Structured Questions 13 Sept 2010

2 hours

READ THE INSTRUCTION FIRST For Examiners


Write your subject class, registration number and name on all Use
the work you hand in.
Section A
Write in dark blue or black pen on both sides of the paper.
You may use a soft pencil for any diagrams, graphs or rough A1
working.
Do not use paper clips, highlighters, glue or correction fluid. A2

Section A A3
Answer all questions.
A4
Section B
Answer any two questions. A5

At the end of the examination, fasten all your work securely Section B
together. B1
The number of marks is given in brackets [ ] at the end of
each question or part question. B2

B3

Total

This question paper consists of 14 printed pages (including this page).


2

Section A
Answer all the questions in this section in the spaces provided.

1 Butanoic acid, CH3CH2CH2COOH (Mr = 88.0), also known as butyric acid have
important uses in the industry, such as perfume and food additives.

In an experiment, a 10.0 cm3 sample of butanoic acid of concentration 0.025


mol dm3 was titrated against 0.010 mol dm-3 barium hydroxide, Ba(OH)2.

(a) Calculate the volume of barium hydroxide, Ba(OH)2 required for the
neutralization of butanoic acid in the experiment.

2 CH3CH2CH2COOH + Ba(OH)2 (CH3CH2CH2COO)2Ba + 2 H2O

No. of moles of butanoic acid = 10.0 x x 10-3 x 0.025 = 2.5 x 10-4


No. of moles of Ba(OH)2 = (2.5 x 10-4) = 1.25 x 10-4
1.25 x10 4
Volume of Ba(OH)2 = x 1000 = 12.50 cm3
0.01

[2]

(b) Write an expression for the acid dissociation constant, Ka for butanoic acid.
Hence, determine its Ka value given that the initial pH of butanoic acid is 3.2.

[H + ][CH 3CH 2CH 2COO ]


Ka =
[CH 3CH 2CH 2COOH ]

Ka = (103.2)2 / (0.025 103.2) = 1.59 105 mol dm3

[2]

[Turn Over
3

(c) Another experiment was conducted to determine the enthalpy change of


reaction between butanoic acid and barium hydroxide.

Given the following data, determine the enthalpy change of reaction for the
above reaction.

Volume of 3.00 mol dm-3 barium hydroxide = 30.0 cm3


Volume of 2.00 mol dm-3 butanoic acid = 40 cm3
Temperature rise = 14.9 C
Specific heat capacity of water = 4.18 J g-1 K-1

2 CH3CH2CH2COOH + Ba(OH)2 (CH3CH2CH2COO)2Ba + 2 H2O

No. of moles of butanoic acid = 2 x 40 x 103 = 0.08

No. of moles of barium hydroxide = 30 x 103 x 3 = 0.09

0.08 moles of butanoic acid requires 0.04 moles of barium hydroxide for
reaction, thus butanoic acid is limiting.

Heat evolved = 70 x 4.18 x 14.9 = 4360 J

4360
H = x 2 = 109 kJ mol
0.08

[2]

(d) Predict and explain how the magnitude of the temperature rise will change
when the reaction in (c) as repeated using 2-chlorobutanoic acid with the same
volume and concentration as butanoic acid.

Temperature rise would be larger. 2-chlorobutanoic acid is a stronger acid and


dissociates more than butanoic acid, thus, less of the energy evolved from
neutralization would be used up to fully dissociate 2-chlorobutanoic acid. Since
more heat would be evolved for the neutralization of 2-chlorobutanoic acid, the
temperature rise would be larger

[2]

[Total: 8]

[Turn Over
4

2 The first seven ionisation energies of Element A from Period 2 are as follows:

No of electrons lost 1st 2nd 3rd 4th 5th 6th 7th

Ionisation energy/ 1410 2866 4576 7473 9442 53250 64340


kJ mol1

(a) Deduce the group that Element A belongs to in the Periodic Table. Explain
your answer.

Largest jump in I.E is from 5th to 6th IE, thus A is in group V. There are 5
valence electrons, the 6th electron is in an inner principal quantum shell nearer
the nucleus, thus require much more energy to remove due to stronger
attraction to the nucleus.

[2]

(b) State the identity of Element A and hence write its electronic configuration.

A is nitrogen. Configuration = 1s2 2s2 2p3

[1]

(c) Magnesium reacts with A to form an ionic compound of the formula Mg3A2.
State and explain which one of the ions in the compound would have a larger
ionic radius.

Ions are Mg2+ and N3

Both ions are isoelectronic, but Mg2+ has more protons than N3. The valence
electrons of Mg2+ would be more strongly attracted towards the nucleus,
leading to a smaller ionic radius.

[2]

[Total: 5]

[Turn Over
5

3 (a) The boiling points of some compounds are given in the table below.

Compounds Mr Boiling Points / C


Lithium chloride 42.5 1382
Lithium iodide 134 462
Ammonia 17 33
Hydrogen fluoride 20 19.5
Iodine 254 185

(ii) Explain the difference in the boiling point between

I. lithium chloride and lithium iodide

q+q
LE
r+ + r
Both cations and anions have the same charge. However, Cl has a smaller ionic
radius than I. Thus, the interionic distance between Li+ and I is greater than
between Li+ and Cl.

Lattice energy of LiCl more exothermic than LiI. Ionic bonds in LiI is stronger
than in LiCl, leading to a larger boiling point as more energy required to break
stronger ionic bonds between Li+ and Cl.

[2]
II. ammonia and hydrogen fluoride

F is more electronegative than N, thus making the H attached to F in HF more


protonic. This leads to stronger hydrogen bonds between the lone pair on F of
one HF molecule with the protonic hydrogen of another HF molecule as
compared to that between NH3.
Thus, more energy required to break the stronger hydrogen bonds between HF
molecules compared to NH3 molecules.
[2]
III. iodine and hydrogen fluoride.

The temporary dipole induced dipole forces between I2 molecules is much


stronger than the hydrogen bonds between HF molecules as I2 has a much
larger electron cloud, leading to greater polarisability. Thus, more energy
required to break the stronger temporary dipole induced dipole forces between I2
molecules.
[2]

[Total: 6]

[Turn Over
6

4 (a) An organic compound B of the molecular formula C4H6O, undergoes a series of


reactions as shown by the table below. State the deductions for each of the following
reactions.

Test Deductions
(I) Brown precipitate is observed when C=C bond is present, mild oxidation with
cold dilute alkaline KMnO4 is added KMnO4 to form a diol.
to 1 cm3 of a solution containing B in
a test-tube.

(II) Silver mirror is observed when Aldehyde group is present. Tollens reagent
Tollens reagent is added to 1 cm3 of oxidize aldehyde to carboxylate, Ag+ reduced
a solution containing B in a test- to Ag mirror.
tube.

(III) 0.1 mole of compound B will react 4.8


with 4.8 dm3 of hydrogen gas in the No. of moles of H2 = 24 = 0.2 moles
presence of a platinum catalyst at Reduction of C=C and aldehyde group to form
room temperature and pressure. alkane and 1o alcohol group.

[4]

(b) Compound B exhibits geometric isomerism. Together with the information obtained in
(a), give the structural formula of B.

B: CH3CH=CHCHO
[1]

(c) State the structural features present in compound B that enables it to exhibit
geometric isomerism. Hence draw the geometric isomers of B.

C=C to restrict free rotation


The two groups attached to the same C of the carbon-carbon double bond are
different.

[Turn Over
7

Trans isomer cis isomer


[2]

(d) Compounds C and D have the same molecular formula C4H6O. Both gives the same
positive test as shown in (a). C and D are structural isomers of one another but they
do not exhibit geometric isomerism. Draw the structures for compounds C and D.

C D

[2]

(e) Suggest a test-tube reaction to differentiate compound B from either C or D. State the
reagents and conditions used and the observations for each of the compounds.

KMnO4, H2SO4, heat


B: Purple KMnO4 decolourises
C&D: Purple KMnO4 decolourises and effervescence observed.

[2]

[Total: 11]

5 Methane, one of the main components in natural gas is an important fuel used
worldwide. It has an enthalpy change of combustion of 890 kJ mol1.

Methane is being depleted at an alarming rate. The current worldwide consumption of


methane is about 4.9 x 109 m3 per year. Thus, alternative sources of methane need to
be sought to sustain the worlds consumption.

One of the alternative sources of methane comes in the form of methane hydrate. It is
also known as methane ice or fire ice in which a large amount of methane is trapped
within the crystal structure of ice. Part of the crystal structure of ice is shown below.

[Turn Over
8

Larger hydrocarbon molecules such as ethane and propane can also form hydrates,
although as the molecule length increases (butanes, pentanes), the tendency to form
hydrates decreases.

Significant deposits of methane hydrate are found under sediments on the ocean floors
and the arctic permafrost of the Earth, about 500 to 2000 m below the ocean surface. It
is estimated that there are 11.3 x 1018 m3 of methane hydrate available worldwide.

Thus, international research and exploration by governmental and industrial entities


evaluated that methane hydrates could be important primary energy sources in the
future due to the depletion of fossil fuels. To utilize the methane in methane ice,
excavators would likely melt the ice underground first, and then extract the methane gas.
When brought to the surface, it is estimated that 1 m3 of methane hydrate contains about
180 m3 of methane, measured at room temperature pressure.

Although methane hydrate is plentiful throughout the world, there are many key
challenges being faced in the utilizing of methane hydrate as a source of methane. With
these challenges in mind, China's Ministry of Land and Resources estimated that the
country could begin using its combustible ice within 10 to 15 years, joining other
countries in methane hydrate exploration.

Water gas, which consists of an equimolar mixture of hydrogen and carbon monoxide is
another type of fuel used in the industries. The enthalpy of combustion of hydrogen and
carbon monoxide are 242 kJ mol1 and 283 kJ mol1 respectively.

Water gas could be produced according to the following equation.

H2O (g) + C(s) H2 (g) + CO (g)

It is a useful product but requires careful handling because of the risk of carbon
monoxide poisoning.

(a) Calculate the amount of energy that can be produced from the methane extracted
from 1 m3 of methane hydrate.

[Turn Over
9

Volume of methane = 180 m3


180 x10 3
No. of moles of methane = = 7500 moles
24
Energy evolved = 7500 x 890 = 6.68 x 106 kJ

[2]

(b) Calculate the volume of methane that can be obtained at room temperature and
pressure from the methane hydrate available worldwide.

Volume of methane = 11.3 x 1018 x 180 = 2.03 x 1021 m3

[1]

(c) Based on the current worldwide consumption of methane, calculate how many
years the methane obtained from methane hydrate could sustain the world.

2.03x10 21
No. of years = = 4.15 x 1011 years
4.9 x 10 9
[1]

(d) Suggest a possible challenge faced by the world in methane hydrate exploration

The methane hydrate is found 500 to 2000 m below the ocean surface; thus,
advanced machinery is required to obtain the methane hydrate.
[1]

(e) Suggest a reason why hydrocarbons form hydrates less readily as the molecule
length increases.

As molecular length of the hydrocarbon chain increases, it becomes difficult for it to


pack within the spaces in the crystal structure of water. As the hydrocarbon chain is
hydrophobic, it is also unable to form favourable interactions with water.

[1]

(f) Calculate the amount of energy that can be produced from 1 m3 of water gas.

1 m3 of water gas contains 500 dm3 of CO and 500 dm3 of H2.

[Turn Over
10

500
No. of moles of H2 and CO = = 20.8 moles
24

Energy evolved = 20.8 (242 + 283) = 1.09 x 104 kJ


[2]

(g) Suggest two reasons why methane is used as a preferred fuel compared to water
gas in industries.

Methane gives out more energy per m3 and methane does not contain the toxic CO
found in water gas.

[2]

[Total:10]

[Turn Over
11

Section B

Answer two of the following three questions. Answer these questions on separate answer paper.

1 Butane can react with Cl2(g) to form monochlorinated butane.

(a) (i) Name the type of reaction and state the condition required for the reaction to occur.

Free radical substitution. Heat or uv light is required.

(ii) Draw the structures of all the possible monochlorinated butanes formed.

H H H H

H C C C C Cl

H H H H

(iii) State and explain the ratio in which they are formed.
1-chlorobutane: 2-chlorobutane is 3:2 as there are 6 hydrogen atoms that could be
replaced to form 1-chlorobutane but only 4 hydrogen atoms that could be replaced
to form 2-chlorobutane.
[6]

(b) The rate for the reaction between chlorobutane and aqueous sodium hydroxide was
studied using two solutions of different initial concentrations of sodium hydroxide and the
same initial concentration of chlorobutane. The results are as shown below:

Time / min Experiment 1 Experiment 2


[Chlorobutane] / mol dm-3 when [Chlorobutane] / mol dm-3
[NaOH] = 0.20 mol dm-3 when [NaOH] = 0.30 mol
dm-3
0 0.0100 0.0100
40 0.0079 0.0070
80 0.0062 0.0049
120 0.0049 0.0034
160 0.0038 0.0024
200 0.0030 0.0017

(i) Plot these data on suitable axes.

[Turn Over
12

0.01
3
[chlorobut ane]/moldm 0.009
0.008
0.007
0.006
0.005
0.004 Expt.1
0.003
0.002
0.001
Expt.2
0
0 20 40 60 80 100 120 140 160 180 200 220 240
77.5s 155 77.5 = 77.5 s
time/s

(ii) From the graphs, deduce the order of reaction with respect to chlorobutane and
aqueous sodium hydroxide.

Chlorobutane: constant t1/2 of 77.5 s, thus first order.

0.01 0.005
Expt 1: 0.20 mol dm-3 NaOH, R = = 5.88 x 10-5 mol dm-3 s-1
85 0

0.01 0.004
Expt 2: 0.30 mol dm-3 NaOH, R = = 8.57 x 10-5 mol dm-3 s-1
70 0

8.57 x10 5
When [NaOH] increase 1.5 x, R increase = 1.46 x
5.88 x10 5

1st order wrt NaOH.

(iii) Hence, deduce the rate equation and calculate a value for the rate constant, stating
its units.

R = k[NaOH][chlorobutane]

8.57 x10 5
k= = 0.0286 mol1 dm3s1
(0.30)(0.01)

[6]

[Turn Over
13

(c) With the aid of a relevant diagram, explain how an increase in temperature will increase
the rate of the above reaction.

Temperature increase, there will be a greater fraction of molecules with energy greater
than activation energy, thus more effective collisions leading to a greater rate of reaction.

[3]

(d) The table below shows the pH values of Period 3 chlorides in water.

Period 3 chloride pH of a 1.0 mol dm-3 solution in


water at 25oC
NaCl 7.0
MgCl2 6.5
AlCl3 3.0
SiCl4 2.0
PCl5 1.0 - 2.0

(i) By writing relevant balanced equations, explain the difference in pH of aqueous


solutions of NaCl, AlCl3 and PCl5.

NaCl + aq Na+ + Cl-


Na+ has low charge to volume ratio than Na+, thus cannot polarize water to give H+.
Thus, solution is at pH 7.

AlCl3 + 6 H2O [Al(H2O)6]3+ + 3 Cl-


[Al(H2O)6]3+ + H2O [Al(H2O)6OH]2+ + H3O+

Al3+ has higher charge to volume ratio than Na+, thus polarize water causing O H
bond to break and give H+. Thus, solution is at pH 3.

PCl3 + 3 H2O H3PO3 + 3 HCl

PCl3 hydrolyses in water to form a strong acid HCl of pH1.0 to 2.0.

[Turn Over
14

(ii) Using the information given in the above table and the Data booklet, predict with
explanations, the pH value of a 1 mol dm-3 solution of BeCl2.

ion charge
ionic radius
Na+ 1
= 10.5
0.095
Be2+ 2
= 64.5
0.031
Al3+ 3
= 60.0
0.050

Be2+ has the highest charge to radius ratio, thus would polarize O H bonds of
water to greater extent, leading to more H+ produced, pH = 2.5

[5]

[Total: 20]

[Turn Over
15

2 At room temperature, dinitrogen tetraoxide dissociates and forms an equilibrium with nitrogen
dioxide in a closed vessel, as shown in the equation below.

N2O4(g) 2NO2(g) )

Pale yellow Dark brown

(a) (i) Draw the dot-cross diagram of N2O4 & NO2.

Bent trigonal planar

(ii) Hence state and explain if the forward reaction is endothermic or exothermic.
Forward reaction is dissociation, breaking of N N bond, thus is endothermic.

[3]

(b) (i) At 1400C, 3 moles of N2O4 was placed in a 2 dm3 vessel and allowed to reach
equilibrium. At equilibrium, 60% of the N2O4 dissociated into NO2. Calculate the
equilibrium constant, Kc at this temperature, stating its units.

60
Amt of N2O4 dissociated = x 3 = 1.8
100
N2O4 (g) 2NO2 (g)
Initial amt 3 0
Change - 1.8 +2(1.8)
Final amt 1.2 3.6
[ ]/moldm-3 0.6 1.8

[NO2 ] 2 1.8
2
Kc = = =5.4 mol dm-3
[N 2O 4 ] 0.6
(ii) State and explain the expected colour of the equilibrium mixture in the vessel when
(I) the volume is increased to 4 dm3
(II) the vessel is immersed in a beaker containing hot water.

(I): equilibrium shifts to the right to less moles of gas by Le Chateliers principle to
partially reduce pressure. More NO2 form, thus is darker brown.

(II): equilibrium shifts to the right to favour forward endothermic reaction by Le


Chateliers principle to partially reduce the temperature by absorbing the excess
heat. More NO2 form, thus is darker brown.

[7]

[Turn Over
16

(c) (i) At 1400C, the activation energy for the forward reaction is 150 kJ mol-1 and that for
the reverse reaction is 91.3 kJ mol-1. Sketch a reaction pathway diagram for this
reaction. Energy/ kJ mol-1

Ea(b) = 91.3 kJmol-1

Ea(f) = 150 kJmol-1


Ea(b) catalysed = 60.3 kJmol-1

H = +58.7 kJmol-1

(ii) Hence determine the value of the enthalpy change of dissociation of N2O4.

H = +58.7 kJmol1

(iii) In the presence of a catalyst, the activation energy for the reverse reaction is
60.3 kJ mol-1. On your sketch in c(i), draw the energy profile of the catalysed
reaction. Suggest a value for the activation energy for the dissociation of N2O4 under
these conditions.
Ea = 58.7 + 60.3 = 119 kJ mol1

[5]

(d) The oxides of Period 3 elements differ considerably in their physical and chemical
properties.

(i) State the formulae of the oxides formed by sodium, aluminium and phosphorus.
Na2O, Al2O3, P4O10

(ii) Explain the acid-base nature of these oxides in terms of their structures and
bonding. Hence write equations for the reactions, if any, of oxides of these elements
with aqueous alkali and acid.
Na2O is an ionic oxide, thus is a basic oxide and thus react with acids
e.g. Na2O + 2 HCl 2 NaCl + H2O

Al2O3 is amphoteric as it is ionic with covalent character, thus react with both acid
and alkali

e.g. Al2O3 + 6 HCl 2 AlCl3 + 3 H2O


Al2O3 + 2 NaOH + 3 H2O 2 NaAl(OH)4

[Turn Over
17

P4O10 is a covalent compound and is thus an acidic oxide. Thus it reacts with a
base.

P4O10 + 12 NaOH 4 Na3PO4 + 6 H2O

[5]

[Total: 20]

3 (a) A compound E has the following composition by mass:

C: 29.3% H: 5.70% Br: 65.0%

Heating E in the presence of ethanolic KOH forms F.

F can decolourise Br2 in CCl4. F can also react with gaseous HBr to form E and G.

G forms H when heated with aqueous NaOH. When compound H is reflux with acidified
K2Cr2O7, J is formed. Both compounds H and J give a yellow precipitate when warmed
with alkaline I2(aq).

White fumes are observed and compound K is formed when SOCl2 is added to H.

K reacts with ethanolic KCN to form compound L which when heated in the presence of
dilute HCl forms M.

M and H are heated in the presence of excess concentrated sulfuric acid forms N.

Deduce the structures of compounds E to N, explaining the chemistry for the reactions
involved. Write equations where appropriate.

[15]

[Turn Over
18

Reaction Deduction
Compound E C H Br
% by 29.3 5.70 65.0
C:29.3% H:5.70% Br:65.0% mass
Ar 12.0 1.0 80.0
Mole 2.44 5.70 0.8125
Ratio 3 7 1

Empirical formula = C3H7Br

Compound E

Heating E in the presence of ethanolic E undergoes elimination of HBr to form an


KOH forms F. alkene F.

Compound F

F can decolourise Br2 in CCl4. Electrophilic addition of Br2 across C=C

F can also react with gaseous HBr to F undergoes electrophilic addition with HBr
form E and G to give E and G

H H H

H C C C H

H Br H
Compound G

G forms H when heated in aqueous Nucleophilic substitution of G to give H, an


NaOH. alcohol.

[Turn Over
19

Compound H

When compound H is reflux with Oxidation of alcohol H. H is not a tertiary


acidified K2Cr2O7, J is formed. alcohol.

Both compounds H and J give a yellow H is a methyl alcohol (secondary alcohol), J


precipitate when warmed with alkaline is a methyl ketone.
I2(aq).

Compound H

Compound J

White fumes are observed and K is a chloroalkane, substitution of OH for


compound K is formed when SOCl2 is Cl occurred.
added to H.

Compound K

K reacts with ethanolic KCN to form Nucleophilic substitution of Cl with CN. L


compound L which when heated in the is has CN group, undergoes hydrolysis with
presence of dilute HCl forms H. hot HCl(aq) to give H, -COOH group formed.

[Turn Over
20

Compound L

H CH3

H C C COOH

H H

Compound M

M and H when heated in the presence M is acid, H is alcohol. Esterification


of excess concentrated sulfuric acid occurred. N is an ester.
forms N.

Compound N

[Turn Over
21

(b) Suggest the reagents and conditions used for the following synthesis and draw the
structures of the intermediates formed in the reaction.

(i) H
I II III CH3 CH2NH2
CH3CH2OH P Q C

OH
I: K2Cr2O7, H2SO4 (aq), heat with distillation
II: KCN, HCl, trace of NaCN as catalyst, cold
III: LiAlH4 dry ether

P = CH3CHO

Q = CH3CH(OH)CN

(ii) CH3 COOH


Br
I II
R

I: Br2, AlBr3, dark


II: KMnO4, dilute H2SO4, heat

R=
CH3
Br

[5]

[Total: 20]

[Turn Over
1

NANYANG JUNIOR COLLEGE


JC 2 PRELIMINARY EXAMINATION
Higher 1

CHEMISTRY 8872/01
Paper 1 Multiple Choice 22 September 2010
50 minutes
Additional Materials: Multiple Choice Answer Sheet
Data Booklet

READ THESE INSTRUCTIONS FIRST

Write in soft pencil.


Do not use staples, paper clips, highlighters, glue or correction fluid.
Write your name, class and tutors name on the Answer Sheet in the spaces provided unless
this has been done for you.

There are thirty questions on this paper. Answer all questions. For each question there are
four possible answers A, B, C and D.
Choose the one you consider correct and record you choice in soft pencil on the separate
Answer Sheet.

Read the instructions on the Answer Sheet very carefully.

Each correct answer will score one mark. A mark will not be deducted for a wrong answer.
Any rough working should be done in this booklet.

This document consists of 15 printed pages and 1 blank page.


[Turn over

H1 Chemistry 8872/01 NYJC J2/10 PX


2

Section A

For each question there are four possible answers, A, B, C and D. Choose the one
you consider to be correct.

1 A mixture of propane, C3H8, and a gaseous alkene of the general formula CnH2n
occupied 24 cm3. 114 cm3 of oxygen was required for complete combustion.
After combustion, 72 cm3 of carbon dioxide was produced. (All volumes were
measured at the same temperature & pressure.)

What is the formula of the alkene present in the mixture?

A C2H4
B C3H6
C C4H8
D C5H10

2 Which named element does not disproportionate in the reaction shown?

A carbon in H2C2O4 H2O + CO + CO2


B chlorine in 3ClO 2Cl + ClO3
C nitrogen in H2O + 2NO2 HNO2 + HNO3
D sulfur in 2FeSO4 Fe2O3 + SO2 + SO3

3 Which one of the following corresponds to the configuration of the four electrons
of highest energy for the ground state of an element in Group IV?

A 1s2 2s2
B 3p4
C 4s2 4p2
D 3d2 4s2

H1 Chemistry 8872/01 NYJC J2/10 PX [Turn over


3

4 The diagram shows a slow stream of a liquid being deflected by a negatively


charged rod.

burette

liquid
negatively charged rod

Which liquid would be deflected as shown in the above diagram?

A bromine
B cyclohexane
C tetrachloromethane
D water

5 The following diagrams show the structures of an element, its principal oxide and
its halide.

= element = element = element


= oxygen = halogen

What could the element be?

A aluminium
B carbon
C phosphorus
D silicon

H1 Chemistry 8872/01 NYJC J2/10 PX [Turn over


4

6 A student used the apparatus below to heat a can containing 300 g of water.

thermometer
can containing
300 g of water

burner containing
propan-1-ol

The following data were recorded:


Mass of propan-1-ol burnt = mg
Change in temperature of water = T oC

Given:
Relative molecular mass of propan-1-ol = 60.0
Enthalpy change of combustion of propan-1-ol = 2021 kJ mol1
Specific heat capacity of water = c kJ kg1 K1

What is the efficiency of this heating process?

m 2021 1000
A 100%
300 c T 60.0
m c T 60.0
B 100%
300 2021 1000
300 c T 60.0
C 100%
m 2021
300 c T 60.0
D 100%
m 2021 1000

H1 Chemistry 8872/01 NYJC J2/10 PX [Turn over


5

7 The major natural source of fluorine is the mineral fluorspar, which is mainly
calcium fluoride, CaF2. The first stage in liberating F2 is to grind the compound
and react it with concentrated sulfuric acid. The products are hydrogen fluoride
and calcium sulfate.
CaF2 + H2SO4 2HF + CaSO4
The appropriate enthalpy changes of formation are given in the table.

Hf (kJ mol1)
CaF2 1220
H2SO4 814
HF 271
CaSO4 1434

What is the enthalpy change of the reaction (in kJ mol1)?

A +58
B 58
C +329
D 329

8 Which graph would confirm that the rate of decomposition of hydrogen peroxide
is first order with respect to the concentration of hydrogen peroxide?

rate A B
rate

[H2O2] [H2O2]

rate C rate D

[H2O2] [H2O2]

H1 Chemistry 8872/01 NYJC J2/10 PX [Turn over


6

9 Lead is the final product formed by a series of changes in which the rate-
determining stage of the radioactive decay of uranium-238. This radioactive
decay is a first-order reaction with a half-life of 4.5 109 years.

What would be the age of a rock sample, originally lead-free, in which the molar
proportion of uranium to lead is now 1:3?

A 1.5 x 109 years


B 2.25 x 109 years
C 9.0 x 109 years
D 13.5 x 109 years

10 What is the effect of a catalyst on the position of equilibrium, rate constant and
equilibrium constant for a reversible reaction?

position of equilibrium rate constant equilibrium constant


A shifts left increases no effect
B shifts right decreases increases
C no change decreases increases
D no change increases no change

11 Two equilibria are shown below.

Reaction I: 2NO2(g) 2NO(g) + O2(g)


Reaction II: NO(g) + 1 O2(g) NO2(g)
2

The numerical value of Kc for reaction I is 0.36. Under the same conditions, what
is the numerical value of Kc for reaction II?

A 0.18 B 1.7 C 2.8 D 5.6

H1 Chemistry 8872/01 NYJC J2/10 PX [Turn over


7

12 Drinking water is often disinfected with Cl2, which hydrolyses to form HClO, a
weak acid but powerful disinfectant.
[HClO]
The fraction of HClO in solution is defined as .
[HClO] + [ClO ]

Given that the acid dissociation constant, Ka, of HClO is 2.9 x 108 mol dm3,
what is the fraction of HClO at pH 7.00?

A 0.125
B 0.223
C 0.500
D 0.775

13 A sample of 1 mol of ethanoic acid is diluted at constant temperature to a


volume V.

Which diagram shows how the pH of the sample varies with V?

A B

pH pH

7 7

0 V 0 V

C D

pH pH

7 7

0 V 0 V

H1 Chemistry 8872/01 NYJC J2/10 PX [Turn over


8

14 Which of the following shows a correct trend for the elements of Period 3?

melting point

Na Mg Al Si P S Cl
ionic radius

Na Mg Al Si P S Cl

electrical conductivity

Na Mg Al Si P S Cl

pH of aq chlorides

D 7

Na Mg Al Si P

H1 Chemistry 8872/01 NYJC J2/10 PX [Turn over


9

15 Fibre glass can be considered to be a mixture of ionic oxides and giant covalent
oxides.

Which of the following is not a constituent of fibre glass?

A Al2O3
B Na2O
C P4O10
D SiO2

16 Which of these compounds can exist as a pair of cis-trans isomers?

A CH3CH=CH2
B (CH3)2C=CH2
C (CH3)2C=CHCH3
D CH3CH=CHCH3

17 Which hydrocarbon, on treatment with hot acidified potassium manganate(VII),


would give ethanoic acid only?

A CH3CH=CH2
B CH3CH=CHCH3
C (CH3)2C=CHCH3
D (CH3)2C=C(CH3)2

18 Which property does benzene have as a consequence of the delocalisation of


electrons in the benzene molecule?

A Benzene is a good conductor of electricity.


B The carbon-carbon bond lengths are between those of CC bonds and C=C
bonds.
C Addition reactions of benzene take place more easily than substitution.
D Substitution in benzene takes place at one particular carbon atom.

H1 Chemistry 8872/01 NYJC J2/10 PX [Turn over


10

19 Which reagent and conditions are used to bring about the reaction shown?
CH3 CH3

Cl
A Cl2 in the dark
B Cl2 with AlCl3
C Cl2 with ultraviolet light
D concentrated HCl heated under reflux

20 Some chlorobutanes were separately treated with hot ethanolic sodium


hydroxide. Two of these gave the same hydrocarbon, C4H6.

From which pair of chlorobutanes was this hydrocarbon obtained?

A CH3CH2CH2CH2Cl and CH3CH2CH2CHCl2


B CH3CH2CH2CH2Cl and CH3CH2CHClCH3
C CH3CH2CH2CH2Cl and ClCH2CH2CH2CH2Cl
D CH3CHClCHClCH3 and ClCH2CH2CH2CH2Cl

21 Which inorganic reagent may be used to distinguish between ethanol and


methanol?

A alkaline aqueous I2
B aqueous NaOH
C K2Cr2O7 in dilute H2SO4
D Na

H1 Chemistry 8872/01 NYJC J2/10 PX [Turn over


11

2 Which set of alcohols correctly shows a primary, a secondary and a tertiary


alcohol?

primary secondary tertiary


A CH2OH CH 2 OH CH 2 OH

CH2 H C OH H C OH

CH3 CH3 CH 2 OH

B CH2OH CH3 CH3


H3C C H H3C C OH H3C C H
CH3 CH3 CH2OH

C CH2OH CH2OH CH2OH


H3C C H H3C C CH2OH H3C C CH2OH
H H CH2OH

D H CH3 CH3
H3C C OH H3C C OH H3C C OH
H H CH3

23 Butanedione, CH3COCOCH3, is a yellow liquid which is responsible for the


cheese-like smell in cheese and from unwashed feet.
How does butanedione react with 2,4-dinitrophenylhydrazine reagent and
Fehlings reagent?

2,4-dinitrophenylhydrazine Fehlings reagent


A positive positive
B positive negative
C negative positive
D negative negative

24 The same carboxylic acid is obtained either by the hydrolysis of a nitrile P or by


the oxidation of an alcohol Q.
Which of the following pairs could be P and Q?

P Q
A CH3CH2CN CH3CH2OH
B (CH3)2CHCN (CH3)3COH
C C6H5CH(CH3)CN C6H5CH2CH(OH)CH3
D C6H5CH2CN C6H5CH2CH2OH

H1 Chemistry 8872/01 NYJC J2/10 PX [Turn over


12

25 Given that halogen groups are electron withdrawing, which of the following
equimolar aqueous solutions has the lowest pH?

A chloroethanoic acid
B ethanoic acid
C fluoroethanoic acid
D propanoic acid

H1 Chemistry 8872/01 NYJC J2/10 PX [Turn over


13

Section B

For each of the questions in this section one or more of the three numbered
statements 1 to 3 may be correct.

Decide whether each of the statements is or is not correct (you may find it helpful to
put a tick against the statements which you consider to be correct).

The responses A to D should be selected on the basis of

A B C D
1, 2 and 3 1 and 2 2 and 3 1 only
are only are only are is
correct correct correct correct

No other combination of statements is used as a correct response.

26 Use of Data Booklet is relevant to this question

The graph shows the trend of the ionisation energies for the outermost fifteen
electrons in a particle, X.
Ionisation energies

0 5 10 15 20
number of electrons removed

Which of the following could be X?

1 Ar
2 P
3 Cl2+

H1 Chemistry 8872/01 NYJC J2/10 PX [Turn over


14

The responses A to D should be selected on the basis of

A B C D
1, 2 and 3 1 and 2 2 and 3 1 only
are only are only are is
correct correct correct correct

No other combination of statements is used as a correct response.

27 The energy profile diagram for a reversible reaction P + Q R is shown below.

energy

P+Q
y
R
progress of reaction

Which of the following statements are correct?

1 The activation energy of the backward reaction is (x + y).


2 The magnitude of the enthalpy change of the backward reaction is (x y).
3 The forward reaction is endothermic.

28 Which statements are correct for a reversible reaction at equilibrium?

1 The rates of forward and reverse reactions are equal.


2 The concentrations of the reactants and products are equal.
3 The rate constants for the forward and reverse reactions are equal.

H1 Chemistry 8872/01 NYJC J2/10 PX [Turn over


15

The responses A to D should be selected on the basis of

A B C D
1, 2 and 3 1 and 2 2 and 3 1 only
are only are only are is
correct correct correct correct

No other combination of statements is used as a correct response.

29 Aureomycin is a powerful antibiotic.


OH O OH O O
OH
C
NH2

Cl HO CH3 H N CH3
CH3
aureomycin

What functional groups are present in its molecular structure?


1 ester
2 tertiary alcohol
3 ketone

30 Malic acid, HO2CCH(OH)CH2CO2H, is found in apples.


Which properties does malic acid have?

1 It can form esters both with ethanoic acid and with ethanol.
2 1 mole of malic acid can react with 3 moles of Na metal.
3 It can react with hot concentrated H2SO4 to form a mixture of two
compounds.

End of Paper

H1 Chemistry 8872/01 NYJC J2/10 PX


NANYANG JUNIOR COLLEGE
JC 2 Preliminary Examinations
Higher 1

CANDIDATE
NAME

TUTORS
CLASS
NAME

CHEMISTRY 8872/02
Paper 2 21 Sep 2010
2 hours
Candidates answer on the Question Paper.
Additional Materials: Data Booklet
Writing Paper
Graph Paper
READ THESE INSTRUCTIONS FIRST
Write your name and class on all the work you hand in.
Write in dark blue or black pen on both sides of the paper.
You may use a soft pencil for any diagrams, graphs or rough working.
Do not use staples, paper clips, highlighters, glue or correction fluid.
Answer all questions.
For Section A, write your answers in this booklet.
For Section B, write your answers on the writing paper provided.
You are advised to show all working in calculations.
At the end of the examination, fasten all your work securely together. Section A
The number of marks is given in brackets [ ] at the end of each question or
part question. 1
4
2
6
3
4
4
6
5
8
6
6
7
6

Total
40
This paper consists of 16 printed pages and 0 blank page
[Turn over
2 For
examiners
Section A use

Answer all the questions.


Write your answers on the spaces provided.

1 When ethanedioate ions, C2O42, are oxidised by acidified aqueous potassium


manganate(VII), the purple solution decolourises and a gas that gives a white
precipitate in limewater is formed.

(a) Write down a balanced equation for the above reaction.

(b) Calculate the volume of 0.034 mol dm3 potassium manganate(VII) required to
oxidise 1.5 103 mol of the salt KHC2O4H2C2O4 completely.

[4]

[Total: 4]
3 For
examiners
2 A fullerene is any molecule composed entirely of carbon, in the form of a hollow use

sphere, ellipsoid, or tube.

Buckminsterfullerene, the molecule C60, was the first fullerene to be discovered.

The bonding in buckminsterfullerene is similar to that in graphite.

(a) State the type of bonding between carbon atoms and between C60 molecules in
buckminsterfullerene.

Type of bonding:

between carbon atoms:

between C60 molecules: ....[2]

(b) Buckminsterfullerene is soluble in many organic (carbon-based) solvents. Suggest


a suitable solvent, explaining your answer.

[2]

(c) Aside from its solubility in organic solvents, suggest another property of
buckminsterfullerene which is similar to that of graphite.

[1]

(d) Aside from its solubility in organic solvents, suggest a property of


buckminsterfullerene which is different from that of graphite.

[1]

[Total: 6]
4 For
examiners
3 Sulfuric acid is manufactured industrially by the Contact process. The raw materials use

are sulfur, air and water.

The flowchart below shows an outline of the Contact process.

Stage 1: S(s) + O2(g) SO2(g)

Stage 2: 2SO2(g) + O2(g) 2SO3(g) H = 197 kJ mol1

Operating conditions
Temperature: 450 C
Pressure: 1 atm
Catalyst: vanadium (V) oxide, V2O5

(a) SO3(g) + H2SO4(l) H2S2O7(l)


oleum

Stage 3: + H2O

(b) concentrated sulfuric acid

(a) Sketch graphs to show how the percentage yield of sulfur trioxide in the equilibrium
mixture in Stage 2 varies with

(i) pressure for two temperatures, 400 C and 600 C, respectively

% yield of sulfur trioxide

Pressure / atm
5 For
examiners
(ii) temperature for two pressures, 0.5 atm and 2 atm, respectively. use

% yield of sulfur trioxide

Temperature / C

[2]

(b) Suggest how the percentage yield of sulfur trioxide produced in Stage 2 can be
increased without varying the operating conditions of the process.

[1]

(c) Write an overall balanced equation for the chemical reactions that happen in Stage
3 to produce concentrated sulfuric acid.

[1]

[Total: 4]
6 For
examiners
4(a) By reference to the CO molecule, explain the meaning of the term bond energy. use

[2]

(b) Write a thermochemical equation, including state symbols, to represent the average
bond energy of C H in methane.

[1]

(c) With help of the given energy cycle, make use of the following enthalpy changes
and appropriate data from the Data Booklet to estimate the average bond energy of
C H in methane.

Enthalpy change of combustion of methane = 882 kJ mol-1.


Enthalpy change of formation of carbon dioxide = 394 kJ mol-1
Enthalpy change of formation of water = 242 kJ mol-1
Enthalpy change of atomisation of carbon = +720 kJ mol-1

C(s) + 2H2(g) + 2O2(g) CH4(g) + 2O2(g)

CO2(g) + 2H2O(l)

[4]

[Total: 6]
7 For
examiners
5 Sodium was first isolated in 1807 and argon was discovered in 1894. Although they use

are both in the third period of the Periodic Table, there is considerable variation of
properties from sodium to argon.

(a) On the axes below, sketch graphs to show the variation in the following properties
of elements across the third period from Na to Ar.

(i) atomic radius

Na Mg Al Si P S Cl Ar
(ii)
first ionisation energy

Na Mg Al Si P S Cl Ar
[2]

(b) The variation in both atomic radius and first ionisation energy of the third period
elements can be explained using nuclear charge, shielding effect and effective
nuclear charge.

Explain the term shielding effect and hence use the three terms in italics above to
explain the variation in atomic radius.

Shielding effect

Variation in atomic radius

[2]
8 For
examiners
(b) The properties of oxides of the elements of the third period depend on the use

difference in electronegativity values between the element and oxygen.

The table below shows the electronegativity of elements of the second and third
period as proposed by Linus Pauling.

Li Be B C N O F Ne
0.98 1.57 2.04 2.55 3.04 3.44 3.98

Na Mg Al Si P S Cl Ar
0.92 1.31 1.61 1.90 2.19 2.58 3.16

(i) Calculate the difference in electronegativity between element and oxygen for
the oxides BeO, MgO, Al2O3 and SiO2.

BeO MgO Al2O3 SiO2

(ii) Hence use your values in (b)(i) to predict the following properties of beryllium
oxide, BeO.

structure and bonding

.......

.......

pH when added to water

.......

Reactions with aqueous HCl and aqueous NaOH respectively

.......

.......
[4]

[Total: 8]
9 For
examiners
6 Trans-9-oxodec-2-enoic acid is a primer pheromone used by the queen bee to use

regulate the honeybee caste system.

O
CH3C(CH2)5 H
C=C
H CO2H

trans-9-oxodec-2-enoic acid

Draw structures of the organic products when trans-9-oxodec-2-enoic acid reacts


with

(a) cold dilute KMnO4

(b) aqueous alkaline I2

(c) HCN with a trace of NaCN

(d) CH3CH(OH)CH3 in the presence of concentrated H2SO4.

[6]

[Total: 6]
10 For
examiners
7(a) Ethanoic acid, CH3CO2H, is a stronger acid than ethanol, CH3CH2OH. use

Explain the above statement.

[3]

(b) Give reagents, conditions and observations to show how you would distinguish
between the following compounds.

CH3CH2CH2Cl CH3CH2CH2OH

Reagents and
conditions

Observations

[3]

[Total: 6]
11
Section B
Answer any two of the three questions.
Write your answer on the writing paper provided.

1(a) Many new cars have air-bags which rapidly inflate during an accident and protect
the front passengers. The air-bag is inflated with nitrogen released by the rapid
decomposition of sodium azide, NaN3.

(i) Write a balanced equation for the likely decomposition of sodium azide.

(ii) What would you consider a likely volume for an inflated air-bag, taking into
consideration that it must be big enough to protect a passenger from
knocking his head onto the windscreen? Explain your estimation.

(iii) Based on your answers in (a)(i) and (ii), calculate the mass of sodium azide
needed to inflate an air-bag in a motorcar during an accident.

(iv) Draw a dot-and-cross diagram for the azide ion and suggest its shape.
[8]

(b) Hydrogen azide, HN3, is a colourless liquid with an irritating odour. It behaves as
an acid.
3
(i) A 0.1 mol dm aqueous solution of hydrogen azide has a pH of 2.9. Explain
whether it is a strong or weak acid.
3 3
(ii) The titration curve when 20 cm of this acid is titrated with 0.2 mol dm
aqueous sodium hydroxide is shown below. Copy the diagram and label the
2 axes. Write down the values of x and y in the spaces provided.

(iii) Suggest a suitable indicator for this titration.


[7]
12
(c) Brefeldin A is an antibiotic.

OH
O

HO O CH3

Give the structures of the organic products formed when brefeldin A is reacted with
the following reagents.

(i) Bromine dissolved in tetrachloromethane

(ii) Hot aqueous sodium hydroxide

(iii) Phosphorus pentachloride

[5]

[Total: 20]
13
2 Water, H2O, covers about two-thirds of the Earths surface and is vital to life. About
0.005% of water molecules consist of an oxygen atom bonded to two atoms of the
hydrogen isotope, deuterium, 21 D.

Deuterium oxide, D2O, is known as heavy water and is used for research in
chemical reactions because deuterium atoms react less quickly than normal
hydrogen atoms, 11 H. Like H2O, pure D2O is weakly ionised.

2D2O D3O+ + OD

For D2O, we can use the term KD instead of KW and pD instead of pH.

(a) Write expressions for the following terms.

(i) KD

(ii) pD
[2]

(b) For pure D2O, KD = 1.35 x 1015. Calculate the values of the following.

(i) [D3O+]

(ii) pD
[2]

(c) When pure H2O and pure D2O are mixed, exchange of H and D atoms takes
place and the following equilibrium is established.

D2O(l) + H2O(l) 2HDO(l) Kc = 3.56 at 298 K

(i) If a mixture containing 1.5 mol of D2O and 1.5 mol of H2O is made up,
how many moles of HDO will be present at equilibrium at 298 K?

(ii) Describe and explain the effect on the position of this equilibrium if
HDO is being drained away once it is formed.

[5]
(d) The following list of compounds react with or dissolve in water:

ethanoic acid, ammonia, sodium hydroxide, sodium chloride.

Water can react as either acid or a base. Choose a compound from the
above list with which water acts as

(i) a Bronsted base,

(ii) a Bronsted acid.

Construct a balanced equation for each reaction. [3]


14
(e) A solution containing ethanoic acid and its salt sodium ethanoate,
CH3CO2Na, acts as a buffer solution.

(i) What do you understand by the term buffer solution?

(ii) Write ionic equations to show how this solution reacts with
I added H+(aq) ions,
II added OH(aq) ions.
[3]

(f) Ethanoic acid can be synthesised from ethane by a 3-stage reaction


sequence:
I II III
CH3CH3 F G CH3CO2H

(i) Suggest the reagents and conditions necessary for reactions I, II and
III.

(ii) Draw the structural formulae of the intermediates F and G.


[5]

[Total: 20]
15
3(a) Bones contain a complex mixture of calcium salts, proteins and other material.
When a bone is strongly heated in a current of air, the only residue is calcium oxide.

CaCO3(s) CaO(s) + CO2(g)

(i) Name the type of reaction that occurs when a bone is strongly heated in air.

(ii) From a sample of 50.0 g of bone, 14.0 g of calcium oxide were obtained.
Calculate the percentage by mass of calcium in the bone.
[2]

(b) Calcium can be obtained from many different sources. Primarily, milk is the richest
source of dietary calcium.

The following information was taken from the side of a carton of milk.
How long does your milk keep?

25 C 0.5 day
18 C 1 day
10 C 2 days
5 C 4 days

(i) Plot a graph of the time the milk keeps against temperature.

(ii) Use your graph to predict the length of time the milk will keep at a
temperature of 15 C.

(iii) The souring of milk is a chemical reaction.


How is the rate of this reaction related to the length of time the milk keeps?

(iv) Explain how and why the rate of souring of milk varies with temperature.

(v) Suggest a reason why milk that has been boiled keeps longer than fresh milk.
[8]
16
(c) Cheese, which is also rich in calcium, contains esters of linoleic acid.

CH3(CH2)4CH=CHCH2CH=CH(CH2)7CO2H
linoleic acid

On the lid of a brand of cheese it is claimed that the cheese contains virtually no
trans fatty acids.

Representing the formula by the abbreviated structure ACH=CHCH2CH=CHB,


draw the structural formula of the isomer of linoleic acid that could be present in the
cheese.
[2]

(d) An organic compound, A, has the following composition by mass.


C, 66.7%; H, 11.1%; O, 22.2%

It has a relative molecular mass of 72.

When A is heated under reflux with an excess of acidified K2Cr2O7, the solution
turns from orange to green and Compound B is formed. B turns moist blue litmus
paper red.

However, heating A with acidified K2Cr2O7 followed by immediate distillation gives a


compound C.

When 2,4-dinitrophenylhydrazine is added to C, an orange-yellow crystalline solid is


produced. C also reacts with Fehlings solution and Tollens reagent.

(i) Calculate the molecular formula of A.

(ii) Identify the compounds A, B and C. Write clearly the deductions that you
made from the information given.
[8]

[Total: 20]

End of Paper
NANYANG JUNIOR COLLEGE
JC 2 PRELIMINARY EXAMINATION
Higher 1

CHEMISTRY 8872/01
Paper 1 Multiple Choice Answers 22 September 2010

1B 11 B 21 A
2D 12 D 22 D
3C 13 A 23 B
4D 14 A 24 D
5D 15 C 25 C
6D 16 D 26 C
7A 17 B 27 D
8C 18 B 28 D
9C 19 B 29 C
10 D 20 D 30 A
NANYANG JUNIOR COLLEGE
JC 2 Preliminary Examinations
Higher 1

CANDIDATE
NAME Answers
TUTORS
CLASS
NAME

CHEMISTRY 8872/02
Paper 2 21 Sep 2010
2 hours
Candidates answer on the Question Paper.
Additional Materials: Data Booklet
Writing Paper
Graph Paper
READ THESE INSTRUCTIONS FIRST
Write your name and class on all the work you hand in.
Write in dark blue or black pen on both sides of the paper.
You may use a soft pencil for any diagrams, graphs or rough working.
Do not use staples, paper clips, highlighters, glue or correction fluid.
Answer all questions.
For Section A, write your answers in this booklet.
For Section B, write your answers on the writing paper provided.
You are advised to show all working in calculations.
At the end of the examination, fasten all your work securely together. Section A
The number of marks is given in brackets [ ] at the end of each question or
part question. 1
4
2
6
3
4
4
6
5
8
6
6
7
6

Total
40
This paper consists of 16 printed pages and 0 blank page
[Turn over
2 For
examiners
Section A use

Answer all the questions.


Write your answers on the spaces provided.

1 When ethanedioate ions, C2O42, are oxidised by acidified aqueous potassium


manganate(VII), the purple solution decolourises and a gas that gives a white
precipitate in limewater is formed.

(a) Write down a balanced equation for the above reaction.

2MnO4(aq) + 5C2O42(aq) + 16H+(aq) 2Mn2+(aq) + 10CO2(g) + 8H2O(l)


[1]

(b) Calculate the volume of 0.034 mol dm3 potassium manganate(VII) required to
oxidise 1.5 103 mol of the salt KHC2O4H2C2O4 completely.

KHC2O4H2C2O4 KHC2O4 + H2C2O4


(2 moles of C2O42 are dissociated)
n(C2O42) = 3.0 103 mol [1]
2
n(MnO4) = 3.0 103 x 5

= 1.2 x 103 mol [1]


1.2 x 10 3
V(MnO4) =
0.034
= 0.0353 dm3 or 35.3 cm3 [1]

[4]

[Total: 4]
3 For
examiners
2 A fullerene is any molecule composed entirely of carbon, in the form of a hollow use

sphere, ellipsoid, or tube.

Buckminsterfullerene, the molecule C60, was the first fullerene to be discovered.

The bonding in buckminsterfullerene is similar to that in graphite.

(a) State the type of bonding between carbon atoms and between C60 molecules in
buckminsterfullerene.

Type of bonding:

between carbon atoms: covalent bonds

between C60 molecules: van der Waals forces / td-td / dispersion forces[2]

(b) Buckminsterfullerene is soluble in many organic (carbon-based) solvents. Suggest


a suitable solvent, explaining your answer.

A specific non-polar solvent which is a liquid at rtp e.g. CCl4

Non-polar solvent will dissolve in non-polar solvent

[2]

(c) Aside from its solubility in organic solvents, suggest another property of
buckminsterfullerene which is similar to that of graphite.

Soft / slippery / possess delocalized electrons

[1]

(d) Suggest a property of buckminsterfullerene which is different from that of graphite.

No electrical conductivity / low melting point or boiling point

[1]

[Total: 6]
4 For
examiners
3 Sulfuric acid is manufactured industrially by the Contact process. The raw materials use

are sulfur, air and water.

The flowchart below shows an outline of the Contact process.

Stage 1: S(s) + O2(g) SO2(g)

Stage 2: 2SO2(g) + O2(g) 2SO3(g) H = 197 kJ mol1

Operating conditions
Temperature: 450 C
Pressure: 1 atm
Catalyst: vanadium (V) oxide, V2O5

(a) SO3(g) + H2SO4(l) H2S2O7(l)


oleum

Stage 3: + H2O

(b) concentrated sulfuric acid

(a) Sketch graphs to show how the percentage yield of sulfur trioxide in the equilibrium
mixture in Stage 2 varies with

(i) pressure for two temperatures, 400 C and 600 C, respectively

% sulfur trioxide
400 C

600 C

Pressure / atm
5 For
examiners
(ii) temperature for two pressures, 0.5 atm and 2 atm, respectively. use

% sulfur trioxide

2 atm

0.5 atm

Temperature / C
Temperature / C

[2]

(b) Suggest how the percentage yield of sulfur trioxide produced in Stage 2 can be
increased without varying the operating conditions of the process.

By removing the sulfur trioxide once it is formed. [1]

(c) Write an overall balanced equation for the chemical reactions that happen in Stage
3 to produce concentrated sulfuric acid.

SO3(g) + H2O(l) H2SO4(l) [1]

[Total: 4]
6 For
examiners
4(a) By reference to the CO molecule, explain the meaning of the term bond energy. use

The energy required to break one mole of covalent bonds in the gaseous state.

[1]

(b) Write a thermochemical equation, including state symbols, to represent the average
bond energy of C H in methane.

CH4(g) C(g) + H(g)[1]

(c) With help of the given energy cycle, make use of the following enthalpy changes
and appropriate data from the Data Booklet to estimate the average bond energy of
C H in methane.

Enthalpy change of combustion of methane = 882 kJ mol-1.


Enthalpy change of formation of carbon dioxide = 394 kJ mol-1
Enthalpy change of formation of water = 242 kJ mol-1
Enthalpy change of atomisation of carbon = +720 kJ mol-1

H
C(s) + 2H2(g) + 2O2(g) CH4(g) + 2O2(g)

394 +2(242) 882

CO2(g) + 2H2O(l)

H = 394 + 2(242) (882) = +4 kJ mol-1

+4 = Bonds broken bonds formed


= [720 + 2(436)] [4E(C H)]
E(C H) = +397 kJ mol-1

1 mark for using energy cycle correctly


2 marks for using bond energy method
1 mark for correct answer

[4]

[Total: 6]
7 For
examiners
5 Sodium was first isolated in 1807 and argon was discovered in 1894. Although they use

are both in the third period of the Periodic Table, there is considerable variation of
properties from sodium to argon.

(a) On the axes below, sketch graphs to show the variation in the following properties
of elements across the third period from Na to Ar.

(i) atomic radius

(ii)

[2]

(b) The variation in both atomic radius and first ionisation energy of the third period
elements can be explained using nuclear charge, shielding effect and effective
nuclear charge.

Explain the term shielding effect and hence use the three terms in italics above to
explain the variation in atomic radius.

Shielding effect
The decrease in nuclear attraction experienced by an electron due to it being
blocked by inner shell of electrons.
or words to that effect

Variation in atomic radius


Across the period, nuclear charge increase, shielding effect remains relatively
constant, effective nuclear charge increase.
[2]
8 For
examiners
(b) The properties of oxides of the elements of the third period depend on the use

difference in electronegativity values between the element and oxygen.

The table below shows the electronegativity of elements of the second and third
period as proposed by Linus Pauling.

Li Be B C N O F Ne
0.98 1.57 2.04 2.55 3.04 3.44 3.98

Na Mg Al Si P S Cl Ar
0.92 1.31 1.61 1.90 2.19 2.58 3.16

(i) Calculate the difference in electronegativity between element and oxygen for
the oxides BeO, MgO, Al2O3 and SiO2.

BeO MgO Al2O3 SiO2

1.87 2.13 1.83 1.54

(ii) Hence use your values in (b)(i) to predict the following properties of beryllium
oxide, BeO.

structure and bonding

Giant ionic lattice structure with strong ionic bonds between Be2+ and O2
ions.

pH when added to water

Reactions with aqueous HCl and aqueous NaOH respectively

BeO + HCl BeCl2 + H2O


BeO + 2NaOH + 2H2O Na2Be(OH)4

accept : BeO + NaOH + H2O NaBe(OH)3


: BeO + 2NaOH Na2BeO2 + H2O

.......
[4]

[Total: 8]
9 For
examiners
6 Trans-9-oxodec-2-enoic acid is a primer pheromone used by the queen bee to use

regulate the honeybee caste system.

O
CH3C(CH2)5 H
C=C
H CO2H

trans-9-oxodec-2-enoic acid

Draw structures of the organic products when trans-9-oxodec-2-enoic acid reacts


with

(a) cold dilute KMnO4

O H H
CH3C(CH2)5CCCO2H n
HO OH

(b) aqueous alkaline I2

n O n
CHI3 and OC(CH2)5 H
C=C
H CO2

(c) HCN with a trace of NaCN

OH n
CH3C(CH2)5 H
C=C
CN
H CO2H

(d) CH3CH(OH)CH3 in the presence of concentrated H2SO4.

O
CH3C(CH2)5 H n shown as propyl ester
C=C o shown as branched ester
H CO2CH(CH3)2

[6]

[Total: 6]
10 For
examiners
7(a) Ethanoic acid, CH3CO2H, is a stronger acid than ethanol, CH3CH2OH. use

Explain the above statement.

Strength of acid depends on stability of conjugate base and CH3CO2 is more


stable than CH3CH2O. n

The negative charge in CH3CO2 is delocalized between the C and O atoms,


stabilizing this conjugate base, making the acid strong. n

In CH3CH2O, the ethyl group is electron-donating and pushes electrons


towards the negatively charged O atom, making the ion more unstable. n
[3]

(b) Give reagents, conditions and observations to show how you would distinguish
between the following compounds.

CH3CH2CH2Cl CH3CH2CH2OH
Test 1: Add ethanolic AgNO3 and warm.
Reagents Test 2: Add Na metal
and
conditions Test 3: Add PCl5 n
Test 4: Add KMnO4
Test 1: White ppt formed. No ppt formed.
Test 2: No gas evolved.
H2 gas evolved. n
Observations n
HCl evolved.
Test 3: No gas evolved.
Test 4: No decolorisation KMnO4 decolorised

[3]

[Total: 6]
11
Section B
Answer any two of the three questions.
Write your answer on the writing paper provided.

1(a) Many new cars have air-bags which rapidly inflate during an accident and protect
the front passengers. The air-bag is inflated with nitrogen released by the rapid
decomposition of sodium azide, NaN3.

(i) Write a balanced equation for the likely decomposition of sodium azide.

2NaN3 2Na + 3N2 .(i) n eqn


or 3NaN3 Na3N + 4N2 .(ii) n balanced

(ii) What would you consider a likely volume for an inflated air-bag, taking into
consideration that it must be big enough to protect a passenger from
knocking his head onto the windscreen? Explain your estimation.

Estimated size of air-bag to cover face and distance to windscreen


3
= 50 x 50 x 30 = 75,000 cm . n reasoning n volume
(Acceptable answer from Examiners Report of 1995 Special Paper Q3(a)
3
is 50 200 dm )

(iii) Based on your answers in (a)(i) and (ii), calculate the mass of sodium azide
needed to inflate an air-bag in a motorcar during an accident.

Based on eqn (i), mass of azide = 75 x 2 x 65 = 135.4 g


3 x 24
n n
or based on eqn (ii), mass of azide = 75 x 3 x 65 = 152.3 g
4 x 24

(iv) Draw a dot-and-cross diagram for the azide ion and suggest its shape.

N N N n

Shape is linear. (2 bond pairs and no lone pair at central N)


n
[8]

(b) Hydrogen azide, HN3, is a colourless liquid with an irritating odour. It behaves as
an acid.
3
(i) A 0.1 mol dm aqueous solution of hydrogen azide has a pH of 2.9. Explain
whether it is a strong or weak acid.

If it is a strong acid, pH = lg 0.1 = 1.0. n reasoning


Since pH given is 2.9 and not 1.0, therefore it is a weak acid. n
12
3 3
(ii) The titration curve when 20 cm of this acid is titrated with 0.2 mol dm
aqueous sodium hydroxide is shown below. Copy the diagram and label the
2 axes. Write down the values of x and y in the spaces provided.
pH
n

n
2.9 y

x n Volume of NaOH/cm3
n 10

(iii) Suggest a suitable indicator for this titration.

Suitable indicator is phenolphthalein. n

[7]

(c) Brefeldin A is an antibiotic.

OH
O

HO O CH3

Give the structures of the organic products formed when brefeldin A is reacted with
the following reagents.

(i) Bromine dissolved in tetrachloromethane

OH Br n
Br O
HO O CH3
13
(ii) Hot aqueous sodium hydroxide

OH n n
ONa OH

HO O CH3

(iii) Phosphorus pentachloride

Cl n
O
n Cl O CH3

[5]

[Total: 20]
14
2 Water, H2O, covers about two-thirds of the Earths surface and is vital to life. About
0.005% of water molecules consist of an oxygen atom bonded to two atoms of the
hydrogen isotope, deuterium, 21 D.

Deuterium oxide, D2O, is known as heavy water and is used for research in
chemical reactions because deuterium atoms react less quickly than normal
hydrogen atoms, 11 H. Like H2O, pure D2O is weakly ionised.

2D2O D3O+ + OD

For D2O, we can use the term KD instead of KW and pD instead of pH.

(a) Write expressions for the following terms.

(i) KD
KD = [D3O+][OD] [1]

(ii) pD
pD = lg [D3O+] [1]
[2]

(b) For pure D2O, KD = 1.35 x 1015. Calculate the values of the following.

(i) [D3O+]
[D3O+] = (1.35 x 1015)1/2 = 3.67 x 10-8 mol dm-3 [1]

(ii) pD
pD = lg(3.67 x 10-8) = 7.43 [1]
[2]

(c) When pure H2O and pure D2O are mixed, exchange of H and D atoms takes
place and the following equilibrium is established.

D2O(l) + H2O(l) 2HDO(l) Kc = 3.56 at 298 K

(i) If a mixture containing 1.5 mol of D2O and 1.5 mol of H2O is made up,
how many moles of HDO will be present at equilibrium at 298 K?

D2O(l) + H2O(l) 2HDO(l)


ninitial / mol 1.5 1.5 0
n / mol x x +2x
nfinal / mol 1.5 x 1.5 x 2x [1]

Kc = [HDO]2/[D2O][H2O] = 3.56 [1]


(2x/V) /(1.5 x/V) = 3.56 (Assume total volume = V dm3)
2 2

2x/(1.5 x) = 1.886
x = 0.7283 mol
n(HDO)eqm = 2(0.7283) = 1.456 = 1.46 mol [1]
15
(ii) Describe and explain the effect on the position of this equilibrium if
HDO is being drained away once it is formed.
Position of equilibrium would shift to the right [1] to replace the
removed HDO. [1]
[5]

(d) The following list of compounds react with or dissolve in water:

ethanoic acid, ammonia, sodium hydroxide, sodium chloride.

Water can react as either acid or a base. Choose a compound from the
above list with which water acts as

(i) a Bronsted base,


H2O + CH3CO2H CH3CO2- + H3O+ [1] ; [1] identify
(ii) a Bronsted acid.
H2O + NH3 NH4+ + OH- [1]

Construct a balanced equation for each reaction. [3]

(e) A solution containing ethanoic acid and its salt sodium ethanoate,
CH3CO2Na, acts as a buffer solution.

(i) What do you understand by the term buffer solution?


Buffer solution is a solution that resists large changes in pH
when small amounts of acid or base are added. [1]

(ii) Write ionic equations to show how this solution reacts with
I added H+(aq) ions,
CH3CO2 + H3O+ CH3CO2H [1]
II added OH(aq) ions.
CH3CO2H + OH CH3CO2 [1]
[3]

(f) Ethanoic acid can be synthesised from ethane by a 3-stage reaction


sequence:
I II III
CH3CH3 F G CH3CO2H

(i) Suggest the reagents and conditions necessary for reactions I, II and
III.
I: Cl2 or Br2, UV light [1]
II: aq NaOH, heat [1]
III: acidified KMnO4, heat [1]

(ii) Draw the structural formulae of the intermediates F and G.

F: CH3CH2Cl or CH3CH2Br; [1]


G: CH3CH2OH [1]
[5]
[Total: 20]
16
3(a) Bones contain a complex mixture of calcium salts, proteins and other material.
When a bone is strongly heated in a current of air, the only residue is calcium oxide.

CaCO3(s) CaO(s) + CO2(g)

(i) Name the type of reaction that occurs when a bone is strongly heated in air.
thermal decomposition

(ii) From a sample of 50.0 g of bone, 14.0 g of calcium oxide were obtained.
Calculate the percentage by mass of calcium in the bone.
40.1
( ) (14.0)
% by mass of Ca in the bone = 40.1 + 16.0 x 100% = 20.0%
50.0

[2]

(b) Calcium can be obtained from many different sources. Primarily, milk is the richest
source of dietary calcium.

The following information was taken from the side of a carton of milk.
How long does your milk keep?

25 C 0.5 day
18 C 1 day
10 C 2 days
5 C 4 days

(i) Plot a graph of the time the milk keeps against temperature.

y-axis: time x-axis: temperature


Join the points with a smooth curve, not straight lines.
The curve has a negative slope.

(ii) Use your graph to predict the length of time the milk will keep at a
temperature of 15 C.

1.35 days

(iii) The souring of milk is a chemical reaction.


How is the rate of this reaction related to the length of time the milk keeps?

1
Rate
time
17
(iv) Explain how and why the rate of souring of milk varies with temperature.

The rate of souring reaction increases with increasing temperature.

At higher temperatures, the average kinetic energy of particles increases.


More particles have energy activation energy. Hence frequency of
activated collisions increases. Rate increases.

(v) Suggest a reason why milk that has been boiled keeps longer than fresh milk.

The catalysts are enzymes which would be denatured at 100 C.

[8]
(c) Cheese, which is also rich in calcium, contains esters of linoleic acid.

CH3(CH2)4CH=CHCH2CH=CH(CH2)7CO2H
linoleic acid

On the lid of a brand of cheese it is claimed that the cheese contains virtually no
trans fatty acids.

Representing the formula by the abbreviated structure ACH=CHCH2CH=CHB,


draw the structural formula of the isomer of linoleic acid that could be present in the
cheese.

A B
C CH2 C
H C C H
H H

[2]

(d) An organic compound, A, has the following composition by mass.


C, 60.0%; H, 13.3%; O, 26.7%

It has a relative molecular mass of 60.

When A is heated under reflux with an excess of acidified K2Cr2O7, the solution
turns from orange to green and Compound B is formed. B turns moist blue litmus
paper red.

However, heating A with acidified K2Cr2O7 followed by immediate distillation gives a


compound C.

When 2,4-dinitrophenylhydrazine is added to C, an orange-yellow crystalline solid is


produced. C also reacts with Fehlings solution and Tollens reagent.

(i) Calculate the molecular formula of A.


C3H8O
18

(ii) Identify the compounds A, B and C. Write clearly the deductions that you
made from the information given.

A is CH3CH2CH2OH.
B is CH3CH2CO2H.
C is CH3CH2CHO.

AB
Observations: B turns moist red litmus paper red.
The colour of the solution turns from orange to green.
Deductions: B is a carboxylic acid.
So, A must be a 1 alcohol.
The orange K2Cr2O7 solution turns to green Cr3+ solution.

AC
Observations: C reacts with 2,4-DNPH to form an orange-yellow crystalline solid.
C also reacts with Fehlings solution and Tollens reagent.
Deductions: C must be an aldehyde.
C reacts with Fehlings solution to form a brick-red ppt.
C reacts with Tollens reagent to form a silver mirror.

[8]

[Total: 20]

End of Paper
PIONEER JUNIOR COLLEGE, SINGAPORE

JC 2 PRELIMINARY EXAMINATION 2010


HIGHER 1

CHEMISTRY 8872/01

Paper 1 Multiple Choice 21 SEPTEMBER 2010


Additional materials: Data Booklet
Multiple Choice Answer Sheet 50 MINUTES

READ THESE INSTRUCTONS FIRST

Write in soft pencil.


Do not use staples, paper clips, highlighters, glue or correction fluid.
Write your name, Centre number and index number on the Answer Sheet in the spaces provided
unless this has been done for you.

There are thirty questions on this paper. Answer all questions. For each question there are four
possible answers A, B, C and D.
Choose the one you consider correct and record your choice in soft pencil on the separate
Answer Sheet.

Read the instructions on the Answer Sheet very carefully.

Each correct answer will score one mark. A mark will not be deducted for a wrong answer.
Any rough working should be done in this booklet.

A calculator may be used.

This document consists of 12 printed pages.

Pioneer Junior College [Turn Over


2
Section A

For each question, there are four possible answers labelled A, B, C and D. Choose the
one you consider to be correct.

1 How many hydrogen atoms are there in 9.0 g of glucose, C6H12O6 ?

A 6.0 x 1022
B 3.6 x 1023
C 6.0 x 1023
D 3.6 x 1024

2 Ten percent of the copper produced in the USA comes from bacterial leaching of low
grade copper ores. In this process acidified water is sprayed onto the ore
chalcopyrite. Bacteria then convert the insoluble ore into a solution containing iron
and copper ions.

4 CuFeS2 + 17 O2 + 4 H+ 4 Cu2+ + 4 Fe3+ + 8 SO42- + 2 H2O

No change occurs in the oxidation state of copper

What changes in oxidation state occur for the iron and the sulfur in this reaction?

Change in oxidation state


Fe S
A -1 -6
B -1 +6
C +1 -8
D +1 +8

3 0.01 mol aqueous sodium sulfate (V), Na2SO4 was found to react completely with
0.02 mol hydroxylamine, NH2OH. The half equation for the reduction of sulfate(VI)
ion is given as shown.

2SO42- (aq) + 10H+ (aq) + 8e- S2O32-(aq) + 5H2O (l)

Which of the following could be the nitrogen containing product in this reaction?

A N2O
B NO
C NO2
D NO3-
3
4 After 10 cm3 of a hydrocarbon had been burnt with 70 cm3 of oxygen, the gaseous
mixture occupied a volume of 50 cm3 which was reduced by 30 cm3 by the addition
of aqueous sodium hydroxide. Assuming all volumes are measured at room
temperature and pressure, which one of the following is a possible formula of the
hydrocarbon?

A C3H6
B C3H8
C C4H8
D C4H10

5 What is the volume of 0.100 mol dm-3 KMnO4 aqueous solution required to oxidise a
25.0 cm3 aliquot of 0.300 mol dm-3 FeSO4 aqueous solution?

A 10.00 cm3
B 12.50 cm3
C 15.00 cm3
D 25.00 cm3

6 The first seven ionisation energies of an element A are shown in the sketch graph.

Ionisation energy

Electrons
1 2 3 4 5 6 7 removed

What could be the identity of element A?

A aluminium
B magnesium
C phosphorus
D silicon
4
7 Which of the species in their gaseous state shown below will be deflected by the
largest angle in an electric field?

11
A B3+
4
B He2+
9
C Be2+
1
D H2

8 Which of the following sets of species all have the same shape?

A BCl3 NH3 PCl3


B BeCl2 CO2 SO2
C CH4 NH4+ SiF4
D CO2 H2O SO2

9 The boiling point of the hydrogen fluoride is higher than Hydrogen iodide as follow:

Boiling point of HF = 19.4oC > Boiling point of HI = 34.36 C

Which of the options best describe the phenomenon?

A HF has higher boiling point due to stronger intermolecular hydrogen bonding than
permanent dipole permanent dipole between HI molecules.
B HF has higher boiling point due to a higher H-F bond energy than H-I
C HF has higher boiling point due to stronger induced dipole induced dipole
intermolecular forces than HI
D HF has a higher boiling point due to HF having stronger ionic bonds as it has a
higher magnitude of lattice energy since fluoride ion in HF is smaller than Iodide
ion in HI.

10 Compound B has the structure shown below:

CN

CH 3 C C OH Compound B

H O

Which of the following options shows the correct number of sigma and pi bonds?

Number of sigma bonds Number of pi bonds


A 10 3
B 10 1
C 11 3
D 11 1
5
11 Bromine is added to compound D shown below. The mixture is immediately placed in
the dark and left until no further change takes place.

CH3

H
C
H C
H Compound D

What are likely to be the main product(s)?


CH3
A
H
H C
C
Br H
Br

B CH2Br

H
C
H C
H

C CH3 CH3

Br
H H
H C and H C
C C
Br H Br H
Br Br Br

D
CH2Br CH2Br

Br
H H
H H C
C and
C C
Br H Br H
Br Br Br
6
12 Which of the following options shows the correct reagents and conditions for
Step 1 and correct structure of Compound E?
Hot K2Cr2O7(aq), H2SO4 (aq)
Step 1
CH2(Cl)CH=CH2 CH2(CN)CH=CH2 Compound E

Step 1 Compound E
A KCN in ethanol, heat CH2(CO2H)2
B KCN in ethanol, heat CH2(CO2H)CH=CH2
C KOH in ethanol, heat CH2(CO2H)2
D KOH in ethanol, heat CH2(CO2H)CH=CH2

13 The following unsaturated hydrocarbon has the structure shown below:

H H C CH3
C C CH2 C
CH3 CH C H
CH3 C CH3
CH3

How many pairs of geometrical isomers of this molecule exist?

A 0
B 1
C 2
D 3

14 A compound F has the properties below:

Fehlings solution, on warming with F, retains its blue colour


With hydrogen cyanide and aqueous sodium cyanide, F produces C5H7NO2
On reaction with warm aqueous alkaline iodine, yellow precipitate is formed

What could compound F be?

A CH2=CHCOCH2OH
B CH3COCH2CHO
C CH3COCOCH3
D CH3COCH=CHOH
7
15 Aldehydes and ketones are produced industrially by the catalytic oxidation of
alkenes. For example, ethanal is manufactured from ethene as shown.

catalyst
CH2=CH2 + O2 CH3CHO

The process is also used industrially with but-2-ene, CH3CH=CHCH3

Which compound is obtained from but-2-ene ?

A CH3CH2CHO
B CH3CH2COCH3
C CH3CH2CH2CHO
D (CH3)2CHCHO

16 Compound G is an organic compound which has the structure shown below:

Cl OH
CH3 C C H
H H

What deduction about compound G is wrong ?

A It is produced between reaction of prop-1-ene with aqueous chlorine.


B It forms white precipitate with hot ethanolic silver nitrate
C It contains a secondary halogenoalkane and primary alcohol functional group.
D It is oxidised to a ketone and carboxylate salt with hot alkaline potassium
manganate

17 Which equation defines the enthalpy change of formation of carbon monoxide?

A C(g) + O(g) CO (g)


B C(g) + O2(g) CO(g)
C C(s) + O2 (g) CO (g)
D C(s) + CO2 (g) 2 CO(g)
8
18 The energy diagram represents the reaction occurring with and without a catalyst.

E1
E4
E2

E3

Which of the following statements is correct?

A E4 is the activation energy for the reverse catalysed reaction


B The forward reaction with catalyst is endothermic
C The enthalpy change of reaction is reduced by using a catalyst
D The enthalpy change of reaction is (E2 - E3)

19 The enthalpy change of neutralisation for the reaction between sodium hydroxide
solution and sulfuric acid is to be calculated.

2NaOH (aq) + H2SO4 (aq) Na2SO4 (aq) + 2H2O(l)

40.0 cm3 of 1.00 mol dm-3 of sodium hydroxide solution is reacted with 10.0 cm3 of
1.00 mol dm-3 of sulfuric acid solution. The temperature rises by 5.4oC.

If the density and specific heat capacity of all solutions are assumed to be 1.00 g cm-3
and 4.18 J g-1 K-1 respectively, what is the enthalpy change of neutralisation of the
reaction?

A - 28.2 kJ mol-1
B - 56.4 kJ mol-1
C -112.8 kJ mol-1
D -225.7 kJ mol-1

20 In which reaction is ammonia acting as an acid?

A 2 NH3 (l) + 2 Na (s) 2 Na+ NH2- (s) + H2 (g)


B 2 NH3 (g) + 3 CuO (s) 3 Cu (s) + N2 (g) + 3 H2O (g)
C 2 NH3 (aq) + Ag+ (aq) [Ag(NH3)2]+ (aq)
D NH3 (g) + H2O (l) NH4+ (aq) + OH- (aq)
9
21 What is the pH of a 0.100 moldm-3 barium hydroxide solution, Ba(OH)2 (aq) at 25oC ?
( Given that ionic product of water, Kw = 1.00 x 10-14 mol2 dm-6 at 25oC )

A 0.70
B 1.00
C 13.0
D 13.3

22 The reaction between hydrogen and nitrogen monoxide can react to form nitrogen
and steam.

2 H2 (g) + 2 NO(g) N2 (g) + 2 H2O (g)

The rate of reaction is first order with respect to hydrogen and second order with
respect to nitrogen monoxide.

0.100 mol dm-3 of H2 and 5.00 mol dm-3 of NO (g) were put into a sealed flask of fixed
volume. It was found that there was 0.0250 mol dm-3 of H2 left 40 minutes later.

Which statement is true?

A An excess concentration of 5.00 mol dm-3 NO (g) is used so that the rate of
reaction will remain constant as reaction proceeds.
B There was 0.0250 mol dm-3 of N2 (g) left after 20 minutes.
C There was 0.0125 mol dm-3 of H2 (g) left after 80 minutes.
D The unit of the rate constant is s-1.

23 In acid solution BrO3- ions will slowly oxidise Br- ions to Br2.

BrO3 - + 5 Br - + 6H+ 3 Br2 + 3H2O

The variation for the initial rate of reaction with the concentrations of the reactants
was investigated in four experiments.

Experiment [BrO3 -] / moldm-3 [Br -] / moldm-3 [H+] / moldm-3 Relative rate


number
1 0.10 0.20 0.15 1
2 0.10 0.10 0.30 2
3 0.20 0.10 0.30 4
4 0.20 0.20 0.30 8

What is the rate equation for this reaction?

A rate = k[BrO3-]2 [Br-] [H+]


B rate = k[BrO3-] [Br-]2 [H+]
C rate = k[BrO3-] [Br-] [H+]2
D rate = k[BrO3-] [Br-] [H+]
10
24 Which of the following sets of oxides when added to water will form an acidic,
an alkaline and a neutral solution ?

A Al2O3 SiO2 P4O10


B Na2O MgO Al2O3
C MgO Al2O3 SiO2
D MgO Al2O3 P4O10

25 Consider the period 3 elements and their compounds with their properties sketched
in the form of graphs below:

Graph 1 Graph 2

Atomic number Atomic number

What does Graph 1 and Graph 2 represent?

Graph 1 Graph 2
A melting point of elements first ionisation energy of elements
B melting point of elements atomic radius of elements
C pH of their chlorides in water first ionisation energy of elements
D pH of their chlorides in water atomic radius of elements
11
Section B

For each of the next 5 questions, one or more of the three numbered statements 1 to 3
may be correct.

Decide whether each of the statements is or is not correct (you may find it helpful to put a
tick against the statements which you consider to be correct).

The responses A to D should be selected on the basis of

A B C D
1, 2 and 3 are 1 and 2 only are 2 and 3 only are 1 only is correct
correct correct correct

No other combination of statements is used as a correct response.

26 Calcium has a lower first ionisation energy than magnesium.

Which statements help to explain this?

1 The outer electrons in calcium experiences greater shielding than that in


magnesium.
2 The outer electron in calcium is further away from the nucleus of the atom than
that in magnesium.
3 The Calcium atom has eight more protons in the nucleus than the magnesium
atom.

27 When propanone is reacted with hydrogen cyanide, the progress of the reaction can
be followed by plotting the concentration of hydrogen cyanide against time. One such
plot is given below:

[hydrogen cyanide] / mol dm-3

time / min
What conclusions can be drawn from this result ?

1 The reaction is zero order with respect to hydrogen cyanide.


2 For mixtures containing same concentration of propanone, the gradient of the line
depends on the initial concentration of hydrogen cyanide.
3 The order of reaction with respect to propanone is one.
12
28 The reaction below shown is reversible. For every one mole of gaseous A2 that
dissociate, two moles of gaseous A atoms are produced.

1 A 2 (g) 2 A (g) H = 0 kJ mol-1

Which of the following will not increase the yield of gaseous A atoms?

1 using a catalyst
2 increase in pressure
3 increase in temperature

29 The diagram represents a simplified section of a catalytic converter as fitted into the
exhaust system of a car. Harmful gases are converted into carbon dioxide, nitrogen
and water vapour.
Platinum and rhodium catalyst

CO CO2
Hydrocarbons H2O
NOx N2
Which reactions between the stated compounds could take place on the surface of
the catalyst?

1 carbon monoxide + hydrocarbons carbon dioxide + water


2 carbon monoxide + oxides of nitrogen carbon dioxide + nitrogen
3 hydrocarbons + oxides of nitrogen carbon dioxide + water + nitrogen

30 Which type of reaction(s) are not shown in the synthesis shown below ?

CH3CH2Cl CH3CH2CN CH3CH2COOH CH3CH2CH2OH

1 Oxidation
2 Reduction
3 Substitution

1B 7B 13 C 19 B 25 A
2D 8C 14 D 20 A 26 B
3A 9A 15 B 21 D 27 D
4B 10 C 16 A 22 B 28 A
5C 11 A 17 C 23 C 29 C
6D 12 B 18 D 24 D 30 D

* End Of Paper *
1

Name: Index No.: CT Group: 09

PIONEER JUNIOR COLLEGE, SINGAPORE

JC 2 PRELIMINARY EXAMINATION 2010


HIGHER 1

CHEMISTRY 8872/02
Paper 2 21 SEPTEMBER 2010
2 hours
Candidates answer Section A on the Question Paper
Additional Materials: Answer Paper
Data Booklet

READ THESE INSTRUCTONS FIRST

Write your Centre number, index number and name on all work you hand in.
Write in dark blue or black pen on both sides of the paper.
You may use a soft pencil for any diagrams, graphs or rough workings.
Do not use staples, paper clips, highlighters, glue or correction fluid.

Section A
Answer all the questions

Section B
Answer two questions on separate answer paper

You are reminded of the need for good English and clear presentation in your answers.

At then end of the examination, fasten all your work securely together.
The number of marks is given in brackets [ ] at the end of each question or part question.

FOR EXAMINERS USE


PAPER 1 SECTION A SECTION B
1 / 16 4 / 20
/ 30 2 / 14 5 / 20
3 / 10 6 / 20
TOTAL / 40 TOTAL / 40
PERCENTAGE / 100 GRADE

This document consists of 14 printed pages.

Pioneer Junior College


2

Section A

Answer all questions in this section

1 This question is about elements and their compounds in period 3. For


Examiners
Use
(a) Both magnesium and phosphorus can form chlorides. Magnesium chloride,
MgCl2, has a melting point of 714 C. Phosphorus (III) chloride, PCl3, a
trigonal pyramidal molecule, can react further with chlorine to form
phosphorus (V) chloride, PCl5, a trigonal bipyramidal molecule.

(i) Construct a dot-and-cross diagram for magnesium chloride:

(ii) Magnesium oxide has a melting point of 2852 C. Explain why


magnesium oxide has a higher melting point than magnesium chloride.

[3]

(b) (i) Explain why PCl3 is not a trigonar planar molecule using the valence
shell electron pair repulsion theory.
3

(ii) Explain why PCl5 can exist but not MgCl5 ? For
Examiners
Use

[4]

When PCl5 dissolves in water, it hydrolyses in water to produce hydrochloric acid,


HCl(aq) and phosphoric acid, H3PO4(aq) with pH = 1

When aluminum chloride is dissolved in water, it also hydrolyses in water to


produce an acidic solution.

Both acidic solutions produced from the reaction of PCl5 and AlCl3 with water can
react with sodium carbonate to give carbon dioxide gas.

(c) (i) Write a balanced equation to show the reaction between PCl5 and
water.

(ii) Explain using equation(s) why aluminium chloride reacts with water to
produce an acidic solution ?

(iii) The aqueous chlorides of the period 3 element give different pH.

Write down the pH of the aqueous chlorides in the table provided.

aqueous aqueous aqueous aqueous aqueous


NaCl MgCl2 AlCl3 SiCl4 PCl3
pH
4

(iv) Construct a dot-and-cross diagram of the carbonate anion in sodium For


carbonate. Examiners
Use

[7]

(d) The electrical conductivity of some Period 3 elements is shown below.

Na Mg Al Si P S

(ii) Explain why there is an increasing electrical conductivity from sodium


to aluminium?

(iii) Explain why there is no electrical conductivity for phosphorus and


sulfur?

[2]
[Total 16]
5

2 For each of the following organic compounds, determine the type of isomerism For
they exhibit and suggest a simple chemical test to distinguish them, indicating the Examiners
observations you would see. Use

(a)
H H OO

3 CC OOH CH 2 CH 3
CH3 CCHC and
H H ethyl ethanoate
butanoic acid

(i) Type of isomerism: ________________________________

(ii) Reagents and conditions: ________________________________

________________________________

Observations for butanoic acid:

_____________________________________________________

_____________________________________________________

Observations for ethyl ethanoate:

_____________________________________________________

_____________________________________________________

[3]
6

(b) For
CH 3 CH3 H H Examiners
Use
C C CH3 C C C
H H H H H

but-2-ene but-1-ene

(i) Type of isomerism: _____________________________________

(ii) Reagents and conditions: ________________________________

________________________________

Observations for but-2-ene:

_____________________________________________________

_____________________________________________________

Observations for but-1-ene:

_____________________________________________________

_____________________________________________________
[4]

(c) But-2-ene can exhibit other types of isomerism besides the one mentioned
in (b)(i)

Draw each isomer belonging to the two other types of isomerism in each of
the two boxes shown below.

(i) (ii)

[2]
7

(d) The diagram shown below is an energy cycle involving but-1-ene at 25oC For
and 1 atmospheric pressure. Examiners
Use
H1
4C(g) + 4H2(g) CH3CH2CH=CH2 (g)

+ 6O2(g) + 6O2(g)

H2 H3

4CO2(g) + 4H2O(l)

(i) Calculate the standard enthalpy change, H1 above using the


following bond enthalpies.

Bond Bond enthalpy / kJ mol-1


H-H 436
C-H 410
C-C 350
C=C 610

(ii) The standard enthalpy change, H 1 does not represent the standard
enthalpy change of formation of but-1-ene. Explain why.
8

(iii) Use the following data below as well as your answer in (d)(i) to For
calculate the standard enthalpy change of combustion of but-1-ene Examiners
Use
H c( C(g) ) = - 1110 kJ mol-1
H c(H2) = - 286 kJ mol-1

[5]
[Total: 14]
9

3 Apricot kernels contain glycoside amygdalin, an organic compound which turns For
into deadly hydrogen cyanide acid, HCN, when the kernel is crushed or injured. Examiners
High doses of hydrogen cyanide can cause coma with seizures and cardiac arrest, Use
leading to death in a matter of minutes. A fatal dose for a human can be as low as
1.50 mg kg-1 of body weight. (1 mg = 1.00 x 10-3 g)

The forensics department of the local law enforcement agency was trying to
determine the cause of death of a 75 kg deceased man who was found at home
on the couch with a few empty packets of apricot kernels lying on the ground.

A 10 cm3 sample of blood (a typical human has 71 cm3 of blood per kg body
mass) obtained from the body and hydrogen cyanide containing haemoglobin was
isolated and dissolved to form 25 cm3 of solution. This sample of the haemoglobin
was oxidised by potassium dichromate, K2Cr2O7, to determine the amount of Fe2+
which is also equal to the amount of hydrogen cyanide present.
1.70 cm3 of 0.00100 mol dm-3 acidified K2Cr2O7 solution was required for complete
reaction.

(a) Write a redox equation between Fe2+ and Cr2O72-.

[1]

(b) Show by oxidation number that the reaction in (a) is a redox reaction.

[1]

(c) Calculate the total amount of hydrogen cyanide, HCN in the 25 cm3 of
solution.

[2]

(d) Calculate the amount of hydrogen cyanide, HCN, in the body of the
deceased man.

[1]
10

(e) Calculate the concentration of HCN in mg kg-1 and hence determine if the For
cause of death was due to hydrogen cyanide poisoning. Examiners
Use

[2]

(f) (i) Given that the 25 cm3 sample of hydrogen cyanide solution in (c) has a
pH of 6.30, show quantitatively that HCN is a weak acid.

(ii) What is the effect of adding universal indicator to this 25 cm3 sample of
hydrogen cyanide solution mentioned in (f)(i)?

[2]

(g) Explain why potassium chloride, KCl , cannot be used to oxidise Fe2+?

[1]
[Total: 10]
11

Section B

Answer two of the three questions in this section on separate answer paper

4 An organic compound, E, has the molecular formula C11H14O2. When E is heated


with aqueous HCl, benzoic acid and F with formula C4H9OH, is formed. When F is
heated with hot Al2O3, G, C4H8 is produced. Heating G with acidified potassium
manganate(VII) produces H, C3H6O. H produced an orange precipitate with
2,4-dinitrophenylhydrazine but does not react with Tollens reagent.

(a) Draw the structures of compounds E to H and explain the above reactions.
Write chemical equations to support your answer. [10]

1-aminopropan-2-ol is produced from ethanol by the following synthesis pathway.


H H
H O reaction 1 H O reaction 2 H O H H
H C C H H C C H J H C C C N
H H H H H H H

1-aminopropan-2-ol
(b) (i) Draw the displayed formula of compound J.

(ii) Write the reagents and conditions for reactions 1 and 2. [3]

Alkene is an unsaturated organic compound and undergoes addition. Benzene


which is also an unsaturated organic compound undergoes substitution.

(c) Explain why benzene undergoes substitution instead of addition? [2]

Chlorofluorocarbons, CFCs, are small alkane molecules where some hydrogen


atoms have been substituted by chlorine and fluorine atoms. CFCs are used as
aerosol propellant in spray can and as refrigerant. A typical CFC is CH3CHClF.

(d) (i) Explain why the synthesis of CH3CHClF from fluoroethane, CH3CH2F
and chlorine produces a mixture of products?

(ii) CH3CHClF can be converted to CH3CH(OH)F. Give the reagents and


conditions for this reaction.

(iii) Explain why C-F bond in CFC does not undergo the reaction in (d)(ii)?

(iv) Why does this CFC present an environmental concern?

(v) Suggest one way to reduce the use of CFCs. [5]


[Total: 20]
12

5 Sulfuric acid, H2SO4, is a strong mineral acid which is an important chemical used
in industry and in the school laboratory.

One step in the manufacture of sulfuric acid by the contact process involves the
reaction between sulfur dioxide and oxygen.

2 SO2 (g) + O2 (g) 2 SO3 (g)

When 72 dm3 of sulfur dioxide gas and 32.0 g of oxygen gas are reacted in a
27000 dm3 reaction vessel at a constant temperature of 25oC and 1 atmospheric
pressure, the percentage conversion of sulfur trioxide was 80% at equilibrium.

(a) (i) Calculate the initial amount of sulfur dioxide gas and initial amount of
oxygen gas at 25oC and 1 atmospheric pressure.

(ii) Explain which reagent is the limiting reagent.

(iii) Calculate the equilibrium constant with its units.

(iv) What will happen to the yield of SO3(g) when the reversible reaction at
equilibrium is conducted at 2 atmospheric pressure? [6]

(b) (i) Sulfuric acid is a dibasic strong acid. Explain the underlined term.

(ii) A 100 cm3 of solution contains 0.980 g of sulfuric acid. Calculate the
pH of the sulfuric acid solution. [4]

The titration curve is obtained when sodium hydroxide is added to sulfuric acid.
The equivalence point is reached when 25.0 cm3 of sodium hydroxide is added.
pH

Phenolphthalein
Equivalence pH = 7

Methyl orange

0 25.0 50.0
vVolume
olume ofofalkali 3
NaOH / cm
(cm3)

Methyl orange and litmus are suitable indicators to determine the equivalence
point of the titration between sulfuric acid and sodium hydroxide solution.
13

Working pH
Indicator range
Methyl 3.2 4.4 Colour of methyl orange Colour of methyl orange
Orange < pH 3.2 (red) > pH 4.4 (yellow)

litmus 6.0 8.0 Colour of litmus Colour of litmus


< pH 6.0 (red) > pH 8.0 (blue)

(c) (i) Explain why methyl orange is a suitable indicator?

(ii) Explain why litmus is a suitable indicator? [3]

Sulfuric acid is needed in many organic reactions. The reaction scheme below
shows the reaction involving sulfuric acid.

M
Concentrated H2SO4,
CH3COOH, heat

Excess concentrated H2SO4,


H2SO4(aq), heat CN OH 170oC
L C CH P+Q
CH2 CH CH2Cl + other
CH3 minor
K product(s)

Concentrated
H2SO4, followed by
H2O, warm

(d) (i) Draw the structures of L, M, N. If other products are also produced,
draw the major product only.

(ii) Draw the structures of P and Q given that P has a higher net dipole
moment.

(iii) Explain why P has a higher net dipole moment than Q? [7]
[Total: 20]
14

6 The standard enthalpy change of formation of boron trifluoride between boron


solid and fluorine gas to produce gaseous boron trifluoride in a reaction vessel is
shown below:
B (s) + 3/2 F2 (g) BF3 (g)

(a) With reference to boron trifluoride, define the standard enthalpy change of
formation of boron trifluoride, Hf(BF3). [1]

The order of reaction with respect to fluorine is one. The half-life of the reaction is
10 days. Given that the initial concentration of fluorine is 0.600 mol dm-3.

(b) Sketch a graph of [F2] against time showing two half-lives in your graph. [2]

(c) Explain what will happen to the rate of formation of gaseous boron trifluoride

(i) when the volume of the reaction vessel is increased ?

(ii) when a bigger piece of solid Boron is used ? [4]

A copper catalyst can increase the rate of formation of gaseous boron trifluoride.

(d) (i) Write the electronic configuration of Copper atom.

(ii) Using relevant diagram, explain how the copper catalyst can increase
the rate of formation of gaseous boron trifluoride [4]

(e) (i) Write an equation to represent the first ionisation energy of Boron.

(ii) Boron has a lower first ionisation energy than Beryllium. Explain why.
[3]
(f) (i) BF3 reacts with NH3 to form molecule S. Draw a structure to represent
the molecule S.

(ii) Explain the criteria for forming the structure you have proposed in (f)(i)

(iii) Show the hydrogen bonding between two molecules of S, showing


dipoles and other relevant information. [5]

BF3 reacts with water to form a weak acid, boric acid, H3BO3 as one of its
products. Excess H3BO3 reacts with aqueous NaOH to form buffer solution
consisting of aqueous H3BO3 and aqueous NaH2BO3.

(g) Write an ionic equation to show how this buffer solution decreases the
acidity when a small amount of acid is added. [1]

END OF PAPER [Total: 20]


1

Name: Index No.: CT Group: 09

PIONEER JUNIOR COLLEGE, SINGAPORE

JC 2 PRELIMINARY EXAMINATION 2010


HIGHER 1

CHEMISTRY 8872/02
Paper 2 21 SEPTEMBER 2010
2 hours
Candidates answer Section A on the Question Paper
Additional Materials: Answer Paper
Data Booklet

READ THESE INSTRUCTONS FIRST

Write your Centre number, index number and name on all work you hand in.
Write in dark blue or black pen on both sides of the paper.
You may use a soft pencil for any diagrams, graphs or rough workings.
Do not use staples, paper clips, highlighters, glue or correction fluid.

Section A
Answer all the questions

Section B
Answer two questions on separate answer paper

You are reminded of the need for good English and clear presentation in your answers.

At then end of the examination, fasten all your work securely together.
The number of marks is given in brackets [ ] at the end of each question or part question.

FOR EXAMINERS USE


PAPER 1 SECTION A SECTION B
1 / 16 4 / 20
/ 30 2 / 14 5 / 20
3 / 10 6 / 20
TOTAL / 40 TOTAL / 40
PERCENTAGE / 100 GRADE

This document consists of 14 printed pages.

Pioneer Junior College


2

Section A

Answer all questions in this section

1 This question is about elements and their compounds in period 3. For


Examiners
Use
(a) Both magnesium and phosphorus can form chlorides. Magnesium chloride,
MgCl2, has a melting point of 714 C. Phosphorus (III) chloride, PCl3, a
trigonal pyramidal molecule, can react further with chlorine to form
phosphorus (V) chloride, PCl5, a trigonal bipyramidal molecule.

(i) Construct dot-and-cross diagram for magnesium chloride:

(ii) Magnesium oxide has a melting point of 2852 C. Explain why


magnesium oxide has a higher melting point than magnesium chloride.

q+ q
ionic bond strength | | (1/2 mark )
r+ + r

Since charge of O2- , q-, in MgO is greater than Cl- in MgCl2


(1/2 mark )

ionic radius of O2- , r- is smaller than ionic radius of Cl-,


(1/2 mark )

Hence MgO has stronger ionic bonds, more energy to break


(1/2 mark )

Hence higher melting point.


[3]

(b) (i) Explain why PCl3 is not a trigonar planar molecule using the valence
shell electron pair repulsion theory.

Phosphorus atom in PCl3 contains three regions of bonding electrons


and one lone pair. ( 1 mark )

Hence there is additional bond pair lone pair repulsion (1/2 mark)
which is greater than bond pair bond pair repulsion that exists in a
3

regular trigonal planar molecule. ( mark ). Hence the observed


shape for water molecule is trigonal pyramidal.

(ii) Explain why PCl5 can exist but not MgCl5 ? For
Examiners
Use
Phosphorus can expand its octet as it has available 3d orbitals to be
used for covalent bonding with 5 other chlorine atoms. ( 1 mark )

For ionic compound, MgCl5 to exist, it requires large amount of


energy ( 1 mark) to remove 5 electrons from Magnesium to form Mg5+ in
MgCl5.
[4]
When PCl5 dissolves in water, it hydrolyses in water to produce hydrochloric acid,
HCl(aq) and phosphoric acid, H3PO4(aq) with a pH = 1

When aluminum chloride is dissolved in water, it also hydrolyses in water to


produce an acidic solution.

Both acidic solutions produced from the reaction of PCl5 and AlCl3 with water can
react with sodium carbonate to give carbon dioxide gas.

(c) (i) Write a balanced equation to show the reaction between PCl5 and
water.

PCl5 + 4 H2O 5 HCl(aq) + H3PO4 (aq) ( 1 mark )

(ii) Explain using equation(s) why aluminium chloride reacts with water to
produce an acidic solution ?

Al3+ has high charge density. [1] It can polarise the O-H bond in water
molecules causing them to break. [1/2 mark]
H3O + is released hence the solution is acidic. [1/2 mark]

[Al(H2O)6]3+ + H2O [Al(H2O)5OH]2+ + H3O + (1 mark)


(iii) The aqueous chlorides of the period 3 element give different pH.

Write down the pH of the aqueous chlorides in the table provided


( 2 marks for this question (- mark for each wrong answer) )

aqueous aqueous aqueous aqueous aqueous


NaCl MgCl2 AlCl3 SiCl4 PCl3
pH 7 6-6.5 3 1-2 1-2
4

(iv) Construct dot-and-cross diagram of the carbonate anion in sodium For


carbonate. Examiners
Use

[7]

( 1 mark )
(d) The electrical conductivity of some Period 3 elements is shown below.

Na Mg Al Si P S

(ii) Explain why there is an increasing electrical conductivity from sodium


to aluminium?

The number of delocalised valence electrons contributed to the sea of


delocalised electrons in the giant lattice structure of metal increased
from one in sodium to three in Aluminium. ( 1 mark )

(iii) Explain why there is no electrical conductivity for phosphorus and


sulfur? [2]
[Total 16]
Phosphorus and Sulfur has a simple molecular structure.
(1/2 mark ).There is no free mobile charge carriers, hence unable to
conduct electricity (1/2 mark)
5

2 For each of the following organic compounds, determine the type of isomerism For
they exhibit and suggest a simple chemical test to distinguish them, indicating the Examiners
observations you would see. Use

(a)
H H OO

3 CC OOH CH 2 CH 3
CH3 CCHC and
H H ethyl ethanoate
butanoic acid

(i) Type of isomerism: Functional group isomerism ( 1 mark )

(ii) Reagents and conditions: Na2CO3(aq) (1 mark)

Observations for butanoic acid:

Effervescence observed, colourless gas evolved forms white


precipitate when bubbled into limewater. ( 1/2 mark )

Observations for ethyl ethanoate:

No observable change ( 1/2 mark )


[3]
6

(b) For
CH 3 CH3 H H Examiners
Use
C C CH3 C C C
H H H H H

but-2-ene but-1-ene

(i) Type of isomerism: Positional isomerism ( 1 mark )

(ii) Reagents and conditions: KMnO4(aq), H2SO4(aq), heat ( 1 mark )

Observations for but-2-ene:

Purple KMnO4 solution decolourised. ( 1/2 mark )

Observations for but-1-ene:

Purple KMnO4 solution decolourised ( 1/2 mark ), effervescence


observed, colourless gas evolved which forms white precipitate when
bubbled into limewater. ( 1 mark )
[4]

(c) But-2-ene can exhibit other types of isomerism besides the one mentioned
in (b)(i)

Draw each isomer belonging to the two other types of isomerism in each of
the two boxes shown below.

(i) (ii)

( 1 mark each )

[2]
7

(d) The diagram shown below is an energy cycle involving but-1-ene at 25oC For
and 1 atmospheric pressure. Examiners
Use
H1
4C(g) + 4H2(g) CH3CH2CH=CH2 (g)

+ 6O2(g) + 6O2(g)

H2 H3

4CO2(g) + 4H2O(l)

(i) Calculate the standard enthalpy change, H1 above using the


following bond enthalpies.

Bond Bond enthalpy / kJ mol-1


H-H 436
C-H 410
C-C 350
C=C 610

H1 = mBE( reactants ) - nBE ( products)


= 4(436) [2BE(C-C) + 8BE(C-H) + BE(C=C) ]
= [1744) [2(350) + 8(410) + 610]
= - 2846
= - 2850 kJ mol-1 ( 2 marks ) (wrong sign = no marks)
( wrong units minus 1 mark )

(ii) The standard enthalpy change, H 1 does not represent the standard
enthalpy change of formation of but-1-ene. Explain why.

Graphite exists as solid under standard conditions of 298 K and 1


atmospheric pressure but the graphite shown in the question is in the
gaseous state. ( 1 mark )
8

(iii) Use the following data below as well as your answer in (d)(i) to For
calculate the standard enthalpy change of combustion of but-1-ene Examiners
Use
H c( C(g) ) = - 1110 kJ mol-1
H c(H2) = - 286 kJ mol-1

H3 = Hc(but-1-ene)

H2 = [4(H c( C(g) ) + 4 H c( H2 )]
= 4(-1110) + 4(-286)
= - 5584 kJ mol-1 (1 mark )

By Hess law
H1 + H3 = H2
H3 = Hc(but-1-ene)
= H2 - H1
= - 5584 (- 2846 )
= -2738
= - 2740 kJ mol-1 (1 mark )

[5]
[Total: 14]
9

3 Apricot kernels contain glycoside amygdalin, an organic compound which turns For
into deadly hydrogen cyanide acid, HCN, when the kernel is crushed or injured. Examiners
High doses of hydrogen cyanide can cause coma with seizures and cardiac arrest, Use
leading to death in a matter of minutes. A fatal dose for a human can be as low as
1.50 mg kg-1 of body weight. (1 mg = 1.00 x 10-3 g)

The forensics department of the local law enforcement agency was trying to
determine the cause of death of a 75 kg deceased man who was found at home
on the couch with a few empty packets of apricot kernels lying on the ground.

A 10 cm3 sample of blood (a typical human has 71 cm3 of blood per kg body
mass) obtained from the body and hydrogen cyanide containing haemoglobin was
isolated and dissolved to form 25 cm3 of solution. This sample of the haemoglobin
was oxidised by potassium dichromate, K2Cr2O7, to determine the amount of Fe2+
which is also equal to the amount of hydrogen cyanide present.
1.70 cm3 of 0.00100 mol dm-3 acidified K2Cr2O7 solution was required for complete
reaction.

(a) Write a redox equation between Fe2+ and Cr2O72-.

6 Fe2+ + Cr2O72- + 14 H+ 6 Fe3+ + 2 Cr3+ + 7H2O


[1]

(b) Show by oxidation number that the reaction in (a) is a redox reaction.

The oxidation number of Fe increases from +2 in Fe2+ to +3 in Fe3+


(1/2 mark ) indicating that Fe undergoes oxidation

The oxidation number of Cr decreases from +6 in Cr2O72- to +3 in Cr3+


indicating that Cr undergoes reduction. (1/2 mark )

[1]

(c) Calculate the total amount of hydrogen cyanide, HCN in the 25 cm3 of
solution.

Amount of Cr2O72- = (1.70/1000) x (0.00100)


= 1.70 x 10-6 mol (1 mark )

Amount of Fe2+ = 6 x amount of Cr2O72- = 6 x 1.70 x 10-6


= 1.02 x 10-5 mol (1 mark )

Hence amount of HCN = 1.02 x 10-5 mol [2]


10

(d) Calculate the amount of hydrogen cyanide, HCN, in the body of the For
deceased man. Examiners
Use
Amount of HCN in the body of the deceased man
= (1.02 x 10-5 / 10) x 71 x 75
= 5.431 x 10-3 mol (1 mark )

[1]

(e) Calculate the concentration of HCN in mg kg-1 and hence determine if the
cause of death was due to hydrogen cyanide poisoning.

Mass of HCN = 5.431 x 10-3 x ( 1.0 + 12.0 + 14.0)


= 0.1466 g
= 146.6 mg ( 1 mark )

Concentration of HCN in mg kg-1 = 146.6 / 75


= 1.954 mg kg-1 ( mark ) > 1.50 mg kg- 1
( mark )
[2]

(f) (i) Given that the 25 cm3 sample of hydrogen cyanide solution in (c) has a
pH of 6.30, show quantitatively that HCN is a weak acid.

[HCN] in the 25 cm3 sample in (c)


= 1.02 x 10-5 / 0.025
= 4.08 x 10-4 moldm-3 (1/2 mark )

[H+] = 10-pH = 10-6.30 = 5.01 x 10-7 moldm-3 (1/2 mark)

[H+] = 5.01 x 10-7 mol dm-3 < [HCN] = 4.08 x 10-4 mol dm-3
Hence HCN is a weak acid as it does not dissociate completely.
(1/2 mark)

(ii) What is the effect of adding universal indicator to this 25 cm3 sample of
hydrogen cyanide solution mentioned in (f)(i)?

Universal indicator turns yellow (1/2 mark )


[2]

(g) Explain why potassium chloride, KCl , cannot be used to oxidise Fe2+?

Cl- can only undergo oxidation. ( 1 mark )


[1]
[Total: 10]
11

Section B

Answer two of the three questions in this section on separate answer paper

4 An organic compound, E, has the molecular formula C11H14O2. When E is heated


with aqueous HCl, benzoic acid and F with formula C4H9OH, is formed. When F is
heated with hot Al2O3, G, C4H8 is produced. Heating G with acidified potassium
manganate(VII) produces H, C3H6O. H produced an orange precipitate with
2,4-dinitrophenylhydrazine but does not react with Tollens reagent.

(a) Draw the structures of compounds E to H and explain the above reactions.
Write chemical equations to support your answer. [10]
O CH3
CH3 O
C C CH3
O CH3 CH3 CH3 C CH2 C
C CH3 CH3
HO CH3 CH3

Compound E Compound F Compound G Compound H

(4 mark)
12

Clue Type of Deduction


reaction
Aqueous HCl, heat Acid ester is present
Hydrolysis

Hot Al2O3 elimination alcohol eliminated to


alkene
Acidic KMnO4, heat oxidation Terminal alkene present
Orange precipitate with condensation Carbonyl compound
2,4-DNPH present
Negative Test for Tollens Not aldehyde ( or ketone )
Reagent

[1/2 each, max 3]


Reaction of ester with hot aqueous hydrochloric acid
O CH3 O
CH3 CH3
C C C CH3
OH
O CH3
+ H 2O + HO
C
CH3

Reaction of Compound F with hot Al2O3


CH3 CH3
CH3
HO
C
CH3
C CH2
+ H2O

CH3

Reaction of Compound G with hot acidic potassium manganate.


CH3 CH3
C CH2 + 4[O] C O + CO2 + H2 O
CH3 CH3

Reaction of compound H with 2,4-DNPH

CH3 H3C
NH2
HN
O N
NO2 HN
+ NO2
+ H2O
CH3 CH3

NO 2

NO 2 (1 mark each, max 3)


13

1-aminopropan-2-ol is produced from ethanol by the following synthesis pathway.


H H
H O reaction 1 H O reaction 2 H O H H
H C C H H C C H J H C C C N
H H H H H H H

1-aminopropan-2-ol
(b) (i) Draw the displayed formula of compound J.

H
H O

H C C C N

H H ( 1 mark )
Compound J

(ii) Write the reagents and conditions for reactions 1 and 2. [3]

1. K2Cr2O7(aq), H2SO4(aq), heat with immediate distillation ( 1 mark)


2. LiAlH4 in dry ether or H2, Ni catalyst, 1500C ( 1 mark )

Alkene is an unsaturated organic compound and undergoes addition. Benzene


which is also an unsaturated organic compound undergoes substitution.

(c) Explain why benzene undergoes substitution instead of addition? [2]

To undergo addition, the delocalised system of benzene must be broken.


(1/2 mark )
This process is energetically unfavorable. (1 mark)
hence benzene undergoes substitution where the aromatic character is
retained ( 1/2 mark)

Chlorofluorocarbons, CFCs, are small alkane molecules where some hydrogen


atoms have been substituted by chlorine and fluorine atoms. CFCs are used as
aerosol propellant in spray can and as refrigerant. A typical CFC is CH3CHClF.

(d) (i) Explain why the synthesis of CH3CHClF from fluoroethane, CH3CH2F
and chlorine produces a mixture of products?

The extent and position of substitution of chlorine atom on any H atom


cannot be controlled. ( 1 mark )
(ii) CH3CHClF can be converted to CH3CH(OH)F. Give the reagents and
conditions for this reaction.

NaOH(aq), heat ( 1 mark )


14

(iii) Explain why C-F bond in CFC does not undergo the reaction in (d)(ii)?

The C-F covalent bond is strong ( 1 mark ) hence the energy provided
is not high enough to break it.

(iv) Why does this CFC present an environmental concern?

CFC molecules will react with O3 in the ozone layer and deplete it.
( 1 mark )
(v) Suggest one way to reduce the use of CFCs. [5]
Replace CFC with hydrofluorocarbon. ( 1 mark )
[Total: 20]
5 Sulfuric acid, H2SO4, is a strong mineral acid which is an important chemical used
in industry and in the school laboratory.

One step in the manufacture of sulfuric acid by the contact process involves the
reaction between sulfur dioxide and oxygen.

2 SO2 (g) + O2 (g) 2 SO3 (g)

When 72 dm3 of sulfur dioxide gas and 32.0 g of oxygen gas are reacted in a
27000 dm3 reaction vessel at a constant temperature of 25oC and 1 atmospheric
pressure, the percentage conversion of sulfur trioxide was 80% at equilibrium.

(a) (i) Calculate the initial amount of sulfur dioxide gas and initial amount of
oxygen gas at 25oC and 1 atmospheric pressure.

Amount of SO2 initially = 72 / 24 = 3 mol ( 1/2 mark )


Amount of O2 initially = 32.0 / 2(16.0) = 1 mol ( 1/2 mark )

(ii) Explain which reagent is the limiting reagent?

O2 is the limiting reagent (1/2 mark) since 1 mol of SO2 should react
with 2 mol of SO2 < 3 mol of SO2 initially. ( 1/2 mark )

(iii) Calculate the equilibrium constant with its units.


2 SO2 (g) + O2 (g) 2 SO3 (g)
Initial amt / mol 3 1 0
Change in amt/ mol 2(-0.8) 1(-0.8) +2(0.8)
Eqm amt/ mol 1.4 0.2 1.6 ( 1 mark)
Eqm [ ] / mol dm-2 1.4/27000 0.2/27000 1.6/27000
= 5.185 x 10-5 = 7.41 x 10-6 = 5.92 x 10-5
Kc = [SO3]2 / [SO2]2 [O2]
= (5.92 x 10-5)2 / (5.185 x 10-5)2 (7.41 x 10-6)
= 17 5920 = 176000 mol-1 dm3
(1 mark, -1/2 mark for unit )
15

(iv) What will happen to the yield of SO3 (g) when the reversible reaction at
equilibrium is conducted at 2 atmospheric pressure?

At a higher pressure of 2 atmospheric pressure, By Le Chateliers


principle, the position of equilibrium will shift to the right with fewer
moles of gas (1mark ) to reduce the pressure.
Hence the yield of SO3(g) increases. ( 1mark )

[6]

(b) (i) Sulfuric acid is a dibasic strong acid. Explain the underlined term.

Sulfuric acid donates two protons and dissociates fully.


( 1 mark ) ( 1 mark )

(ii) A 100 cm3 of solution contains 0.980 g of sulfuric acid. Calculate the
pH of the sulfuric acid solution.

Amount of sulfuric acid = 0.980 / 2(1.0) + 32.1 + 4(16.0)


= 9.989 x 10-3
= 9.99 x 10-3 mol (1/2 mark )

[H2SO4] = 9.989 x 10-3 / (100/1000)


= 9.989 x 10-2
= 9.99 x 10-2 mol dm-3 ( mark )

[H+] = 2 x 9.989 x 10-2


= 1.997 x 10-2 mol dm-3 ( mark )

pH = - log10[H+] = - log10(1.997 x 10-2) = 0.699 ( mark )

[4]
16

The titration curve is obtained when sodium hydroxide is added to sulfuric acid.
The equivalence point is reached when 25.0 cm3 of sodium hydroxide is added.
pH

Phenolphthalein
Equivalence pH = 7

Methyl orange

0 25.0 50.0
vVolume
olume ofofalkali (cm3) 3
NaOH / cm

Methyl orange and litmus are suitable indicators to determine the equivalence
point of the titration between sulfuric acid and sodium hydroxide solution.

Working pH
Indicator range
Methyl 3.2 4.4 Colour of methyl orange Colour of methyl orange
Orange < pH 3.2 (red) > pH 4.4 (yellow)

litmus 6.0 8.0 Colour of litmus Colour of litmus


< pH 6.0 (red) > pH 8.0 (blue)

(c) (i) Explain why methyl orange is a suitable indicator?

The working range of methyl orange fall within the almost vertical
section of the titration curve (1 mark )

And there is a distinct colour change from red to yellow at the end
points. (1/2 mark )
[3]
(ii) Explain why litmus is a suitable indicator?

The working range of litmus of pH 6.0 to 8.0 coincides with the


equivalence pH of 7.0 of the titration curve. ( 1 mark )

There is a distinct colour change from red to blue at the end points.
(1/2 mark ) [3]
17

Sulfuric acid is needed in many organic reactions. The reaction scheme below
shows the reaction involving sulfuric acid.

M
Concentrated H2SO4,
CH3COOH, heat

Excess concentrated H2SO4,


H2SO4(aq), heat CN OH 170oC
L C CH P+Q
CH2 CH CH2Cl + other
CH3 minor
K product(s)

Concentrated
H2SO4, followed by
H2O, warm

(d) (i) Draw the structures of L, M, N. If other products are also produced,
draw the major product only.
18

(ii) Draw the structures of P and Q given that P has a higher net dipole
moment.

Answer given above ( 1 mark each )


(iii) Explain why P has a higher net dipole moment than Q.

P has two electron withdrawing groups ( CN and Cl) on the same side
of the C=C group, ( 1 mark ) hence it has a net dipole moment.

Q has two electron withdrawing on opposite side of the C=C group,


which will cancel part ( 1 mark ) of the other dipole moment, resulting
in a lower dipole moment.
[7]
[Total: 20]
19

6 The standard enthalpy change of formation of boron trifluoride between boron


solid and fluorine gas to produce gaseous boron trifluoride in a reaction vessel is
shown below:
B (s) + 3/2 F2 (g) BF3 (g)

(a) With reference to boron trifluoride, define the standard enthalpy change of
formation of boron trifluoride, Hf(BF3)

The standard enthalpy change of formation of boron trifluoride is the heat


change when one mole of boron trifluoride is formed from its constituent
boron element and fluorine element under standard conditions of 298 K and
1 atmospheric pressure. ( 1 mark )
[1]

The order of reaction with respect to fluorine is one. The half-life of the reaction is
10 days. Given that the initial concentration of fluorine is 0.600 mol dm-3.

(b) Sketch a graph of [F2] against time showing two half-lives in your graph.
[ F2 ] / moldm-3 1 mark for showing correct shape and
variables with correct units on axis.
0.600
1 mark for showing correct numerical
values.
0.300

0.150
0 [2]

(c) Explain what will happen to the rate of formation of gaseous boron trifluoride

(i) when the volume of the reaction vessel is increased ?

when volume of the reaction vessel is increased, [F2] decrease


( 1/2 mark) and there will be less effective collisions per unit time (1/2)
and the rate of formation will decrease. ( 1 mark )

(ii) when a bigger piece of solid boron is used ?

When a bigger piece of solid boron is used, the surface area of boron
reactant become larger (1/2 mark) and there will be greater number of
effective collisions per unit time.(1/2 mark )
hence the rate of formation will increase ( 1 mark ) [4]
20

A copper catalyst can increase the rate of formation of gaseous boron trifluoride.

(d) (i) Write the electronic configuration of Copper atom.


1s2 2s2 2p6 3s2 3p6 3d10 4s1 ( 1 mark )

(ii) Using relevant diagram, explain how the copper catalyst can increase
the rate of formation of gaseous boron trifluoride [4]

( 1/2 mark ) for correct axis and correct shape,


( 1/2 mark ) showing two Ea and legends.
A catalyst provides an alternative pathway with a lower activation
energy ( 1/2 mark ) for a reaction to proceed
More particles have kinetic energy greater or equal to the activation
energy. ( 1/2 mark )
More effective collisions per unit time ( 1 mark )
the rate of reaction increases.
(e) (i) Write an equation to represent the first ionisation energy of Boron.

B(g) B+ (g) + e- ( 1 mark )

(ii) Boron has a lower first ionisation energy than Beryllium. Explain why.

Be: 1s2 2s2 B: 1s2 2s2 2p1


This is because the 2p electron (1/2 mark ) of boron to be removed is
of higher energy (1/2 mark ) than the 2s electron (1/2 mark) of Be as it
is more shielded by the additional inner 2s subshell electrons
( 1/2 mark) in B, hence less energy is needed to remove the outermost
electron from boron than beryllium.
[3]
21

(f) (i) BF3 reacts with NH3 to form molecule S. Draw a structure to represent
the molecule S. (1 mark)

(ii) Explain the criteria for forming the structure you have proposed in (f)(i)

B has 3 valence electrons which are used to form bonds with 3 fluorine
atoms. The resulting compound is electron deficient (1 mark). Lone
pair of electrons on a neighbouring N atom from NH3 forms a dative
bond (1 mark) with B thus achieving octet.

(iii) Show the hydrogen bonding between two molecules of S, showing


dipoles and other relevant information. [5]

F H Show lone pair on F


atom, correct dipoles
F B N H and hydrogen bond
( 2 mark )
F H

F H

F B N H

F H

BF3 reacts with water to form a weak acid, boric acid, H3BO3 as one of its
products. Excess H3BO3 reacts with aqueous NaOH to form buffer solution
consisting of aqueous H3BO3 and aqueous NaH2BO3.

(g) Write an ionic equation to show how this buffer solution decreases the
acidity when a small amount of acid is added.

H2BO3- + H+ H3BO3 ( 1 mark ) [1]


[Total: 20]

END OF PAPER
RAFFLES INSTITUTION
PRELIMINARY EXAMINATION 2010

Higher 1

CHEMISTRY 8872/01
Paper 1 Multiple Choice 27 September 2010
50 minutes

Additional Materials: OMR Answer Sheet


Data Booklet

READ THESE INSTRUCTIONS FIRST

DO NOT open this booklet until you are told to do so.

Write in soft pencil.


Do not use staples, paper clips, highlighters, glue or correction fluid.
Write your name, class and index number in the spaces provided on the Answer
Sheet.

There are thirty questions on this paper. Answer all questions. For each question
there are four possible answers A, B, C and D.
Choose the one you consider to be correct and record your choice with a soft pencil
on the separate Answer Sheet.

Read the instructions on the Answer Sheet very carefully.

Each correct answer will score one mark. A mark will not be deducted for a wrong
answer. Any rough working should be done in this booklet.

This document consists of 11 printed pages.


Section A

For each question there are four answers, A, B, C and D. Choose the one you
consider to be correct.

1. Which of the following has the greatest mass?

A 1.2 x 1024 atoms of iron


B 3.0 x 1023 copper(II) ions
C 3.0 moles of nitrogen molecules
D 6.0 x 1023 atoms of helium gas

2. 10.0 cm3 of 0.200 mol dm3 of K2XO4 reacts with 40.0 cm3 of 0.100 mol dm3
of iron(II) sulfate solution. If Fe2+ is oxidised to Fe3+ by K2XO4, what is the final
oxidation state of X?

A +2 B +3 C +4 D +5

3. In which sequence are Br, Rb+ and Sr2+ ions ranked in order of increasing
radii?

smallest largest
A Sr2+ Br Rb+
B Br Rb+ Sr2+
C Rb+ Sr2+ Br
D Sr2+ Rb+ Br

4. Which of the following corresponds to the third ionisation energy of


chromium?

A Cr2+(g) Cr 3+(g) + e
B Cr 3+(g) + e Cr 2+(g)
C Cr(g) Cr 3+(g) + 3e
D Cr 2+(s) Cr 3+(g) + e

5. Which of the following species has a square planar shape?

A CH4 B BrF4 C AlCl3 D PBr5

2
6. Which of the following shows compounds ranked in order of increasing boiling
points?

A NaCl, CH3OH, CH3CH3


B CH3OH, CH3CH3, NaCl
C CH3CH3, CH3OH, NaCl
D CH3OH, NaCl, CH3CH3

7. The value of the enthalpy change for the process represented by the
equation,

O2(g) + 2e O2(g)

is equal to

A sum of the first and second electron affinity of oxygen


B sum of the enthalpy change of atomisation and the first and second
electron affinity of oxygen
C sum of the first ionisation energy and the first electron affinity of oxygen
D sum of the bond energy of (O=O) and the first and second electron
affinity of oxygen

8. Tetrachloroethene is a colourless liquid widely used for the cleaning of fabrics.


The equation below shows the production of tetrachlorethene from ethene,
hydrogen chloride and oxygen.

C2H4(g) + 4HCl(g) + 2O2(g) C2Cl4(l) + 4H2O(l)

Using the following information, calculate the enthalpy change of reaction for
the above reaction.

Compound Hf / kJ mol1
C2H4 +52.3
HCl 92.3
C2Cl4 52.2
H2O 285.8

A +298.0 kJ mol1
B 298.0 kJ mol1
C +878.5 kJ mol1
D 878.5 kJ mol1

3
9. A theoretical reaction involves A 2B.

Which statement is always correct?

A The overall order of reaction is one.


B Adding a suitable catalyst increases the rate constant of the reaction.
C Removing B as it is formed helps to increase the rate of reaction.
D Increasing the concentration of A increases the rate of the reaction.

10. The graph represents the decomposition of a sample of hydrogen peroxide,


H2O2 in the presence of manganese(IV) oxide.

2.5

2.0

1.5
3
[H2O2]/mol dm

1.0

0.5

0.0
Time/s
0 200 400 600

What conclusion can be drawn from the graph?

A The reaction is zero order with respect to H2O2.


B The rate of decomposition of H2O2 is constant.
C The halflife of H2O2 in the sample is 100 s.
D The rate equation for the decomposition of H2O2 is rate = k[H2O2].

11. The numerical value of the equilibrium constant for the reaction

H2(g) + I2(g) 2HI(g)

is 60 at 450 oC. The number of moles of HI in equilibrium with 2.0 moles of


hydrogen and 0.3 moles of iodine at 450 oC is

A 0.01 B 0.1 C 6 D 36

4
12. Phosphorus trichloride and chlorine react according to the following equation:
PCl3(g) + Cl2(g) PCl5(g)
A mixture of PCl3 and Cl2 was placed in a syringe and brought to equilibrium.
At time t, the plunger of the syringe was suddenly withdrawn such that the
resulting volume was twice the initial volume.
Which graph correctly shows the variation of concentration of PCl5 with time?

A [PCl5] B [PCl5]

time time
t t
C [PCl5] D [PCl5]

t time t time

13. H3PO4 is a triprotic acid which has three acid dissociation constants. The table
below shows the first, second and third acid dissociation constants, Ka,
respectively.

Dissociation Equilibrium in aqueous solution Ka/mol dm3


First H3PO4(aq) + H2O(l) H3O+(aq) + H2PO4(aq) 7.5 x 103
Second H2PO4(aq) + H2O(l) H3O+(aq) + HPO42(aq) 6.2 x 108
Third HPO42(aq) + H2O(l) H3O+(aq) + PO43(aq) 2.2 x 1013

Which of the following ranks the conjugate bases in order of increasing


basicity?

lowest highest
A PO43 HPO42 H2PO4
B H2PO4 HPO42 PO43
C HPO42 PO43 H2PO4
D HPO42 H2PO4 PO43

5
14. Which pair of 1 mol dm3 aqueous solutions, when added together, will give a
buffer solution of pH > 7?

A 25 cm3 of hydrochloric acid and 50 cm3 of aqueous ammonia


B 50 cm3 of hydrochloric acid and 25 cm3 of aqueous ammonia
C 25 cm3 of ethanoic acid and 50 cm3 of sodium hydroxide
D 50 cm3 of ethanoic acid and 25 cm3 of sodium hydroxide

15. Which of the following is an incorrect trend of the elements of the Period 3 of
the Periodic Table?

A The radii of the atoms decrease.


B The oxides of the elements change from basic to acidic.
C The melting points of the elements decrease steadily.
D The compounds of the elements change from ionic to covalent.

16. Elements can form oxides with giant structures. The oxide formed can
conduct electricity in the molten state.

Which of the following contains two such elements?

A magnesium, phosphorus
B sodium, aluminium
C silicon, phosphorus
D aluminium, silicon

17. The graph shows how a property of the elements Na to Cl varies with proton
number.

proton number
11 12 13 14 15 16 17

What is the property?

A Electronegativity
B First ionisation energy
C Ionic radius
D Melting point

6
18. Which class of compounds will the hydrogen atoms to carbon atoms ratio be
the highest in its general formula?

A alcohols
B aldehydes
C arenes
D halogenoalkanes

19. What is the total number of different esters with molecular formula of C 4H8O2?

A 2 B 3 C 4 D 6

20. What is the total number of geometric isomers that can be formed from the
product of the reaction of the following compound with excess concentrated
sulfuric acid at 170 oC?
OH

CH2CHCH2CH2CH3

CH2 CH CH2

A 2 B 4 C 8 D 16

21. Which hydrocarbon, on treatment with hot acidified potassium


manganate(VII), would give propanoic acid only?

A CH3CH2CH CHCH3
B CH3CH2CH CHCH2CH3
C (CH3)2C C(CH3)2

22. On dehydration followed by reaction with bromine, compound X produced a


mixture of two dibromo compounds that are structural isomers.

What could compound X be?

A (CH3)3COH
B (CH3)2CHCH2OH
C CH3CH2CH(OH)CH3
D CH3CH2CH2CH2OH

7
23. Phenylethanoic acid, C6H5CH2COOH can be synthesised from
(bromomethyl)benzene, C6H5CH2Br in the laboratory in two steps.

Step 1 Step 2
C6H5CH2Br X C6H5CH2COOH

What are reagents and conditions that can be used for this synthesis?

Step 1 Step 2
A NaOH(aq), heat Acidified K2Cr2O7, heat
B Ethanolic NaOH, heat Acidified KMnO4, heat
C Ethanolic NaCN, heat HCl(aq), heat
D Cold HCN, trace amount of NaCN H2SO4(aq), heat

24. Upon heating, compound X gives yellow precipitate with alkaline aqueous
iodine. 2 moles of compound X also give effervescence of 1 mole of
hydrogen gas upon reaction with sodium metal. It does not change the
colour of hot, acidified sodium dichromate(VI) from orange to green.

What could X be?

A O CH2CH3

CH3CCH2CCH3

OH
B O

CH3CH2CCH2CHCH3

OH
C OH
O CHCH3

CH3CH2CCH2CCH3

OH
D OH
O
C(CH3)2
CH3CCH2CCH3

OH

8
25. Prednisone is a useful drug in the treatment of inflammatory disorders,
allergies and malignancies such as leukemia and multiple myeloma.
O
OH
O

OH

O
Prednisone
Prednisone

Which statement about Prednisone is correct?

A It exhibits cistrans isomerism.


B It gives a reddish brown precipitate with Fehlings solution.
C It gives steamy white fumes upon reaction with PCl5.
D It reacts with cold, alkaline KMnO4 to give a diol.

9
Section B

For each of the following questions, one or more of the three numbered statements 1
to 3 may be correct.

Decide whether each of the statements is or is not correct (you may find it helpful to
put a tick against the statements which you consider to be correct).

The responses A to D should be selected on the basis of

A B C D
1, 2 and 3 are correct 1 and 2 are correct 2 and 3 are correct 1 only is correct

No other combination of statements is used as a correct response.

26. For the following pairs, in which one does the second compound have a
smaller bond angle than the first?

1 BF4 and OF2


2 PH4+ and NH3
3 AlCl3 and BeCl2

27. The rate equation of a reaction between X and Y is as follows:

rate = k[X]

Which of the following statements are true of the reaction?

1 Doubling [X] will double the rate of reaction.


2 X is involved in the rate-determining step of the reaction mechanism.
3 A graph of [X] against time gives a straight line passing through the origin.

28. The following reaction between carbon monoxide and steam occurs in the
presence of a suitable catalyst.

CO(g) + H2O(l) CO2(g) + H2(g) H = 40 kJ mol1

A higher equilibrium yield of hydrogen would be expected by using

1 a lower temperature
2 a higher pressure
3 a more finely powdered catalyst.

10
The response A to D should be selected on the basis of

A B C D
1, 2 and 3 are correct 1 and 2 are correct 2 and 3 are correct 1 only is correct

No other combination of statements is used as a correct response.

29. Which of the following tests can be used to distinguish between the two
organic compounds below?
CH 2CHO

CH3CH CHCH2OH

1 Na metal
2 Aqueous bromine, in the dark
3 2,4-Dinitrophenylhydrazine

30. A derivative of Rofecoxib, an efficient drug against arthritis, has the following
structure.

O
Rofecoxib

Which of the following statement about this compound are correct?

1 It undergoes hydrogenation with hydrogen gas and nickel catalyst.


2 It reacts with cold dilute alkaline KMnO4 to give a diol.
3 The compound is planar.

END OF PAPER

11
RAFFLES INSTITUTION
PRELIMINARY EXAMINATION 2010

Higher 1

CANDIDATE NAME

CLASS INDEX NUMBER

CHEMISTRY 8872/02
Paper 2 20 September 2010
2 hours
Candidates answer Section A on the Question Paper.

Additional Materials: Data Booklet


Answer Paper

READ THESE INSTRUCTIONS FIRST


DO NOT open this booklet until you are told to do so.

Write your name, class and index number in the spaces provided above.
Write in dark blue or black pen on both sides of the paper.
You may use a soft pencil for any diagrams, graphs or rough working.
Do not use staples, paper clips, highlighters, glue or correction fluid.

Section A (40 marks)


Answer all questions.

Section B (40 marks)


Answer any two questions on separate answer
paper. For Examiners Use
You are to begin each question on a fresh Paper 1
sheet of paper.
Paper 2
A Data Booklet is provided. Section A

At the end of the examination, fasten all your B6


work securely together. B7
The number of marks is given in brackets [ ] at B8
the end of each question or part question.
Total
You are reminded of the need for good English
and clear presentation in your answers.

This document consists of 14 printed pages.

1
Section A (40 marks)

Answer all questions in the spaces provided.

1 This question is about the mining of copper.

The largest copper mine in the world is in Atacama, Chile, where the
mineral chalcopyrite, CuFeS2, is extracted.

(a) What is the percentage by mass of copper in chalcopyrite?

[1]

(b) The richest ore dug out of the ground in Atacama, however, is
only about 1% copper by mass.

Assuming that chalcopyrite is the only copper-component of the


ore, what is the mass of chalcopyrite in 100 g of ore?

[2]

(c)(i) During the roasting stage of the extraction process, chalcopyrite


is heated to between 500 C and 700 C in air to form iron(II)
oxide, copper(II) sulfide and sulfur dioxide gas, SO2.

Write the balanced chemical equation during the roasting of


chalcopyrite.

(ii) Calculate the maximum volume of sulfur dioxide (at room


temperature and pressure) that will be generated from the
roasting of 1 tonne of chalcopyrite. [1 tonne = 1000 kg]

[3]
[Total: 6]

2
2 (a) Elements W, X, Y and Z have four consecutive proton numbers
in the Periodic Table. The table below shows the first 4 ionisation
energies of elements W, X, Y and Z.

Element 1st I.E 2nd I.E 3rd I.E 4th I.E


W 1685 3374 6050 8408
X 2081 3952 6122 9371
Y 496 4562 6910 9543
Z 738 1451 7733 10543

(i) Using the data given above, identify the group that element
W belongs to.

(ii) Explain the significant decrease in the first ionisation


energy from element X to Y. .

(iii) What is the formula of the compound that W and Z are


likely to form?

[4]

(b) Four ions of unknown elements are given below.

21 + 20 20 2 21 3
10 A 11 B 9C 9D

(Note that A, B, C and D are not the atomic symbols of the


elements concerned)

(i) Which of the above ions are isotopic?

3
(ii) A source beam of ions is passed through an electric field.
Sketch the deflection paths of the four ions in the diagram
below. Show clearly their relative angles of deflection.

- - - - - - -

Source beams
of ions

+ + + + + + +
[3]
[Total: 7]

3 Ammonium sulfate is largely used as a fertilizer for alkaline soils. The


reaction of solid ammonium sulfate in water is known as dissolution. The
enthalpy change for the dissolution of ammonium sulfate in water, H1
can be determined by using the energy cycle given below.

H4
2NH4+(g) + SO42(g) (NH4)2SO4(s)
H3
2 x H2 H1
2NH4+(aq) + SO42(aq)

H2 = 132 kJ mol1
H3 = 1099 kJ mol1
H4 = 1370 kJ mol1

(a) Define the enthalpy term that is represented by H4.

[1]

(b) (i) Based on the above energy cycle and the data given
above, calculate H1.

4
(ii) Draw a labelled energy profile diagram of the reaction in
H1, showing clearly the activation energy and the
enthalpy change of the reaction.

(iii) In an experiment, 10.0 g of ammonium sulfate was placed


in an insulated beaker containing 200 cm3 of water at
25.0 oC. Using your answer to (b) (i), and given that 4.0 J
of energy is required to raise 1 cm3 of the solution by 1 K,
calculate the expected temperature change of the solution.

[6]

(c) Would you expect the lattice energy of NH4SeO4 to be higher or


lower compared to that of NH4SO4? Explain briefly.

[2]
[Total: 9]

5
4 Some ammonium iodide crystals are placed in a sealed vessel and
heated at a temperature of 500 C and decomposition occurs.

The following equilibrium is established.

NH4I(s) NH3(g) + HI(g)

The value of equilibrium constant, Kc at this temperature is 7.00 x 105.

(a) Write an expression for the equilibrium constant, Kc of the


reverse reaction, i.e. the formation of ammonium iodide, stating
its numerical value.

[2]

(b) Calculate the amount of each gas present at equilibrium when


0.50 moles of ammonia gas and 0.25 moles of hydrogen iodide
gas are mixed in a 10 dm3 closed vessel at 500 C.

[4]

(c) Predict and explain the effect on the yield of ammonium iodide if
the pressure of the vessel in (b) is decreased at 500 C.

[2]
[Total: 8]

6
5 (a) Compounds containing the allyl group, CH2=CHCH2, have
pungent smells and are found in onions and garlic.

Allyl alcohol, CH2=CHCH2OH, may be converted into propanal,


CH3CH2CHO, by using a ruthenium(IV) catalyst in water.

ruthenium(IV)catalyst
ruthenium(IV) catalyst
CH2=CHCH2OH CH3CH2CHO

The reactant and the product are isomers of each other.

(i) What type of isomers are the reactant and product?

(ii) Explain whether allyl alcohol, CH2=CHCH2OH, can exhibit


geometric isomerism.

(iii) Suggest a chemical test that you would carry out on the
reactant and product to distinguish them.

[5]

7
(b) The reaction scheme below shows the reactions involving an
allylic compound, X.

HO COOH

Step 2

few steps
HO CH2CH CH2 Compound W

Compound X
Step 1

SOCl2, heat CH2CH2CHO


HO
Compound Z
HCN,
trace amount
of NaOH

Compound Y

(i) Draw the structures of compounds W, Y and Z in the boxes


below.

W Y Z

8
(ii) Suggest the reagents and conditions necessary for the
following steps.

Step 1:

Step 2:

[5]
[Total: 10]

9
Section B (40 marks)

Answer two questions from this section on separate answer paper.

6 (a) The bicarbonate buffering system is an important buffer system in


the acid-base homeostasis of living things, including humans.

As a buffer, it tends to maintain a relatively constant blood


plasma pH and counteract any force that would alter it. Thus
blood pH is regulated to stay within the narrow range of 7.35 to
7.45, making it slightly alkaline. If the pH of the body gets too low
(below 7.4), a condition known as acidosis results. This can be
very serious, because many of the chemical reactions that occur
in the body, especially those involving proteins, are pH-
dependent.

In this buffering system, carbon dioxide combines with water to


form carbonic acid (H2CO3), which in turn rapidly dissociates to
form hydrogen and bicarbonate (HCO3) ions according to the
reaction below:
H2CO3 HCO3 + H+ Ka = 7.95 x 107 mol dm3

(i) Define pH.

(ii) Identify the conjugate acid-base pair in the above buffering


system.

(iii) With the aid of an equation, explain how this buffering


system helps to maintain relatively constant pH when H+
ions are added to the blood plasma.

(iv) Calculate pH of a 0.10 mol dm3 solution of carbonic acid.

(v) 25 cm3 of carbonic acid was reacted with 30 cm3 of


0.60 mol dm3 of aqueous sodium hydroxide. Calculate the
pH of the reaction mixture. (You may wish to consider
carbonic acid as a monobasic acid.)

(vi) Suggest a suitable indicator for the titration in (a) (v).

(vii) Is carbonic acid a strong or weak acid? Explain your


answer.
[10]

10
(b) The composition of the human body can be looked at from
different points of view. By mass, oxygen, hydrogen and carbon
are present in larger abundances. Other elements which are
present include sodium, magnesium, phosphorus, sulfur and
chlorine. Aluminium, which is another element found in Period 3,
seems to serve no function in living cells. Rather, it is harmful in
large amounts.

(i) When sodium chloride is added to water, the solution


formed has a pH of 7. When aluminium chloride is added to
water, the resulting solution has a pH of 3. Explain these
observations, and why these two chlorides differ in their
reaction with water.

(ii) Beryllium, which shares a diagonal relationship with


aluminium, undergoes similar chemical reactions as
aluminium. By means of balanced equations, indicate
whether or not beryllium oxide reacts with:

I NaOH(aq)
II HCl(aq)

(ii) Magnesium oxide is only slightly soluble in water but it


dissolves in aqueous magnesium chloride. Explain this
observation with the aid of balanced equations.
[8]

(c) The chloride of an element X in Period 3 is a liquid which has a


boiling point of 58 C.

After dissolving 0.0100 mol of the chloride with water, the


resulting solution required 100 cm3 of 0.400 mol dm3 silver
nitrate for complete precipitation of the chloride ion.

To which group of the Periodic Table does X belong?


[2]

[Total: 20]

11
7 (a) Compound P, C5H9ClO2, which is sparingly soluble in water, is
readily soluble in aqueous sodium hydroxide.

Heating compound P in aqueous sodium hydroxide and adding


excess acid to the product give compound Q, C5H10O3. Q reacts
with warm alkaline aqueous iodine to give a yellow precipitate.

When compound Q is refluxed with concentrated sulfuric acid, it


forms 3 isomers, R, S and T with a molecular formula of C5H8O2.
R is a sweet smelling liquid and S and T exist as a pair of
geometric isomers. S also reacts with aqueous bromine to give a
number of products, one of which is shown below:

O
H3C CH CH CH2 C OH
Br OH

S undergoes reaction with acidified potassium manganate(VII) to


give U, C3H4O4, and ethanoic acid. 1 mole of U liberates 1 mole
of CO2 on reaction with excess sodium carbonate.

(i) Deduce the structures of compounds P, Q, R, S, T and U,


explaining how you arrive at your answers.

(ii) Write balanced equations for the reaction of:

I P with aqueous sodium hydroxide;


II Q with warm alkaline aqueous iodine.
[13]

12
(b) Iodination of propanone is done in aqueous acidic solution
according to the equation:

CH3COCH3(aq) + I2(aq) CH3COCH2I(aq) + H+(aq) + I(aq)

In investigations of this reaction, the following results were


obtained.

Experiment Initial concentrations of Initial rate of


No. reactants/ mol dm3 reaction/
mol dm3
min1
[CH3COCH3] [H+] [I2]
1 0.4 0.4 0.002 1.01 x 104
2 0.4 0.8 0.002 2.02 x 104
3 1.2 0.4 0.002 3.03 x 104
4 1.2 0.8 0.004 6.06 x 104

(i) Using the data above, deduce the orders of reaction with
respect to propanone, iodine and acid.

(ii) Construct a rate equation for the reaction and use it to


calculate a value for the rate constant.

(iii) With the aid of an appropriate diagram, explain how the


rate of reaction will be affected by a decrease in
temperature.
[7]

[Total: 20]

13
8 (a) A compound W, C6H8O, when reacted with excess hydrogen in
the presence of nickel catalyst gives compound X, C6H12O. X
reacts with sodium metal to give hydrogen gas.

W gives an orange precipitate with 2,4 dinitrophenylhydrazine but


there is no reaction with Tollens reagent .

When compound W reacts with bromine, the reddish-brown


bromine decolourised and forms compound Y, C6H8Br2O. With
HCN, W forms compound Z, C7H9NO.

Deduce the structures of compounds W, X, Y, and Z. Write


balanced equations for the reactions involved where appropriate.
[10]

(b) Chlorofluorocarbons, CFCs, are widely used as refrigerants,


propellants (in aerosol applications), and solvents. The
manufacture of such compounds is being phased out by the
Montreal Protocol because of their contribution to ozone
depletion.

A typical chlorofluorocarbon is 2-chloro-1,1,1,2-tetrafluoroethane


(CHFClCF3).

(i) Which covalent bond in the above CFC is the weakest?


Give a reason for your answer.

(ii) How will the boiling point of the above CFC differ from that
of CClF2CF2CHClF?
[4]

(c) Chlorine dioxide, ClO2, is now commonly used in water treatment


due to its greater solubility in water compared to chlorine.

(i) Draw the dotandcross diagram for chlorine dioxide and


suggest the shape and the size of its bond angle.
(ii) Suggest a reason why chlorine dioxide is a good oxidizing
agent.
(iii) Explain why chlorine dioxide is relatively more soluble in
water compared to chlorine. Draw a diagram to illustrate
your answer.
[6]

[Total: 20]

14
END OF PAPER

15
Suggested Solutions to 2010 H1 Chemistry Prelims

1 2 3 4 5 6 7 8 9 10
A C D A B C B D B D
11 12 13 14 15 16 17 18 19 20
C C B A C B C A C B
21 22 23 24 25 26 27 28 29 30
B C C A C B B D A B
Suggested solutions to 2010 H1 Prelims Paper 2

1a
63.5
Percentage by mass of copper = 100% 34.6%
(63.5 55.8 2 32.1)
1b

Mass of ore = 100 g,


1.0
Mass of copper = 100 g 1.00 g
100

34.6% of chalcopyrite 1.00 g copper


100% of ore 2.89 g of chalcopyrite

1c(i)
2CuFeS2 (s) + 3O2 (g) 2FeO (s) + 2CuS (s) + 2SO2 (g)

1c(ii)
1000 103 g
Amt of chalcopyrite = 5.45 103 mol
(63.5 55.8 2 32.1)
Amt of chalcopyrite = Amt of sulfur dioxide
Volume of sulfur dioxide = 5.45 10 24dm 1.30 10 dm 131 m
3 3 5 3 3

2a(i)
Group VII

2a(ii)
Valence electron removed from Y must be from a higher energy principal quantum shell and
experiences significant shielding effect by electrons in the inner shells that outweighs the increase
in nuclear charge

OR increase in shielding effect outweighs increase in nuclear charge as there is an additional


inner filled quantum shell of electrons in Y

effective nuclear charge decreases from X to Y hence less energy is required to remove the
valence electron as compared to that of X.

2a(iii)
ZW 2

2b(i)
C2- and D3-
2b(ii)

- - - - - - -

Source beams A BCD


of ions

+ + + + + + +

3a
H4 represents lattice energy.
Lattice energy of an ionic compound is the energy released when one mole of the solid ionic
compound is formed from its constituent gaseous ions under standard conditions.

3b(i)
By Hess Law, H1 = - H4 + 2H2 + H3 = -(-1370) + 2(-132) + (-1099) = +7 kJ mol-1

3b(ii)
energy

Ea

+ 2-
2NH4 (aq) + SO4
H1 (aq)

(NH4)2SO4(s)

progress of reaction

3b(iii)
Amt of (NH4)2SO4 used = 10.0/(132.1) = 0.0757 mol

Q=mcT
(H1 x n) = (200)(4.0)( T)
T = (7000x0.0757)/800 = 0.662 oC

Temperature dropped by 0.662 oC


3c
q + q-
Lattice energy
r+ + r-
The lattice energy of NH4SeO4 would be less exothermic compared to that of NH4SO4. This is
because the SeO42- ion has a bigger anionic radius compared to SO42- ion, resulting in a lower
charge density.

4a

NH3(g) + HI(g) NH4I(s)

Kc = _____1_____ = 1/(7.00x10-5) = 1.43 x 104


[NH3][HI]

4b

NH3(g) + HI(g) NH4I(s)


Amt 0.5 0.25 -
Eqm amt 0.5-x 0.25-x x
Eqm conc 0.5-x 0.25-x
10 10

Kc = _____1_____ __ = 1.43 x 104


(0.5-x) (0.25-x)
10 10

(0.5-x)(0.25-x) = 0.006993
x2 -0.75x + 0.118 = 0
x = 0.525 (rejected) or 0.225

Amt of NH3 = (0.5-0.225) = 0.275 mol;


Amt of HI = (0.25-0.225) = 0.025 mol

4c
When the pressure is decreased at constant temperature, the concentrations of the gases
decrease.

By Le Chateliers Principle, the system will try to increase the pressure by favouring the forward
reaction to increase the number of moles of gases.

Hence the position of equilibrium will shift to the left and yield of ammonium iodide will decrease.

5a(i)
Structural isomerism or functional group isomerism

5a(ii)
The allyl alcohol cannot exhibit geometric isomerism.
This is due to the presence of 2 identical hydrogen atoms that are bonded to the same doubly-
bonded carbon atom, thus the compound cannot show cis or trans isomers.
5a(iii)
Any one of the following tests is acceptable.

Acceptable distinguishing test Observations


2,4-dinitrophenylhydrazine, Orange ppt observed for CH3CH2CHO.
room temperature No ppt observed for CH2CHCH2OH.
Tollens reagent, heat Silver mirror observed for CH3CH2CHO.
No silver mirror observed for CH2CHCH2OH.
Fehlings reagent, heat Reddish brown ppt observed for CH3CH2CHO.
No reddish brown ppt observed for CH2CHCH2OH.
Na(s), room temperature Effervescence of colourless gas that extinguishes
lighted splint with pop sound is observed for
CH2CHCH2OH.
No effervescence and no gas evolved for
CH3CH2CHO.
PCl5, room temperature Steamy white fumes that turns moist blue litmus
red is observed for CH2CHCH2OH.
No steamy white fumes evolved for CH3CH2CHO.
Br2(l) in CCl4, room temperature Reddish brown solution is decolourised for
CH2CHCH2OH.
Reddish brown solution remains reddish brown for
CH3CH2CHO.
Br2(aq), room temperature Yellowish orange solution is decolourised for
CH2CHCH2OH.
Yellowish orange solution remains yellowish
orange for CH3CH2CHO.

5b(i)

W Z

HO CH2CH2CH2OH HO CH2CH2CH2Cl

HO CH2CH2CH(OH)CN

5b(ii)

Step 1: K2Cr2O7, H2SO4(aq), heat with immediate distillation

Step 2: KMnO4, H2SO4(aq), heat


B6

6a(i)
The pH of a solution is a measure of acidity or alkalinity of the solution. It is the negative
logarithm to base ten of the hydrogen ion concentration in mol dm3 i.e. pH = lg [H+(aq)]

6a(ii)
Acid is H2CO3, conjugate base is HCO3-

6a(iii)
HCO3- + H+ H2CO3

6a(iv)

Ka = [H+]2 / [H2CO3] => [H+] = (Ka [H2CO3])1/2 = (7.95 x 10-7 x 0.10) 1/2

= 2.82 x 10-4 mol dm-3


pH = 3.55

6a(v)
Amt of carbonic acid used = 25/1000 x 0.10 =2.50 x 10-3 mol
Amt of NaOH(aq) = 30/1000 x 0.60 = 0.018 mol

NaOH(aq) is in excess.

[OH-] in excess = (0.018-2.50 x 10-3 )/ 55/1000 = 0.281 mol dm-3

pH = 13.4

6a(vi)

Phenolphthalein

6a(vii)
Carbonic acid is a weak acid as [H+] is not equal to [carbonic acid]. Thus carbonic acid did not fully
dissociate to give H+ ions.

6b(i)
NaCl dissolves (hydrates) readily in water to form aquated ions and neutral solutions of pH 7.

NaCl(s) + aq Na+(aq) + Cl-(aq)

AlCl3 dissolves in water to form acidic solutions (due to the very high charge density of Al3+). The
small, highly polarising cation weakens the OH bonds of the water molecules in its surrounding
sphere of coordination and results in the release of hydrogen ions in solution. pH of the resulting
solution is about 3.

AlCl3(s) + 6H2O(l) [Al(H2O)6]3+(aq) + 3Cl(aq) (3)


[Al(H2O)6]3+(aq) + H2O(l) [Al(H2O)5(OH)]2+(aq) + H3O+(aq) (4)

6b(ii)
I : BeO + 2OH- + H2O [Be(OH)4]2-

II: BeO + 2H+ Be2+ + H2O


6b(iii)
MgO(s) + H2O(l) Mg(OH)2(s)
MgO is sparing soluble in water as MgO has a very exothermic lattice energy.

MgCl2(s) + 6H2O(l) [Mg(H2O)6]2+(aq) + 2Cl(aq) (1)


2+ + +
[Mg(H2O)6] (aq) + H2O(l) [Mg(H2O)5(OH)] (aq) + H3O (aq) (2)
Due to the high charge density of Mg2+ , it polarises the water molecules to a slight extent. MgCl2
undergoes slight hydrolysis to give a weakly acidic solution.

The acidic solution formed reacts with MgO, thus dissolving it.
MgO(s) + 2H+(aq) Mg2+(aq) + H2O(l)

6c

Amount of XCln = 100/1000 x 0.400 = 0.0400 mol

Amt of XCln : Cl- = 0.0100 : 0.0400 = 1 : 4

X is in Group IV.
B7a

B7a(i) Explanations and equations


P, C5H9ClO2 C to H ratio is not close to 1:1, W does not
contain an aromatic ring.

P sparingly soluble in water but soluble in P has COOH group and a large hydrophobic
NaOH group.

Heating P in aq NaOH and adding excess acid Nucleophilic substitution


to the product produces Q, C5H10O3. Q which will have an alcohol group on it.
Or eqn

Q reacts with warm aqueous alkaline iodine to Q has CH3CH(OH)- group.


give a yellow ppt Or eqn

Q forms 2 isomers, R, S and T, C5H8O2 when Q undergoes dehydration/elimination to give


it is refluxed with conc sulfuric acid. S which has a C=C double bond.
Or eqn

S and T can exhibit cis-trans isomerism. S and T each has a C=C double bond with 2
different substituents on each C of the double
bond.

S reacts with aq Br2 to give a number of S undergoes addition reaction.


products one of which is
O
H3C CH CH CH2 C OH
Br OH

S undergoes reaction with acidified potassium Strong oxidation. The C=C double bond is
manganate (VII) to give U, C3H4O4 and cleaved and is located between the 2nd and 3rd
ethanoic acid. carbon in the carbon chain of S.
Or eqn

1 mol U reacts with excess with excess U is a dicarboxylic acid.


Na2CO3 to give 1 mole of CO2 Or eqn

R is a sweet smelling liquid. Q undergoes self-esterification to form ester R.


Or eqn

OH O O
H3C CH CH2 CH2 C OH H3C CH CH CH2 C OH + H2O
(Q) (S)

O O O O O O
H3C CH CH HCH
3C2 CH CH
C OH CH
Br22 + C
+ 4[O] H2OOH
HO C CH2 HC3C OH
CH CH CCHCH
+ HO 2 3C OH + HBr

(S) (S) + Br2 + H2O (T) Br OH


O O O O
H3C CH CH CH2 C OH+ 4[O] HO C CH2 C OH+ HO C CH3
(S) (T)
(S) (U)

O O O O
HO C CH2 C OH+ Na2CO3 Na+-O C CH2 C O-Na++ CO2 + H2O
(T) (U)

OH O H3C O
O
CH C
H3C CH CH2 CH2 C OH + H2O
H2C CH2
(Q) (R)

Structures

OH O H3C O
Cl O OH O O
CH C
P: H3C CH CH2 CH2 C Q: H3C HCH
OH + 2NaOH 3C CH CH22 CH
CH2 CH C 2OH
C O-Na+ + H2O
H2C CH2
(P) + NaCl + H2O
(Q) (R)
H3C O +H +
O
CH C O O O
OH + H2O
H2C CH2 H3C CH CH CH2 COHOH+ 4[O] O HO C CH2 C OH+ HO
R: S/T: H3C CH CH2 CH2 C OH
(R) (S) (T)
(Q)
O O O O
HO C CH2 C OH+ Na2CO3 Na+-O C CH2 C O-Na++ CO2 + H2O
U:
(T)

B7a(ii)

Cl O OH O
H3C CH CH2 CH2 C OH+ 2NaOH H3C CH CH2 CH2 C O-Na+
(P) + NaCl + H2O
B7a(iii) + H+

OH O O O
OH O
H3C CH CH2 CH2 C OH Na+-O C CH2 CH2 C O-Na+
H3C CH CH2 CH2 C OH
(Q) + 7NaOH + 4I2 + CHI3 + 5NaI + 6H2O
yellow (Q)
ppt
7b(i)
Compare expt 1 and 3, when [propanone] tripled, the initial rate increased by 3x first order
reaction with respect to propanone.

Compare expt 1 and 2, when [H+] doubled, the initial rate increased by 2x first order reaction
with respect to H+.

Compare expt 3 and 4, when [H+] and [I2] doubled, the rate increased by 2x zero order reaction
with respect to iodine.

Order of reaction with respect to


Propanone = 1; acid = 1; iodine =0

7b(ii)
Using Expt 1:
Rate = k[propanone] [H+]
1.01 x 10-4 = k(0.4)(0.4)
k = 6.31 x 10-4 mol-1dm3min-1

7b(iii)
number of molecules total no. of molecules with
with a given energy energy Ea at temperature T1
T2 > T1
T1
total no. of molecules with
T2 energy Ea at temperature T2

0 kinetic energy
Ea

The shaded area under the Maxwell-Boltzmann distribution curve is proportional to the number of
molecules possessing energy greater than or equal to the activation energy, E a.
When temperature decreases (from T2 to T1), the number of reacting molecules possessing
energy greater than or equal to the activation energy decreases.
The frequency of effective collisions decreases. Hence, the rate of reaction decreases.
B8(a)

Explanations:
W, C6H8O W is not a saturated compound.

W reacts with excess hydrogen in presence of W is reduced to X.


nickel catalyst to give X, C6H12O Or eqn

X reacts with sodium metal to give H2(g) X contains an alcohol functional group.
Or eqn

W gives an orange ppt with 2,4-DNPH Condensation / Addition-elimination reaction


W must be an aldehyde or ketone.
Or eqn

W does not react with Tollens Reagent W is not an aldehyde. W is likely to be a


ketone.

W decolourised reddish-brown bromine and W undergoes addition to form Y. W is an


formed Y, C6H8Br2O. alkene.
Or eqn

With HCN, W forms compound Z, C7H9NO. W undergoes addition to form Z.


Or eqn

O + 2H2 OH

W X

OH
+ Na + H2
X
NO2 CH3
N CHH
2C
O N HC CH
NO
2 2
+ H CH2 CH2 + H2O
W

O + Br2 Br O

W
Br Y

N
C
O + HCN
OH

W Z
Structures

W: O Y: Br O
cyclohex-3-en-1-one

Br
N
X: Z: C
OH
OH

8b(i)
C- Cl bond is the weakest.
Cl has the largest atomic orbital, the valence orbital used in bonding is larger and more diffuse so
that the overlap of the C- Cl orbitals is least effective, hence C- Cl bond length is the longest/C-Cl
bond energy is the smallest

8b(ii)
CHFClCF3 has a lower boiling point than CClF2CF2CHClF.
CHFClCF3 has a smaller and less polarisable electron cloud, thus it has weaker van der Waals
forces of attraction between the molecules. Less energy is required to overcome these weaker
interactions and thus CHFClCF3 has a lower boiling point.

8c(i)

shape: bent Bond angle: 104.50

8c(ii)
Chlorine dioxide has an unpaired electron thus it has a tendency to take in electrons to achieve a
stable electronic structure.

8c(iii)
Lone pair of electrons on oxygen in chlorine dioxide is able to undergo H-bonding with water.
RIVER VALLEY HIGH SCHOOL
YEAR 6 PRELIMINARY EXAMINATION
CANDIDATE
NAME

CLASS 6

CENTRE INDEX
NUMBER S NUMBER

H1 CHEMISTRY 8872/01
Paper 1 Multiple Choice 22 September 2010
50 minutes

Additional Materials: Multiple Choice Answer Sheet


Data Booklet

READ THESE INSTRUCTIONS FIRST

Write in soft pencil.


Do not use staples, paper clips, highlighters, glue or correction fluid.
Write your name, class, Centre number and index number on the Answer Sheet in the spaces
provided.

There are thirty questions on this paper. Answer all questions. For each question there are
four possible answers A, B, C and D.
Choose the one you consider correct and record your choice in soft pencil on the separate
Answer Sheet.

Read the instructions on the Answer Sheet very carefully.

Each correct answer will score one mark. A mark will not be deducted for a wrong answer.
Any rough working should be done in this booklet.

________________________________________________________________________
This document consists of 12 printed pages.

River Valley High School 8872/01/PRELIM/10 Turn over


Preliminary Examination 2010
2

Section A

For each question there are four possible answers, A, B, C and D. Choose the one you
consider to be correct.

1 3 g of hydrogen gas reacted with 160 g of bromine gas to give hydrogen bromide, HBr,
under standard conditions. How many molecules are present at the end of the reaction?

A 6.02 x 1023 A 9.03 x 1023


C 12.0 x 1023 C 15.1 x 1023

2 10 cm3 of a hydrocarbon was completely burnt in 100 cm3 of excess oxygen. When
cooled to room temperature, the volume of residual gas obtained was 80 cm3. When the
residual gas was passed through aqueous sodium hydroxide, the volume decreased to
20 cm3.
Which one of the following represents the structure of the hydrocarbon? (All volumes are
measured at r.t.p.)

A C6H12 B C6H10
C C6H8 D C6H6

3 25.0 cm3 of 0.05 mol dm3 KClO4(aq) required 50.0 cm3 of 0.20 mol dm3 TiCl3(aq) to
reach end-point. Given that Ti(III) ion is oxidised to Ti(IV) ion in this reaction, which one
of the following formulae represents the reduction product of the ClO4 ion?

A Cl B ClO
C ClO2 D ClO3

4 The table below gives the successive ionisation energies for an element X.
What could be the formula of the chloride of X?
1st 2nd 3rd 4th 5th 6th
ionisation energy
950 1800 2700 4800 6000 12300
/kJ mol1

A XCl B XCl2
C XCl3 D XCl4

River Valley High School 8872/01/PRELIM/10


Preliminary Examination 2010
3

5 Consider the following particles.

particle number of neutrons number of nucleons


P 16 33
Q 18 35
R 17 34
S 16 31

Which one of the following species has more electrons than neutrons and more neutrons
than protons?

A P B Q+
C R2 D S3

6 Which of the following ions would be deflected the least when a beam is passed through
an electric field?

24
A 12 Mg2+ B
27
13 Al 3+

C
16
8 O2 D
31
15 P3

7 Which of the following does not have a planar structure?

A NO3 B H2C2O4
C ICl4 D H2C=C=CH2

8 Which quantity would best indicate the relative strengths of the hydrogen bonds between
the molecules in liquid hydrogen halides?

A enthalpy changes of vapourisation


B enthalpy changes of formation
C enthalpy changes of solution
D bond dissociation energies

9 Which equation represents the standard enthalpy change of atomisation of bromine?

A Br2(l) 2Br(g)
B Br2(g) 2Br(g)
C Br2(l) Br(g)
D Br2(g) Br(g)

River Valley High School 8872/01/PRELIM/10 Turn over


Preliminary Examination 2010
4

10 It is often said that the rate of a typical reaction is roughly doubled by raising the
temperature by 10 C.
What of the following statements best explains this observation?

A Raising the temperature by 10 C doubles the average energy of each reactant


particle.
B Raising the temperature by 10 C doubles the average speed of the reactant
particles.
C Raising the temperature by 10 C doubles the frequency of collisions of the reactant
particles.
D Raising the temperature by 10 C doubles the number of reactant particles having
more than a certain minimum energy.

11 The reaction of hydrogen peroxide with iodide ions in an acidic solution can be monitored
by an initial rates method.
H2O2(aq) + 2H+(aq) + 2I (aq) 2H2O(l) + I2(aq)
The rate equation was found to be as follows:
Rate = k [H2O2][I ]
What of the following statements is incorrect?

A The overall order of the reaction is 2.


B The value of the rate constant, k, is dependent on temperature.
C The rate is independent of the hydrogen ion concentration.
D The rate doubles when the concentration of all the reactants is doubled.

12 An equilibrium can be represented by the following equation.


P(aq) + Q(aq) = 2R(aq) + S(aq)
In a certain mixture, the equilibrium concentration of Q is 10 mol dm3.
What will the new equilibrium concentration of Q be if 5 mol of pure Q is dissolved in the
mixture?

A between 10 mol dm3 and 15 mol dm3


B between 5 mol dm3 and 10 mol dm3
C 15 mol dm3
D 10 mol dm3

River Valley High School 8872/01/PRELIM/10


Preliminary Examination 2010
5

13 In an acid-base titration, a 0.20 mol dm3 solution of a base is added to 25 cm3 of a


0.10 mol dm3 solution of an acid.
The pH value of the solution is plotted against the volume of base added, V, as shown in
the diagram.

13

11

9
pH
7

12.5 25.0
V /cm3

This diagram could represent a titration between

A CH3COOH(aq) and NH3(aq).


B HCl(aq) and Ba(OH)2(aq).
C H2SO4(aq) and NaOH(aq).
D HNO3(aq) and NH3(aq).

14 An element X is a solid with a low conductivity at room temperature. It forms only one
chloride, which is a liquid at room temperature and a non-conductor of electricity. The
chloride of X hydrolyses in water, forming a white solid and a strongly acidic solution.
Which of the following elements could X be?

A aluminium
B phosphorus
C silicon
D sulfur

River Valley High School 8872/01/PRELIM/10 Turn over


Preliminary Examination 2010
6

15 Which property of the oxides of Period 3 elements (sodium to phosphorus) decreases


from Na2O to Al2O3 and also from SiO2 to P4O10?

A melting point
B covalent character
C pH of the resultant solution when the oxide is mixed with water
D solubility in aqueous alkali

16 Consider the following reaction :


Cl2(g) + 2H2O(l) + SO2(g) 2HCl(aq) + H2SO4(aq)
Cl2(g) + H2S(g) 2HCl(aq) + S(s)
SO2(g) + 2H2S(l) 2H2O(l) + 3S(s)
What can be deduced about the strength of the three reacting gases as reducing agents?

strongest weakest
A hydrogen sulfide chlorine sulfur dioxide
B chlorine sulfur dioxide hydrogen sulfide
C hydrogen sulfide sulfur dioxide chlorine
D sulfur dioxide hydrogen sulfide chlorine

17 Which of the following is the least likely product formed in the termination step in the
reaction of ethane with chlorine in the presence of uv light?

A HCl B CH3CH2Cl
C CH3CHCl2 D CH3CH2CH2CH3

18 How many isomeric alkenes are there with the molecular formula C4H8?

A 2 B 3
C 4 D 5

19 When compound Y was heated with acidified potassium manganate(VII), a gas was
produced which gave a white precipitate when passed into limewater.
Which of the following could not be Y?

A CH3OH B C6H5CH2CH3
C CH3CCHCHCH3 D CH3CH2CHCH2

River Valley High School 8872/01/PRELIM/10


Preliminary Examination 2010
7

20 In which of the following reactions is elimination the major type of reaction?

21 Aldehydes and ketones are produced industrially by the catalytic oxidation of alkenes.
For example, ethanal is manufactured from ethene as shown.

catalyst
H2C=CH2 + O2 CH3CHO
The process is also used industrially with but-2-ene, CH3CH=CHCH3.
Which compound is obtained from but-2-ene?

A CH3CH2CHO B CH3COCH2CH3
C CH3CH2CH2CHO D (CH3)2CHCHO

River Valley High School 8872/01/PRELIM/10 Turn over


Preliminary Examination 2010
8

22 Acidified potassium dichromate(VI) of concentration 1 mol dm3 was added in parts to


three separate test-tubes containing 10 cm3 of 1 mol dm3 of unknown solutions X, Y and
Z respectively. Each mixture was warmed and the observations are recorded below.

colour observed upon warming when


1 part of 3.3 cm3 of
st
2nd part of 3.3 cm3 of
solution
K2Cr2O7 was added K2Cr2O7 was added
X orange orange
Y green orange
Z green green

Which of the following could be the identities of X, Y and Z?

X Y Z
A C6H5CH2OH CH3CH2CH2OH CH3CH2CH2CHO
B CH3CH2CH2CH3 CH3C(OH)CHCH3 CH3CHCHCH2OH
C CH3CH2CH2COOH CH3CH2CH2CHO CH3CH2COCH3
D (CH3)2C(OH)CH2CH3 CH3CH(OH)CH2CH3 CH3CH2CH2OH

23 Dichlorodifluoromethane, CCl2F2, is widely used in aerosol propellants and as a


refrigerant.
Which statement helps to explain why dichlorodifluoromethane is chemically inert?

A The carbon-fluorine bond energy is large.


B The carbon-fluorine bond has a low bond polarity.
C Fluorine is highly electronegative.
D Fluorine compounds are non-flammable.

24 Which pair of reactions could have the same common intermediate?


W CH3CH2CH3 intermediate (CH3)2CHCN
X CH3CH(OH)CH3 intermediate (CH3)2C(OH)CN
Y CH3CH=CH2 intermediate CH3CH(OH)CH3
Z CH3CO2CH2CH2CH3 intermediate CH3CH2CH2Br

A W and X B W and Y
C X and Z D Y and Z

River Valley High School 8872/01/PRELIM/10


Preliminary Examination 2010
9

25 Tamoxifen is widely used in the treatment of breast cancer.

Tamoxifen

What is the number of sp2 and sp3 carbon atoms respectively after subjecting Tamoxifen
to hydrogen gas under heat and in the presence of nickel.

sp2 sp3
A 6 20
B 8 18
C 18 8
D 20 6

River Valley High School 8872/01/PRELIM/10 Turn over


Preliminary Examination 2010
10

Section B

For each of the questions in this section, one or more of the three numbered statements 1 to 3
may be correct.

Decide whether each of the statements is or is not correct (you may find it useful to put a tick
against the statements that you consider to be correct).

The responses A to D should be selected on the basis of

A B C D
1, 2 and 3 are 1 and 2 only are 2 and 3 only are 1 only is
correct correct correct correct

No other combination of statements is used as a correct response.

26 In which sequences are the molecules quoted in order of increasing bond angle within
the molecule?

1 H2O NH3 CH4


2 H2O SF6 BF3
3 CH4 CO2 SF6

27 The equilibrium constant, Kc, for the reaction


X(g) + Y(g) = Z(g)
varies with temperature as shown in the diagram below.

Kc

T
Which conclusions can be drawn from the given information?

1 The equilibrium mixture contains a greater proportion of Z at higher pressures.


2 The equilibrium mixture contains a greater proportion of Z at higher temperatures.
3 The reaction is endothermic in the forward direction.

River Valley High School 8872/01/PRELIM/10


Preliminary Examination 2010
11

The responses A to D should be selected on the basis of

A B C D
1, 2 and 3 are 1 and 2 only are 2 and 3 only are 1 only is
correct correct correct correct

No other combination of statements is used as a correct response.

28 The diagram below represents the reaction profile of a particular reaction.


k
reactants 1
products
k1

energy

reactants Ea

H
products

reaction pathway
Which of the following will cause both of the rate constants of the forward and backward
reactions, k1 and k1, to be increased?

1 introducing a catalyst
2 heating the equilibrium mixture
3 increasing the concentrations of both the reactants and products

29 Compounds containing CN ions are toxic. It is known that 50 mg of CN ions will be fatal
to human beings.
Which of the following compounds do not contain free CN ions in their aqueous
solutions?

1 CH3CN
2 HCN
3 NH4CN

River Valley High School 8872/01/PRELIM/10 Turn over


Preliminary Examination 2010
12

The responses A to D should be selected on the basis of

A B C D
1, 2 and 3 are 1 and 2 only are 2 and 3 only are 1 only is
correct correct correct correct

No other combination of statements is used as a correct response.

30 Which reagents will convert ethanol into bromoethane?

1 red phosphorus and bromine


2 sodium bromide and sulfuric acid
3 aqueous bromine

END OF PAPER

River Valley High School 8872/01/PRELIM/10


Preliminary Examination 2010
RIVER VALLEY HIGH SCHOOL
YEAR 6 PRELIMINARY EXAMINATION
CANDIDATE
NAME

CLASS 6

CENTRE INDEX
NUMBER S NUMBER

H1 CHEMISTRY 8872/02
Paper 2 21 September 2010
2 hours
Candidates answer Section A on the Question Paper.
Additional Materials: Data Booklet

READ THESE INSTRUCTIONS FIRST

Write your name, class, Centre number and index number on all the work you hand in.
Write in dark blue or black pen on both sides of the paper.
You may use a soft pencil for any diagrams, graphs or rough working.
Do not use staples, paper clips, highlighters, glue or correction fluid.

Section A
Answer all the questions.

Section B
Answer two questions on separate answer paper.

A Data Booklet is provided. Do not write anything on it.

The number of marks is given in brackets [ ] at the end of each question or part question.
At the end of the examination, fasten all your work securely together.

For Examiners Use

Section A

B5

B6

B7

Total
________________________________________________________________________________________________________
This document consists of 12 printed pages.

River Valley High School 8872/02/PRELIM/10 [Turn over


Preliminary Examination 2010
2

Section A

Answer all questions in this section.

1 The Kolbe reaction is an organic reaction named after Adolph Kolbe. It is formally a
decarboxylative dimerisation and proceeds by a free radical reaction mechanism.
When the salt of a monocarboxylic acid, RCOOH, is electrolysed, an alkane (RR)
is produced via the following steps.
Step 1 : RCOO R. + CO2 + e
Step 2 : 2R. RR
Electrolysis of a mixture of salts of two mono-carboxylic acids A (RCOOH) and B
(RCOOH) produces three straight-chain alkanes C, D and E, which can be
separated by fractional distillation.

(a) A solution containing 0.400 g of A required 19.60 cm3 of 0.100 mol dm3
barium hydroxide for complete neutralisation.
Calculate the relative molecular mass of A.

[2]

(b) A gaseous sample of 0.300 g of C occupied a volume of 84.0 cm3 at 25 C


and 1 atm.
Calculate the relative molecular mass of C.

[2]

River Valley High School 8872/02/PRELIM/10


Preliminary Examination 2010
3

(c) A 1.00 g sample of D was burned in an excess of oxygen and the residual
gas was passed through silica gel (to absorb the water vapour) and then
bubbled through concentrated NaOH(aq). It was found that the mass of silica
gel increased by 1.55 g and the NaOH(aq) bottle increased in mass by
3.03 g.

(i) Calculate the number of moles of carbon dioxide, and of water,


produced.

(ii) Hence, calculate the C:H ratio in D.

[2]

(d) Using your results from (a)(c), suggest the structures of A to E.

A:

B:

C:

D:

E: [3]

River Valley High School 8872/02/PRELIM/10 [Turn over


Preliminary Examination 2010
4

(e) When acid B is in the liquid state, its relative molecular mass is found to be
almost twice that in the aqueous state.
With the aid of a diagram, explain this observation.

Diagram:

Explanation: ..

....

....

....

.... [3]

(f) The Ka values for three monocarboxylic acids are given below.

acid Ka

CH3COOH 4.73

CCl3COOH 0.63

C6H5COOH 4.20

Explain the differences in the Ka values.

....

....

....

....

....

....

....

....

.... [3]

[Total: 15]

River Valley High School 8872/02/PRELIM/10


Preliminary Examination 2010
5

2 (a) Ammonia is an extremely valuable inorganic substance used in the fertiliser


industry, the manufacture of explosives, and many other applications.
Ammonia is mainly produced via the Haber process:
N2(g) + 3H2(g) = 2NH3(g)

(i) Write an expression for the equilibrium constant, Kc, for the reaction in
the Haber process.

(ii) Calculate the value of Kc given the following equilibrium composition in


a 2 dm3 reaction vessel at 1000 K.

amount at
substance
equilibrium /mol

nitrogen 14.0

hydrogen 18.0

ammonia 1.6

(iii) The reaction between nitrogen and hydrogen is slow because nitrogen
is chemically inert.
Suggest why nitrogen is so unreactive.

[4]

River Valley High School 8872/02/PRELIM/10 [Turn over


Preliminary Examination 2010
6

(b) Ammonia can also act as a precursor for the synthesis of the cationic
component of ionic liquids.
Ionic liquids are salts which exist in the liquid state at temperatures below
100 C. There is increased interest in these ionic liquids because their
physical properties (e.g. electrical conductivity, ability to dissolve inorganic or
organic compounds) can be easily manipulated by varying the types of
cations and anions present.
An example of an ionic liquid is tetrahexylammonium tetrafluoroborate:

(i) Suggest a dot-and-cross diagram to show the bonding in the


tetrafluoroborate anion, BF4, and state its shape.

Diagram:

Shape: ..

(ii) Explain why ionic liquids are good solvents for both polar and non-
polar substances.

(iii) Explain why ionic liquids can conduct electricity.

[5]

[Total: 9]

River Valley High School 8872/02/PRELIM/10


Preliminary Examination 2010
7

3 The use of the Data Booklet is relevant in this question.


The oxides of elements in Period 3 of the Periodic Table show different properties.

(a) Explain, in terms of structures and bonding, why

(i) the melting point of SiO2 is much higher than that of SO2,

(ii) the melting point of MgO is higher than that of Na2O.

[4]

(b) Write equations for the reactions when Na2O and P4O10 are separately added
to water. State the pH of the resulting solutions formed.

Na2O

Equation:

pH:

P4O10

Equation:

pH: [3]

(c) Al2O3 is said to be amphoteric.

Using Al2O3 as an example, write two equations to illustrate the term


amphoteric.

[2]

[Total: 9]

River Valley High School 8872/02/PRELIM/10 [Turn over


Preliminary Examination 2010
8

4 (a) Jasmonic acid, methyl jasmonate and jasmone are volatile natural
compounds extracted from jasmine flowers.

O O O

COOH CO2CH 3

jasmonic acid methyl jasmonate jasmone

Describe, by means of simple chemical tests, how you would distinguish


between the three compounds shown above.

[4]

(b) State the reagents and conditions for converting jasmonic acid to methyl
jasmonate in the laboratory.

[1]

(c) Jasmonic acid and methyl jasmonate function as volatile signaling molecules
that are released when plants suffer injury.
State and explain which of the compounds you would expect to be more
volatile.

[2]

[Total: 7]

River Valley High School 8872/02/PRELIM/10


Preliminary Examination 2010
9

Section B

Answer two of the three questions in this section on separate answer paper.

5 Ethanol is an increasingly important bio-fuel as it can be produced from renewable


sources as such sugar-cane and other plants.

(a) Define, with the aid of an equation, the standard enthalpy change of
combustion of ethanol. [2]

(b) The figure below shows a diagram of a calorimeter used by a student to


determine the enthalpy change of combustion of ethanol.

thermometer

small metal can

water

wick

spirit lamp containing


ethanol

(i) When 0.96 g of ethanol was combusted in a calorimeter, the energy


evolved heated 100 cm3 of water from 28.0 C to 65.3 C.
Given that this process is only 70% efficient, use these data and
values from the Data Booklet to calculate the enthalpy change of
combustion of ethanol.

(ii) Suggest a possible reason why only 70% of the heat produced from
the combustion is used to warm the water.
[4]

(c) An accurate value for the standard enthalpy change of combustion of ethanol
is 1370 kJ mol1.
Suggest why the value calculated in (b)(i) is still less than the accurate value
despite having taken into account the efficiency of heat transfer to the water. [1]

(d) (i) Use the bond energies given in the Data Booklet to calculate another
value for the standard enthalpy change of combustion of ethanol.

(ii) Suggest a reason for the discrepancy between this value and the
accurate value given in (c).
[3]

River Valley High School 8872/02/PRELIM/10 [Turn over


Preliminary Examination 2010
10

(e) Bromoethane can be hydrolysed by aqueous sodium hydroxide to form


ethanol. Results of an investigation into the kinetics of this reaction are given
below.

experiment [CH3CH2Br] [NaOH(aq)] relative initial


number /mol dm3 / mol dm3 rate

1 0.10 0.20 1.00

2 0.20 0.10 1.00

3 0.30 0.20 3.00

(i) Use the data in the table to deduce the order of reaction with respect to
CH3CH2Br and with respect to NaOH. Hence write an overall rate
equation for the reaction.

(ii) Calculate a value for the rate constant, giving its units.

(iii) Explain what is meant by the term activation energy.

(iv) Describe how the activation energy is affected by the presence of a


catalyst, and explain how this increases the rate of reaction.

(v) Explain, in molecular terms, how the initial reaction rate would be
affected by changing the concentration of reactants in a reaction.
[10]
[Total: 20]

River Valley High School 8872/02/PRELIM/10


Preliminary Examination 2010
11

6 (a) What do you understand by the Bronsted-Lowry theory of acids and bases? [2]

(b) For each of the following reactions (1) and (2):

(i) identify the two acids and the two bases present;

(ii) suggest, with reasons, which ion or molecule is the stronger acid, and
which is the stronger base.

(1) NH3 + H2O = NH4+ + OH Kc = 1.8 105 mol dm3


(2) C6H5O + CH3COOH = C6H5OH + CH3COO Kc = 1.3 x 106 mol dm3 [5]

(c) (i) The pH of a 0.1 mol dm3 solution of CH3COOH(aq) is 2.4, whereas
the pH of a 0.1 mol dm3 solution of HCl(aq) is 1.0.
Calculate the ratio of hydrogen ion concentrations in these two
solutions.

(ii) However, when 100 cm3 of 0.1 mol dm3 CH3COOH(aq) is reacted with
an excess of zinc powder, the same volume of hydrogen (120 cm3) is
evolved as when 100 cm3 of 0.1 mol dm3 HCl(aq) is used.
Explain why this is so.
[5]

(d) The standard enthalpy change of neutralisation for the reaction of NaOH(aq)
with HCl(aq) and with CH3COOH(aq) are 57.9 kJ mol1 and 56.1 kJ mol1
respectively.

(i) Explain the difference in the values of the standard enthalpy change of
neutralisation.

(ii) Suggest a value for the standard enthalpy change of neutralisation for
the reaction of NaOH(aq) with H2SO4(aq). Explain your answer.

(iii) State and explain the effect of an increase in temperature on the pH of


a solution of CH3COOH(aq).

(iv) The concentration of a solution of CH3COOH(aq) can be determined


accurately by titrating with a standard solution of NaOH(aq) but not a
standard solution of NH3(aq).
Explain why NH3(aq) cannot be used and suggest a suitable indicator
for the titration of CH3COOH(aq) with NaOH(aq).
[8]
[Total: 20]

River Valley High School 8872/02/PRELIM/10 [Turn over


Preliminary Examination 2010
12

7 (a) Compound A has the molecular formula C4H8.


On heating A with acidified potassium manganate(VII), compound B,
C3H6O2, is formed.
B gives effervescence with both sodium metal and aqueous sodium
carbonate.
A also reacts with cold alkaline potassium manganate(VII) to form compound
C, C4H10O2. Strong oxidation of C gives propanoic acid.
One mole of C reacts with two moles of phosphorus pentachloride to give
compound D. A reacts with chlorine to also give D.
Deduce the structures of A, B, C and D. Explain your reasoning and write
equations where appropriate to show the reactions that are occurring. [10]

(b) Compound T, C9H10O2, is used in artificial flavourings.


When T is heated under reflux with dilute aqueous acid, U, C7H6O2 and V,
C2H6O, are produced.
U is insoluble in cold water but dissolves readily in aqueous sodium
hydroxide.
On reacting V with aqueous alkaline iodine, a yellow precipitate, W, is
produced.
When V is distilled with acidified potassium dichromate(VI), a liquid X is
collected. X forms a silver mirror when warmed with Tollens reagent. X also
forms an orange precipitate with 2,4-dinitrophenylhydrazine.
Deduce the structures of T, U, V, W and X and explain the chemistry of the
reactions involved. [10]

[Total: 20]

END OF PAPER

River Valley High School 8872/02/PRELIM/10


Preliminary Examination 2010
RIVER VALLEY HIGH SCHOOL
YEAR 6 PRELIMINARY EXAMINATION

H1 CHEMISTRY 8872/01
Paper 1 Multiple Choice
Suggested Answers

1 D 6 A 11 D 16 C 21 B 26 D
2 C 7 D 12 A 17 A 22 D 27 A
3 A 8 A 13 D 18 B 23 A 28 B
4 C 9 C 14 C 19 C 24 B 29 D
5 D 10 D 15 C 20 A 25 C 30 B

River Valley High School 8872/01ANS/PRELIM/10


Preliminary Examination 2010
RIVER VALLEY HIGH SCHOOL
YEAR 6 PRELIMINARY EXAMINATION

H1 CHEMISTRY 8872/02
Paper 2
Suggested Answers

1 (a) 19.60
nBa(OH)2 required = 0.100
1000
= 0.00196 mol
Ba(OH)2 2RCOOH
nRCOOH used = 0.00196 2
= 0.00392 mol
0.400
Mr of A =
0.00392
= 102

(b) 84.0
nC =
24000
= 0.00350 mol
0.300
Mr of C =
0.00350
= 85.7

(c) (i) 1.55


nH2O =
2 1.0 + 16.0
= 0.0861 mol
3.03
nCO2 =
12.0 + 2 16.0
= 0.0689 mol

(ii) Mole ratio of C:H = 0.0689 : 2 0.0861


=2:5

(d) A: CH3CH2CH2CH2COOH

B: CH3CH2COOH

C: CH3CH2CH2CH2CH2CH3

River Valley High School 8872/02ANS/PRELIM/10


Preliminary Examination 2010
2

D: CH3CH2CH2CH3

E: CH3CH2CH2CH2CH2CH2CH2CH3

(e)
hydrogen bond
+
O H O
+ +
CH2CH3C CCH2CH3
+
O H O

When B is in its liquid state, dimerisation occurs whereby hydrogen bonds


are formed between the two acid molecules as shown in the diagram. Hence,
the molecular mass is almost twice the molecular mass of a single acid
molecule.
However, when acid B is in its aqueous state, there is a much bigger
proportion of water molecules compared to acid molecules and hydrogen
bonds are formed between the acid molecules and water molecules instead,
hence the molecular mass is that of a single acid molecule.

(f) CH3COOH has the lowest Ka and is the weakest acid as the methyl group is
electron donating and intensifies the negative charge of the COO group,
making the ethanoate anion the least stable.
C6H5COOH has a higher Ka than CH3COOH as the negative charge of the
COO group is delocalised into the benzene ring, making the benzoate anion
more stable than the ethanoate anion.
CCl3COOH has the highest Ka and is the strongest acid as the electron-
withdrawing Cl atoms disperses the negative charge of the COO group,
making the CCl3COO anion the most stable.

2 (a) (i) [NH3 ]2


Kc =
[N2 ][H2 ]3

(ii) 2
1.6

Kc = 2
3
14.0 18.0

2 2
= 1.25 104 mol2 dm6

(iii) The N%N (triple) bond is very strong / has a high bond energy.

(b) (i)

River Valley High School 8872/02ANS/PRELIM/10


Preliminary Examination 2010
3

Tetrahedral

(ii) Ionic liquids can form ion-dipole interactions with polar molecules and
van der Waals forces/instantaneous dipole-induced dipole interactions
with non-polar molecules.

(iii) In the liquid state, the ions are mobile and can migrate to the
electrodes when a potential difference is applied.

3 (a) (i) SiO2 has a giant molecular/covalent structure while SO2 has a simple
molecular/covalent structure. More energy is required to overcome the
stronger covalent bonds between the Si and O atoms in SiO2 than the
weaker van der Waals forces between the SO2 molecules.

(ii) Both MgO and Na2O have giant ionic structures. Since Mg2+ has a
higher charge and is smaller than Na+ (Mg2+ has a higher charge
density than Na+), more energy is required to overcome the stronger
electrostatic attraction between the oppositely charged Mg2+ and O2
ions in MgO than that between the Na+ and O2 ions in Na2O.

(b) Na2O(s) + H2O(l) 2NaOH(aq) pH = 12


P4O10(s) + 6H2O(l) 4H3PO4(aq) pH = 2

(c) Al2O3(s) + 6H+(aq) 2Al3+(aq) + 3H2O(l)


Al2O3(s) + 2OH(aq) + 3H2O(l) 2[Al(OH)4](aq) or
Al2O3(s) + 2NaOH(aq) + 3H2O(l) 2Na[Al(OH)4](aq)
(state symbols not required in this question)

4 (a) To distinguish jasmonic acid from the rest, use any one of the following tests:
Add Na2CO3(aq). Effervescence (of a colourless odourless gas which
gives a white ppt with limewater) occurs with jasmonic acid, but no gas is
evolved with methyl jasmonate and jasmone.
Add Na. Effervescence (of a colourless odourless gas which extinguishes
a lighted splint with a pop sound) occurs with jasmonic acid, but no gas
is evolved with methyl jasmonate and jasmone.
Add PCl5 or SOCl2. White fumes formed with jasmonic acid, but no white
fumes with methyl jasmonate and jasmone.
To distinguish methyl jasmonate from the rest:
Add acidified K2Cr2O7(aq) and heat. Orange solution of K2Cr2O7 turns
green with methyl jasmonate, but K2Cr2O7 remains orange for jasmonic
acid and jasmone.

(b) CH3COOH with a few drops of concentrated H2SO4, heat under reflux

(c) Methyl jasmonate is more volatile.


Less energy is required to overcome the weaker van der Waals forces
between methyl jasmonate molecules than the stronger hydrogen bonding
between jasmonic acid molecules.

River Valley High School 8872/02ANS/PRELIM/10


Preliminary Examination 2010
4

5 (a) The standard enthalpy change of combustion of ethanol is the energy


change when one mole of ethanol is completely burnt in oxygen under
standard conditions of 298 K and 1 atm.
CH3CH2OH + 7/2O2(g) 2CO2(g) + 3H2O(l)
(state symbols required)

(b) (i) Heat absorbed by water = 100 4.18 (65.3 28.0)


= 15600 J
= 15.6 kJ
100
Heat evolved from combustion of ethanol = 15.6
70
= 22.3 kJ
0.96
nethanol burnt =
2 12.0 + 6 1.0 + 16.0
= 0.0209 mol
22.3
Hc (ethanol) =
0.0209
= 1070 kJ mol1

(ii) Heat produced from the combustion could be lost to the surroundings
during the heat transfer from the flame to the water/ from the warm
water (especially without the use of a lid). or
Some of the heat produced from the combustion is absorbed by the
metal can.

(c) Combustion of ethanol may be incomplete due to inadequate supply of


oxygen near the flame.

(d) (i) CH3CH2OH(l) + 7/2O2(g) 2CO2(g) + 3H2O(l)

Hc,(CH3CH2OH) = [E(CC) + 5E(CH) + E(CO) + E(OH) + 7/2E(O=O)]


[4E(C=O) + 6E(OH)]
= [(+350) + 5(+410) + (+360) + (+460) + 7/2(+496)] [4(+740) + 6(+460)]

= 764 kJ mol1

(ii) The value for Hc,(CH3CH2OH) obtained from bond energies is not
accurate as the bond energies quoted from the Data Booklet represent
average bond energies derived from the full range of molecules that
contain the particular bonds.
CH3CH2OH and H2O are in the liquid state under standard conditions
but calculation in (d)(i) makes use of bond energies which relate to
breaking of covalent bonds in gaseous molecules.

(e) (i) Using experiments 1 and 3:


As [CH3CH2Br] is tripled while [NaOH] is kept constant, rate is tripled.
Hence the reaction is first order with respect to CH3CH2Br / the order of

River Valley High School 8872/02ANS/PRELIM/10


Preliminary Examination 2010
5

reaction with respect CH3CH2Br is 1.


Let rate = k[CH3CH2Br][NaOH]n
Using experiments 1 and 2:
Rate(1) k (0.10)(0.20)n 1.00
= =
Rate(2) k (0.20)(0.10)n
1.00
1
( 2) = 1
n

2
n=1
Hence the reaction is first order with respect to NaOH / the order of
reaction with respect NaOH is 1.
Rate = k[CH3CH2Br][NaOH]

(ii) Using experiment 1,


1.00 = k(0.10)(0.20)
k = 50.0 mol1 dm3 s1

(iii) The activation energy is the minimum amount of energy that reactant
particles must possess before a reaction could occur.

(iv) A catalyst provides an alternative reaction pathway with a lower


activation energy. As a result, more reactant particles possess the
energy required for an effective collision and the frequency of effective
collisions increases. The rate of reaction is thus increased.

(v) The rate of reaction generally increases with the concentration of one
or more of the reactant species. As the concentration of the reactant is
increased, the number of reactant particles per unit volume increases.
Consequently, there will be an increase in the frequency of collisions
between the reactant particles.

6 (a) An acid is a substance that can donate a proton to another substance, i.e.
proton donor, while a base is a substance that can accept a proton from
another substance, i.e. proton acceptor.

(b) (i) (1) acids H2O and NH4+; bases NH3 and OH
(2) acids CH3COOH and C6H5OH; bases C6H5O and CH3COO

(ii) (1) The low value of Kc(1) implies that NH4+ and OH react readily to
give NH3 and H2O / NH4+ and OH is relatively less stable than NH3
and H2O respectively. Hence NH4+ is a stronger acid than H2O and
OH is a stronger base than NH3.
(2) The high value of Kc(2) implies that C6H5O and CH3COOH react
readily to give C6H5OH and CH3COO / C6H5O and CH3COOH is
relatively less stable than C6H5OH and CH3COO respectively. Hence
CH3COOH is a stronger acid than C6H5OH and C6H5O is a stronger
base than CH3COO.

River Valley High School 8872/02ANS/PRELIM/10


Preliminary Examination 2010
6

(c) (i) [H+] in CH3COOH = 102.4


= 3.98 103 mol dm3
[H+] in HCl = 101.0
= 1.00 101 mol dm3
Ratio of [H+] in CH3COOH : [H+] in HCl = 3.98 103 : 1.00 101
1 : 25

(ii) The same volume of H2 is evolved in both cases as the ionic


equilibrium of CH3COOH = CH3COO + H+ is pulled completely over to
the right by zinc reacting with the H+ formed, so CH3COOH is
completely reacted.
100
Volume of H2 evolved = 0.1 24000
1000
= 120 cm3

(d) (i) CH3COOH is a weak acid that ionises partially in aqueous solution
while HCl is a strong acid that ionises completely in aqueous solution.
Since ionisation of CH3COOH is endothermic, some of the heat
released from the neutralisation is absorbed to bring about further
ionisation of the weak acid. Therefore, the enthalpy change of
neutralisation of NaOH with CH3COOH is less exothermic than that of
NaOH with HCl.

(ii) Hneut = 57.9 kJ mol1


Hneut for H2SO4 and HCl are expected to be the same as they are
both strong acids that dissociate completely to give H+ in solution and
the enthalpy change involves only heat evolved from the formation of
one mole of water.

(iii) pH of the acid solution decreases with increasing temperature. As the


acid dissociation process is endothermic, the position of equilibrium of
the dissociation process shifts to the right as temperature increases,
causing [H+] to increase.

(iv) There is no appropriate indicator for a titration of CH3COOH with NH3


as there is no sharp change in pH throughout the titration to give a
sharp colour change in the indicator, hence the end-point cannot be
determined accurately.
Phenolphthalein.

River Valley High School 8872/02ANS/PRELIM/10


Preliminary Examination 2010
7

7 (a) A has molecular formula CnH2n and undergoes oxidation with acidified
KMnO4(aq) to give B with one less carbon atom A is a terminal alkene
CH3CH2CH=CH2 + 5[O] CH3CH2COOH + CO2 + H2O
B undergoes acid-metal reaction with Na and acid-carbonate reaction with
Na2CO3 B is a carboxylic acid
CH3CH2COOH + Na CH3CH2COONa + H2
2CH3CH2COOH + Na2CO3 2CH3CH2COONa + CO2 + H2O
A undergoes mild oxidation with cold alkaline KMnO4(aq) to give C C is a
diol
CH3CH2CH=CH2 + H2O + [O] CH3CH2CH(OH)CH2OH
(C undergoes oxidation to give propanoic acid with loss of one carbon atom
A is a terminal alkene
CH3CH2CH(OH)CH2OH + 4[O] CH3CH2COOH + CO2 + 2H2O)
1 mol of C (a diol) undergoes nucleophilic substitution with 2 mol of PCl5 to
give D D is an alkyl halide with two Cl atoms
CH3CH2CH(OH)CH2OH + 2PCl5 CH3CH2CHClCH2Cl + 2POCl3 + 2HCl
A undergoes electrophilic addition with Cl2 to give D
CH3CH2CH=CH2 + Cl2 CH3CH2CHClCH2Cl
A: CH3CH2CH=CH2
B: CH3CH2COOH
C: CH3CH2CH(OH)CH2OH
D: CH3CH2CHClCH2Cl

(b) T has a molecular formula with C:H 1:1 T contains a benzene ring
T undergoes acidic hydrolysis with HCl(aq) to give U and V T is an ester
and U and V are carboxylic acid (or benzoic acid) and alcohol (or ethanol)
respectively
U is insoluble in cold water due to the large hydrophobic benzene ring, but
undergoes acid-base reaction with NaOH(aq) U is benzoic acid
V gives a positive tri-iodomethane test with alkaline I2(aq) V has the
structure RCH(CH3)OH V is ethanol
V undergoes oxidation with acidified K2Cr2O7(aq) to give X V is an alcohol
(or ethanol)
X undergoes oxidation with Tollens reagent X is an aldehyde (or ethanal)
X undergoes condensation with 2,4-DNPH X is a carbonyl
compound/aldehyde (or ethanal)
T: C6H5COOCH2CH3
U: C6H5COOH
V: CH3CH2OH
W: CHI3
X: CH3CHO

River Valley High School 8872/02ANS/PRELIM/10


Preliminary Examination 2010
ST. ANDREWS JUNIOR COLLEGE
Higher 1
PRELIMINARY EXAMINATION 2010

Name Class

CHEMISTRY 8872/ 01
PAPER 1 21 September 2010

50 minutes
Additional Materials: Multiple Choice Answer Sheet
Data Booklet

READ THESE INSTRUCTIONS FIRST


Sections A and B (30 marks)
There are thirty questions in these sections. Answer all questions. For each question there
are four possible answers A, B, C, and D.
Choose the one you consider correct and record your choice in soft pencil on the separate
Answer Sheet.

Each correct answer will score one mark. A mark will not be deducted for a wrong answer.

This paper consists of 10 printed pages.


2

Section A (25 marks)


For each question there are four possible answers, A, B, C and D. Choose the one
you consider to be correct.

1. Which statement about 44 g of CO2 is correct?


A It contains the same number of atoms as 14 g of 14N.
B It contains twice the number of atoms as 16 g of 16O2.
C It contains the same number of molecules as 14 g of 14N2.
D It contains the same number of molecules as 32 g of 16O2.

2. 15 cm3 of an unknown hydrocarbon was burnt completely in 200 cm3 of excess


oxygen. The volume of gases in the final mixture was 170 cm3. After passing the
gaseous products through an alkaline solution, the volume decreased by 60 cm3.
Given that all volumes were measured at room temperature and pressure, what is
the formula of the hydrocarbon?
A C4H8 B C4H10
C C5H10 D C5H12

3. Vanadium, V, in ammonium vanadate(V), NH4VO3, can be reduced to a few


oxidation states with zinc metal. A 200 cm3 solution of 0.05 mol dm-3 ammonium
vanadate(V) required 0.327 g of zinc metal to react completely. Given that zinc is
oxidised to Zn2+, what is the final oxidation state of vanadium in the product?
A +1 B +2
C +3 D +4

4. Which of the following reactions is a disproportionation reaction?


A Fe2O3 + 3CO 2Fe + 3CO2
B 2CrO42- + 2H+ Cr2O72- + H2O
C 2H2O2 2H2O + O2
D 2H2S + 3O2 2H2O + 2SO2

5. Which of the following pairs consists of a planar and non-planar molecule?


A NO3-, C3H8 B XeF4, CO32-
C NH4+, ClO3- D C2H4, H2O
3

6. Aluminium fluoride has a melting point of 1291 C while aluminium chloride


sublimes at 178 C. Which of the following statements accounts for this difference?
A Aluminium ion has a high charge density.
B The intermolecular interactions between aluminium fluoride molecules are
stronger than that between aluminium chloride molecules.
C Al F bond is stronger than Al Cl bond.
D Fluoride ion has a smaller electron cloud than chloride ion.

7. The graph below shows the second ionisation energies (in kJ mol-1) of 8
consecutive elements, A to H, whose atomic numbers are less than 20.

What is the formula of the compound formed between A and D?


A D2A B DA2
C D3A D DA

8. Alpha particles, He2+, are commonly emitted by larger radioactive nuclei. A mixture
of an unknown ion, X, O2+ and alpha particles are passed through an electric field,
as shown in the diagram below.

source

alpha particle 2+
O X

Which of the following represents X?


A Li+ B Be2+
C B3+ D Al3+

[Turn Over
4

9. A study of the kinetics of the decomposition reaction of an unknown sample, Y, was


conducted and the following data was obtained.
[Y] / mol dm-3 Relative initial rate
0.5 1
1.0 1
1.5 1

Which of the following graphs is correct?


A [Y] B [Y]

time time

C [Y] D [Y]

time time

10. Use of the Data Booklet is relevant to this question.


When 8.00 g of ammonium nitrate, NH4NO3, was added to a polystyrene cup
containing 100 cm3 of water, the temperature of the water changed from 25 C to
15 C. What is the enthalpy change of reaction of ammonium nitrate dissolving in
water?
A - 4.18 kJmol-1
B - 41.8 kJmol-1
C + 4.18 kJmol-1
D + 41.8 kJmol-1
5

11. Lattice energy of an ionic salt XY is more exothermic than MgO. Which of the
following about the ionic radii of X and Y and the charges of their ions is correct
when compared with that of magnesium and oxygen?
Ionic radius Charge of ions
X Y X Y
A Same Same Lower Lower
B Smaller Smaller Same Same
C Larger Larger Same Same
D Larger Larger Lower Lower

12. The value of the enthalpy change for the process represented by the equation:
O2(g) + 4e- 2O2-(g) is equal to
A the sum of the first and second electron affinity of oxygen
B the sum of the enthalpy change of atomisation and the first electron affinity of
oxygen
C the sum of the enthalpy change of atomisation, first and second electron affinity
of oxygen
D twice the sum of the enthalpy change of atomisation, first and second electron
affinity of oxygen

13. An equilibrium is represented by the following equation:

2SO2 (g) + O2 (g) 2SO3 (g) H = a kJ mol1

Which of the following changes would affect both the percentage yield of sulphur
trioxide and the value of the equilibrium constant?
A Adding vanadium(V) oxide as the catalyst
B Decreasing the temperature
C Increasing the mass of sulphur dioxide
D Reducing the pressure

[Turn Over
6

14. Silver chloride dissolves in dilute ammonia to form a silver(I) diamine complex and
the equilibrium equation is shown below.
AgCl(s) + 2NH3(aq) Ag(NH3)2+(aq) + Cl-(aq)
What are the units for the equilibrium constant?
A mol dm-3 B no units
C mol-1 dm3 D mol-2 dm6

15. What is the pH of a 0.20 mol dm-3 aqueous solution of methylamine, CH3NH2, at
25C? Given that Kb = 5.0 x 10-4 mol dm-3.
A 2.0 B 10.0
C 12.0 D 16.0

16. Which of the following mixtures is not a buffer solution?


A 25 cm3 of 0.05 mol dm-3 HCl mixed with 25 cm3 of 0.1 mol dm-3 CH3COONa
B 25 cm3 of 0.1 mol dm-3 NaOH mixed with 25 cm3 of 0.05 mol dm-3 CH3COOH
C 25 cm3 of 0.05 mol dm-3 HCl mixed with 25 cm3 of 0.1 mol dm-3 NH3
D 25 cm3 of 0.1 mol dm-3 NH4Cl mixed with 25 cm3 of 0.2 mol dm-3 NH3

17. Which salt is produced from a strong acid and a strong base?
A CH3COONa
B NH4NO3
C NaHCO3
D NaHSO4

18. Element L, found in period 3, forms an oxide which is insoluble in water but
dissolves in both HCl and NaOH. What is the pH of a solution of its chloride?
A 3
B 6
C 7
D 9
7

19. Which graph shows the trend in electrical conductivity across Period 3 elements?
A Electrical conductivity B Electrical conductivity

Elements Elements
Na Mg Al Si P O Cl Ar Na Mg Al Si P O Cl Ar

C Electrical conductivity D Electrical conductivity

Elements Elements
Na Mg Al Si P O Cl Ar Na Mg Al Si P O Cl Ar

20. Which structure correctly represents 4-chloro-3-hydroxybutanoic acid?


Cl

OH
A B

OH O

C D

21. Propanal, CH3CH2CHO, reacts with HCN in a nucleophilic addition reaction. What is
the hybridisation state of the carbon bonded to the nitrile (-CN) group in the
product?
A sp B sp2
C sp3 D no hybridisation present

[Turn Over
8

22. Which of the following organic compounds will most likely undergo a nucleophilic
substitution reaction by OH- to give an alcohol?
A chloroethane
B chlorobenzene
C 2-bromomethylbenzene
D 2-butene

23. A series of tests was carried out on Compound X and the following observations
were recorded in the table below.
Tests / Reagents & Conditions Observations
Alkaline aqueous iodine, Heat Yellow ppt observed with a medicinal smell
detected
Acidified KMnO4, Heat Purple KMnO4 decolourised

What could Compound X be?


A CH3CHO B CH3CH2CH(CH3)COCH3
C CH3CH2C(CH3)2OH D C6H5CH(CH3)CH2OH

24. Which carbon atom in the molecule shown will readily be attacked by an
electrophile?

25. Which of the following is the most suitable reagent for the conversion reaction
below?

A NaBr B SOBr2
C Br2 D AlBr3
9

Section B (5 marks)
For each of the questions in this section one or more of the three numbered
statements 1 to 3 may be correct. Decide whether each of the statements is or is not
correct (you may find it helpful to put a tick against the statements which you
consider to be correct).

The responses A to D should be selected on the basis of


A B C D
1,2 and 3 are 1 and 2 only are 2 and 3 only are 1 only is correct
correct correct correct

No other combination of statements is used as a correct response.

26. Which of the following statements can be explained by hydrogen bonding?


1 Butan-1-ol has a higher boiling point than 2-methylpropan-1-ol.
2 Carboxylic acids often form dimers in the gaseous state.
3 A clogged water pipe is likely to burst during winter.

27. 3-chloropropene is heated with aqueous NaOH to form CH2(OH)CH = CH2. The
kinetics of this reaction are given below.
Experiment No. [3-chloropropene] [NaOH] Initial rate
/ mol dm-3 / mol dm-3 / mol dm-3 s-1
1 0.01 0.02 1.4 x 10-3
2 0.01 0.03 1.4 x 10-3
3 0.04 0.20 5.6 x 10-3

Which of the following statements are correct?


1 Rate = k [3-chloropropene]
2 The units of rate constant are mol-1 dm3 s-1.
3 The overall order of reaction is 2.

28. Which of the following reagents can be used to distinguish the following two
compounds?
C6H5CHO CH3CH2COOH
1 Tollens reagent
2 Aqueous sodium carbonate
3 Sodium metal

[Turn Over
10

The responses A to D should be selected on the basis of


A B C D
1,2 and 3 are 1 and 2 only are 2 and 3 only are 1 only is correct
correct correct correct

No other combination of statements is used as a correct response.

29. A compound has the structural formula shown below.

How can it be classified?


1 An ester
2 A carboxylic acid
3 An alcohol

30. Which of the following compounds gives


(i) white ppt with cold ethanolic silver nitrate;
(ii) fumes of HCl with PCl5?
1

*******************END OF PAPER********************
ST. ANDREWS JUNIOR COLLEGE
Higher 1
PRELIMINARY EXAMINATION 2010

Name Class

CHEMISTRY 8872/ 02
PAPER 2 16 September 2010

Candidates are to answer Section A on the Question Paper. 2 hours


Additional Materials: Writing paper
Data Booklet

READ THESE INSTRUCTIONS FIRST


Write your name and class on all the work you hand in.
Write in dark blue or black pen on both sides of the paper.
You may use a soft pencil for any diagrams, graphs or rough working.
Do not use staples, paper clips, highlighters, glue or correction fluid.

Section A
Answer all questions.

Section B
Answer any two questions on separate writing paper.

At the end of the exam, fasten all your work securely together.
The number of marks is given in brackets [] at the end of each question or part question.

For examiners use


Section A / 40

B6

B7

B8

Total / 80

This paper consists of 15 printed pages and 1 blank page.


2

Section A
Answer all questions.

1 Sodium peroxodisulfate, Na2S2O8, is commonly used as a bleaching


agent in hair cosmetics.
(a) Given that peroxodisulfate oxidises iodide solution to form black
crystals, while it is reduced to form SO 42 , construct a balanced
equation.

[1]
A quality control experiment was set up to monitor the current level of
peroxodisulfate in the air of the laboratory to ensure that it does not reach
hazardous levels (above 1000 mg dm-3).
(b) 25 cm3 of air was bubbled into 250 cm3 of water and 25 cm3 of this
solution was taken to titrate against a standard solution of
potassium iodide, with known concentration of 0.0025 mol dm-3.
(i) Assuming that all of the peroxodisulfate particles were
dissolved into the solution, and that it took 10.25 cm3 of iodide
solution to fully react with the peroxodisulfate present,
calculate the mass of peroxodisulfate in 1 dm3 of air.

(ii) Comment on whether the laboratory needs to be temporarily


closed down for disinfection.

...


[3]
3

1 (c) The reaction between peroxodisulfate and iodide is slow. However,


when trace amounts of iron(II) nitrate was added, the reaction took
place readily. With the aid of a relevant diagram, explain the
observations above and state the role of iron(II) nitrate.

..

..

..
[4]
[Total: 8]

2 (a) Steel became an important resource with global modernisation. It is


used as materials for building skyscrapers, bridges and ships. But
iron upon exposure to the elements, would begin to corrode,
forming rust. Since ships hulls are in constant contact with salt
water and sunlight, bars of zinc were welded to the hull as an
attempt to protect the iron from corrosion.
FeO + Zn Fe + ZnO
Another material, such as aluminium, is also able to perform similar
function.
(i) Write a balanced reaction between iron(II) oxide and
aluminium.

[Turn Over
4

2 (a) (ii) Using the data below, hence calculate the enthalpy change for
the reaction proposed.
Hf(FeO) = 272 kJ mol-1
Hf(Al2O3) = 1676 kJ mol-1

[2]
(b) This reaction could also be used to extract iron from its oxide.
Assuming that excess iron(II) oxide was used during the reaction,
suggest a chemical reaction that could be used to remove the
aluminium product from the reaction vessel, with the aid of a
relevant chemical equation.

[3]
[Total: 5]
5

3 An investigation was done to determine the kinetics of the reversible


endothermic reaction between compounds A and B.

The result of the investigation was tabulated and the following graphs
were obtained.
-3 -3
[A] / mol dm [A] / mol dm

0.08 0.16

0.06 0.12

0.04 0.08

-3
0.02 [B] = 2.0 mol dm -3
[B] = 1.0 mol dm
0.04

2 4 6 8 10 time / min 4 8 12 16 20 time / min

(a) Determine the order of reaction with respect to [A] and [B]
respectively.

[2]

(b) Comment on the use of the high concentration of B in the above


experiments.


[1]
[Turn Over
6

3 (c) Calculate the rate constant, given that the initial rate for the graph
on the left is 1.39 x 10-2 mol dm-3 min-1, stating its units clearly.

[2]
(d) Suggest a change in condition to increase the amount of D
collected.


[1]
[Total: 6]

4 (a) The following compound, compound F, was synthesised from


benzene through a series of reactions.
OH

a
b

Cl

Cl
compound F

Identify two different functional groups present in the compound.

1.

2.
[2]
7

4 (b) In the table below, write down the type of hybridisation expected,
the bond angles a and b, and give a sketch of a hybrid orbital
exhibited by each of the respective carbon.

Type of Sketch of a hybrid


Carbon Bond angle
hybridisation orbital

[3]
(c) During the synthesis of the compound F, a by-product, compound
G, was formed.
O

Cl

Cl
compound G
Suggest a simple chemical test to distinguish between compounds
F and G. State the reagents and conditions needed, the
observations that you would make, and write a balanced equation
for the positive test.

[3]
[Total: 8]
[Turn Over
8

5 (a) Complete the reaction scheme below by writing the structural


formulae of the organic compounds H L and stating the reagents
and conditions needed for Steps I, II and III in the following tables.

CH3CHO
Step III

CHI3 + Compound H CH3CH2OH


iodine in
alkaline solution,
heat Step I

CH3CH2 H
H+/KMnO4
C C heat Compound I + Compound J
CH3 CH3

Step II

CH3CH2 CN
C OH
dilute H2SO4
CH3 heat Compound K

conc. H2SO4,
2 Compound K heat Compound L + 2 H2O
9

Compound H Compound I

Compound J Compound K

Compound L

Step Reagent Condition

II

III

[8]

[Turn Over
10

5 (b) Compound K, upon dehydration with excess concentrated sulfuric


acid at 180 oC, produces 3 different organic isomers M, N, and O.
Isomer M is a structural isomer of isomers N and O.
(i) Draw the displayed formulae of isomers M, N, and O.

(ii) Using your structures of isomers N and O, suggest and


explain which has the higher boiling point


[5]
[Total: 13]

End of Section A
11

Section B

Answer 2 out of 3 questions.

6 (a) A student wants to identify an unknown Group I metal, P, within a


sample of hydrogen carbonate, PHCO3. After doing some
research, she found a decomposition reaction that would suit her
purpose. The decomposition reaction is shown as:
2 PHCO3(s) P2CO3(aq) + H2O(l) + CO2(g)
She heated a 5.00 g sample of PHCO3 in a boiling tube, and
bubbled the gas evolved into a solution of calcium hydroxide
(concentration 1.0 mol dm-3). A white precipitate formed was found
to have a mass of 1.70 g. Calculate the molar mass of PHCO3 and
hence identify P. [3]

(b) A standard solution of propanoic acid was made by pipetting


20.0 cm3 of propanoic acid, labelled solution Q, which was then
made up to 250 cm3 with deionised water (labelled solution R).
Solution R was then used to react with solid PHCO3. It was
3
determined that 34.50 cm of solution R was needed to react with
0.293 g of solid PHCO3 completely. Calculate the concentration of
propanoic acid in solution Q. [2]

(c) Given that solution Q contains only propanoic acid and its pH was
2.50 under room temperature and pressure (r.t.p.) conditions,
calculate the acid dissociation constant of propanoic acid. [1]

(d) Both propanoic acid and butan-1-ol have the same relative
molecular mass, but have different pH values when the same
amount was dissolved in 1 dm3 of solution. Explain, with the aid of
relevant equations, which would have a lower pH value. [3]

[Turn Over
12

6 (e) Butan-1-ol, with a molecular formula of C4H10O, can be converted


into its various structural, positional and functional isomers through
the aid of a catalyst and heat. A sample of butan-1-ol was heated
to form 2 different structural isomers (compounds S and T).

Compound S can decolourise hot acidified potassium


manganate(VII) solution. The oxidised product, labelled compound
U, could form an orange precipitate with 2,4-dinitrophenylhydrazine
solution.

A single isomer C4H9OCl was formed when compound T when


reacted with a limited supply of chlorine gas under intense light.
(i) Identify and draw the displayed formulae of compounds S, T
and U.
(ii) State the observations that could be made and illustrate your
answer with the aid of balanced equations, when
I compound S was added with hot alkaline iodine solution;
II compound T was added to PCl5 at room temperature
and pressure conditions; and
III butanal, an oxidised product from butan-1-ol, was added
with hot Fehlings solution.
[11]
[Total: 20]

7 (a) Aluminium chloride is used as a catalyst in organic chemistry. Use


dot-and-cross diagrams to illustrate the structures of aluminium
chloride and its dimer, Al2Cl6, and state the shapes and bond
angles around the aluminium atom(s) in each structure. [4]

(b) The reaction between gaseous chlorine and benzene is catalysed


by the presence of AlCl3. Describe, with the aid of a balanced
equation, why the reaction would not likely take place when
aqueous chlorine was used instead. [2]
13

7 (c) Another commonly used catalyst for this reaction is FeCl3 and it
54 56
exists as various isotopes such as FeCl3 and FeCl3. Write the
electronic configuration of Fe in 56FeCl3. [1]

(d) A student investigated the kinetics of the reaction between FeCl3,


Cl2 and benzoic acid and the following information was obtained.

Experiment No. [FeCl3] [Cl2] / mol dm-3 [benzoic acid] Rate


/ mol dm-3 / mol dm-3 / mol dm-3 min-1
1 0.100 0.075 0.250 1.39 x 10-5
2 0.100 0.100 0.250 1.85 x 10-5
3 0.150 0.075 0.300 2.50 x 10-5
4 0.150 0.100 0.250 2.78 x 10-5

(i) Determine the order of reaction with respect to FeCl3, Cl2, and
benzoic acid. Hence write the rate equation and determine
the rate constant of the reaction using information from
experiment 3.
(ii) Sketch a labelled energy profile diagram for this reaction. [7]

(e) Suggest reagents and conditions for the 4-step synthesis of


3-oxo-butanoic acid from propene. [6]

[Total: 20]

[Turn Over
14

8 (a) Sulfur, S8, in its yellow solid state, exists in a variety of oxidation
states in its compounds. One of them is that of sulfur dioxide,
which is readily formed during a volcanic eruption or during the
combustion of fossil fuels. Using the relevant information from the
Data Booklet, and given the following data, calculate the bond
energy of S=O in sulfur dioxide.

S8(s) S8(g) Ho = +120 kJ mol-1


Hf(SO2(g)) = 296 kJ mol-1 [2]

(b) The key reaction during the Contact process, for manufacturing
sulfuric acid, is as follows:

A 3 dm3 vessel was used and 4 moles of sulfur dioxide and 4 moles
of oxygen were introduced into the vessel. Given that the
o
exothermic reaction, under the conditions of 450 C, 2 atm pressure
and presence of vanadium(V) oxide, ensured a 96% conversion,
calculate the Kc of the reaction. [3]

(c) Predict and explain the changes to the yield and the Kc value if the
following changes were made:
(i) temperature was reset to 200 oC; and
(ii) 2 moles of SO2 were added. [5]

(d) A 25 cm3 portion of the equilibrium mixture in the above part was
extracted and dissolved in 1 dm3 of distilled water, forming a
mixture labelled as mixture V. Given that sulfurous acid, H2SO3, is
a weak dibasic acid, calculate the volume of 1.5 mol dm-3 sodium
hydroxide solution required to completely neutralise the mixture V
and suggest a possible indicator to be used. [4]
15

8 (e) Suggest a chemical test to distinguish the compounds in each of


the following pairs, stating clearly all observations and balanced
equations for positive tests. [6]

Compound W Compound X
(i) CH3

Cl

(ii) H H O

HO C CH
C C
H HH H

End of Paper 2
16

BLANK PAGE
Paper 1

1 D 7 B 13 B 19 B 25 B
2 A 8 D 14 B 20 B 26 C
3 D 9 C 15 C 21 C 27 D
4 C 10 D 16 B 22 A 28 A
5 A 11 B 17 D 23 A 29 C
6 D 12 D 18 A 24 A 30 B

Paper 2

1 Sodium peroxodisulfate, Na2S2O8, is commonly used as a bleaching


agent in hair cosmetics.
(a) Given that peroxodisulfate oxidises iodide solution to form black
crystals, while it is reduced to form SO 42 , construct a balanced
equation.
S2O82- + 2I- 2SO42- + I2
A quality control experiment was set up to monitor the current level of
peroxodisulfate in the air of the laboratory to ensure that it does not reach
hazardous levels (above 1000 mg dm-3).
(b) 25 cm3 of air was bubbled into 250 cm3 of water and 25 cm3 of this
solution was taken to titrate against a standard solution of
potassium iodide, with known concentration of 0.0025 mol dm-3.
(i) Assuming that all of the peroxodisulfate particles were
dissolved into the solution, and that it took 10.25 cm3 of iodide
solution to fully react with the peroxodisulfate present,
calculate the mass of peroxodisulfate in 1 dm3 of air.
amount of iodide used : 0.01025 x 0.0025 = 2.5625 x 10-5 mol
amount of peroxodisulfate present in 25 cm3 of solution =
1.28125 x 10-5 mol
amount of peroxodisulfate present in 25 cm3 of air =
1.28125 x 10-4 mol
mass of peroxodisulfate present in 1 dm3 of air =
1.28125 x 10-4 x 40 x (192.2)
= 0.985 g
(ii) Comment on whether the laboratory needs to be temporarily
2

closed down for disinfection.


As it is still below 1000 mg dm-3, the laboratory need not be closed
down.
(c) The reaction between peroxodisulfate and iodide is slow. However,
when trace amounts of iron(II) nitrate was added, the reaction took
place readily. With the aid of a relevant diagram, explain the
observations above and state the role of iron(II) nitrate.

Uncatalysed reaction
Number of molecules
Fraction of molecules
having energy E Ea
having energy E
Catalysed reaction
Number of molecules
TK having energy E Ea

Ea Ea
Energy

Reaction was slow because all were negative ions / anions /


activation energy of the reaction was very high.
Iron (II) ions acted as a catalyst
Lowering the activation energy by providing an alternative pathway
Increasing the proportion of particles with energy equal or greater than
the new activation energy
More effective collisions took place
Increase in the rate of reaction

2 (a) Steel became an important resource with global modernisation. It is


used as materials for building skyscrapers, bridges and ships. But
iron upon exposure to the elements, would begin to corrode,
forming rust. Since ships hulls are in constant contact with salt
water and sunlight, bars of zinc were welded to the hull as an
3

attempt to protect the iron from corrosion.


FeO + Zn Fe + ZnO
Another material, such as aluminium, is also able to perform similar
function.
(i) Write a balanced reaction between iron(II) oxide and
aluminium.
3FeO + 2Al Al2O3 + 3 Fe
(ii) Using the data below, hence calculate the enthalpy change for
the reaction proposed.
Hf(FeO) = 272 kJ mol-1
Hf(Al2O3) = 1676 kJ mol-1

H rxn = Hf(Al2O3) 3 x Hf(FeO)


= 1676 3 x (272) = 860 kJ mol-1
(b) This reaction could also be used to extract iron from its oxide.
Assuming that excess iron(II) oxide was used during the reaction,
suggest a chemical reaction that could be used to remove the
aluminium product from the reaction vessel, with the aid of a
relevant chemical equation.
React the solid mixture with NaOH(aq)
Al2O3 + 2NaOH + 3H2O 2 NaAl(OH)4
FeO and Fe does not undergo any reaction with NaOH.
4

3 An investigation was done to determine the kinetics of the reversible


endothermic reaction between compounds A and B.

The result of the investigation was tabulated and the following graphs
were obtained.
-3 -3
[A] / mol dm [A] / mol dm

0.08 0.16

0.06 0.12

0.04 0.08

-3
0.02 [B] = 2.0 mol dm -3
[B] = 1.0 mol dm
0.04

2 4 6 8 10 time / min 4 8 12 16 20 time / min

(a) Determine the order of reaction with respect to [A] and [B]
respectively.
Base on the graph on the left given,

[A] / mol dm-3 0.08 0.06 0.04 0.02


Time 0 2 4 8

The half life is constant for either graph, hence order of reaction
with respect to [A] is 1

Comparing the two graphs,


Graph on the left = time taken for [A] = 0.08 mol dm-3 to drop to
0.04 mol dm-3 is 4 min.
Graph on the right = time taken for [A] = 0.08 mol dm-3 to drop to
0.04 mol dm-3 is 8 min.
This implies that rate has been halved when [B] is halved, hence
order of reaction with respect to [B] is 1.
(b) Comment on the use of the high concentration of B in the above
experiments.
This was to ensure that the rate of reaction is only dependent on
the change in [A] or words to that effect.
(c) Calculate the rate constant, given that the initial rate for the graph
on the left is 1.39 x 10-2 mol dm-3 min-1, stating its units clearly.
[A] = 0.08 mol dm-3, [B] = 2.0 mol dm-3

Rate
k= = 8.68 x 10-2 mol-1 dm3 min-1
[ A][ B ]
5

(d) Suggest a change in condition to increase the amount of D


collected.
Increase temperature / increase [A] / increase [B] / decrease [C]

4 (a) The following compound, compound F, was synthesised from


benzene through a series of reactions.
OH

a
b

Cl

Cl
compound F

Identify two different functional groups present in the compound.


1. chlorobenezene
2. secondary chloroalkane
3. secondary alcohol
4. benzene
(b) In the table below, write down the type of hybridisation expected,
the bond angles a and b, and give a sketch of a hybrid orbital
exhibited by each of the respective carbon.

Carbon type of bond angle sketch of a hybrid


hybridization orbital
dumb-bell shape,
marked with a sp2 120o with one lobe as a
circle
dumb-bell shape,
with one lobe as a
circle, the other
marked with b sp3 109.5o
lobe must be
thinner than the
one above

(c) During the synthesis of the compound F, a by-product, compound


G, was formed.
6

Cl

Cl
compound G
Suggest a simple chemical test to distinguish between compounds
F and G. State the reagents and conditions needed, the
observations that you would make, and write a balanced equation
for the positive test.
Possible answers
Using 2,4-dinitrophenylhydrazine, warm
the ketone would form an orange precipitate, but the
alcohol would not
OR
using hot acidified potassium dichromate
the alcohol would turn the dichromate from orange to
green, but the ketone would not

AND
Balanced equation

5 (a) Complete the reaction scheme below by writing the structural


formulae of the organic compounds H L and stating the reagents
and conditions needed for Steps I, II and III in the following tables.
7

CH3CHO
Step III

CHI3 + Compound H CH3CH2OH


iodine in
alkaline solution,
heat Step I

CH3CH2 H
H+/KMnO4
C C Compound I + Compound J
heat
CH3 CH3

Step II

CH3CH2 CN
C OH
dilute H2SO4
CH3 heat Compound K

conc. H2SO4,
2 Compound K heat Compound L + 2 H2O

Compound H Compound I

Compound J Compound K

Compound L
8

Step Reagent Condition


I LiAlH4 dry ether
HCN trace amount of NaOH / NaCN,
II
10 20 oC
III acidified K2Cr2O7 heat and distill

(b) Compound K, upon dehydration with excess concentrated sulfuric


acid at 180 oC, produces 3 different organic isomers M, N, and O.
Isomer M is a structural isomer of isomers N and O.
(i) Draw the displayed formulae of isomers M, N, and O.

(ii) Using your structures of isomers N and O, suggest and


explain which has the higher boiling point
isomer O has the higher boiling point
more polar as compared to isomer N
9

Section B

6 (a) A student wants to identify an unknown Group I metal, P, within a


sample of hydrogen carbonate, PHCO3. After doing some
research, she found a decomposition reaction that would suit her
purpose. The decomposition reaction is shown as:
2 PHCO3(s) P2CO3(aq) + H2O(l) + CO2(g)
She heated a 5.00 g sample of PHCO3 in a boiling tube, and
bubbled the gas evolved into a solution of calcium hydroxide
(concentration 1.0 mol dm-3). A white precipitate formed was found
to have a mass of 1.70 g. Calculate the molar mass of PHCO3 and
hence identify P.
Ca(OH)2 + CO2 CaCO3 + H2O
1.70 g of CaCO3 = 0.016983 mol
amount of CO2 present = 0.016983 mol
amount of PHCO3 present = 0.033966 mol
molar mass of PHCO3 = 147.2 g mol-1
relative atomic mass of P = 86.2, Identify of P is Rb.
(b) A standard solution of propanoic acid was made by pipetting
20.0 cm3 of propanoic acid, labelled solution Q, which was then
made up to 250 cm3 with deionised water (labelled solution R).
Solution R was then used to react with solid PHCO3. It was
determined that 34.50 cm3 of solution R was needed to react with
0.293 g of solid PHCO3 completely. Calculate the concentration of
propanoic acid in solution Q.
PHCO3 + CH3CH2COOH CH3CH2COOP+ + H2O + CO2
amount of PHCO3 present = 0.002 mol
amount of propanoic acid present in 34.50 cm3 of solution R =
0.002 mol
3
amount of propanoic acid present in 20.0 cm of solution Q =
0.014493 mol
[propanoic acid] = 0.72464 = 0.725 mol dm-3 (3 s.f.)
(c) Given that solution Q contains only propanoic acid and its pH was
2.50 under room temperature and pressure (r.t.p.) conditions,
calculate the acid dissociation constant of propanoic acid.
[H+] = 10-2.5 mol dm-3
10

[H + ][CH 3 CH 2 COO - ]
Ka =
[CH 3 CH 2 COOH]
= 1.38 x 10-5 mol dm-3
(d) Both propanoic acid and butan-1-ol have the same relative
molecular mass, but have different pH values when the same
amount was dissolved in 1 dm3 of solution. Explain, with the aid of
relevant equations, which would have a lower pH value.
pH is a measure of H+ concentration in water.

CH3CH2COO- is a more stable anion, as it would delocalise the


negative charge over 3 centers, forming a resonance structure,
but CH3CH2CH2O- is unable to do so. Hence propanoic acid would
form a stronger acid as compared to butan-1-ol and hence a lower
pH .
(e) Butan-1-ol, with a molecular formula of C4H10O, can be converted
into its various structural, positional and functional isomers through
the aid of a catalyst and heat. A sample of butan-1-ol was heated
to form 2 different structural isomers (compounds S and T).

Compound S can decolourise hot acidified potassium


manganate(VII) solution. The oxidised product, labelled compound
U, could form an orange precipitate with 2,4-dinitrophenylhydrazine
solution.

A single isomer C4H9OCl was formed when compound T when


reacted with a limited supply of chlorine gas under intense light.
(i) Identify and draw the displayed formulae of compounds S, T
and U.

Information Deduction
Compound S can decolourise hot acidified Can be oxidised
potassium manganate (VII) solution. Primary or secondary alcohol
The oxidised product, labeled compound Condensation reaction
11

U, could form orange precipitate with 2,4- Product is a ketone


dinitrophenylhydrazine solution.
A single isomer C4H9OCl was formed Substitution reaction
when compound T when reacted with a All H are in the same chemical
limited supply of chlorine gas under background.
intense light.

Structure of S Structure of T Structure of U

(ii) State the observations that could be made and illustrate your
answer with the aid of balanced equations, when
I compound S was added with hot alkaline iodine solution;
Yellow precipitate will form
CH3CH(OH)CH2CH3 + 4I2 + 6OH-
CH3CH2COO- + CHI3 + 5NaI + 5H2O
II compound T was added to PCl5 at room temperature and
pressure conditions; and
white fumes will be formed
(CH3)3COH + PCl5 (CH3)3CCl + POCl3 + HCl
III butanal, an oxidised product from butan-1-ol, was added with
hot Fehlings solution.
Brick red precipitate will be formed
CH3CH2CH2CHO + 2Cu2+ + 5OH-
CH3CH2CH2COO- + Cu2O + 3H2O

7 (a) Aluminium chloride is used as a catalyst in organic chemistry. Use dot-


and-cross diagrams to illustrate the structures of aluminium chloride and
its dimer, Al2Cl6, and state the shapes and bond angles around the
aluminium atom(s) in each structure.
12

(b) The reaction between gaseous chlorine and benzene is catalysed by the
presence of AlCl3. Describe, with the aid of a balanced equation, why the
reaction would not likely take place when aqueous chlorine was used
instead.

The catalyst would be hydrolysed and not present to lower the activation
energy of the reaction, thus resulting in a kinetically unfavourable
reaction.
(c) Another commonly used catalyst for this reaction is FeCl3 and it exists as
54 56
various isotopes such as FeCl3 and FeCl3. Write the electronic
configuration of Fe in 56FeCl3.
1s2 2s2 2p6 3s2 3p6 3d5
(d) A student investigated the kinetics of the reaction between FeCl3, Cl2 and
benzoic acid and the following information was obtained.

Experiment No. [FeCl3] [Cl2] / mol dm-3 [benzoic acid] Rate


/ mol dm-3 / mol dm-3 / mol dm-3 min-1
1 0.100 0.075 0.250 1.39 x 10-5
2 0.100 0.100 0.250 1.85 x 10-5
3 0.150 0.075 0.300 2.50 x 10-5
4 0.150 0.100 0.250 2.78 x 10-5

(i) Determine the order of reaction with respect to FeCl3, Cl2, and
benzoic acid. Hence write the rate equation and determine
the rate constant of the reaction using information from
experiment 3.
13

Comparing experiments 1 and 2,


1
[Cl2] increase by 1 times while rate increases by
3
1
1 times
3
order w.r.t. [Cl2] is 1.
Comparing experiments 2 and 4,
[FeCl3] increase by 1.5 times while rate increases by
1.5 times
order w.r.t. [FeCl3] is 1.
Comparing experiments 3 and 4,
1
[Cl2] increase by 1 times and [benzoic acid] decrease
3
1 5
by 1.2 times, while rate increases by 1 X times
3 6
order w.r.t. [benzoic acid] is 1.
Rate = k [FeCl3] [Cl2] [benzoic acid]
k = 7.41 x 10-3 mol-1 dm3 min-1
14

(ii) Sketch a labelled energy profile diagram for this reaction.

(e) Suggest reagents and conditions for the 4-step synthesis of


3-oxo-butanoic acid from propene.
H H
H H H
H H
C C C C
H C H Br2(aq) H C Br
O
H H H

KCN(alc)
heat

H H
H H H
H H
H C C
C C
H C CN
H C COOH dilute H SO OH
O 2 4
H heat H
H

H H
H
C C
H+ / KMnO4 H C COOH
heat O
H

8 (a) Sulfur, S8, in its yellow solid state, exists in a variety of oxidation
states in its compounds. One of them is that of sulfur dioxide,
which is readily formed during a volcanic eruption or during the
15

combustion of fossil fuels. Using the relevant information from the


Data Booklet, and given the following data, calculate the bond
energy of S=O in sulfur dioxide.

S8(s) S8(g) Ho = +120 kJ mol-1


Hf(SO2(g)) = 296 kJ mol-1
1
S8 (g) + O2(g) SO2(g)
8
bonds to be broken 1 S-S and 1 O=O: 264 + 496 = 760 kJ
bonds to be formed 2 S=O = 2x kJ
enthalpy change of reaction
= sublimation of sulfur + bonds to be broken bonds to be formed
-296 = 1/8 x (+120) + 760 2x
x = +535.5 kJ mol-1 (536 3 s.f.)
(b) The key reaction during the Contact process, for manufacturing
sulfuric acid, is as follows:

A 3 dm3 vessel was used and 4 moles of sulfur dioxide and 4 moles
of oxygen were introduced into the vessel. Given that the
o
exothermic reaction, under the conditions of 450 C, 2 atm pressure
and presence of vanadium(V) oxide, ensured a 96% conversion,
calculate the Kc of the reaction.
amount of sulfur dioxide at equilibrium = 0.04 x 4 = 0.16 mol
amount of oxygen at equilibrium = 4 0.5 * (4 0.16) = 2.08 mol
amount of sulfur trioxide at equilibrium = 0.96 x 4 = 3.84 mol

[SO 3 ] 2 [3.84 ] 2
Kc = = 3 = 830 mol-1 dm3
2
[SO 2 ] [O 2 ] [ 0.16 2 2.08
] [ ]
3 3
(c) Predict and explain the changes to the yield and the Kc value if the
following changes were made:
(i) temperature was reset to 200 oC; and
temperature was lowered
system favors the exothermic reaction
heat given off to counteract the drop in temperature
equilibrium shifts to the right
both kf and kb decreases
16

but kf decrease by a lesser extent


Kc value increases
(ii) 2 moles of SO2 were added.
additional moles of reactants was added
system tries to remove the additional reactants,
favoring the formation of more products
equilibrium shifts to the right
Kc value remains unchanged
Kc dependent on temperature only
(d) A 25 cm3 portion of the equilibrium mixture in the above part was
extracted and dissolved in 1 dm3 of distilled water, forming a
mixture labelled as mixture V. Given that sulfurous acid, H2SO3, is
a weak dibasic acid, calculate the volume of 1.5 mol dm-3 sodium
hydroxide solution required to completely neutralise the mixture V
and suggest a possible indicator to be used.
H2SO3 + 2NaOH Na2SO3 + 2H2O

H2SO4 + 2NaOH Na2SO4 + 2H2O


amount of SO2 present = 25 / 3000 x 0.16 mol = 1.3333 x 10-3 mol
amount of SO2 present = 25 / 3000 x 3.84 mol = 0.032 mol amount
of NaOH needed = (0.032 + 1.3333 x 10-3) x 2 = 0.066667 mol
volume of NaOH used = 0.066667 / 1.5 = 0.0444 dm3 = 44.4 cm3
possible indicators phenolphthalein
(e) Suggest a chemical test to distinguish the compounds in each of
the following pairs, stating clearly all observations and balanced
equations for positive tests.
17

Compound W Compound X
(i) CH3

Cl

ethanolic silver nitrate / aq NaOH, heat, acidify followed by AgNO3.


Compound W will form a white precipitate while Compound X will not form a white
precipitate.

(ii) H H O

HO C CH
C C
H HH H

Iodine in alkaline solution, heat


Compound W will not form the yellow precipitate, but Compound X will form a yellow
precipitate.
1

SERANGOON JUNIOR COLLEGE


General Certificate of Education Advanced Level
Higher 1

CHEMISTRY 8872/01
JC2 Preliminary Examination 25 Aug 2010
Paper 1 MCQ 50 min
Additional Materials: OMS
Data Booklet

READ THESE INSTRUCTIONS FIRST

Write in soft pencil.


Do not use staples, paper clips, highlighters, glue or correction fluid on the OMS.

Follow the instructions given to you on how to fill in the OMS.


Write your name and index number on the OMS in the spaces provided.
Shade correctly the item index number, starting with the number 2 and the last 4 numbers
of your FIN/NRIC number.
Eg. If your NRIC is S8906660Z, shade 26660 for the item index number.

There are thirty questions on this paper. Answer all questions. For each question there are
four possible answers A, B, C and D.
Choose the one you consider correct and record your choice in soft pencil on the separate
OMS

Each correct answer will score one mark. A mark will not be deducted for a wrong answer.
Any rough working should be done in this booklet.

This document consists of 14 printed pages and 0 blank page

SRJC 2010 PRELIM/01 [Turn over


2

Answer ALL Questions

1 A sample of 0.020 mol of an oxochloride of sulfur, SO2Cl2, reacted completely with 250 cm3 of
water according to the following equation.

SO2Cl2 (l) + 2 H2O (l) H2SO4 (aq) + 2 HCl (aq)

What would be the volume of 0.1 mol dm-3 aqueous sodium hydroxide required to neutralise
10.0 cm3 of the above resulting solution?

A 16.0 cm3
B 32.0 cm3
C 48.0 cm3
D 64.0 cm3

2 Basic hydrolysis of acetonitrile, CH3CN, takes place according to the following equation:
- +
CH3CN + NaOH + H2O CH3COO Na + NH3
A sample of acetonitrile was boiled with aqueous NaOH and the NH3 gas evolved was passed
into 50 cm3 of 1.00 mol dm-3 of HCl. The excess acid required 26.00 cm3 of 1.00 mol dm-3
NaOH solution for neutralisation. What was the mass of acetonitrile used?
(Given Mr of CH3CN = 41.0)

A 0.492 g
B 0.984 g
C 1.07 g
D 1.97 g

3 In a redox experiment, 15.0 cm3 of 0.2 mol dm-3 of acidified manganate(VII) ions was used to
oxidise 25.0 cm3 of 0.6 mol dm-3 of an aqueous solution of An+.

What is the change in oxidation number for element A?

A 1
B 2
C +1
D +5

SRJC 2010 PRELIM/01 [Turn over


3

4 Beams of particles travelling at the same speed from different sources are subjected to an
electric field as shown in the diagram below. The path for a beam of neutrons has already been
drawn.

neutrons
C
B
- +
D

source

Which of the following correctly represents the particles labelled B, C and D?

B C D
4
A He+ 1
H+ electrons
4
B He+ 2 +
H 1 +
H
1 + 2 + 3 +
C H H H
1 + 2 +
D H H electrons

5 Which diagram represents the arrangement of electrons in the 3rd and 4th quantum shell of a
Co2+ ion?
__ __ __ __ __ __ __ __ __ __
A
3s 3p 3d 4s
__ __ __ __ __ __ __ __ __ __
B
3s 3p 3d 4s
__ __ __ __ __ __ __ __ __ __
C
3s 3p 3d 4s
__ __ __ __ __ __ __ __ __ __
D
3s 3p 3d 4s

6 Which one of the following statements best explains why the boiling point of butanone (79.6 oC)
is higher than that of pentane (36.1 oC)?

A The relative molecular mass of butanone is higher than that of pentane.


B The butanone molecule has a larger surface area than the pentane molecule.
C The covalent bonds in butanone molecules are stronger than those in the pentane
molecules.
D There are stronger dipole interactions between butanone molecules but only weak
interactions between pentane molecules.

SRJC 2010 PRELIM/01 [Turn over


4

7 The decomposition of hydrogen peroxide was found to be a first order reaction. Which of the
following graphs is correct?

A B
Rateof of
Rate decomposition
decomposition of Rate
Rate of of decomposition
decomposition of
of H O
N2O5 2 2 N2of
O5H2O2

[N[H
2O2
[H O
] 2]
25O2] [H
[N[H OO ] ]2]
0 0 22O252

C D

Rate of decomposition of
N2O5
[H2O2] Rate of[H 2O2]
decomposition of
N2O5

[Ntime [Ntime
time
2O5]
0 2O5] 0

8 The graph below shows how the percentage of gaseous products present at equilibrium varies
with temperature and pressure.

% products T oC
at equilibrium
(T + 10) oC

pressure

Which of the following equilibrium correctly represents the graph above?

A H2 (g) + I2 (g) 2HI (g) H = + 53 kJ mol-1


B N2O4 (g) 2 NO2 (g) H = + 57 kJ mol-1
C N2 (g) + 3 H2 (g) 2 NH3 (g) H = - 92 kJ mol-1
D 4 NH3 (g) + 5 O2 (g) 4 NO (g) + 6 H2O (g) H = - 950 kJ mol-1

SRJC 2010 PRELIM/01 [Turn over


5

9 The graph below shows the change in concentrations of the equilibrium mixture with time for
the following reaction:
F (g) + 2 G (g) H (g)

Concentration /mol dm-3

time
T1 T2

Which of the following shows the correct conditions applied to cause the change at time T1 and
T2?

T1 T2
A decrease in volume increase in [H]
B decrease in volume decrease in [H]
C increase in volume increase in [H]
D increase in volume decrease in [H]

SRJC 2010 PRELIM/01 [Turn over


6

10 The following graph shows the pH changes when 0.15 mol dm-3 of Ba(OH)2 is added to 20 cm3
of 0.15 mol dm-3 of ethanoic acid.

pH
L

13

J
3

Volume of Ba(OH)2 / cm3


Which of the following statements is true about this titration?

A The equivalence volume for this titration is 20 cm3.


B A buffer region is observed at point J.
C A good indicator that can be used in this titration is universal indicator.
D pH at point K is smaller than 7.

11
H H
C C
H
O
C
H
Compound M
Compound M was reacted with hydrogen gas in the presence of palladium catalyst. What is the
total number of sigma and pi bonds found in the product formed?

No. of sigma bonds No. of pi bonds


A 21 3
B 25 2
C 26 1
D 28 0

12 3-methylpentane was reacted with chlorine gas in the presence of ultraviolet light. What is the
total number of possible isomers formed, assuming mono-substitution took place?

A 4
B 5
C 6
D 14

SRJC 2010 PRELIM/01 [Turn over


7

13 Which of the following shows the correct products formed when buta-1,3-diene undergoes
Reaction 1 and Reaction 2 separately?

Reaction 1: With excess cold alkaline KMnO4 followed by heating.


Reaction 2: With hot acidified KMnO4.

Reaction 1 Reaction 2

OH H OH H
A H C C C C H CO2 and H2O
H OH H OH

OH OH
B CO2 and H2O
O C C O

C CO2 and H2O CO2 and H2O

O O O
OH OH
D C C C C
HO OH O C C O
O

14 Which of the following is the major product formed when CH3CH=C(NO2)CH2CH=C(CH3)2


reacts with HBr?

CH3 NO2 H CH3


A H C C CH2 C C CH3
H Br Br H
CH3 NO2 H CH3
B H C C CH2 C C CH3
Br H H Br
CH3 NO2 H CH3
C H C C CH2 C C CH3
H Br H Br
CH3 NO2 H CH3
D H C C CH2 C C CH3
Br H Br H

SRJC 2010 PRELIM/01 [Turn over


8

15 Which one of the following will be formed when butane-1,3-diol is distilled with hot acidified
potassium dichromate(VI)?
A CH3COCH2CHO
B CH3COCH2COOH
C CO2 and H2O
D CH3CH(OH)CH2COOH

16 1 mol of organic compound N undergoes elimination with ethanolic sodium hydroxide to form
2 mol of HBr.

What could compound N be?

A B

Br CH3
CH2CHCH3 Br C CH2CH2Br
Br CH3

C D

Br CH3
CH2Br BrCH2 C CH2Br
CH3

SRJC 2010 PRELIM/01 [Turn over


9

17 Experiments are carried out on three compounds R, S and T.

CH3(CH2)16CO2CH2 CN
BrCH2CH2CH2Br
CH3(CH2)16CO2CH
CH3(CH2)16CO2CH2
R S T

A sample of 0.01 mol of each compound is heated under reflux with 50 cm3 of 1.0 mol dm-3
aqueous NaOH (in excess) until hydrolysis is completed and any ammonia produced is
expelled from solution. The excess NaOH is found to require 20 cm3, 30 cm3 and 40 cm3 of
1.0 mol dm-3 HCl for neutralization.

Which sequence of compounds matches the above results?

20 cm3 of HCl 30 cm3 of HCl 40 cm3 of HCl


required required required

A R S T
B R T S
C S R T
D T S R

18 Which reagent gives the same visible observation with propanoic acid and propanal?

A Dilute aqueous sodium carbonate


B Alkaline aqueous iodine
C Sodium metal
D Tollens reagent

SRJC 2010 PRELIM/01 [Turn over


10

19 Terephthalaldehyde is used as an intermediate for manufacturing of dyes and fluorescent


whitening agents. It can be synthesized from 4-methylbenzoic acid via a 3-step synthesis.

COOH CHO
3 steps

CH3 CHO

4-methylbenzoic acid Terephthalaldehyde

Which of the following reagents and conditions are needed for the conversion?

Step1 Step2 Step3


A Cr2O72- /H+, heat to distill LiAlH4, r.t.p Hot KMnO4 / H+
B Cr2O72- /H+, heat to distill NaBH4, r.t.p Hot KMnO4 / H+
C Hot KMnO4 / H+ LiAlH4, r.t.p Cr2O72- / H+, heat to distill
D Hot KMnO4 / H+ NaBH4, r.t.p Cr2O72- / H+, heat to distill

20 A compound V gives compound W, C8H8O, on oxidation under appropriate conditions. W gives


a precipitate on warming with Fehlings reagent.

Which of the following could be V?

A CH3
C

O
B CH2CHO

C CH3

HOCH2
D CH2CH2OH

SRJC 2010 PRELIM/01 [Turn over


11

21 The structure of an organic compound X is shown below:

O COOH

Compound X

Which one of the following statements is true?

A The six-membered carbon ring in X is planar.


B X reacts with cold HCN under basic conditions.
C X does not give an orange precipitate with 2,4-dinitrophenylhydrazine.
D Reaction of sodium carbonate with X produces a gas which turns aqueous red litmus blue.

22 Which of the following statement is true when the compound below reacts with hot aqueous
NaOH?

Cl CH2OCOCH3

COOCH3

A White fumes are observed.


B A white precipitate is observed.
C Neutralisation has taken place.
D The isolated final product has a low melting point.

23 A food chemist wants to create the odour of green apples for a product. An ester with this
odour has the formula C2H5CO2CH(CH3)2. In which of the following will the substances react
together to produce this ester?

A C2H5OH and (CH3)2CHCOOH


B CH3COOH and CH3CH(OH)CH2CH3
C C2H5COOH and C2H5CH2OH
D C2H5COOH and (CH3)2CHOH

24 An unknown element forms a chloride which is able to give an acidic solution and an oxide
which can dissolve readily in an acidic solution.

What could this element be?

A Calcium
B Aluminium
C Silicon
D Phosphorus

SRJC 2010 PRELIM/01 [Turn over


12

25 Use of the Data Booklet is relevant to this question.


The graph below shows the second ionisation energies of elements 1 to 4.
The elements are adjacent to each other in the Periodic Table.

Second
ionisation energy
(arbitrary units)
2

4
3

Atomic number

Which statement about the elements 1 to 4 is correct?

A The melting point of the elements increases in the order of 1 > 2 > 3 > 4
B The atomic radius of the elements decreases in the order of 2 < 3 < 4 < 1
C The electrical conductivity of the elements increases in the order 1 < 2 < 3 < 4
D The third ionisation energy of the elements decreases in the order of 3 < 2 < 1 < 4

For questions 26 30, one or more of the numbered statements 1 to 3 may be correct. Decide
whether each of the statements is or is not correct. The responses A to D should be selected on the
basis of

A B C D
1, 2 and 3 1 and 2 only 2 and 3 only 1 only
are correct are correct are correct is correct

No other combination of statements is to be used.

26 Which of the following contain(s) hydrogen bonds?

1 NH4Cl (s)
2 NH3 (l)
3 HNO3 (l)

SRJC 2010 PRELIM/01 [Turn over


13

A B C D
1, 2 and 3 1 and 2 only 2 and 3 only 1 only
are correct are correct are correct is correct

27 The kinetics of the reaction between compounds Y and Z were shown in the graph below:

[Y][I2]

2y

[ [Z]
J]=22ax
y

[Z]J =] =x 2x
[[H] a

time / min
5 10

Which of the following conclusion(s) can be drawn from the graph?


1 The reaction is first order with respect to compound Y.
2 The reaction is second order with respect to compound Z.
3 The overall order of reaction is 2.

28 In an acid-base titration, 25.0 cm3 of 0.150 mol dm-3 HCl was titrated with 0.150 mol dm-3 NH3.
The table below shows the data of four acid-base indicators.

Indicator Approximate pH range

Congo red 3.0 5.0


Bromocresol green 3.8 5.5
Cresol red 7.2 8.8

Which of the following correctly represent(s) the appropriate indicator(s) used in the above
titration?

1 Congo red
2 Bromocresol green
3 Cresol red

SRJC 2010 PRELIM/01 [Turn over


14

A B C D
1, 2 and 3 1 and 2 only 2 and 3 only 1 only
are correct are correct are correct is correct

29 The structure of compound B is shown below:


Br
Br
OH

O
Compound B

What types of reaction will compound B undergo?

1 Electrophilic substitution and condensation


2 Nucleophilic substitution and elimination
3 Neutralisation and nucleophilic addition

30 Compound C can be formed from ethanal in three steps:


O

CH3CHO 3 steps CH3


O
Ethanal
O
CH3
O
Compound C

Which of the following reagent(s) and condition(s) is / are required?

1 Dilute H2SO4, reflux


2 Excess concentrated H2SO4, reflux
3 HCN in aqueous NaOH, cold

-END OF PAPER-

SRJC 2010 PRELIM/01 [Turn over


1
SERANGOON JUNIOR COLLEGE
General Certificate of Education Advanced Level
Higher 1

CANDIDATE
NAME

CLASS INDEX NUMBER

CHEMISTRY 8872/02
Preliminary Examination 23 Aug 2010
Paper 2 2hr
Additional Materials: Data Booklet
Writing Papers

READ THESE INSTRUCTIONS FIRST

Write your name and class on all the work you hand in.
Write in dark blue or black pen on both sides of the paper.
You may use a soft pencil for any diagrams, graphs or rough work.

SECTION A:
Answer all questions in the space provided.

SECTION B:
Answer any two questions on separate answer paper.

At the end of the examination, fasten all your work securely together.
The number of marks is given in brackets [ ] at the end of each question or part question.

ForExaminersUse
MCQP1 /30

A1 /10

A2 /10

A3 /10

A4 /10

SectionB1 /20

SectionB2 /20

SectionB3 /20

Total /110

This document consist of 16 printed pages and 0 blank page

SRJC 2010 [Turn Over


2

Section A
Answer all the questions in this section in the spaces provided

1 The simplest hydride of boron is boron hydride of molecular formula B2H6. It has been suggested
that gaseous boron hydride can be used as a rocket propellant, using either oxygen or fluorine as
oxidants.

Combustion of boron hydride with oxygen yields a high melting point solid boron (III) oxide, B2O3
and water as the only products.

(a) (i) Write the balanced chemical equation for the reaction between boron hydride and
oxygen.

(ii) Using the information given below, calculate the standard enthalpy change for the
above reaction.

Substance Standard enthalpy change of formation /


kJ mol-1
B2O3 -1273
B2H6 +36
H2O -286

(iii)
2B3+ (g) + 3O (g) + 6e

2x H6
2B2+ (g) + 3O (g) + 4e 3 x EA of oxygen

2 x H5
2B+ (g) + 3O (g) + 2e 2B3+ (g) + 3O2- (g)

2 x H4
2B (g) + 3O (g)

3 x H3 Hlatt
2B (g) + 3/2 O2 (g)

2 x H2
2B (s) + 3/2 O2 (g)

H1
B2O3 (s)

SRJC 2010 [Turn Over


3

Using data from the Data Booklet and information below, identify the
standard enthalpy terms H1, H2, H3 and H4 on the left hand side of
the Born Haber Cycle.

H1 is

H2 is ...

H3 is ..

H4 is ..

Hence determine the lattice energy of boron (III) oxide.

Enthalpy term H / kJ mol-1


Standard enthalpy change of -1273
formation of boron (III) oxide
Standard enthalpy change of +573
atomisation of boron
Standard enthalpy change of +248
atomisation of oxygen
Sum of first and second electron +702
affinity for oxygen

[5]

SRJC 2010 [Turn Over


4
(b) Paracetamol is a common analgesic and pyretic drug often used as a painkiller and for
treating mild fever. In a study carried out to determine the rate of removal of
paracetamol from the body, the reaction was found to have a constant half-life of 2.7
hours.
H
N

O
HO
Paracetamol

(i) Sketch a graph to show how 1 mol dm-3 of paracetamol decreases to 25% of its initial
concentration.

(ii) Write the rate equation for the reaction and calculate the value of the rate constant, k.

..

(iii) Explain why Paracetamol is not very soluble in water. [5]

.
.

[Total: 10 marks]

SRJC 2010 [Turn Over


5

2 Alcohol B has the following structure:

OH
CH3 C CH3
C3H 7

(a) Draw the displayed formula of the possible isomers of alcohol B. Label your
isomers for use in (b). [2]

(b) To the corresponding isomers of alcohol B in (a), concentrated sulfuric acid


was added and products with molecular formula, C6H12 were obtained. Draw
the structural formula of the products formed. [2]

SRJC 2010 [Turn Over


6

(c) The boiling point of three organic compounds is given as follow:

Compound Boiling Point

Butene 273 K

Butan-1-ol 390 K

Butanoic acid 437 K

Briefly explain the difference in their boiling point. [2]

..................................

(d) Organic compound C contains carbon, hydrogen and oxygen only.

10 cm3 of compound C undergoes complete combustion to produce 40 cm3 of


carbon dioxide and 50 cm3 of water vapour. 1 mol of C produces 0.5 mol of
hydrogen gas when sodium metal is added.

No effervescence was observed when compound C was reacted with sodium


carbonate but white fumes of gas were seen when phosphorous
pentachloride was added to compound C.

Upon reaction with acidified potassium manganate (VII), compound D was


formed. Compound D reacts with 2,4-DNPH but did not react with Tollens
reagent. Compound C and D both give yellow precipitate when reacted with
alkaline aqueous iodine.

Compound C was passed through hot alumina and compound E was obtained
as one of the products. When compound E undergoes strong oxidation,
bubbles of gas were observed.

Draw the structures of Compound C, D and E. [4]

[Total: 10 marks]

SRJC 2010 [Turn Over


7
3 Shikimic acid, is an important biochemical intermediate in plants and
microorganisms.

COOH

HO OH
OH

Shikimic acid

(a) Draw the structural formulae of the organic products formed when shikimic acid is
treated with HBr followed by hot aqueous NaOH. [2]

(b) Apart from shikimic acids, cinnamic acid, zingerone and vanillin are other
compounds that can be naturally isolated from plants and herbs.

CH3
COOH
HO
OCH3

Cinnamic acid Zingerone

CHO

HO
OCH3
Vanillin

(for this question ignore the chemical effect of OCH3)

SRJC 2010 [Turn Over


8
(i) State one reagent and condition that will react with vanillin and not zingerone and
describe what would be observed.

Reagent and condition: ...

Observation: ..............................

(ii) State one reagent and condition, other than Br2 and oxidising agent, that will react
with cinnamic acid and not zingerone and describe what would be observed. [4]

Reagent and condition: ...

Observation: ...........................

(c) The carbonyl functional group undergoes a series of reactions which follow the
following pattern:

catalyst X
C O + HX C
OH

where X can be CN, OCH3, etc.

Using the above information, draw the structural formula of the product formed for
the following reactions:

C CHO
(i) O OH
Addition of HCN to , followed by a dehydration process.

(ii) Addition of CH3OH to vanillin, followed by an oxidation reaction. [2]

(Note: OCH3 is inert to oxidation)

SRJC 2010 [Turn Over


9
(d) Shikimic acid undergoes complete combustion to form carbon dioxide and water.
Construct a reaction pathway diagram for the combustion process. In your
diagram, clearly indicate the enthalpy change of combustion and activation
energy. [2]

[Total: 10 marks]

SRJC 2010 [Turn Over


10
4 (a) (i) On the grid below, sketch the trend of the ionic radii of elements in Period 3 from
sodium to chlorine.

ionic
radii / nm

Na Mg Al Si P S Cl Element

(ii) Explain the trend of the ionic radii for elements in Period 3. [3]

........................................................................................................................

........................................................................................................................

........................................................................................................................

.......................................................................................................................

.......................................................................................................................

SRJC 2010 [Turn Over


11
(b) The first six ionisation energies of three successive elements in a Periodic Table are
given below:

Ionisation Energy in kJ mol-1

Element 1st 2nd 3rd 4th 5th 6th

F 786 1580 3230 4360 16100 19800

G 1060 1900 2920 4960 6270 21300

J 1000 2260 3390 4540 7000 8500

(i) State and explain the Group which element G belongs to.

Group: ..................................

Explanation: ..................................................................................................................

.......................................................................................................................................

......................................................................................................................................

(ii) Explain why the first ionisation energy of G is

higher than F:

.......................................................................................................................................

.......................................................................................................................................

and higher than J:

.......................................................................................................................................

.......................................................................................................................................

(iii) Suggest a chemical formula of an oxide of element J and predict the type of bonding
present in the oxide of J.

Formula of oxide of J: ..............................................

Type of bonding present in oxide of J: ......................................

(iv) With aid of an equation, predict a pH value for the resulting solution obtained when
the oxide of J is dissolved in water.

Equation: ....................................................................................

pH = ..................................... [7]

[Total: 10 marks]

SRJC 2010 [Turn Over


12

Section B
Answer two questions from this section on a separate piece of paper.

5 (a) The kinetics of the acidcatalysed reaction of propanone with iodine can be investigated
experimentally by varying the concentrations of the three substances involved.

CH3COCH3(aq) + I2(aq) CH2ICOCH3(aq) + HI(aq)



The following results were obtained in an experiment.

[CH3COCH3] = 0.10 mol dm3


[I2] / mol dm3

[H+] = 0.10 mol dm3

[H+] = 0.20 mol dm3

time / min

[H+] = 0.10 mol dm3


[I2] / mol dm3

[CH3COCH3] = 0.10 mol dm3

[CH3COCH3] = 0.15 mol dm3

time / min

time / min

(i) Deduce, with reasoning, the order of reaction with respect to (1) I2, (2) H+ and (3)
CH3COCH3.

(ii) Hence, write a rate equation for the reaction and determine a value for the rate
constant k, including its units.

(iii) Predict, with reasoning, the effect of a catalyst on the activation energy and rate
constant of a reaction. [10]

SRJC 2010 [Turn Over


13

(b) Propanoic acid and citric acid are some of the acids used in the formulation of hair
shampoo. A chemistry student wanted to formulate a shampoo with maximum buffer
capacity so that there will be minimal damage to the hair.

In an attempt to prepare a buffer for the shampoo, she titrated 20.0 cm3 of 0.100 mol dm-3
sodium hydroxide against 0.20 mol dm-3 citric acid.

(i) Deduce, with reasoning, whether propanoic acid or 2-chloropropanoic acid is a


stronger acid.

(ii) Calculate the pH of 0.100 mol dm-3 sodium hydroxide.

(iii) Calculate the volume of citric acid required to completely neutralise 20.0 cm3 of
sodium hydroxide.

(iv) When excess citric acid is added, a buffer solution is obtained.


Explain, with the aid of equations, how the mixture can act as a buffer upon addition
of a small amount of acid and alkali.
(Citric acid could be represented by C5H7O4COOH)

(v) The pH of 1 mol dm-3 of propanoic acid is 4 while the pH of 1 mol dm-3 of hydrochloric
acid is 1. Explain the difference in their pH.

(vi) Suggest with reasoning, if the same volume of hydrogen gas will be produced if
propanoic acid and hydrochloric acid, of the same concentration and volume are to
react with 2.0 g of zinc metal under the same condition. [10]

[Total: 20 marks]

SRJC 2010 [Turn Over


14

66 (a) Compound K has a composition of 46.15% C, 7.69% H and 46.15% O. It is formed when
compound L (a bromo-containing organic substance) is refluxed with aqueous sodium
hydroxide followed by acidification.

Effervescence was observed when sodium carbonate was added to K. Upon treatment of
K with acidified potassium dichromate (VI), compound M is produced. Compound M reacts
with 2,4-dinitrophenylhydrazine to form an orange hydrazone, N. On reaction of M with
Fehlings reagent, the reddish brown deposit was however not observed.

A yellow precipitate was observed when K and M were warmed with alkaline aqueous
iodine. When K is refluxed with concentrated sulfuric acid, a sweet smelling compound P,
C8H12O4 is formed.

Deduce the structures of K, L, M, N and P. Maximum credit cannot be awarded just by


drawing of the structures only. [10]

(b) Suggest a simple chemical test to distinguish between the following compounds:
O
O
O
O

and [3]

(c) Antimony pentachloride (SbCl5) decomposes in a gas phase reaction at high temperature.

SbCl5 (g) SbCl3 (g) + Cl2 (g) H = negative

At some temperature, dynamic equilibrium is established. An equilibrium mixture in a


5 dm-3 container is found to contain 6.91 g of SbCl5, 16.45 g of SbCl3 and 5.11 g of Cl2.

(i) What is meant by the term dynamic equilibrium?

(ii) Determine the Kc expression and hence calculate Kc for the above equilibrium.

(iii) Using Le Chateliers Principle, state and explain the effect on the position of
equilibrium and the equilibrium composition when:

1) temperature is decreased at constant pressure,


2) pressure is decreased at constant temperature. [7]

[Total: 20 marks]

SRJC 2010 [Turn Over


15
7 (a) The oxides of sodium to phosphorus (Na2O, MgO, Al2O3, SiO2 and P4O10) differ in their
bonding and reactions with water.

(i) Copy and complete the following graph on your answer script.
pH

Na2O MgO Al2O3 SiO2 P4O10


You are to indicate clearly on your graph the pH of the resulting solution when
each oxide reacts with water.

(ii) With the aid of equations, state the pH of the solution obtained when aluminium
chloride is dissolved in water.

(iii) A student was instructed to perform tests to deduce the identities of two chemical
bottles with missing labels. The bottles contain either gallium oxide or selenium
oxide.

The student first labelled the two bottles as Q and R. Then, he conducted solubility
tests and record his observations as follows:

Sample Q dissolves in both hydrochloric acid and aqueous sodium hydroxide


while sample R dissolves in aqueous sodium hydroxide only.
Suggest the identities of Q and R and write equations to show the acid-base
behaviour of the oxides. [8]
[Gallium is 31Ga; Selenium is 34Se]

(b) Ethanol is a chemical which is widely used as a solvent, fuel and an important
intermediate in large scale synthesis of organic compounds.

When 2.00 g of ethanol was burned under a container of water, the heat released was
found to increase the temperature of 150 g of water from 28oC to 65oC.

(i) Write a balanced equation, with state symbols, to represent the standard enthalpy
change of combustion of ethanol.
(ii) Using the data given above, calculate the enthalpy change of combustion of
ethanol.
(iii) Using bond energy values from the Data Booklet, calculate another value for the
standard enthalpy change of combustion of ethanol. Suggest an explanation for
the discrepancy between your answer and the calculated value in b(ii). [6]

SRJC 2010 [Turn Over


16

(c) Glycolic acid is the smallest -Hydroxy acids (AHAs). It is also one of the main
ingredients used in the manufacture of skin care products due to its excellent capability
to penetrate human skin.

O
HO
OH

Glycolic Acid

(i) Show how glycolic acid can be synthesized from methanol via three steps. In your
answer, show clearly the reagents, conditions and intermediates involved.

(ii) State the product(s) formed when hot acidified potassium manganate (VII) is added
to glycolic acid.

(iii) Explain, with the aid of a diagram, why the relative molecular mass of glycolic acid is
152 when dissolved in a suitable solvent. [6]

[Total: 20 marks]

END OF PAPER

SRJC 2010 [Turn Over


2

2010 JC2 H1 Prelim Paper 1 Suggested Solution

1 A sample of 0.020 mol of an oxochloride of sulfur, SO2Cl2, reacted completely with 250 cm3 of
water according to the following equation.

SO2Cl2 (l) + 2 H2O (l) H2SO4 (aq) + 2 HCl (aq)

What would be the volume of 0.1 mol dm-3 aqueous sodium hydroxide required to neutralise
10.0 cm3 of the above resulting solution?

A 16.0 cm3
B 32.0 cm3
C 48.0 cm3
D 64.0 cm3

Solution: B
2 NaOH + H2SO4 Na2SO4 + 2 H2O
10
Amt of H2SO4 in 10.0 cm3 of solution= 0.020 = 0.0008mol
250
Amt of NaOH required= 0.0008 2 = 0.0016mol

NaOH + HCl NaCl + H2O


10
Amt of HCl in 10.0 cm3 of solution= 0.020 2 = 0.0016mol
250
Amt of NaOH required = 0.0016mol
0.0032
Total Volume of NaOH required = = 0.032dm 3
0.1

2 Basic hydrolysis of acetonitrile, CH3CN, takes place according to the following equation:
- +
CH3CN + NaOH + H2O CH3COO Na + NH3
A sample of acetonitrile was boiled with aqueous NaOH and the NH3 gas evolved was passed
into 50 cm3 of 1.00 mol dm-3 of HCl. The excess acid required 26.00 cm3 of 1.00 mol dm-3
NaOH solution for neutralisation. What was the mass of acetonitrile used?
(Given Mr of CH3CN = 41.0)

A 0.492 g
B 0.984 g
C 1.07 g
D 1.97 g
3

Solution: B
Eqn 1: NH3 + HCl NH4Cl
Eqn 2: NaOH + HCl NaCl + H2O

From eqn 2:
26.00
Amt of NaOH required = 1 = 0.026mol
1000
Amt of HCl in excess = 0.026mol
50.00
Initial amt of HCl = 1 = 0.05mol
1000
Amt of HCl reacted = 0.05 0.026 = 0.024mol

From Eqn 1:
Amt of HCl reacted = Amt of NH3 = Amt of CH3CN = 0.024mol

Mass of CH3CN= 0.024 x 41 = 0.984 g

3 In a redox experiment, 15.0 cm3 of 0.2 mol dm-3 of acidified manganate(VII) ions was used to
oxidise 25.0 cm3 of 0.6 mol dm-3 of an aqueous solution of An+.

What is the change in oxidation number for element A?

A 1
B 2
C +1
D +5

Solution: C
[R]: MnO4- + 8H+ + 5e Mn2+ + 4H2O ---- (1)
[O]: An+ A(n+x) + xe -------(2)

15
Amt of MnO4- = 0.2 = 0.003 mol
1000
25
Amt of oxide of A = 0.6 = 0.015 mol
1000

MnO4- 5 An+

Hence,
x 5
=
5 1
x=1

Since A lost one electron, change in oxidation state = +1


4

4 Beams of particles travelling at the same speed from different sources are subjected to an
electric field as shown in the diagram below. The path for a beam of neutrons has already been
drawn.

neutrons
C
B
- +
D

source

Which of the following correctly represents the particles labelled B, C and D?

B C D
4
A He+ 1
H+ electrons
4
B He+ 2 +
H 1 +
H
1 + 2 + 3 +
C H H H
1 + 2 +
D H H electrons

Solution: D
Positively charged 2H+ and 1H+ will be deflected towards the negative terminal and electron (negatively
charged) will be deflected towards the positive terminal.

Since angle of deflection is proportional to (e /m) values,


e/m (1H+) = 1/1 = 1
e/m (2H+) = 1/2 = 0.5

1
H+ is lighter hence angle of deflection would be 2 times greater than 2H+.

5 Which diagram represents the arrangement of electrons in the 3rd and 4th quantum shell of a
Co2+ ion?
__ __ __ __ __ __ __ __ __ __
A 3s 3p 3d 4s
__ __ __ __ __ __ __ __ __ __
B 3s 3p 3d 4s
__ __ __ __ __ __ __ __ __ __
C 3s 3p 3d 4s
__ __ __ __ __ __ __ __ __ __
D 3s 3p 3d 4s
5

Solution: C

Co: 1s2 2s2 2p6 3s2 3p6 3d7 4s2


Co2+: 1s2 2s2 2p6 3s2 3p6 3d7

6 Which one of the following statements best explains why the boiling point of butanone (79.6 oC)
is higher than that of pentane (36.1 oC)?

A The relative molecular mass of butanone is higher than that of pentane.


B The butanone molecule has a larger surface area than the pentane molecule.
C The covalent bonds in butanone molecules are stronger than those in the pentane
molecules.
D There are stronger dipole interactions between butanone molecules but only weak
interactions between pentane molecules.
Solution: D
The Mr of butanone and pentane are both 72. Permanent dipole-dipole interaction exist between the
butanone due to the presence of polar C=O bond. Pentane only has weak temporary induced dipole
interactions.

7 The decomposition of hydrogen peroxide was found to be a first order reaction. Which of the
following graphs is correct?

A B
Rateof of
Rate decomposition
decomposition of Rate
Rate of of decomposition
decomposition of
Nof
O
2 5
H 2O 2 N2of
O5H2O2

[N[H
2O2
[H O
] 2]
25O2] [H
[N[H OO ] ]2]
0 0 22O252

C D

Rate of decomposition of
N2O5
[H2O2] Rate of[H 2O2]
decomposition of
N2O5

[Ntime [Ntime
time
2O5]
0 2O5] 0

Solution: A
6

8 The graph below shows how the percentage of gaseous products present at equilibrium varies
with temperature and pressure.

% products T oC
at equilibrium
(T + 10) oC

pressure

Which of the following equilibrium correctly represents the graph above?

A H2 (g) + I2 (g) 2HI (g) H = + 53 kJ mol-1


B N2O4 (g) 2 NO2 (g) H = + 57 kJ mol-1
C N2 (g) + 3 H2 (g) 2 NH3 (g) H = - 92 kJ mol-1
D 4 NH3 (g) + 5 O2 (g) 4 NO (g) + 6 H2O (g) H = - 950 kJ mol-1

Solution: C
When pressure is increased at constant temperature,
By Le Chateliers Principle, the position of equilibrium shifts to the right to decrease total amount of
gases.
When temperature is increased at constant pressure,
By Le Chateliers Principle, the position of equilibrium shifts to the left to absorb heat, favouring the
backward endothermic reaction.
7

9 The graph below shows the change in concentrations of the equilibrium mixture with time for
the following reaction:
F (g) + 2 G (g) H (g)

Concentration /mol dm-3

time
T1 T2

Which of the following shows the correct conditions applied to cause the change at time T1 and
T2?

T1 T2
A decrease in volume increase in [H]
B decrease in volume decrease in [H]
C increase in volume increase in [H]
D increase in volume decrease in [H]

Solution: A
At T1, [F], [G] and [H] increased at the same time and it is due to the decrease in volume which
increases the pressure of the system.

At T2, [F] and [G] increased while [H] decreased implied that the backward reaction has been
favoured and hence there must have been an increase in [H].
8

10 The following graph shows the pH changes when 0.15 mol dm-3 of Ba(OH)2 is added to 20 cm3
of 0.15 mol dm-3 of ethanoic acid.

pH
L

13

J
3

Volume of Ba(OH)2 / cm3


Which of the following statements is true about this titration?

A The equivalence volume for this titration is 20 cm3.


B A buffer region is observed at point J.
C A good indicator that can be used in this titration is universal indicator.
D pH at point K is smaller than 7.
Solution: B
Acidic buffer is found before the equivalence point in a strong base weak acid titration as there is
excess acid and salt present.
11
H H
C C
H
O
C
H
Compound M
Compound M was reacted with hydrogen gas in the presence of palladium catalyst. What is the
total number of sigma and pi bonds found in the product formed?

No. of sigma bonds No. of pi bonds


A 21 3
B 25 2
C 26 1
D 28 0

Solution: D
Both C=C and aldehyde group in compound M will be reduced.
H H H H
H
H C C
H
H H HH
H O C H
H H
The product is
H H H which consist of 28 sigma and 0 pi bond.
9

12 3-methylpentane was reacted with chlorine gas in the presence of ultraviolet light. What is the
total number of possible isomers formed, assuming mono-substitution took place?

A 4
B 5
C 6
D 14

Solution: A

CH3
CH3 CH2 C CH2 CH3
H 3-methylpentane
4 possible products are formed:
1-chloro-3-methylpentane; 2-chloro-3-methylpentane; 3-chloro-3-methylpentane;
3(chloromethyl)pentane
10

13 Which of the following shows the correct products formed when buta-1,3-diene undergoes
Reaction 1 and Reaction 2 separately?

Reaction 1: With excess cold alkaline KMnO4 followed by heating.


Reaction 2: With hot acidified KMnO4.

Reaction 1 Reaction 2

OH H OH H
A H C C C C H CO2 and H2O
H OH H OH

OH OH
B CO2 and H2O
O C C O

C CO2 and H2O CO2 and H2O

O O O
OH OH
D C C C C
HO OH O C C O
O

Solution: C
H H H
H
C C C C
H H
Mild [O]
Cold alkaline MnO4- strong [O]
H H H OH
H OH OH
H C C C C
H C C
OH O
OH OH O
+ CO2 + H2O
heat
strong [O]
O

HO C C C C OH
O O
O CO2 + H2O
11

14 Which of the following is the major product formed when CH3CH=C(NO2)CH2CH=C(CH3)2


reacts with HBr?

CH3 NO2 H CH3


A H C C CH2 C C CH3
H Br Br H
CH3 NO2 H CH3
B H C C CH2 C C CH3
Br H H Br
CH3 NO2 H CH3
C H C C CH2 C C CH3
H Br H Br
CH3 NO2 H CH3
D H C C CH2 C C CH3
Br H Br H

Solution: C

15 Which one of the following will be formed when butane-1,3-diol is distilled with hot acidified
potassium dichromate(VI)?
A CH3COCH2CHO
B CH3COCH2COOH
C CO2 and H2O
D CH3CH(OH)CH2COOH

Solution: A

2o alcohol will be oxidised to a ketone while 1o alcohol will be oxidised to an aldehyde.


12

16 1 mol of organic compound N undergoes elimination with ethanolic sodium hydroxide to form
2 mol of HBr.

What could compound N be?

A B

Br CH3
CH2CHCH3 Br C CH2CH2Br
Br CH3

C D

Br CH3
CH2Br BrCH2 C CH2Br
CH3

Solution: B
Halogenoarene does not undergo elimination.
The 2 Br in option D do not undergo elimination because there is a lack of H on the neighbouring
carbon.

17 Experiments are carried out on three compounds R, S and T.

CH3(CH2)16CO2CH2 CN
BrCH2CH2CH2Br
CH3(CH2)16CO2CH
CH3(CH2)16CO2CH2
R S T

A sample of 0.01 mol of each compound is heated under reflux with 50 cm3 of 1.0 mol dm-3
aqueous NaOH (in excess) until hydrolysis is completed and any ammonia produced is
expelled from solution. The excess NaOH is found to require 20 cm3, 30 cm3 and 40 cm3 of
1.0 mol dm-3 HCl for neutralization.

Which sequence of compounds matches the above results?

20 cm3 of HCl 30 cm3 of HCl 40 cm3 of HCl


required required required

A R S T
B R T S
C S R T
D T S R
13

Solution: C

Volume of HCl used = Volume of excess OH-


Volume of OH- reacted with R: S: T = 20:30:10
Volume of excess OH- that reacted with HCl (R, S, T) = (50-20): (50-30) : (50-10) = 30: 20 : 40

18 Which reagent gives the same visible observation with propanoic acid and propanal?

A Dilute aqueous sodium carbonate


B Alkaline aqueous iodine
C Sodium metal
D Tollens reagent

Solution: B
Both do not undergo oxidation with alkaline aqueous iodine, hence brown iodine remains in both
cases.

19 Terephthalaldehyde is used as an intermediate for manufacturing of dyes and fluorescent


whitening agents. It can be synthesized from 4-methylbenzoic acid via a 3-step synthesis.

COOH CHO
3 steps

CH3 CHO

4-methylbenzoic acid Terephthalaldehyde

Which of the following reagents and conditions are needed for the conversion?

Step1 Step2 Step3


A Cr2O72- /H+, heat to distill LiAlH4, r.t.p Hot KMnO4 / H+
B Cr2O72- /H+, heat to distill NaBH4, r.t.p Hot KMnO4 / H+
C Hot KMnO4 / H+ LiAlH4, r.t.p Cr2O72- / H+, heat to distill
D Hot KMnO4 / H+ NaBH4, r.t.p Cr2O72- / H+, heat to distill

Solution: C

Methyl side chain can only be oxidised using KMnO4 (a stronger oxidising agent than Cr2O72-) while
acids can only be reduced using LiAlH4 (a stronger reducing agent than NaBH4).
14

20 A compound V gives compound W, C8H8O, on oxidation under appropriate conditions. W gives


a precipitate on warming with Fehlings reagent.

Which of the following could be V?

A CH3
C

O
B CH2CHO

C CH3

HOCH2
D CH2CH2OH

Solution: D
A cannot be oxidized further.
B doesnt form C8H8O upon oxidation.
CH3 CH3

C undergoes oxidation to form OHC or HOOC which does not gives ppt
with Fehlings.
CH2CHO

D undergoes oxidation to form which is an aliphatic aldehyde (gives ppt with Fehlings).

21 The structure of an organic compound X is shown below:

O COOH

Compound X

Which one of the following statements is true?

A The six-membered carbon ring in X is planar.


B X reacts with cold HCN under basic conditions.
C X does not give an orange precipitate with 2,4-dinitrophenylhydrazine.
D Reaction of sodium carbonate with X produces a gas which turns aqueous red litmus blue.
15

Solution: B
The six-membered ring is planar about the carbonyl carbon and tetrahedral about the rest of the
carbon atoms.
X gives an orange ppt with 2,4-DNPH as it contains carbonyl functional group.
X (contains acid functional group) reacts with Na2CO3 to produce CO2 which is an acidic gas. It
will turn aqueous blue litmus red.

22 Which of the following statement is true when the compound below reacts with hot aqueous
NaOH?

Cl CH2OCOCH3

COOCH3

A White fumes are observed.


B A white precipitate is observed.
C Neutralisation has taken place.
D The isolated final product has a low melting point.

Solution: C
Final product:

Cl CH2OH

COO-Na+

Product is an ionic salt which has high melting point.


Basic hydrolysis of esters followed by neutralisation of the carboxylic acid group has occurred.
There are no white fumes and ppt formed.

23 A food chemist wants to create the odour of green apples for a product. An ester with this
odour has the formula C2H5CO2CH(CH3)2. In which of the following will the substances react
together to produce this ester?

A C2H5OH and (CH3)2CHCOOH


B CH3COOH and CH3CH(OH)CH2CH3
C C2H5COOH and C2H5CH2OH
D C2H5COOH and (CH3)2CHOH

Solution: D
16

24 An unknown element forms a chloride which is able to give an acidic solution and an oxide
which can dissolve readily in an acidic solution.

What could this element be?

A Calcium
B Aluminium
C Silicon
D Phosphorus

Solution: B
Students must be able to find an element which can give acidic properties for both chloride and oxide.
Only aluminium possesses such amphoteric characteristic.

25 Use of the Data Booklet is relevant to this question.


The graph below shows the second ionisation energies of elements 1 to 4.
The elements are adjacent to each other in the Periodic Table.

Second
ionisation energy
(arbitrary units)
2

4
3

Atomic number

Which statement about the elements 1 to 4 is correct?

A The melting point of the elements increases in the order of 1 > 2 > 3 > 4
B The atomic radius of the elements decreases in the order of 2 < 3 < 4 < 1
C The electrical conductivity of the elements increases in the order 1 < 2 < 3 < 4
D The third ionisation energy of the elements decreases in the order of 3 < 2 < 1 < 4

Solution: C
The atomic number increases from Group VIII, I, II and III.
17

A B C D
1, 2 and 3 1 and 2 only 2 and 3 only 1 only
are correct are correct are correct is correct

26 Which of the following contain(s) hydrogen bonds?

1 NH4Cl (s)
2 NH3 (l)
3 HNO3 (l)

Solution: C
Option 1: The N in NH4+ does not contain lone pair of electrons for H-bonding.

H H
H N H N
H H
Option 2:

O O H O H
N O
N
O O
Option 3:

27 The kinetics of the reaction between compounds Y and Z were shown in the graph below:

[Y][I2]

2y

[ [Z]
J]=22ax
y

[Z]J =] =x 2x
[[H] a

time / min
5 10

Which of the following conclusion(s) can be drawn from the graph?


1 The reaction is first order with respect to compound Y.
2 The reaction is second order with respect to compound Z.
3 The overall order of reaction is 2.

Solution: C
The reaction is second order with respect to compound Z because rate of reaction increases four
times when its concentration is doubled. The reaction is independent of compound Y because a
straight line graph was obtained. i.e. rate is independent of [Y] Therefore the overall order of reaction
is 2.
18

28 In an acid-base titration, 25.0 cm3 of 0.150 mol dm-3 HCl was titrated with 0.150 mol dm-3 NH3.
The table below shows the data of four acid-base indicators.

Indicator Approximate pH range

Congo red 3.0 5.0


Bromocresol green 3.8 5.5
Cresol red 7.2 8.8

Which of the following correctly represent(s) the appropriate indicator(s) used in the above
titration?

1 Congo red
2 Bromocresol green
3 Cresol red

Solution: B
For a strong acid-weak base titration, region of rapid pH change lies between pH 3-7.
Only congo red and bromocresol green pH transition range lies within the region of rapid pH change.

29 The structure of compound B is shown below:


Br
Br
OH

O
Compound B

What types of reaction will compound B undergo?

1 Electrophilic substitution and condensation


2 Nucleophilic substitution and elimination
3 Neutralisation and nucleophilic addition

Solution: D

Statement 1 is correct:
- presence of benzene ring thus electrophilic substitution can occur.
- Presence of ketone functional group thus condensation reaction is possible with 2,4-DNPH.
19

Statement 2 is wrong:
- Halogenoarenes cannot undergo nucleophilic substitution and elimination.
- The alcohol has no neighbouring H atoms.Thus elimination is not possible.

Statement 3 is wrong:
- There are no acidic groups present.
- Ketones can undergo nucleophilic addition with HCN.

30 Compound C can be formed from ethanal in three steps:


O

CH3CHO 3 steps CH3


O
Ethanal
O
CH3
O
Compound C

Which of the following reagent(s) and condition(s) is / are required?

1 Dilute H2SO4, reflux


2 Excess concentrated H2SO4, reflux
3 HCN in aqueous NaOH, cold

Solution: A

O
CH3
O
O
CH3

CH3CHO CH3CH(OH)CN CH3CH(OH)COOH


O

-END -
2010 JC2 H1 Prelim Paper Suggested Solution

1 The simplest hydride of boron is boron hydride of molecular formula B2H6. It has
been suggested that gaseous boron hydride can be used as a rocket propellant,
using either oxygen or fluorine as oxidants.

Combustion of boron hydride with oxygen yields a high melting point solid
boron (III) oxide, B2O3 and water as the only products.
(a) (i) Write the balanced chemical equation for the reaction between [1]
boron hydride and oxygen.
B2H6 + 3O2 B2O3 + 3H2O Comment: Moststudentsabletowrite
thebalancedequation.

(ii) Using the information given below, calculate the standard [1]
enthalpy change for the above reaction.

Substance Standard enthalpy change of formation /


kJ mol-1
B2O3 -1273
B2H6 +36
H2O -286

Standard enthalpy change of reaction

= [-1273 + 3(-286)] (+36)


Comment: Somestudentsusedthe
= -2167 kJ mol-1 wrongformula
H=Hf(reactants)Hf(products)instead
(iii) [3] ofH=Hf(products)Hf(reactants).A
numberofstudentalsodidnotmultiply
2B3+ (g) + 3O (g) + 6e theHf(H2O)by3astherewere3moles
ofwaterproduced.
2x H6
2B2+ (g) + 3O (g) + 4e 3 x EA of oxygen

2 x H5
2B+ (g) + 3O (g) + 2e 2B3+ (g) + 3O2- (g)

2 x H4
2B (g) + 3O (g)

3 x H3 Hlatt
2B (g) + 3/2 O2 (g)

2 x H2
2B (s) + 3/2 O2 (g)

H1
B2O3 (s)

Using data from the Data Booklet and information below,


identify the standard enthalpy terms H1, H2, H3 and H4
on the left hand side of the Born Haber Cycle.

Hence determine the lattice energy of boron (III) oxide.

Enthalpy term H / kJ mol-1


Standard enthalpy change of -1273
formation of boron (III) oxide
Standard enthalpy change of +573
atomisation of boron
Standard enthalpy change of +248
atomisation of oxygen
Sum of first and second electron +702
affinity for oxygen

H1 is standard enthalpy change of formation of boron (III) oxide Comment: Moststudentswerenot


abletoidentifythestandardenthalpy
changesfromtheequationsgiveninthe
H2 is standard enthalpy change of atomisation of boron energycycle.Anumberofstudentsalso
gavethevaluesofthestandardenthalpy
H3 is standard enthalpy change of atomisation of oxygen changesinsteadidentifyingthestandard
enthalpychanges.Manyalsodidnot
attemptthisquestion.
H4 is first ionisation energy of boron

2B3+ (g) + 3O (g)


rd
3 I.E
= 2(3660)

2+
2B (g) + 3O (g)
nd st nd
2 I.E 1 and 2 E.A
= 2(2420) = 3(702) =2106kJmol-1
2B+ (g) + 3O (g) 2B3+ (g) + 3O2- (g)
st
1 I.E
= 2(799)
2B (g) + 3O (g)
Ha(O) Hlattice of B2O3
= 3(248)=744 kJmol-1
2B (g) + 3/2 O2 (g)
Ha(B)
-1
= 2(573)=1146 kJmol
2B (s) + 3/2 O2 (g)
Hf= -1273 kJmol-1

B2O3 (S)
Hlattice (B2O3) = -1273[1146+744 +2(799) +2(2420) +2(3660)+2106)]= -19027 kJ mol-1 Comment: Moststudentswerenot
abletodothisastheywerenotableto
identifythecorrectstandardenthalpy
changeofreaction.Manydidnot
attemptthisquestion.
(b) Paracetamol is a common analgesic and pyretic drug often
used as a painkiller and for treating mild fever. In a study
carried out to determine the rate of removal of paracetamol
from the body, the reaction was found to have a constant half-
life of 2.7 hours.
H
N

O
HO
Paracetamol
(i) Sketch a graph to show how 1 mol dm-3 of paracetamol [2]
decreases to 25% of its initial concentration.
Let the initial concentration of paracetamol be x.
candidate can start with 1 mol dm-3or any appropriate concentration Comment: Moststudentswereableto
[paracetamol] drawthisgraph.Somestudentsdidnot
labeltheaxescorrectlyormissedoutthe
labelsoftheconcentrationstoillustrate
thehalflives.

Time/h
2.7 5.4
(ii) Write the rate equation for the reaction and calculate the value [2]
of the rate constant, k.
Rate = k [paracetamol] Comment: Moststudentswereableto
writetherateequationbutmajority
ln 2
k= werenotablecalculatethekvalueas
theyhaveforgottenabouttheformula
t1 thatrelateshalflifetotherateconstant.
2

ln 2
=
2.7
= 0.257 h 1

(iii) Explain why Paracetamol is not very soluble in water. [1]

The intermolecular forces of attraction between Paracetamol are primarily Van


der Waals forces of attraction which is not able to displace the stronger Comment: Manystudentswereable
hydrogen bonding in water. Hence it is not soluble in water. toexplainthisquestion.Anumberof
studentsmentionediondipole
interactionswhichisincorrectasitis
Or presence of hydrophobic benzene ring. usedtoexplainthesolubilityofionic
compounds.
Total: 10 marks
2 Alcohol B has the following structure:
OH
CH3 C CH3
3 7 C H

(a) Draw the displayed formula of the possible isomers of alcohol [2] Comment: Majorityofthestudentsdid
notgavethetwopossibleisomersof
B. Label your isomers for use in (b). alcohol.Studentsfailedtorecognisethat
thereischainisomerismfromthe
H H molecularformulaofC3H7.Insteadmany
H O H H O H studentsdrewtheotherpossible
alcoholswiththesamemolecular
H C C C H H C C C H formulawithalcoholB.
H H H H
H H H H

H C C C H H C C C H

H H H H H
H

Alcohol 1 Alcohol 2

(b) To the corresponding isomers of alcohol B in (a), concentrated [2]


sulfuric acid was added and products with molecular formula,
C6H12 were obtained. Draw the structural formula of the
products formed. Comment: Asstudentswerenotable
toanswerparta,mostalsoanswered
H
H partbwrongly.Thosewhomanagedto
H O H drawthecorrectisomersinpartaonly
H O H drewoneproductfromelimination,
H C C C H
H C C C H failingtorecognisethatwateris
H H eliminatedfromadjacentcarbonatoms
H H H H
H H andwouldthereforeeachalcoholwill
H C C C H give2products.
H C C C H
H H
H H H H

H CH3 CH3 H CH3 CH3 CH3


CH3
C C C C C C
H CH2CH2CH3 C H C CH3 C
CH3
H CH2CH3 H CH3 CH3

Productsofalcohol1Productsofalcohol2

(c) The boiling point of three organic compounds is given as [2]


follow:

Compound Boiling Point

Butene 273 K

Butan-1-ol 390 K

Butanoic acid 437 K

Briefly explain the difference in their boiling point.


Butene, butan-1-ol and butanoic acid are simple molecular substances.

Butene has weak intermolecular van der waals forces attraction while Comment: Anumberofstudentswere
notabletoidentifythatbutanehas
butan-1-ol and butanoic acid has stronger intermolecular hydrogen bonding. intermolecularVDWandthatbutanol
andbutanoicacidhasintermolecular
hydrogenbondingandthoughtthatit
Intermolecular hydrogen bonding in butanoic acid is more extensive than that wasduetodifferencesinthenumberof
of butan-1-ol. electrons.Studentsalsofailtorecognise
thatbutanoicacidwereabletoform2
hydrogenbondsandbutanolonlyform1
Energy required: hydrogenbondthustheyhaving
differentextensivenessofhydrogen
Butanoic acid > Butan-1-ol > Butene bonding.Studentsalsofailedtomention
differentamountofenergyrequiredto
overcometheintermolecularforcesof
(d) Organic compound C contains carbon, hydrogen and oxygen [4] attraction.
only.

10 cm3 of compound C undergoes complete combustion to


produce 40 cm3 of carbon dioxide and 50 cm3 of water vapour.
1 mol of C produces 0.5 mol of hydrogen gas when sodium
metal is added.

No effervescence was observed when compound C was


reacted with sodium carbonate but white fumes of gas were
seen when phosphorous pentachloride was added to
compound C.

Upon reaction with acidified potassium manganate (VII),


compound D was formed. Compound D reacts with 2,4-DNPH
but did not react with Tollens reagent. Compound C and D
both give yellow precipitate when reacted with alkaline
aqueous iodine.

Compound C was passed through hot alumina and compound


E was obtained as one of the products. When compound E
undergoes strong oxidation, bubbles of gas were observed.

Draw the structures of Compound C, D and E.


From the combustion data:
10 cm3 of compound C gives 40 cm3 of CO2 and 50 cm3 H2O
Which means
1 mol of compound C gives 4 mol of CO2 and 5 mol H2O

CO2 C ; H2O 2H

Compound C contains 4 C and 10 H


Since 1 mol of C produces 0.5 mol of hydrogen gas when sodium metal is
added presence of one alcohol group C4H10O Comment: Manystudentsdidnot
determinethemolecularformulaofC
whichwasderivedfromtheinformation
giveninthequestiontohelpthem
deducethestructureofC.
CH3 H Yellow ppt
Comment: Quiteanumberofstudents
C C -
I2/OH I2/OH-
Hot alumina wereabletodeducethecorrect
H CH3 CH3 structuresofCandD.Somewerenot
CH3 abletodetermineE.Anumberof
MnO4-/H+
H C CH2CH3 O C CH2CH3 studentsthoughtthatChasonly3C
insteadof4C.
OH D
H Hot alumina C Tollens
H 2,4-DNPH
C C
H CH2CH3 Reaction occurs No Reaction
E
Strong oxidation

CO2 gas produced


due to terminal
alkene

Total: 10 marks
3 Shikimic acid, is an important biochemical intermediate in
plants and microorganisms.

COOH

HO OH
OH

Shikimic acid

(a) Draw the structural formulae of the organic products formed [2]
when shikimic acid is treated with HBr followed by hot aqueous
NaOH.
COO-Na+ Comment: Moststudentsdidnotget
HO thecorrectstructure.Somewereableto
recognisethatthecompound
electrophilicadditionbutnotableto
recognisethatthereisalsonucleophilic
substitutionandneutralisation.Some
HO OH
alsodrewtheminorproductfromthe
OH addition.

(b) Apart from shikimic acids, cinnamic acid, zingerone and vanillin
are other compounds that can be naturally isolated from plants
and herbs.

CH3
COOH
HO
OCH3

Cinnamic acid Zingerone

CHO

HO
OCH3
Vanillin

(for this question ignore the chemical effect of OCH3)


(i) State one reagent and condition that will react with vanillin and [2]
not zingerone and describe what would be observed.
Comment: ThosewhousedTollens
Reagent and condition: Tollens reagent, warm reagentstatedthewrongconditionof
refluxinsteadofwarm.Onreflux,no
silvermirrorwillbeobserved.Quitea
Observation: Silver mirror numberofstudentssuggestedusing
Fehlingsreagentwhichisusedto
identifyaliphaticaldehydeandthus
(ii) State one reagent and condition, other than Br2 and oxidising [2] cannotbeusedinthiscaseasvanillinis
agent, that will react with cinnamic acid and not zingerone and anaromaticaldehyde.

describe what would be observed.


Comment: Manystudentswerenot
Reagent and condition: Na2CO3 (aq), rtp abletousethecorrectreagentand
suggestedusingNainstead.Nawill
reagentwithbothcompoundsandthus
Observation: effervescence produced (giving white ppt on passing gas notsuitable.
produced through aqueous Ca(OH)2 )

(c) The carbonyl functional group undergoes a series of reactions


which follow the following pattern:

catalyst X
C O + HX C
OH

Where X can be CN, OCH3, etc.

Using the above information, draw the structural formula of the


product formed for the following reactions:
(i) [1]
C CHO
O OH
Addition of HCN to , followed by a
dehydration process.

Comment: Mostwerenotableto
deducethecorrectstructure.

O O CN

(ii) Addition of CH3OH to vanillin, followed by an oxidation [1]
reaction.

(Note: OCH3 is inert to oxidation)


Comment: Mostwerenotableto
O answerthisquestion.

C
OCH3

HO
OCH3

(d) Shikimic acid undergoes complete combustion to form carbon [2]


dioxide and water. Construct a reaction pathway diagram for
the combustion process. In your diagram, clearly indicate the
enthalpy change of combustion and activation energy.

Transition State

Activation Energy

Shikimic acid
Enthalpy change of
combustion

CO2 and H2O

Comment: Mostwereabletodrawthe
Reaction Pathway correctenergyprofilediagram,anumber
gavetheendothermicgraphinstead.

Total: 10 marks
4 (a) (i) On the grid below, sketch the trend of the ionic radii of elements in Period 3
from sodium to chlorine.
[1]
(ii) Explain the trend of the ionic radii for elements in Period 3. [3]

ionic
radii / nm

optional

Na Mg Al Si P S Cl Element
Comment: Mostwereabletodrawthe
[1m]. Correct sloping and relative position. correcttrendbutanumberdrawa
generaldownwardslopinggraphandnot
realisingthatanionicradiusislargerthan
Na, Mg and Al (and Si) form cations while P, S and Cl form anions. Anionic cationicradius.
radii are larger than cationic radii because they have 1 more quantum shell Comment: Moststudentsdidnot
of electrons. explainthedifferenceintheanionicand
cationicradius.

Generally, ionic radius decreases across the period. Nuclear charge increase Comment: Manystudentsdidnot
explainthegeneraldecreasingradius
across the period, while the ions are isoelectronic to one another. Thus, trendforbothcationsandanions.
valence electrons experience stronger electrostatic forces of attraction from Insteadtheyexplainedthedifferenceof
cationicradiuswithatomicradius.
the nucleus across the period, pulling them closer towards the nucleus /
reducing the distance between valence electrons and nucleus.
(b) The first six ionisation energies of three successive elements in a Periodic
Table are given below:

Ionisation Energy in kJ mol-1

Element 1st 2nd 3rd 4th 5th 6th

F 786 1580 3230 4360 16100 19800

G 1060 1900 2920 4960 6270 21300

J 1000 2260 3390 4540 7000 8500

(i) State and explain the Group which element G belongs to. [2]

Group V
Drastic increase in ionization energy occur from 5th IE to 6th IE indicating
the 6th electron removed is found in the inner quantum shell, hence, there are 5
electrons in the valence shell of G

(ii) Explain why the first ionisation energy of G is [2]

higher than F:
and higher than J:

F: ns2np2
G: ns2np3
J: ns2np4

1st IE of G is higher than F because nuclear charge of G is higher than F and Comment: Manystudentshad
forgottenhowtoexplainthedifferences
the valence electrons of F and G are found in the same subshell, thus the inionisationenergyacrosstheperiod.
valence electron on G experiences stronger electrostatic forces of attraction
from the nucleus.

1st IE of G is higher than J because of inter-electron repulsion between the


paired electrons occupying the same p-orbital in J. Lesser energy is Comment: Manystudentsfailto
recognisethattherewasinterelectron
required to remove it. repulsionbetweenthepairedelectrons
andexplaineditintermsofnuclear
(iii) Suggest a chemical formula of an oxide of element J and [2] charge.
predict the type of bonding present in the oxide of J.
Formula of oxide of J: JO2 or JO3 Comment: Moststudentswerenot
Type of bonding present in oxide of J: Covalent bonding abletorecognisethatelementJisanon
metalandwillformacovalent
compoundwithoxygen.SinceGisin
(iv) With aid of an equation, predict a pH value for the resulting [2] groupV,JshouldbeingroupVIasthey
solution obtained when the oxide of J is dissolved in water. areconsecutiveelements.Studentswere
notabletolinkthequestionstogether.

JO2 + H2O H2JO3 or JO3 + H2O H2JO4 Comment: Asstudentswerenotable


pH = 1, 2 or 3 (any one) togivethecorrectchemicalformulaof
theoxide,theywerenotabletowrite
theequationandthusthepH.
Total: 10 marks
Paper 2 Section B
5 (a) The kinetics of the acidcatalysed reaction of propanone with iodine can be
investigated experimentally by varying the concentrations of the three
substances involved.

CH3COCH3(aq) + I2(aq) CH2ICOCH3(aq) + HI(aq)



The following results were obtained in an experiment.

[CH3COCH3] = 0.10 mol dm3


[I2] / mol dm3

[H+] = 0.10 mol dm3

[H+] = 0.20 mol dm3

time / min

[H+] = 0.10 mol dm3


[I2] / mol dm3

[CH3COCH3] = 0.10 mol dm3

[CH3COCH3] = 0.15 mol dm3

time / min

time / min

(i) Deduce, with reasoning, the order of reaction with respect to (1) I2, (2) H+ [5]
and (3) CH3COCH3.
Since [I2] vs time graph is a straight line, rate of reaction is independent
of [I2].
Order of reaction is zero wrt I2. Comment: Studentsdidnotgive
reasonfortheorderofreactionwrtI2
Using the 1st graph,

Rate when [H+] = 0.20 mol dm3 :


-(0.008-0.002) / (0-120) = 5 x 10-5 moldm-3min-1
Rate when [H+] = 0.10 mol dm3 :
-(0.008-0.002) / (0-240) = 2.5 x 10-5 moldm-3min-1
When [H+] doubles, the rate also doubles.
Order of reaction is one wrt H+. Comment: Studentsstatethiswithout
anysupportingargument.
Using the 2nd graph,

Rate ([CH3COCH3] = 0.10 mol dm3):


-(0.008-0.002) / (0-240) = 2.5x10-5 moldm-3min-1

Rate ([CH3COCH3] = 0.15 mol dm3):


-(0.008-0.002)/(0-160) = 3.75x10-5 moldm-3min-1

When [CH3COCH3] increases by 1.5X, the rate also increases by 1.5X.


Order of reaction is one wrt CH3COCH3. Comment: Studentsstatethiswithout
anysupportingargument.

(ii) Hence, write a rate equation for the reaction and determine a value for the [3]
rate constant k, including its units.
Rate = k[CH3COCH3][H+] Comment: Incorrectexpressionfor
rateequation
Using graph 1,
5x10-5 = k (0.1)(0.2)
k = 2.50 x 10-3 mol-1dm3 min-1

(iii) Predict, with reasoning, the effect of a catalyst on the activation energy [2]
and rate constant of a reaction.
A catalyst lowers the activation energy by providing an alternative
pathway for reaction to take place. Comment: Studentsfailedtomention
alternativereactionpathway
The forward reaction is favoured which results in a larger rate constant.

(b) Propanoic acid and citric acid are some of the acids used in the formulation of
hair shampoo. A chemistry student wanted to formulate a shampoo with
maximum buffer capacity so that there will be minimal damage to the hair.

In an attempt to prepare a buffer for the shampoo, she titrated 20.0 cm3 of
0.100 mol dm-3 sodium hydroxide against 0.20 mol dm-3 citric acid.
(i) Deduce, with reasoning, whether propanoic acid or 2-chloropropanoic acid [2]
is a stronger acid.
Electron withdrawing Cldisperses the negative charge on oxygen.
This stabilises the conjugate base and 2-chloropropanoic acid is a
stronger acid compared to propanoic acid.

(ii) Calculate the pH of 0.100 mol dm-3 sodium hydroxide. [1]


pOH= -log [OH-] = - log (0.1) = 1
pH = 14-1 = 13 Comment: Studentstooklg(0.100)
andcalculatedthewrongpHof1.

(iii) Calculate the volume of citric acid required to completely neutralise [2]
20.0 cm3 of sodium hydroxide.
Amt of NaOH = 20/1000 x 0.1 = 0.002 mol
Citric acid NaOH
Vol of citric acid = 0.002 / 0.2 = 10.0 cm3
(iv) When excess citric acid is added, a buffer solution is obtained. [2]
Explain, with the aid of equations, how the mixture can act as a buffer Comment: Studentsgavethewrong
upon addition of a small amount of acid and alkali. equation
(Citric acid could be represented by C5H7O4COOH)
When H+ is added,
H+ + C5H7O4COO- C5H7O4COOH Comment: Emphasisonthesingle
+
arrowasH andOH isremovedbythe
buffersolution.
When OH- is added,
C5H7O4COOH + OH- C5H7O4COO- + H2O

(v) The pH of 1 mol dm-3 of propanoic acid is 4 while the pH of 1 mol dm-3 of [1]
hydrochloric acid is 1. Explain the difference in their pH. Comment: Thisisasimplequestion
Propanoic acid is a weak acid and dissociates partially to produce low butsomestudentsmakeuseofelectron
withdrawingconceptwhichistotally
[H+] while hydrochloric acid dissociates completely to produce high [H+]. wrongforthisquestion.

Since pH=-log [H+]

Hence pH of propanoic acid is 4 while pH of hydrochloric acid is 1.

(vi) Suggest with reasoning, if the same volume of hydrogen gas will be [2]
produced if propanoic acid and hydrochloric acid, of the same
concentration and volume are to react with 2.0 g of zinc metal under the
same condition.

(1) CH3COOH CH3COO- + H+


(2) HCl H+ + Cl-
(3) 2H+ + Zn Zn2+ + H2
Comment: Moststudentsgotthis
Same volume of H2 gas will be produced in both cases. questionbysayingweakacidgivelower
volumeofH2gasforthereaction.

Reason:

Since H+ reacts with zinc, for (1), [H+] will decrease and by LCP,
equilibrium position of (1) will shift right, hence all CH3COOH will be
reacted eventually producing the same amount of H2 gas.
6 (a) Compound K has a composition of 46.15% C, 7.69% H and 46.15% O. It is
formed when compound L (a bromo-containing organic substance) is refluxed with
aqueous sodium hydroxide followed by acidification.

Effervescence was observed when sodium carbonate was added to K. Upon


treatment of K with acidified potassium dichromate (VI), compound M is produced.
Compound M reacts with 2,4-dinitrophenylhydrazine to form an orange hydrazone,
N. On reaction of M with Fehlings reagent, the reddish brown deposit was
however not observed.

A yellow precipitate was observed when K and M were warmed with alkaline
aqueous iodine. When K is refluxed with concentrated sulfuric acid, a sweet
smelling compound P, C8H12O4 is formed.

Deduce the structures of K, L, M, N and P. Maximum credit cannot be awarded Comment: Moststudentsfailedto
[10] writethefunctionalgroups.
just by drawing of the structures only.

C H O
% by mass 46.15 7.69 46.15
Ar 12 1 16
Amount 3.85 2.67 2.88
Simplest ratio 4 8 3

The empirical formula of K is C4H8O3. Comment: Studentscalculatedthe


informationinthetablebutfailtowrite
outtheempiricalformula.
L undergoes nucleophilic substitution and neutralisation with aqueous
sodium hydroxide, followed by acidification to form K. Comment: Somestudentsareunsure
andtheywroteelectrophilicsubstitution
L contains halogenoalkane. whichiswrong.

K undergoes neutralisation with sodium carbonate to produce


effervescence.
K contains carboxylic acid group.

K and M undergoes oxidation with alkaline solution with iodine.


OH Comment: Studentsfailedtoindicate
O R C
thestructures.
CH3
R
They contain the structure
C CH3 H
or

K undergoes oxidation with K2Cr2O7 to form M


K is a secondary alcohol Comment: Somestudentsindicate
primaryalcohols.

M undergoes condensation with 2,4 DNPH but do not undergo oxidation


with Fehlings reagent.
M contains a ketone

A undergoes self esterification when refluxed with concentrated sulphuric


acid to form E.
P is an ester
K contains an alcohol and an acid group.
OH

CH3 C CH2COOH

H
K
Br

CH3 C CH2COOH

H
L
O
CH3 C CH2COOH
M


O2N Comment: Studentsdidnotknowhow
todrawstructureofhydrazine.

N N NO2
C H
CH3 CH2COOH
N

CH3 H

CH2 C O
O C
O C CH2C O
H CH3
P

Max 5 for reasonings and Max 5 for structures


(b) Suggest a simple chemical test to distinguish between the following [3] Comment: Studentsfailedtorecognise
thattheyneedtohydrolysetheester
compounds: beforetheycandoanytest.
O
O
O
O

and
Test: Add acidified potassium manganate to separate test tubes of each
compound and reflux.
Observations:
O

:No decolourisation of purple KMnO4.


O

: Decolourisation of purple KMnO4.

(c) Antimony pentachloride (SbCl5) decomposes in a gas phase reaction at high


temperature.

SbCl5 (g) SbCl3 (g) + Cl2 (g) H = negative

At some temperature, dynamic equilibrium is established. An equilibrium


mixture in a 5 dm-3 container is found to contain 6.91 g of SbCl5, 16.45 g of
SbCl3 and 5.11 g of Cl2.

(i) What is meant by the term dynamic equilibrium? [1]

A dynamic equilibrium is one where rate of forward reaction equals rate


of backward reaction in a reversible reaction. Comment: Allstudentsfailedto
indicatethisintheiranswer.

(ii) Determine the Kc expression and hence calculate Kc for the above [3]
equilibrium.

[SbCl3 ][Cl2 ]
Kc =
[SbCl5 ]
[SbCl5] = 4.614 x 10-3 mol dm-3
[SbCl3] = 0.014398 mol dm-3
[Cl2] = 0.014394 mol dm-3

Kc = 0.0449 mol dm-3 Comment: Studentsmadecareless


mistakeintheircalculation
(iii) Using Le Chateliers Principle, state and explain the effect on the position of
equilibrium and the equilibrium composition when: Comment: Somestudentsfailedto
statetheequilibriumcomposition.
1) temperature is decreased at constant pressure,
2) pressure is decreased at constant temperature. [3]

According to Le Chateliers Principle, the position of equilibrium shifts to the


right to release heat, favouring the exothermic reaction. Comment: Studentsfailedtoindicate
thisintheiranswerortheygotitwrong.
[SbCl3] and [Cl2] will increase while [SbCl5] decrease until a new
equilibrium is reached.

According to Le Chateliers Principle, the position of equilibrium shifts to the


right to increase no of gaseous molecules.

[SbCl3] and [Cl2] will increase while [SbCl5] decrease until a new
equilibrium is reached.
7 (a) The oxides of sodium to phosphorus (Na2O, MgO, Al2O3, SiO2 and P4O10) differ
in their bonding and reactions with water.

(i) Complete the following graph on your answer script. [2]

You are to indicate clearly on your graph the pH of the resulting solution Comment: Studentsmixedupand
when each oxide reacts with water. drew

pH

13


2
Na2O MgO Al2O3 SiO2 P4O10
(ii) With the aid of equations, state the pH of the solution obtained when [2] Comment: Studentsonlygaveone
equation.
aluminium chloride is dissolved in water.
Somestudentswrotetheequationfor
AlCl3 + 6H2O [Al (H2O)6]3+ + 3Cl- aluminiumoxideinsteadofaluminium
chloride.
[Al(H2O)6]3+ [Al(H2O)5(OH)]2+ + H+
pH = 3
(iii) A student was instructed to perform tests to deduce the identities of two
chemical bottles with missing labels. The bottles contain either gallium
oxide or selenium oxide.

The student first labelled the two bottles as Q and R. Then, he conducted
solubility tests and record his observations as follows:

Sample Q dissolves in both hydrochloric acid and aqueous sodium


hydroxide while sample R dissolves in aqueous sodium hydroxide only.
Suggest the identities of Q and R and write equations to show the acid- Comment: Somestudentswrote
GalliumandSeleniumforQandR
base behaviour of the oxides. [4] respectivelywhichiswrong
[Gallium is 31Ga; Selenium is 34Se] Comment: Studentsarestillnot
Q: Ga2O3 / gallium oxide R: SeO2 / selenium oxide familiarinwritingtherequiredequations.

Ga2O3 (s) + 6HCl (aq) 2GaCl 3 (aq) + 3H2O (l)


Ga2O3 (s) + 2NaOH (aq) + 3H2O (l) 2NaGa(OH)4 (aq)
SeO2 (s) + 2NaOH (aq) Na2SeO3 (aq) + H2O (l)
(b) Ethanol is a chemical which is widely used as a solvent, fuel and an important
intermediate in large scale synthesis of organic compounds.

When 2.00 g of ethanol was burned under a container of water, the heat
released was found to increase the temperature of 150 g of water from 28oC to
65oC.

(i) Write a balanced equation, with state symbols, to represent the standard [1]
enthalpy change of combustion of ethanol.
Comment: Studentsgotthisstate
CH3CH2OH (l) + 3O2 (g) 2CO2 (g) + 3H2O (l) symbolwrong,

(ii) Using the data given above, calculate the enthalpy change of combustion [2]
of ethanol.
Comment: Studentsused2gwhichis
Heat absorbed by water, Q = mc T = (150)(4.18)(6528) wrong.

= 2.3199 x 104 J = 23.20 kJ


2.00
Amount of ethanol burnt = = 0.04348 mol
46
Comment: Signwasnotwrittento
23.20
Enthalpy change of combustion = = 534 kJ mol-1 . indicatetheexothermicreaction.
0.04348
(iii) Using bond energy values from the Data Booklet, calculate another value [3]
for the standard enthalpy change of combustion of ethanol. Suggest an
explanation for the discrepancy between your answer and the calculated
value in b(ii).
Hcombustion = B.E.(reac tan ts ) - B.E.( products )
= [(1x350) + (5x410) + (1x460) + (360x1) + (3 x 496) [(740x4) + (460x6)]
= 4708 5720 = -1012 kJ mol-1

Bond energies used in this calculation are average values.


Or Answer did not consider the condensation of water.
(c) Glycolic acid is the smallest -Hydroxy acids (AHAs). It is also one of the main
ingredients used in the manufacture of skin care products due to its excellent
capability to penetrate human skin.

O
HO
OH

Glycolic Acid

(i) Show how glycolic acid can be synthesized from methanol via three steps. [3]
In your answer, show clearly the reagents, conditions and intermediates
involved.
Step 1
O Comment: Asexpectedstudentswrote
KMnO4whichiswrongasitwilloxidise
Reagents: acidified K2Cr2O7 themethanolintoCO2andH2O.
CH3OH H H Conditions: Distlillation

Step 2
O
CN Reagents: HCN with trace
HO
H H amount of NaOH / NaCN
Conditions: cold 10 20oC
Step 3

O
HO CN HO
OH
Reagents: aq HCl / aq H2SO4
Conditions: Heat/ reflux

Alternative:
Step 1: Limited amount of Cl2 or excess methanol
(free radical substitution) Comment: IfFreeRadicalSubstitution
isused,limitedCl2oeexcessmethanol
Step 2: alcoholic NaCN, heat (nucleophilic substitution) mustbeindicated.
Step 3: dilute HCl, heat (acidic hydrolysis)

(ii) State the product(s) formed when hot acidified potassium manganate (VII) [1]
is added to glycolic acid.
CO2 and H2O

(iii) Explain, with the aid of a diagram, why the relative molecular mass of [2]
glycolic acid is 152 when dissolved in a suitable solvent.
Dimerisation occurred. . Comment: Diagramwaspoorlydrawn
aseitherlonepairofelectronsorpartial
+ chargewasnotpresent.
H H O H
O
HO C C C C OH
O H O H
H
+

END OF CORRECTION TEMPLATE


TEMASEK JUNIOR COLLEGE

CHEMISTRY 8872/01
Paper 1 Multiple Choice

Tuesday 21st SEPTEMBER 2010 50 minutes

Additional materials: Data Booklet


Multiple Choice Answer Sheet

READ THESE INSTRUCTIONS FIRST

Do not open this booklet until you are told to do so.

Write in soft pencil.


Do not use staples, paper clips, highlighters, glue or correction fluid.
Write your name, index number and CG on the Answer Sheet in the spaces provided.

There are thirty questions on this paper. Answer all questions. For each question, there are four
possible answers labelled A, B, C, and D.
Choose the one you consider correct and record your choice in soft pencil on the separate Answer
Sheet.

Read the instructions on the Answer Sheet very carefully.

Each correct answer will score one mark. A mark will not be deducted for a wrong answer.
Any rough working should be done in this booklet.
You may use a calculator.

This document consists of 13 printed pages.


[Turn over
2
Section A

For each question there are four possible answers, A, B, C and D. Choose the one you consider to
be correct.

1 10 11
Boron consists of the isotopes 5B and 5B . In an experiment, the relative abundance of
11
5 B atomsin the isotopic mixture is found to be 60%. What is the relative atomic mass of
boron based on the experiment?

A 10.2 B 10.4 C 10.5 D 10.6

2 Which of the following formulae represents a particle with the composition 1 proton, 1 neutron
and 2 electrons?
(D represents deuterium, 2H)

A D B D- C H- D He

3 Oxygen reacts with platinum (VI) fluoride, PtF6, as follows:


O2 + PtF6 O2+ PtF6-

It was suggested that xenon should react similarly and, in this way, the first noble gas
compound was produced.
Xe + PtF6 Xe+ PtF6-

What is the most likely reason for the suggestion being made?

A O and Xe have similar atomic radii.

B O and Xe have similar atomic affinities.

C O and Xe have similar electronic configurations.

D O2 and Xe have similar first ionisation energies.

TJC 2010 8872/01/PRELIM 2010 [Turn over


3

4 Which graph represents the number of unpaired p orbital electrons for atoms with proton
numbers 13 to 18?

A B

C D

H
5 Which one of the following is the correct shape of hydroxylamine, H N O H ?

About N About O

A Trigonal pyramidal Bent

B Trigonal pyramidal Linear

C Trigonal planar Linear

D Trigonal planar Bent

TJC 2010 8872/01/PRELIM 2010 [Turn over


4

6 Which of the following pairs contains delocalized electrons in both systems?

A cyclohexene, sodium

B sodium, graphite

C graphite, silicon

D benzene, ethene

7 Which of the following statements is correct?

A XeF4 and CH3CH=CH2 are planar molecules

B BCl3 and BrCl3 have the same molecular shape.

C Benzene and ethanol are nonpolar liquids.

D AlH4- and BF4- contain dative covalent bonds.

8 The boiling points of ammonia, hydrogen chloride and carbon dioxide are given below.

Gas Formula Boiling point / oC

ammonia NH3 33

hydrogen chloride HCl 85

carbon dioxide CO2 78

Which of the following statements best explains the above data?

A NH3 has a higher boiling point than HCl because the hydrogen bonding in NH3 is
stronger than the permanent dipolepermanent dipole interactions in HCl.

B The strength of the induced dipoleinduced dipole interactions increases in the order
HCl < CO2 < NH3.

C The strength of the permanent dipolepermanent dipole interactions increases in the


order HCl < CO2 < NH3.

D CO2 has a higher boiling point than HCl because the permanent dipolepermanent
dipole interactions in CO2 are stronger than the permanent dipolepermanent dipole
interactions in HCl.

TJC 2010 8872/01/PRELIM 2010 [Turn over


5

9 Three substances, R, S, T, have physical properties as shown.

Substance mp/oC bp/oC Electrical conductivity

of solid of liquid

R 801 1413 poor good

S 2852 3600 poor good

T 3550 4827 good not known

What could be the identities of R, S and T?

R S T

A NaF KCl Cu

B NaBr BaO SiO2

C NaCl MgO C[graphite]

D NaBr CaO C[diamond]

10 10 cm3 of 0.2 mol dm-3 K2XO4 will just react with 40 cm3 of 0.1 mol dm-3 iron(II) sulfate
solution.
If Fe2+ is oxidized to Fe3+ by K2XO4, what is the final oxidation state of X?

A +2 B +3 C +4 D +5

TJC 2010 8872/01/PRELIM 2010 [Turn over


6

11 2CHClF2 (g) CF2=CF2 (g) + 2HCl (g)


The standard enthalpy changes of formation of HCl (g) is -92.3 kJ mol-1, that of CHClF2 (g) is
-485.2 kJ mol-1 and that of CF2=CF2 (g) is -658.3 kJ mol-1.
Which of the following describes how the yield of CF2=CF2 (g) at equilibrium varies as
temperature increases?

A B

Yield Yield

temperature temperature

C D
Yield Yield

temperature
temperature

12 20 cm3 of 0.10 mol dm-3 of aqueous NaOH is added to 25 cm3 of 0.15 mol dm-3 of hydrochloric
acid.
What is the approximate pH of the final solution?

A 1.4 B 2.8 C 4.5 D 7.0

TJC 2010 8872/01/PRELIM 2010 [Turn over


7

13 The graph shows the change in pH when ethanoic acid is gradually added to 10 cm3 of 0.10
mol dm-3 sodium hydroxide. An indicator is used to determine the end-point for the titration.

Which of the following contains the correct concentration of ethanoic acid and indicator used
for the titration?

A 0.05 mol dm-3 ethanoic acid, screened methyl orange

B 0.05 mol dm-3 ethanoic acid, phenolphthalein

C 0.10 mol dm-3 ethanoic acid, screened methyl orange

D 0.10 mol dm-3 ethanoic acid, phenolphthalein

14 Iodine-131 is a radioactive isotope with a half-life of 8 days.


Given that radioactive decay is a first-order reaction, what fraction of the isotope would remain
after 48 days?

A 1 B 1 C 1 D 1
16 96 26 28

TJC 2010 8872/01/PRELIM 2010 [Turn over


8

15 Hydrogen peroxide reacts with acidified iodide ions, liberating iodine. In investigations of this
reaction, the following results were obtained.

initial concentration of reactants/ mol dm-3 initial rate of formation of


- + iodine/ mol dm-3 s-1
[H2O2] [I ] [H ]
0.010 0.010 0.10 2 x 10-6
0.030 0.010 0.10 6 x 10-6
0.030 0.020 0.10 1.2 x 10-5
0.030 0.020 0.20 1.2 x 10-5

Which of the following statements follows from these results?

A The overall order of this reaction is 2.

B The unit of the rate constant k is mol-2 dm6 s-1.

C This reaction can be catalysed by H+.

D The rate equation for the reaction can be written as rate = k [H2O2]2 [I-]

16 The chloride of an element R of the third period is a liquid which has a boiling point of 76 C
and fumes in air.
After mixing 0.010 mol of the chloride with water, the resulting solution required 100 cm3 of
0.30 mol dm-3 silver nitrate for complete precipitation of the chloride ion.
To which Group of the Periodic Table does R belong?

A I B III C IV D V

TJC 2010 8872/01/PRELIM 2010 [Turn over


9

17 The graph shows how a property of the elements Na to Cl varies with proton number.

11 12 13 14 15 16 17
Proton number

What is the property?

A ionic radius

B first ionisation energy

C melting point

D electronegativity

18 Which of the following synthetic routes does not produce ethanoic acid?

A aqueous alkaline iodine H+


CH3CH(OH)CH3
heat r.t.p.

B excess conc H2SO4 acidic K2Cr2O7


CH3CH(OH)CH2CH3
180 oC heat

C alcoholic KCN dil HCl


CH3Br
reflux heat

D [Ag(NH3)2]+ H+
CH3CHO
heat r.t.p.

TJC 2010 8872/01/PRELIM 2010 [Turn over


10

19 1,2-dibromo-3-chloropropane (DBCP) has been used in the control of earthworms in


agricultural land. Which of the following would be the best synthesis of this compound?

A CH3CH2CH2Cl + 2Br2 DBCP + 2HBr

B CH2BrCHBrCH3 + Cl2 DBCP + HCl

C ClCH2CH=CH2 + PBr5 DBCP + PBr3

D CH2=CHCH2Cl + Br2 DBCP

20 In the hydrogenation of HO2C-CH2-CH=CH-CHO using platinum catalyst, the volume of


hydrogen (measured at s.t.p.) that reacts with 1 mole of the compound could be

A 22.4 dm3 B 24.0 dm3 C 44.8 dm3 D 67.2 dm3

21 The ester X, which is used in perfumes, has the molecular formula C6H12O2. It is found that
one of the products of acid hydrolysis of compound X gives a yellow precipitate with hot
aqueous alkaline iodine.
What is the structural formula of compound X?

A CH3CO2CH2CH(CH3)2

B (CH3)2CHCO2CH2CH3

C CH3CH2CO2CH2CH2CH3

D CH3CH2CH(CH3)CO2CH3

22 When an organic compound X was treated with phosphorous pentachloride, fumes of


hydrogen chloride were evolved. When X was warmed with acidified aqueous potassium
dichromate(VI), the solution turned green.
Which of the following was X?

A CH3CH2CHO

B CH3CH(OH)CH3

C CH3CH2CO2H

D CH3COCH3

TJC 2010 8872/01/PRELIM 2010 [Turn over


11

23 Which one of the following gives the correct order of acid strength (strongest first) for ethanoic
acid, chloroethanoic acid, and ethyl ethanoate?

Strongest Weakest
A ethanoic acid chloroethanoic acid ethyl ethanoate
B chloroethanoic acid ethyl ethanoate ethanoic acid
C chloroethanoic acid ethanoic acid ethyl ethanoate
D ethyl ethanoate chloroethanoic acid ethanoic acid

24 Bromine in an inert solvent is added separately to hexane, hexene, benzene and


methylbenzene. In which of the following pairs will the observations be the same?

A Hexane and benzene

B Hexane and hexene

C Hexene and methylbenzene

D Hexene and benzene

25 Citrus fruits like lemons contain vitamin C which is essential to prevent scurvy. Vitamin C has
the following structure.
HO OH

O O OH
OH
Which one of the following statements about vitamin C is incorrect?

A Vitamin C cannot exhibit geometric isomerism.

B 1 mole of vitamin C reacts completely with 4 moles of Na metal to give 2 moles of H2


gas.

C It can react with 2,4-dinitrophenylhydrazine to give an orange precipitate.

D It can react with hot sodium hydroxide.

TJC 2010 8872/01/PRELIM 2010 [Turn over


12

Section B

For each of the questions in this section, one or more of the three numbered statements 1 to 3 may
be correct.
Decide whether each of the statements is or is not correct (you may find it helpful to put a tick
against the statements which you consider to be correct).
The responses A to D should be selected on the basis of

A B C D

1, 2 and 3 1 and 2 2 and 3


1 only is correct
are correct only are correct only are correct

No other combination of statements is used as a correct response.

26 Which reactions cannot represent standard enthalpy changes at 298 K?

1 NH3(g) + HCl(g) NH4Cl(s)

2 C(g) + 8H(g) C3H8(g)

3 CH4(g) + 2O2(g) CO2(g) + 2H2O(g)

27 For which of the following reactions does the enthalpy value represent both a standard
enthalpy change of combustion and a standard enthalpy change of formation?

1 C(s) + O2(g) CO2(g)

2 2C(s) + O2(g) 2CO(g)

3 C(s) + 2O(g) CO2(g)

TJC 2010 8872/01/PRELIM 2010 [Turn over


13

28 Consider the following changes:


H1: M(s) M(g)
H2: M(s) M2+(g) + 2e -
H3: M(g) M+(g) + e-
H4: M+(g) M2+(g) + e-
H5: M(g) M2+(g) + 2e -

The second ionisation energy of M could be calculated from the energy values associated with

1 H1 + H5

2 H5 - H3

3 H2 - H1 - H3

29 When a compound W was heated in the tube containing reagent X, a colour change was
observed in the tube. A precipitate was noted when reagent Y was heated with the products
from reaction of W and X. Which of the following combinations could be W, X and Y?

W X Y

1 CH2=CH2 aqueous bromine CH3CO2-Ag+

2 CH2=CHCH3 acidified KMnO4(aq) Fehlings solution

3 CH3CH2Br NaOH(aq) alkaline aqueous iodine

30 The two compounds P and Q shown below are important flavours in citrus fruits. These
compounds are commonly used in the food and perfume industries.
CH 3 CH3

CHO CH 2OH

H3C CH 2 H 3C CH 3
P Q

Which statements concerning P and Q are correct?

1 Q can be distinguished from P by using sodium metal.

2 P can be converted to Q using lithium aluminium hydride in dry ether.

3 Reaction of P with hot acidified KMnO4 gives two organic products.

TJC 2010 8872/01/PRELIM 2010 [Turn over


14

Answers

1 D 11 A 21 B

2 B 12 A 22 B

3 D 13 B 23 C

4 D 14 C 24 A

5 A 15 A 25 C

6 B 16 D 26 C

7 D 17 A 27 D

8 A 18 B 28 C

9 C 19 D 29 D

10 C 20 C 30 D

TJC 2010 8872/01/PRELIM 2010 [Turn over


TAMPINES JUNIOR COLLEGE

PreliminaryExaminations2010

CHEMISTRY (Higher 1) 8872/1

PAPER 1 Multiple Choice

Friday 17 September 2010 50 minutes

Additional materials: Multiple Choice Answer Sheet

Data Booklet

INSTRUCTIONS TO CANDIDATES
Write in soft pencil.

Write your name and Civics Group on the Answer Sheet in the spaces provided.

There are thirty questions in this paper. Answer all questions. For each question, there are
four possible answers, A, B, C and D.
Choose the one you consider correct and record your choice in soft pencil on the separate
Answer Sheet.

Each correct answer will score one mark. A mark will not be deducted for a wrong answer.

Any rough working should be done in this booklet.

This document consists of 15 printed pages.


[Turn over

8872_01/1
Section A
For each question there are four possible answers, A, B, C and D. Choose the one
you consider to be correct.

1. 10 cm3 of a hydrocarbon CxHy was exploded with an excess of oxygen. There


was a contraction of 30 cm3. When the product was treated with a solution of
potassium hydroxide, there was a further contraction of 40 cm3. What is the
molecular formula of the hydrocarbon?
[All gas volumes are measured at r.t.p.]

A C4H8
B C4H10
C C3H6
D C3H8

2. Consider the following half-equations.


MnO4- + 8H+ + 5e- Mn2+ + 4H2O
Fe2+ Fe3+ + e-
What volume of 0.01 moldm-3 KMnO4 is required to oxidize 15 cm3 of an
acidified aqueous solution of 0.1 mol dm-3 Fe2+ ions.
A 15 cm3
B 30 cm3
C 45 cm3
D 150 cm3

3. Which of the following statements about the reaction below is correct?


IO3 (aq) + 2I (aq) + 6H+(aq) + 3Cl-(aq) 3ICl(aq) + 3H2O(l)

A The oxidation number of chlorine changes from -1 to 0.


B The oxidation number of chlorine changes from -1 to +1.
C The oxidation number of iodine in IO3- changes from +5 to -3.
D The oxidation number of iodine in IO3- changes from +5 to +1.

8872_01/2
4. The table below give the enthalpy change of combustion of enthyne (C2H2),
hydrogen (H2) and ethane (C2H6).

Substance Hc / kJmol1

C2H2 1300

H2 285

C2H6 1560

What is the enthalpy change of the following reaction?


C2H2(g) + 2H2(g) C2H6(g)
A +310 kJmol1
B +155 kJmol1
C 155 kJmol1
D 310 kJmol1

5. Given the standard enthalpy change of formation of Fe3O4 (s) and Al2O3 (s) is
-1117.1 kJ mol-1 and -1669.8 kJ mol-1 respectively, what is the standard
enthalpy change of reaction for the following reaction?

3 Fe3O4 (s) + 8 Al (s) 9 Fe (s) + 4 Al2O3 (s)

A +3327.9 kJ mol-1
B -3327.9 kJ mol-1
C -1109.3 kJ mol-1
D - 552.7 kJ mol-1

8872_01/3
6. For which of the following reactions does the enthalpy value represent both a
standard enthalpy change of combustion and a standard enthalpy change of
formation?

A CO(g) + O2(g) CO2(g)


B 2C(s) + O2(g) 2CO(g)
C C(s) + O2(g) CO2(g)
D C(s) + 2O(g) CO2(g)

7. Lead is the final product formed by a series of changes in which the rate
determining step is the radioactive decay of uranium-238. This radioactive
decay is a first order reaction with a half life of 4.5 x 109 years.

What would be the age of a rock sample, originally lead free, in which the
molar proportion of uranium to lead is now 1:3?

A 1.5 x 109 years


B 2.25 x 109 years
C 4.5 x 109 years
D 9.0 x 109 years

8. It is often said that the rate of a typical reaction is roughly doubled by raising
the temperature by 10C.

Which of the following best explains this observation?

A Raising the temperature by 10C doubles the number of molecules


having energy more than a certain minimum energy.

B Raising the temperature by 10C doubles the average energy of each


molecule.

C Raising the temperature by 10C doubles the average speed of each


molecule.

D Raising the temperature by 10C doubles the number of molecular


collisions in a given time.

8872_01/4
9. Under which one of the following sets of conditions will the highest yield of
ammonia at equilibrium be obtained from the following reaction?

N2 (g) + 3H2(g) 2 NH3 (g) H = -92 kJ mol-1


Temperature Pressure
A high high
B high low
C low high
D low low

10. Nitrogen dioxide decomposes on heating according to the following equation


2NO2(g) 2NO(g) + O2(g)
When 4 moles of nitrogen dioxide was put into a 1 dm3 container and heated, the
equilibrium mixture contained 0.8 moles of oxygen. What is the numerical value
of the equilibrium constant, Kc, at the temperature of the experiment?

0.8 0.8
A
2.4

0 .8 2 0 .8
B
42

1.6 0.8
C
2.4 2

1.6 2 0.8
D
2.4 2

8872_01/5
11. What is the final pH of the solution formed by mixing equal volumes of two
HCl solutions, one with pH 1.0 and the other with pH 3.0.

A 1.0 B 1.3 C 2.0 D 2.5

12. A titration was carried out between a weak acid, HA, and aqueous sodium
hydroxide. The graph obtained was shown below:

pH

volume of NaOH
A suitable indicator for the above titration is
Indicator pH range
A Thymol blue 1.5 to 2.5
B Bromocresol green 3.8 to 5.5
C Bromothymol blue 6.0 to 7.5
D Thymolphthalein 9.3 to 10.5

13. Silicon carbide is a major industrial abrasive and a refractory material. Which
type of structure explains these properties?

A A simple covalent structure with covalent bonds between silicon and


carbon atoms.
B A layered structure with covalent bonds between silicon and carbon
atoms and weak van der Waals forces between the layers.
C A giant covalent structure with strong covalent bonds between silicon
and carbon atoms forming a 3-dimensional network.
D A giant ionic lattice with strong ionic bonds holding the oppositely
charged ions together.

8872_01/6
14. The boiling points (b.p.) of ammonia, hydrogen chloride and carbon dioxide
are given below.

gas formula b.p. / C

ammonia NH3 33

hydrogen chloride HCl 85

carbon dioxide CO2 78

Which of the following statements best explains the above data?


A NH3 has a higher b.p. than HCl because the hydrogen bonding
between NH3 molecules are stronger than the permanent dipole
permanent dipole interactions between HCl molecules.

B The strength of the induced dipole induced dipole interactions


increases in the order HCl < CO2 < NH3.

C The strength of the permanent dipole permanent dipole interactions


increases in the order HCl < CO2 < NH3.

D CO2 has a higher b.p. than HCl because the permanent dipole
permanent dipole interactions between CO2 molecules are stronger
than the permanent dipole permanent dipole interactions between
HCl molecules.

8872_01/7
15. The ions P3, S2 and Cl have radii 0.242 nm, 0.184 nm and 0.181 nm
respectively. This decrease in radius in going from P3 to Cl is due to

A an increase in the total number of electrons and in the nuclear charge.

B an increase in the total number of electrons with nuclear charge


remaining constant.

C a constant total number of electrons and an increase in nuclear


charge.

D a constant total number of electrons and nuclear charge.

16. Which of the following elements form an oxide with a giant covalent structure,
and a chloride which is readily hydrolysed?

A Magnesium C Phosphorus

B Sodium D Silicon

17. Which of the following oxide is unlikely to dissolve in aqueous sodium


hydroxide?

A MgO B Al2O3 C SiO2 D SO2

18. How many isomers are possible for the unsaturated compound of molecular
formula C3H5F?

A 3 B 4 C 5 D 6

8872_01/8
19. Vitamin A plays a role in many functions throughout the human body such as
vision, gene transcription, bone metabolism and skin health. The diagram
below shows the structure of vitamin A.

What is the total number of cis-trans isomers of this molecule?


A 8 B 10 C 16 D 32

20. The table below shows the rate of hydrolysis of the halogen-containing
compounds P to S. The rate of hydrolysis is measured by the speed at which
the silver halide precipitate is formed.

P Q R S

Cl CH2Cl CH2Br CH2I

No precipitate Precipitate Precipitate formed Precipitate


formed with formed after 10 after 2 minutes of forms
prolonged minutes of warming reaction instantaneously
heating with heating reaction mixture with with ethanolic
ethanolic mixture with ethanolic AgNO3. AgNO3.
AgNO3. ethanolic AgNO3.

Which statement explains the rate of reaction?


A The CX bond length increases from compound P to S.
B The solubility of the compounds increases from P to S.
C The mechanism for the reaction changes from P to S.
D The reaction conditions are increasingly milder from P to S.

8872_01/9
21. A compound has the following properties:

It reacts with phosphorus pentachloride, giving off white fumes of HCl.


It reacts with aqueous bromine.
It reacts with ethanol
It reacts with sodium hydroxide to form an ionic compound.
The compound is
A CH3CH=CHCOOH

B CH3CH=CHCH2OH

C CH3CH=CHCHO

D CH3CH2CH2COOH

22. Which of the following reagents will distinguish between compounds S and T?

O
H3 C O

H3 C OH
OH

S T

A Aqueous bromine

B Alkaline aqueous iodine

C Acidified potassium dichromate (VI)

D 2,4 dinitrophenylhydrazine

8872_01/10
23. Which observation will NOT be obtained with the compound
CH3CH(OH)CH3?

A a pale yellow precipitate when warmed with alkaline aqueous iodine


B an orange precipitate when warmed with 2,4-dinitrophenylhydrazine
C fumes of hydrogen chloride gas when heated with phosphorus
pentachloride

D decolourisation of hot, acidified purple potassium manganate(VII)

24. Pentan-2-one is treated with cold hydrogen cyanide in the presence of traces
of sodium cyanide. The product X of this reaction is then boiled with
hydrochloric acid to give Y. What are X and Y likely to be?

A H OH H H
H OH
H3C C C C CH3 H3 C C C C CH3
H CN H H COOH H

B H H H OH H H H OH
H3 C C C C C H H3C C C C C H
H H H CN H H H COOH

H OH H OH
H3C C C CH3 H3C C C CH3
H CN H COOH

D H H OH
H H OH
H 3C C C C CH3 H 3C C C C CH3
H H CN H H COOH

8872_01/11
25. Experiments are carried out on three compounds X, Y, and Z.

CH3(CH2)16CO2 CH2

CH3(CH2)16CO2 CH BrCH2CH2CH2Br
CH3(CH2)16CO2 CH2 CN

X Y Z

A sample of 0.01 mol of each compound is heated under reflux with 50 cm3
of 1 mol dm-3 NaOH (in excess) until hydrolysis is complete and any
ammonia produced is expelled from solution.

The excess NaOH is then titrated in each case and is found to require
20cm3, 30 cm3 and 40 cm3 of 1 mol dm-3 HCl for neutralisation.

Which sequence of compounds matches these results?

20 cm3 30 cm3 40 cm3


A X Y Z

B X Z Y

C Y Z X

D Z Y X

8872_01/12
Section B
For each of the questions in this section one or more of the three numbered
statements 1 to 3 may be correct.
Decide whether each of the statements is or not correct (you may find it helpful to
put a tick against the statements which you consider to be correct).
The responses A to D should be selected on the basis of
A B C D

1,2 and 3 are 1 and 2 only are 2 and 3 only are 1 only is correct
correct correct correct

No other combination of statements is used as a correct response.

26. The energy profile diagram for the reaction P + Q R is shown below:

Energy

E1

R
E2

P+Q
Reaction coordinate

1 The activation energy of the forward reaction isE2+E1

2 The activation energy of the reverse reaction is E1

3 The enthalpy change of the reverse reaction is E2

27. In which sequence(s) are the molecules quoted in order of increasing bond
angle within the molecule?

1 H2O NH3 CH4


2 H2O SF6 BF3
3 CH4 CO2 SF6

8872_01/13
A B C D

1,2 and 3 are 1 and 2 only are 2 and 3 only are 1 only is correct
correct correct correct

28. Why is a solution of aluminium chloride acidic?


1 Chloride ions react with water to form hydrochloric acid.

2 Aluminium ions have a high charge density.

3 The H-O bonds are weaker in [Al(H2O)6]3+ than in H2O.

29. What deductions can be made about the psychoactive drug, W, in cannabis
as shown in the structure below?

CH2CH2CH2CH2CH3

HO OH
CH2

H2C C CH3
CH3 W

1 W reacts with concentrated sulphuric acid at 180C to form a product


that decolourises aqueous bromine at room temperature.

2 W can form cis-trans isomers

3 W reacts with sodium metal to form 2 moles of hydrogen gas.

8872_01/14
A B C D

1,2 and 3 are 1 and 2 only are 2 and 3 only are 1 only is correct
correct correct correct

30. Which of the following statements about benzene are correct?


1 The carbon atoms form a regular planar hexagon.

2 Addition reactions of benzene take place more easily than substitution.

3 Each carbon-carbon bond in benzene is stronger than a carbon-carbon


double bond in ethene.

---End of Paper---

8872_01/15
TAMPINES JUNIOR COLLEGE
JC 2 PRELIMINARY EXAM
Higher 1

CANDIDATE
NAME

TUTORS
CLASS
NAME

CHEMISTRY 8872/02
Paper 2 14 Sep 2010
Candidates answer Section A on the Question Paper. 2 hours
Additional Materials: Answer Paper
Data Booklet
Graph Paper

READ THESE INSTRUCTIONS FIRST

Write your name and class on all the work you hand in.
Write in dark blue or black pen on both sides of the paper.
You may use a soft pencil for any diagrams, graphs or rough working.
Do not use staples, paper clips, highlighters, glue or correction fluid.

Section A For Examiners


Answer all questions on the Question Paper. Use

Section B
A1
Answer two questions on separate answer paper.

A2
At the end of the examination, fasten all your work securely together.
The number of marks is given in brackets [ ] at the end of each question or part
question. A3

A4

A5

A6

Total
(Sect A)

This document consists of 14 printed pages.


[Turn over
2
Section A For
examiners
Answer all questions in this section in the spaces provided. use only

Question 1 [8 marks]

Gold compounds are present in solution in sea water. It is believed that gold was
present in concentrations as high as 8.00 x 107 g per tonne. (1 tonne = 1000 kg)

About 70 % of the Earths surface is covered by water. The oceans of planet Earth
have an area of about 360 x 106 km2. The average depth of the Earths oceans is
thought to be 3.7 km.

(a) (i) Use the data above to estimate the volume of the earths oceans in dm3.
(1 km3 = 1 x 1012 dm3)

(ii) Calculate the concentration of gold in sea water, in mol dm3, given that the
density of sea water is 1 kg dm3.

(iii) Hence, estimate the total mass of gold thought to be in the oceans.

[3]
3
1(b) Gold can also be extracted from low-grade ore around the world by use of the For
examiners
Elsner Reaction. use only

4 Au + 8 NaCN + O2 + 2H2O 4NaAu(CN)2 + 4NaOH

In this process, the powdered ore is combined with a dilute aqueous solution of
sodium cyanide and air is bubbled through it. The NaAu(CN)2 compound is
soluble in water and the gold is then displaced by adding zinc dust to precipitate
the gold.

(i) What is the change in oxidation state of gold in the Elsner Reaction?

________________________________________________________________

(ii) Suggest why the gold can be precipitated out of the NaAu(CN)2 compound by
zinc.

___________________________________________

___________________________________________
[2]

(c) When 1.00 g of impure gold ore was reacted with aqueous sodium cyanide, the
amount of sodium hydroxide produced from the Elsner reaction was neutralized
with 20 cm3 of 0.05 mol dm3 hydrochloric acid present in excess.

The excess acid was then titrated with standard 0.02 mol dm3 sodium hydroxide
and 29.70 cm3 of base is required to reach end-point.

Find the percentage by mass of gold in the impure gold ore.

[3]
4
Question 2 [4 marks] For
examiners
The following table shows Ka values for three organic acids. use only

Acid Ka / mol dm3

CH3COOH 1.75 x 105

CH2ClCOOH 1.36 x 103

CHCl2COOH 5.53 x 102

(i) State and explain the trend in Ka values for the three acids. [2]

___________________________________________

___________________________________________

___________________________________________

___________________________________________

(ii) How would you expect the acidity of benzoic acid to compare with that of the
ethanoic acid? Explain. [2]

___________________________________________

___________________________________________

___________________________________________

___________________________________________

___________________________________________

___________________________________________
5
Question 3 [4 marks] For
examiners
use only
A student used the apparatus below to determine the enthalpy change of combustion of
propan-1-ol. It was found that 0.60 g of propan-1-ol was used to raise the temperature
of 200 g of the water. The initial temperature of the water was 21.0C. The specific heat
capacity of water is 4.18 J g-1 K-1.

thermometer

copper can
containing
200 g of
water
burner containing
propan-1-ol
(i) Calculate the number of moles of propan-1-ol burnt. [1]

(ii) Given that the enthalpy of combustion of propan-1-ol is -2021 kJ mol-1, calculate
the final temperature of water at the end of the experiment. [2]

(iii) The actual temperature of the water measured at the end of experiment is 38.0C.
Give an explanation for the difference. [1]
6
Question 4 [5 marks] For
examiners
use only
Aluminium oxide reacts with chlorine trifluoride, ClF3, according to the following
equation:

4ClF3(g) + 2Al2O3(s) 4AlF3(s) + 2Cl2(g) + 3O2(g) Hr = -1420 kJ mol-1

(i) Write the equation which corresponds to the standard enthalpy change of
formation of chlorine trifluoride. [1]

__________________________________________________________________

(ii) Some standard enthalpy changes are given below:

Enthalpy change of formation of AlF3(s) 1350 kJ mol-1

Enthalpy change of combustion of Al(s) 840 kJ mol-1

Using the data given in the question, calculate the enthalpy change of formation
of chlorine trifluoride. [2]

(iii) Using your answer in (ii) and relevant data from the Data Booklet, estimate the
average bond energy of the Cl-F bond. [2]
7
Question 5 [7 marks] For
examiners
use only
For the reaction scheme below, give the reagents and conditions for steps I to III in the
table provided, and draw the structural formulae of B to E in the boxes provided. [7]

CH3
COOH
Oxidation NC C OH
COOH
CH3

Step I Step III


B

C9H10 H2SO4,
CH(OH)CH3
K2Cr2O7,
CH3
heat
Step II
A C

C9H12O C9H10O
I2, NaOH,
warm
I2, NaOH,
warm

2,4DNPH

C8H7O2Na

Reagents & Conditions

Step I

Step II
D
Step III
C15H14N4O4
8
Question 6 [12 marks] For
examiners
use only

(a) Aluminium, silicon, and phosphorus are three elements from period 3 of the
Periodic Table. Describe and explain very briefly the differences in melting point
of these three elements in terms of their structures and bonding.
[6]

___________________________________________

___________________________________________

___________________________________________

___________________________________________

___________________________________________

___________________________________________

(b) Aluminium, silicon, and phosphorus combine directly with chlorine to form
chlorides. Write balanced equations for the reactions, if any, of these chlorides
with water. [3]

___________________________________________

___________________________________________

___________________________________________

___________________________________________

___________________________________________
9
For
examiners
6(c) Construct dotand-cross diagrams to illustrate the bonding in the molecules of use only

NH3, NH2 and NH4+, and predict their shapes and bond angles.
Present your answers in the following table. [3]

Shape Bond Dot-and-Cross Diagram


Angle
(i) NH3

(ii) NH2

(iii) NH4+
10

Section B
Answer two of the following three questions.
Answer these questions on separate answer paper.

Question 1 [20 marks]

(a) Compound X undergoes hydration and the rate of hydration was followed by
measuring the change in concentration of Compound X with time.

Two experiments were carried starting with different concentrations of H+. The
following results were obtained.

Time/ Expt 1, with [H+] = 0.10 mol dm-3 Expt 2, with [H+] = 0.05 mol dm-3
minutes [Compound X]/ mol dm-3 [Compound X]/ mol dm-3
0 0.0050 0.0050
15 0.0040 0.0045
30 0.0032 0.0040
45 0.0026 0.0036
60 0.0021 0.0032
75 0.0017 0.0029
90 0.0014 0.0026

(i) Using the same axes, plot graphs of [Compound X] against time for the two
experiments.

(ii) Use your graphs to determine the order of reaction with respect to [H+] and
to [Compound X], showing your working clearly.

Hence give the rate equation for the reaction.

(iii) Calculate the initial rate from experiment 1, and use it, together with your
rate equation, to calculate the rate constant for the reaction, including units.
[8]
11
Cont Q1
(b) Suggest simple chemical tests by which the following pairs of compounds can be
distinguished from each other. You should state the reagents and conditions for
each test and the observations that would distinguish one compound from the
other.

(i)
H CH3

H3C-HC=CH-CH2-C-CH3 (CH3)2C=CH-C-CH3
OH OH

(ii)

[4]

(c) Write down the balanced chemical equations, reagents and essential conditions
for each of the conversions suggested.

(i) ethene to cyanoethane (C2H5CN) [4]


(ii) 2-bromobutane to 2-methylbutanoic acid [4]
12
Question 2 [20 marks]
(a) An organic compound D, C10H11O2Br, does not react with sodium
hydrogencarbonate solution but reacts slowly on heating in sodium hydroxide to
form a water-soluble substance E, C3H5O3Na and an insoluble compound F,
C7H8O.
F on oxidation gives benzoic acid. E with sodium produces hydrogen and G,
C3H4O3Na2. E also reacts with hot alkaline aqueous iodine to give a yellow
precipitate.
Deduce the structural formulae of D to G, explaining clearly your reasoning for all
the reactions described above. [8]

(b) Lemon juice contains citric acid, a weak monobasic acid which dissociates
incompletely in water:

C5H7O4COOH (aq) C5H7O4COO- (aq) + H+ (aq)

(i) The pH of lemon juice is 2.11. Calculate the molar concentration of


hydrogen ions, [H+] in lemon juice.

(ii) The concentration of citric acid in lemon juice is determined by titrating


20.0 cm3 of the solution with 0.150 mol dm-3 NaOH. The end-point was
reached when 33.30 cm3 of NaOH was added.
Calculate the concentration of citric acid in lemon juice.

(iii) Explain the difference in the results obtained in parts (i) and (ii).

(iv) Suggest, with reason, a suitable indicator for the titration.

(v) Write an expression for the acid dissociation constant, Ka, of citric acid.

(vi) Citric acid and sodium citrate constitute a buffer system.


With the aid of suitable equations explain how such a system works
I. when small amounts of alkali are added
II. when small amounts of acid are added.
[8]

(c) The oxides Al2O3, SiO2 and P4O10 differ considerably in their chemical properties:
the first react with acids whereas all three react with alkalis.

Write balanced equations for all the reactions involved. [4]


13
Question 3 [20 marks]
(a) Under different conditions, acidified potassium dichromate(VI) reacts with A,
C4H8O, to yield B or C. When treated with hot concentrated acidified potassium
manganate(VII), both B and C give the same organic product CH3CO2H and
carbon dioxide gas. However, only B and not C will undergo reaction with 2,4-
dinitrophenylhydrazine.

(i) Deduce the structures of A to C.

(ii) What type of isomerism is shown by A? Illustrate your answer with suitable
diagrams.
[6]

(b) 0.50 g of hydrated iron (II) sulfate (FeSO4.7H2O) was dissolved in dilute H2SO4
and titrated with 0.02 mol dm-3 potassium manganate (VII). What volume of
potassium manganate (VII) was required to complete the titration?
[3]

(c) Enthalpy change of reaction 1, H1, can be determined using enthalpy change of
reaction 2, H2, and enthalpy change of reaction 3, H3, in the energy cycle
below.

Na2O(s) + H2O(l) +2HCl(aq) 2NaOH(aq) + 2HCl(aq)


H1

H2 H3

2NaCl(aq) + 2H2O(l)
(i) H2 can be calculated from the reaction between Na2O(s) and HCl(aq).
When 6.2 g of Na2O(s) is dissolved in 250 cm3 of 1.0 mol dm-3 HCl(aq), the
temperature of the solution rose by 17 oC. Calculate H2.
[Specific heat capacity of water is 4.18 J g-1 K-1]

(ii) The enthalpy change of neutralisation between NaOH(aq) and HCl(aq) is


known to be - 57.3 kJ mol-1. Define the term enthalpy change of
neutralisation. Hence, calculate H3.

(iii) Using your answer in (i) and (ii), calculate H1.


[6]
14
Cont Q3
(d) For MnO4- ions to act as an oxidizing agent, it has to be acidified with sulfuric
acid. Sulfuric acid is manufactured by the Contact Process which involves the
following reaction:

2SO2(g) + O2(g) 2SO3(g)

The percentage of SO3 obtainable at equilibrium is plotted against the operating


temperature.
% SO3 at equilibrium

100

0
Temperature/ oC

(i) Explain whether the formation of SO3 is an exothermic or endothermic


process.

(ii) Write an expression for the equilibrium constant, Kc.

(iii) Given that in a 3 dm3 vessel, there are 0.2 moles of SO2, 0.3 moles of O2
and 4 moles of SO3 at equilibrium. Calculate the Kc value.
[5]

-- End of Paper --
VICTORIA JUNIOR COLLEGE
JC 2 PRELIMINARY EXAMINATIONS
Higher 1

CHEMISTRY 8872/01
Paper 1 Multiple Choice 24 September 2010

50 mins

Additional Materials: Multiple Choice Answer Sheet


Data Booklet

READ THESE INSTRUCTIONS FIRST

Write in soft pencil.


Do not use staples, paper clips, highlighters, glue or correction fluid.
Write your NRIC/FIN number, name and CT group on the Answer Sheet.

There are thirty questions. Answer all questions. For each question there are four possible
answers A, B, C and D.
Choose the one you consider correct and record your choices in soft pencil on the separate
Answer Sheet.

Read the instructions on the Answer Sheet very carefully.

Each correct answer will score one mark. A mark will not be deducted for a wrong answer.
Any rough working should be done in this booklet.

This document consists of 13 printed pages and 1 blank page.

VJC 2010 8872/01/PRELIM/10 [Turn over


2

1 Which one of the following has the same number of particles as one mole of copper
atoms?

A The number of ions in 2 dm3 of 0.25 mol dm3 of aqueous nitric acid
B The number of delocalised electrons in one mole of magnesium metal
C The number of atoms in 71 g of chlorine gas
D The number of ions in 58.5 g of sodium chloride

2 Arsenic, As (Ar = 74.9), is a poison commonly used in pesticides. To determine the


mass of arsenic present in a sample of pesticide, all the arsenic was first converted to
arsenate ion, AsO43-. 1.25 10-3 mol of AgNO3 was then added to precipitate AsO43- as
Ag3AsO4. The excess Ag+ ions needed 18.2 cm3 of 0.0108 mol dm-3 KSCN to form
silver thiocyanate, AgSCN.

What is the mass of arsenic present in the sample of pesticide?

A 0.015 g C 0.031 g
B 0.026 g D 0.079 g

3 Use of the Data Booklet is relevant to this question.

What do the ions 36S2- and 37Cl- have in common?

A Both ions have more electrons than neutrons.

B Both ions contain the same number of nucleons in their nuclei.

C Both ions have an outer electronic configuration 2s2 2p6.

D Both ions have 20 neutrons in their nuclei.

4 Consider the table below.

Particle Number of neutrons Number of nucleons


Q- 16 33
R+ 19 39
S2- 17 33
T2+ 18 35

Which of the following pairs consists of particles that are isoelectronic?

A Q- and S2- C S2- and T2+


B R+ and S2- D Q- and T2+

VJC 2010 8872/01/PRELIM/10


3

5 Elements L and M have the following successive ionisation energies in kJ mol-1.

1st 2nd 3rd 4th 5th 6th 7th


L 580 1800 2700 11600 14800 18400 23300

M 1310 3400 5300 7500 11000 13300 20300

What could be the formula of the compound formed by these two elements?

A LM4 C L3M2
B L2M3 D L4M

6 Which of the following species has a trigonal planar shape?

A CO32- C NH3
B SCl2 D BrF2-

7 Which one of the following statements about the properties of graphite is incorrect?

A It can be used as a lubricant because of the weak forces of attraction between


layers of atoms.

B It is a good conductor of electricity in the direction parallel to each layer.

C The carbon to carbon distance between different layers is the same as that
within each plane.

D Each carbon atom is bonded to other carbon atoms in a trigonal planar


arrangement.

8 Use of the Data Booklet is relevant to this question.

An energy cycle is given below.

75 kJ mol-1
C(graphite) + 2H2(g) CH4(g)

H
C(g) + 2H2(g)

C(g) + 4H(g)

What is the value of H in kJ mol-1?

A +350 B +693 C +843 D +1130


VJC 2010 8872/01/PRELIM/10 [Turn over
4

9 A student used the apparatus below to heat a can containing 300 g of water.

thermometer
can containing
300 g of water

burner containing
propan-1-ol

The following data were recorded:


Mass of propan-1-ol burnt = mg
Change in temperature of water = T oC

You are given that:


Relative molecular mass of propan-1-ol = 60.0
Enthalpy change of combustion of propan-1-ol = 2021 kJ mol1
Specific heat capacity of water = c J g1 K1

What is the efficiency of this heating process?

m 2021 1000 300 c T 60.0


A 100% C 100%
300 c T 60.0 m 2021

m c T 60.0 300 c T 60.0


B 100% D 100%
300 2021 1000 m 2021 1000

10 Solutions 1 and 2 contain different concentrations of a monobasic acid, X. For each


solution, the pH was measured and a 10.0 cm3 portion was titrated with 0.100 mol dm-3
NaOH solution. The results are given in the table below:

pH of solution Volume of NaOH


needed for titration / cm3
Solution 1 2.9 10
Solution 2 2.7 20

Which of the following can be deduced from the given information?

A The concentration of H+ in solution 2 is twice that in solution 1.

B Methyl orange can be used to detect the endpoint of each titration.

C A mixture of X and its salt constitutes a buffer solution.

D More NaOH is required for neutralisation if X is replaced by HCl.

VJC 2010 8872/01/PRELIM/10


5

11 Azomethane decomposes when heated according to the following equation.

CH3N=NCH3(g) CH3CH3(g) + N2(g)

This reaction is first order with respect to azomethane with a half-life of 20 min.

In one experiment, 16 cm3 of azomethane was allowed to decompose under constant


temperature and pressure. What is the time taken for the volume of the reaction
mixture to reach 30 cm3?

A 20 min B 37.5 min C 40 min D 60 min

12 Two gases A and B react according to: A(g) + 3B(g) AB3(g)

The following results were obtained at 350 K.

Rate of formation of AB3


Experiment [A] / mol dm-3 [B] / mol dm-3
/ mol dm-3 min-1
1 0.100 0.100 0.00200
2 0.100 0.200 0.00800
3 0.300 0.300 0.01800

What is the rate equation for this reaction?

A Rate = k[A] C Rate = k[B]2


B Rate = k[B] D Rate = k[A][B]3

13 Two unlabelled beakers containing equal volumes of 0.5 mol dm-3 hydrochloric acid
and 0.5 mol dm-3 ethanoic acid were given.

Which statement is correct?

A The enthalpy change of neutralisation for the two solutions is the same.
B The rate of reaction with Mg ribbon is the same.
C The volume of H2 collected when reacted with Mg is the same.
D The initial pH of the two acids is the same.

14 Which statement about indicators is always correct?

A The pKa value of the indicator is within its pH range.


B The pH range is greater for indicators with higher pKa values.
C The colour red indicates an acidic solution.
D The mid-point of the pH range of an indicator is 7.

VJC 2010 8872/01/PRELIM/10 [Turn over


6

15 The graphs below show the variation of the percentage of gaseous products present
at equlibrium, with temperature and pressure.

% products (T + 10) oC
at
equilibrium
T oC

pressure

Which one of the following systems could the graphs represent?

A N2(g) + 3H2(g) 2NH3(g) H = 92 kJ mol1

B 3O2(g) + 4NH3(g) 2N2(g) + 6H2O(g) H = 1248 kJ mol1

C 2N2(g) + O2(g) 2N2O (g) H = +82 kJ mol1

D CO2(g) + C(s) 2CO (g) H = +173 kJ mol1

16 The chloride of an element R of the third period is a liquid which has a boiling point of
76 oC and fumes in air.

VJC 2010 8872/01/PRELIM/10


7

After mixing 0.010 mol of the chloride with water, the resulting solution required 0.030
mol of silver nitrate for complete precipitation of the chloride ion.

To which Group of the Periodic Table does R belong?

A V B IV C III D II

17 Beryllium resembles aluminium in its chemical properties.

Which property of beryllium compounds is unlikely to be correct?

A Beryllium oxide can undergo reaction with aqueous sodium hydroxide.


B Beryllium chloride can form the dimer Be2Cl4.
C Beryllium chloride is an ionic compound.
D Beryllium chloride dissolves in water to give an acidic solution.

18 COCH 3 CH 3
O

CH=CHCH 3 COCH 3

compound A compound B

Which of the following reagents will distinguish compound A from compound B?

A hot acidified KMnO4 C ethanolic KCN


B aqueous bromine D Fehlings solution

19 Which of the following 2-stage processes will not yield the final product as shown?
-O C -O C
NC 2 2

A OH- PCl5
Br OH Cl
reflux rt

HO HO Cl

NC NC HO2 C

excess conc. H2SO4 H+ / MnO4-


Br Br Br
B heat reflux

CO2 H
VJC 2010 HO 8872/01/PRELIM/10 [Turn over
8

NC HO 2C
NC

Na HCl (aq)
C Br
rt reflux
O O
HO

CH 2NH 2
NC
H
N

D H2 / Pt in a sealed tube
Br Br
rt heat

HO HO
Br

VJC 2010 8872/01/PRELIM/10


9

20 Which of the following compounds react with hot acidified KMnO4 and the resultant
product formed will give a positive test with both 2,4-dinitrophenylhydrazine and PCl5?

21 Why does the reaction C2H5X + OH C2H5OH + X occur faster if X is iodine


instead of bromine?

A The CBr bond is stronger than the CI bond.

B The CBr bond is more polar than the CI bond.

C The I ion is less hydrated in water than the Br ion.

D The overlapping of CBr orbitals is less effective than that of CI orbitals.

22 Compound R has the structural formula shown below.

OH Br
O
HO
OH

Which of the following reagents will not undergo any reaction with compound R?

A aqueous sodium carbonate C ethanolic ammonia


B phosphorus pentachloride D Bradys reagent

VJC 2010 8872/01/PRELIM/10 [Turn over


10

23 Compound M can be synthesised as shown below.

HO OH
Compound M

I II III
HOCH2 CH2CH=CH2 M

What are the reagents used for steps I, II and III?

I II III
1. concentrated H2SO4
A NaOH(aq), I2(aq) HCl(aq)
2. H2O
1. concentrated H2SO4
B OH/KMnO4(aq) HCl(aq)
2. H2O
C H+/KMnO4(aq) PCl5 NaOH(aq)
D H+/KMnO4(aq) PCl5 H2O

24 Acarol is sold as an insecticide for use on fruit and vegetables.

OH O
Br C C O CH(CH3)2

Br
Acarol

The final stage in its manufacture is an esterification.

Which alcohol is used to form the ester? OH

A di(4bromophenyl)methanol, Br C Br

B ethanol H

C propan1ol

D propan2ol

VJC 2010 8872/01/PRELIM/10


11

25 One industrial preparation of ethanoic acid is the direct carbonylation of methanol using
a rhodium catalyst.

rhodium
CH3OH + CO CH3CO2H
catalyst

CO2H
Which compound could be used to produce HC CH2CO2H by this method?
CH2CO2H

OH
A HC CH2CO2H
CO2H

CO2H
B HC CO2H
CH2OH

OH
C HC CH2OH
CH2OH

CH2OH
D HC CH2OH
CH2OH

VJC 2010 8872/01/PRELIM/10 [Turn over


12

Section B

For each of the questions in this section, one or more of the three numbered statements 1 to
3 may be correct.

Decide whether each of the statements is or is not correct (you may find it helpful to put a
tick against the statements that you consider to be correct).

The responses A to D should be selected on the basis of

A B C D

1, 2 and 3 1 and 2 2 and 3 1 only


are only are only are is
correct correct correct correct

No other combination of statements is used as a correct response.

26 Carbon disulphide, CS2, is a volatile flammable liquid used in the manufacture of


cellophane. On combustion, CS2 is oxidised as follows.

CS2(g) + 3O2(g) CO2(g) + 2SO2(g)

Which statements are correct?

1 When 20 cm3 of CS2 is reacted with 100 cm3 of oxygen, 60% of the final gas
volume will dissolve in aqueous alkali.
2 38.1 g of CS2 will react exactly with 33.6 dm3 of O2 under standard
temperature and pressure.
3 When 30 cm3 of CS2 is reacted with 60 cm3 of O2, 30 cm3 of CO2 will be
produced.

27 Which trends across the third period (Na to S) are always true?

1 The bonding in the oxide changes from ionic to covalent.

2 The first ionisation energy generally increases.

3 The ionic radius decreases.

VJC 2010 8872/01/PRELIM/10


13

The responses A to D should be selected on the basis of

A B C D

1, 2 and 3 1 and 2 2 and 3 1 only


are only are only are is
correct correct correct correct

No other combination of statements is used as a correct response.

kf
28 The diagram shows the energy profile of a reaction, reactants products
kb
where kf denotes the rate constant of the forward reaction and kb denotes the rate
constant of the backward reaction.

Potential
energy

Ea
products

reactants

Reaction profile

Which statements are correct?

1 kf and kb will increase in the presence of a catalyst.

2 Heating the equilibrium mixture will increase kf but decrease kb.

3 kf will increase when the concentrations of reactants are increased.

29 Which compounds could undergo elimination reaction when treated with hot ethanolic
sodium hydroxide?

1 (CH3)3CBr

2 (CH3)2CHBr

3 (CH3)3CCH2Br

VJC 2010 8872/01/PRELIM/10 [Turn over


14

30 Which reactions would produce benzene1,3dicarboxylic acid?

CN

1 + aqueous H2SO4, reflux

CO2CH3

CH=CHCH3

2 + hot acidified potassium manganate(VII)

CO2CH3

CH(OH)CH3

3 + akaline aqueous iodine followed by acidification

COCH3

2010 Prelim H1 P1 Chemistry Answers

Paper 1: Multiple Choice

1 A 2 B 3 D 4 A 5 B
6 A 7 C 8 B 9 D 10 C
11 D 12 C 13 C 14 A 15 D
16 A 17 C 18 B 19 D 20 C
21 A 22 D 23 A 24 D 25 C
26 B 27 B 28 D 29 B 30 A

VJC 2010 8872/01/PRELIM/10


15

BLANK PAGE

VJC 2010 8872/01/PRELIM/10 [Turn over


2010 H1 CHEMISTRY PRELIMINARY EXAMINATION PAPER 2 ANSWERS

Section A

1(a)
Melting Point
Si

Al
Mg

Na S
P
Cl
Ar

Na, Mg, Al:


High melting point due to strong metallic bond.
Melting point increases from Na to Al due to increase in valence electrons.

Si:
High melting point due to giant molecular structure.
It involves breaking of strong covalent bonds.

P4, S8, Cl2, Ar:


Low melting point due to weak dispersion forces between molecules/atoms.
Dispersion forces depends on the number of electrons in molecules. Hence,
melting point decreases in the order S8 > P4 > Cl2 > Ar.

(b)(i)
1s2 2s2 2p6 3s2 3p4

(ii)
y y

3pz
x x

3px
3s 3py
z z

(iii)
E is sulfur.
First I.E. of E is lower than that of D as less energy is required to remove the 3p4
electron which experiences interelectronic repulsion.

1
(c)(i)
6Fe2+ + Cr2O72 + 14H+ 6Fe3+ + 2Cr3+ + 7H2O

(ii)
No. of moles of Fe2+ used
= 0.050 x 40.0 x 103
= 2.00 x 103 mol

From equation, no. of moles of Cr2O72


= 2.00 x 103 / 6
= 3.33 x 104 mol

Total no. of moles of Cr2O72


= 0.10 x 50.0 x 103
= 5.00 x 103 mol

No. of moles of Cr2O72 reacted with 100 cm3 of polluted water sample
= 5.00 x 103 3.33 x 104
= 4.67 x 103 mol

(iii)
No. of moles of Cr2O72 reacted with 1 dm3 of polluted water sample
= 4.67 x 103 x 10
= 0.0467 mol (allow ecf)

(iv)
O2 + 4H+ + 4e 2H2O
Cr2O72 + 14H+ + 6e 2Cr3+ + 7H2O

Equivalent no. of moles of O2


6
= x 0.0467
4
= 0.0700 mol

(v)
Cr2O72 + 14H+ + 6e 2Cr3+ + 7H2O
C6H12O6 + 6H2O 6CO2 + 24H+ + 24e
4Cr2O72 + 32H+ + C6H12O6 8Cr3+ + 22H2O + 6CO2

2(a)(i)
Heat evolved
100
= 200 x 4.18 x 46.9 x [ m if missing 100/80]
80
= 49000 J or 49.0 kJ

Amount of cyclobutane used


1.00
=
12.0 x 4 + 1.0 x 8
= 0.0179 mol

2
Hc(cyclobutane)
49.0
=
0.0179
= 2740 kJ mol1

(ii)
H1 = Hc(cyclobutane) 2Hc(CH2=CH2)
= 2740 2(1422) (ecf)
= +104 kJ mol1

(b)(i)
The heat change when one mole of ethanol is formed from its constituent
elements in their standard states at 298 K and 1 atm.

(ii)
H = BE(reactants) BE(products)
= 3(436) + (496) (350 + 5 x 410 + 360 + 460)
= 1664 kJ mol1

(iii)
276 kJ mol1
2C(s) + 3H2(g) + O2(g) CH3CH2OH(l)

2Hat +39 kJ mol1


1664 kJ mol1
2C(g) + 3H2(g) + O2(g) CH3CH2OH(g)

By Hess Law,
2Hat = 276 + 39 (1664)
= +1427 kJ
Hat = +713.5 kJ mol1

(c)(i)
Step 1
Reagents: H+ / K2Cr2O7
Conditions: distil

Step 2
Reagents: HCN, NaCN catalyst
Conditions: room temperature

(ii)
B: CH3CH(OH)CO2H

C: CH3CH(OH)CO2CH2CH3

3
3(a)(i)(I)
Reagents: HCN, NaCN catalyst
Conditions: room temperature

(II)
Reagents: alkaline KMnO4
Conditions: room temperature

(ii)
Geometric isomerism
H H H CHO
C C C C
CHO H


cis trans

(iii)
An orange precipitate is seen.
H
NO2
N
OH O NHNH2 OH N
NO2 NO2
CH3 CH3
+

+ H2 O
OH NO2 OH

(iv)
Reagents: alkaline aqueous I2
Conditions: warm (< 70 oC)
Observation: Yellow ppt (CHI3) formed for acetylhydroquinone and no yellow ppt
formed for vanillin.

[OR Tollens reagent, heat, Ag mirror formed for vanillin only (no reaction with
Fehlings solution)]

(b)(I)
OH

HO
CH2CH2OH
+
Na O2CCH2CH=C(CH3)2

4
(II)
O

Cl CH2CO2H
CH2(CO2H)2
(CH3)2CO

Section B

4a(i)
[CH3OH]
Kc =
[CO][H2 ]2

(ii)
CO(g) + 2H2(g) CH3OH(g)
Initial / mol 2 6 0
Change / mol 1.7 3.4 1.7
Equilibrium / mol 0.3 2.6 1.7

(iii)
1.7/4
Kc = (ecf)
(0.3/4)(2. 6/4) 2
= 13.4 mol2 dm6 (ecf)

(iv)
rate

forward reaction

backward reaction

0 time

forward reaction
backward reaction

(b)(i)
When temperature increases, the equilibrium amount of CH3OH decreases.
The backward reaction is favoured.
By Le Chateliers Principle, the backward reaction is endothermic.
Hence, the sign of H of the forward reaction is negative.

5
(ii)
Too low a temperature will result in a slow reaction rate,
rendering the process economically unviable / not cost effective.

(iii)
Addition of a catalyst.

(c)(i)
An acid is a proton donor.
A base is a proton acceptor.

(ii)
HClO + H2O ClO + H3O+

(iii)
[H3O + ][ClO ]
Ka =
[HClO]

(iv)
Ka = (2.50 x 105)2 / 0.0200
= 3.13 x 108 mol dm3

(d)(i)
A buffer solution resists pH change when a small amount of acid or base is added
to it.

(ii)
Solution contains a high concentration of HClO and ClO.
When a small amount of acid is added, ClO neutralises it.
ClO + H+ HClO
When a small amount of base is added, HClO neutralises it.
HClO + OH ClO + H2O

5(a)
A undergoes addition reaction with Br2 in CCl4.
A contains one C=C double bond since it reacts with Br2 in a 1:1 ratio.

A undergoes mild oxidation with cold alkaline KMnO4 to give B, a diol.


Further oxidation of B gives C which has a carboxylic acid group as C can react
with Na2CO3.
B contains a 1o alcohol.
C is not a carbonyl compound as it does not react with 2,4DNPH.
B also contains a 3o alcohol.

A undergoes oxidative cleavage with hot acidified KMnO4


to form a single organic product with one less carbon.
Hence, A is a terminal alkene.

6
A is .

OH
CH2OH

B is .

OH
COOH

C is .

D undergoes addition
to give cyanohydrin E.
D is a ketone.
O

D is .

OH
CN

E is where it undergoes acidic hydrolysis will give C.

A undergoes Wacker Process to give F.


F contains an aldehyde group since it gives a positive test with Tollens reagent.
F does not contain an alkene group since it gives a negative test with Br2(aq).
CHO

F is .

(b)(i)
Comparing expts 3 and 4, (OR When [Br] is doubled, rate is doubled.)
order wrt Br = 1.

Comparing expts 2 and 3 after making total volume the same,


when [BrO3] is doubled, rate is doubled.
Order wrt BrO3 = 1.
Comparing expts 1 and 2,

7
1 a
2 25 120
= +
where a is the order with respect to H .
1 50 60
a=2
Order wrt H+ = 2.

Rate = k[Br][BrO3][H+]2

(ii)

[Br]

t1/2 constant

time

Graph 2:
Show twice [Br] on the yaxis.
Show two constant halflife.

(iii)
number of T1
molecules
with a given
energy

Ea Ea energy

At the same temperature T1, a catalyst provides an alternative reaction pathway


such that the new activation energy Ea is lower.
There is a larger number of molecules with energy Ea.
Hence, number of effective collisions per unit time increases and
rate increases.

8
6(a)(i)
_
+
Na Cl x

xx
x Cl x
x x
x

xx Al x xx x
x Cl x
x Cl x
xx xx

(ii)
Dative bond is formed where lone pair of electrons on N is donated into the empty
orbital of Al.
CH3
CH3 CH3
N

Al
Cl Cl
Cl

(iii)
Bond angle around N atom in (CH3)3N: 107o
Bond angle around N atom in the product: 109.5o (or 109o)

(iv)
MgCl2 has giant ionic structure with ionic bonding between Mg2+ and Cl ions.
PCl3 has simple molecular structure with dipoledipole attractions between
molecules.
As ionic bonding is stronger than dipoledipole attractions, the melting point of
MgCl2 is higher than PCl3.
PCl3 has covalent bonding instead of ionic bonding due to the low electronegativity
difference between P and Cl atoms. Thus, sharing of electrons occurs. [OR PCl3
has covalent bonding instead of ionic bonding as P has high electronegativity.
Thus, sharing of electrons occurs.]

OR
MgCl2 has ionic bonding instead of covalent bonding due to the high
electronegativity difference between Mg and Cl atoms. Thus, transferring of
electrons occurs. [OR MgCl2 has ionic bonding instead of covalent bonding as Mg
has low electronegativity. Thus, transferring of electrons occurs.]

(v)
MgCl2 is an electrical conductor when molten due to presence of mobile ions while
PCl3 is not due to absence of mobile ions when molten.
[N.B. Both MgCl2 and PCl3 are able to conduct electricity in aqueous solution due
to presence of mobile ions.]

9
(b)(i)
Y: Al
Z: P

The oxide of Y reacts with both NaOH and HCl. Hence, it is an amphoteric oxide.
Al2O3(s) + 6HCl(aq) 2AlCl3(aq) + 3H2O(l)
Al2O3(s) + 2NaOH(aq) + 3H2O(l) 2NaAl(OH)4(aq)
The chloride of Y undergoes hydration and hydrolysis
to form an acidic solution. Hence, it reacts with NaOH.
AlCl3(s) + 6H2O(l) [Al(H2O)6]3+(aq) + 3Cl(aq)
[Al(H2O)6]3+(aq) + H2O(l) [Al(H2O)5(OH)]2+(aq) + H3O+(aq)

Phosphorus chloride and phosphorus oxide will react with water but silicon dioxide
is insoluble in water.
PCl3(l) + 3H2O(l) H3PO3(aq) + 3HCl(aq)
OR PCl5(s)+ 4H2O(l) H3PO4(aq) + 4HCl(aq)
P4O6(s) + 6H2O(l) 4H3PO3(aq)
OR P4O10(s) + 6H2O(l) 4H3PO4(aq)

(ii)
Both the chloride and oxide of Z react with water to form solution with pH = 2.

(c)
CN CN
HCN PCl5
CH3 C CH3 CH3 C CH3 CH3 C CH3
NaCN room temp.
O catalyst OH Cl
room temp.
HCl(aq)
heat

CO2H
CH3 C CH3
Cl
[Steps 2 and 3 can be interchanged.]

10
YISHUN JUNIOR COLLEGE
JC2 PRELIMINARY EXAMINATION 2010

CHEMISTRY 8872/01

HIGHER 1 24 August 2010


50 minutes
Paper 1 Multiple Choice

Additional materials: Multiple choice Answer Sheet

Data Booklet

YISHUN JUNIOR COLLEGE YISHUN JUNIOR COLLEGE YISHUN JUNIOR COLLEGE YISHUN JUNIOR COLLEGE YISHUN JUNIOR COLLEGE
YISHUN JUNIOR COLLEGE YISHUN JUNIOR COLLEGE YISHUN JUNIOR COLLEGE YISHUN JUNIOR COLLEGE YISHUN JUNIOR COLLEGE
YISHUN JUNIOR COLLEGE YISHUN JUNIOR COLLEGE YISHUN JUNIOR COLLEGE YISHUN JUNIOR COLLEGE YISHUN JUNIOR COLLEGE
YISHUN JUNIOR COLLEGE YISHUN JUNIOR COLLEGE YISHUN JUNIOR COLLEGE YISHUN JUNIOR COLLEGE YISHUN JUNIOR COLLEGE
YISHUN JUNIOR COLLEGE YISHUN JUNIOR COLLEGE YISHUN JUNIOR COLLEGE YISHUN JUNIOR COLLEGE YISHUN JUNIOR COLLEGE
YISHUN JUNIOR COLLEGE YISHUN JUNIOR COLLEGE YISHUN JUNIOR COLLEGE YISHUN JUNIOR COLLEGE YISHUN JUNIOR COLLEGE
YISHUN JUNIOR COLLEGE YISHUN JUNIOR COLLEGE YISHUN JUNIOR COLLEGE YISHUN JUNIOR COLLEGE YISHUN JUNIOR COLLEGE
YISHUN JUNIOR COLLEGE YISHUN JUNIOR COLLEGE YISHUN JUNIOR COLLEGE YISHUN JUNIOR COLLEGE YISHUN JUNIOR COLLEGE
YISHUN JUNIOR COLLEGE YISHUN JUNIOR COLLEGE YISHUN JUNIOR COLLEGE YISHUN JUNIOR COLLEGE YISHUN JUNIOR COLLEGE
YISHUN JUNIOR COLLEGE YISHUN JUNIOR COLLEGE YISHUN JUNIOR COLLEGE YISHUN JUNIOR COLLEGE YISHUN JUNIOR COLLEGE
YISHUN JUNIOR COLLEGE YISHUN JUNIOR COLLEGE YISHUN JUNIOR COLLEGE YISHUN JUNIOR COLLEGE YISHUN JUNIOR COLLEGE
YISHUN JUNIOR COLLEGE YISHUN JUNIOR COLLEGE YISHUN JUNIOR COLLEGE YISHUN JUNIOR COLLEGE YISHUN JUNIOR COLLEGE
YISHUN JUNIOR COLLEGE YISHUN JUNIOR COLLEGE YISHUN JUNIOR COLLEGE YISHUN JUNIOR COLLEGE YISHUN JUNIOR COLLEGE

READ THESE INSTRUCTIONS FIRST

Write in soft pencil.


Do not use staples, paper clips, highlighters, glue or correction fluid.
Write your name and CTG on the OMS form and shade your register number.

There are 30 questions on this paper. Answer all questions. For each question there are
4 possible answers A, B, C and D.
Choose the one you consider correct and record your choice in soft pencil on the OMS.

Read the instructions on the Answer Sheet very carefully.

Each correct answer will score one mark. A mark will not be deducted for a wrong answer.
Any rough working should be done in this question paper.

This question paper consists of 14 printed pages


2

Section A
For each question there are four possible answers, A, B, C, and D. Choose the one you
consider to be correct.

1 The diagram shows the mass spectrum of a sample of zinc. Calculate the relative
atomic mass of zinc.

A 65 B 65.25 C 65.5 D 65.66

2 A sample of the hydrocarbon C6H12 is completely burned in dry oxygen and the products
are collected as shown.
[Ar : H, 1.0 ; C, 12.0 ; O, 16.0]

The increases in mass of the collecting vessels P and Q of the apparatus are MP and
MQ, respectively. What is the ratio MP / MQ?

A 2.4 C 1.2

B 0.82 D 0.41
8872/JC2 Prelim/YJC/P1/2010
3

3 The bond angles in the PH3 molecule are likely to be

A 90 C 109

B 104 D 120

4 A weedkiller can be prepared by heating a bleach solution.

3NaClO 2NaCl + NaClO3


bleach weedkiller

What are the oxidation states of chlorine in these three compounds?

A 1 1 +5

B +1 1 +5

C +1 1 +7

D +2 +1 +7

5 In an experiment, 0.00130 mole of a metallic salt reacted completely with 20.80 cm3 of
0.0250 mol dm-3 MnO4.
The half equation for the reduction of MnO4 is shown below:
MnO4 + 8H+ + 5e Mn2+ + 4H2O
Given the final oxidation number of the metal in the salt was +5, what would the
original oxidation number of the metal be?

A +1

B +2

C +3

D +4

8872/JC2 Prelim/YJC/P1/2010
4

6 The sketch below shows the variation of first ionisation energy with proton number for
six elements of consecutive proton numbers between 1 and 18 (H to Ar).

What is the identity of element X?

A Mg B Al C Si D P

7 What is the order of increasing energy of the listed orbitals in the atom of titanium?

A 3s 3p 3d 4s

B 3s 3p 4s 3d

C 3s 4s 3p 3d

D 4s 3s 3p 3d

8 Hydrogen bonding can occur between molecules of methanal, HCHO, and molecules of
liquid Y. What could liquid Y be?

A CH3CO2CH3

B CH3COCH3

C CH3CHO

D CH3CH2OH

8872/JC2 Prelim/YJC/P1/2010
5

9 The diagram represents the reaction pathway for the following reaction.
W (g) + X (g) Y (g) + Z (g)

What statement can be made about the reverse reaction:

Y (g) + Z (g) W (g) + X (g)?

A It will have a larger activation energy and a positive H.

B It will have a larger activation energy and a negative H.

C It will have a smaller activation energy and a positive H.

D It will have a smaller activation energy and a negative H.

10 For which of the following reactions does the value of Hrxn represent both a standard
enthalpy change of combustion and a standard enthalpy change of formation of a
substance?

A 2S (s) + O2 (g) 2SO2 (g)

B S (g) + O2 (g) SO2 (g)

C S (s) + O2 (g) SO2 (g)

D SO (g) + O2 (g) SO2 (g)

8872/JC2 Prelim/YJC/P1/2010
6

11 The equilibrium constant, Kc, to form ethyl ethanoate from ethanol and ethanoic acid at

60C, is 4.00. The reaction is shown below:

C2H5OH + CH3COOH CH3COOC2H5 + H2O

When 1.00 mol each of ethanol and ethanoic acid are allowed to reach equilibrium at
60C, what is the number of moles of ethyl ethanoate formed?
1 2 1 3
A B C D
3 3 4 4

12 Which statement concerning the equilibrium reaction below is true?

2CrO42 (aq) + 2H+ (aq) Cr2O72 (aq) + H2O(l)

A An increase in acid concentration will result in an increase in the concentration


of Cr2O72 (aq).

B A redox reaction is taking place.

C The addition of a catalyst will result in an increase in the concentration of


Cr2O72 (aq).

D The equilibrium constant, Kc, has no units.

13 Which of the following reagents can be mixed to form an acidic buffer?

A HCN and NaCN

B HNO3 and NaNO3

C NaOH and NaCl

D HCl and NaOH

8872/JC2 Prelim/YJC/P1/2010
7

14 Which solid-line curve most accurately represents the distribution of molecular speeds
in a gas at 500 K if the dotted-line curve represents the corresponding distribution for
the same gas at 300 K?

A C

B D

15 Element R in period three of the periodic table forms an oxide which has a high melting
point and is insoluble in water. Identify element R.

A sodium

B silicon

C sulfur

D chlorine

8872/JC2 Prelim/YJC/P1/2010
8

16 Which one of the following chlorides of the elements of period 3 is a volatile liquid which
is readily hydrolysed by water?

A MgCl2 C Al2Cl6
B PCl5 D SiCl4

17 Which of the following alkane is the most volatile?

A C

B D

18 Identify hybridisation state of carbons in the following molecule.

A B

COCH3
D

CH2Cl

A B C D

A sp2 sp3 sp3 sp2

B sp2 sp3 sp2 sp3

C sp3 sp2 sp2 sp3

D sp3 sp2 sp3 sp2

8872/JC2 Prelim/YJC/P1/2010
9

19. Which compound cannot be obtained from propene in a single reaction?

H CH3
C C
H H
propene

H CH3 H CH3
A C
H C C H H C C CN

H H H H

B H CH3 D H CH3
H C C OH H C C Br
OH H H H

20. Compound S has the structure shown below:

Cl CH3

C C

Cl H

Which reagent, when reacted with compound S, causes the most damage to the ozone
layer?

A Steam C Cl2

B cold KMnO4 D HCl

8872/JC2 Prelim/YJC/P1/2010
10

21. Pheromone is a chemical compound used by the honeybee to attract other bees
towards itself.
CH CHCHBrCH=C(CH3)2

Pheromone

How many cis-trans isomers does this compound have?

A 2 C 4
B 3 D 6

22. The compound shown below, ibuprofen, is a powerful painkiller.

CH3 CH3
CH3 C CH2 CH
OH OH
CO2H

Which of the following reacts completely with one mole of ibuprofen?

A 3 mol of SOCl2 C 2 mol of PCl5

B 3 mol of HCl D 2 mol of NaOH

8872/JC2 Prelim/YJC/P1/2010
11

23. The unsaturated diketone shown is excreted by the bombardier beetle.


O
H3C

O
How does the compound above reacts with 2,4-dinitrophenylhydrazine reagent
(2,4-DNPH), Tollens reagent and Fehlings reagent?

2,4-DNPH Tollens Fehlings


A negative negative negative

B positive negative negative

C positive positive negative

D positive positive positive

24. A halogen compound T gave little or no precipitate when boiled under reflux with
ethanolic silver nitrate. Which of the following formulae is likely to be T?

A CH3CHClCH2CH3

B Cl CH3

C CH2CH2Cl

D CH3CH2COCl

8872/JC2 Prelim/YJC/P1/2010
12

25 A student obtained ethene from ethanol by carrying out an experiment using the
following set-up.

What type of reaction takes place on the hot pumice?

A combustion

B reduction

C elimination

D substitution

8872/JC2 Prelim/YJC/P1/2010
13

Section B

For each of the questions in this section, one or more of the three numbered statements 1 to
3 may be correct.

Decide whether each of the statements is or is not correct (you may find it helpful to place a
tick against the statements that you consider to be correct).

The responses A to D should be selected on the basis of

A B C D
1, 2 and 3 are 1 and 2 only 2 and 3 only 1 only is
correct are correct are correct correct

No other combination of statements is used as a correct response.

26 Which pairs of compounds contain one that is giant ionic and one that is simple
molecular?

1 Al2O3 and Al2Cl6

2 SiO2 and SiCl4

3 P4O10 and PCl3

27 k1
Given, reactant products
k1

Which of the following changes will cause both of the rate constants, k1 and k1 to
increase?

1 Introducing a catalyst.

2 Heating the equilibrium mixture.

3 Increasing the concentrations of the reactants.

8872/JC2 Prelim/YJC/P1/2010
14

28 The lattice energy of an ionic solid Un+Vn is dependent on

1 the lattice structure of UV.

2 charges on the ions Un+ and Vn.

3 ionic radii of Un+ and Vn.

29. Vitamin A has the structure

CH3 H CH3 H CH3


H3C
C C C C CH2OH
C C C C
H H H H
CH3
It is likely that this substance will

1 decolorise bromine water.

2 react with sodium carbonate solution.

3 react with aqueous sodium hydroxide.

30. Below are the structures of compound X and Y:

O O

O O

X Y
Which sets of reagents and conditions can be used to distinguish between compounds
X and Y?

1 Acidified KMnO4, reflux.

2 Aqueous alkaline iodine, heat.

3 2,4-dinitrophenylhydrazine.

8872/JC2 Prelim/YJC/P1/2010
CANDIDATES NAME: ______________________ CTG: _________

YISHUN JUNIOR COLLEGE


JC2 PRELIMINARY EXAMINATION 2010

CHEMISTRY 8872/02

HIGHER 1 24 August 2010


2 Hours
Paper 2

Additional materials: Answer Paper

Data Booklet

YISHUN JUNIOR COLLEGE YISHUN JUNIOR COLLEGE YISHUN JUNIOR COLLEGE YISHUN JUNIOR COLLEGE YISHUN JUNIOR COLLEGE
YISHUN JUNIOR COLLEGE YISHUN JUNIOR COLLEGE YISHUN JUNIOR COLLEGE YISHUN JUNIOR COLLEGE YISHUN JUNIOR COLLEGE
YISHUN JUNIOR COLLEGE YISHUN JUNIOR COLLEGE YISHUN JUNIOR COLLEGE YISHUN JUNIOR COLLEGE YISHUN JUNIOR COLLEGE
YISHUN JUNIOR COLLEGE YISHUN JUNIOR COLLEGE YISHUN JUNIOR COLLEGE YISHUN JUNIOR COLLEGE YISHUN JUNIOR COLLEGE
YISHUN JUNIOR COLLEGE YISHUN JUNIOR COLLEGE YISHUN JUNIOR COLLEGE YISHUN JUNIOR COLLEGE YISHUN JUNIOR COLLEGE
YISHUN JUNIOR COLLEGE YISHUN JUNIOR COLLEGE YISHUN JUNIOR COLLEGE YISHUN JUNIOR COLLEGE YISHUN JUNIOR COLLEGE
YISHUN JUNIOR COLLEGE YISHUN JUNIOR COLLEGE YISHUN JUNIOR COLLEGE YISHUN JUNIOR COLLEGE YISHUN JUNIOR COLLEGE
YISHUN JUNIOR COLLEGE YISHUN JUNIOR COLLEGE YISHUN JUNIOR COLLEGE YISHUN JUNIOR COLLEGE YISHUN JUNIOR COLLEGE
YISHUN JUNIOR COLLEGE YISHUN JUNIOR COLLEGE YISHUN JUNIOR COLLEGE YISHUN JUNIOR COLLEGE YISHUN JUNIOR COLLEGE
YISHUN JUNIOR COLLEGE YISHUN JUNIOR COLLEGE YISHUN JUNIOR COLLEGE YISHUN JUNIOR COLLEGE YISHUN JUNIOR COLLEGE
YISHUN JUNIOR COLLEGE YISHUN JUNIOR COLLEGE YISHUN JUNIOR COLLEGE YISHUN JUNIOR COLLEGE YISHUN JUNIOR COLLEGE
YISHUN JUNIOR COLLEGE YISHUN JUNIOR COLLEGE YISHUN JUNIOR COLLEGE YISHUN JUNIOR COLLEGE YISHUN JUNIOR COLLEGE
YISHUN JUNIOR COLLEGE YISHUN JUNIOR COLLEGE YISHUN JUNIOR COLLEGE YISHUN JUNIOR COLLEGE YISHUN JUNIOR COLLEGE

For Examiners Use


INSTRUCTIONS TO CANDIDATES
READ THESE INSTRUCTIONS FIRST Section A
Write your
Write your Name
name and
andCTG
CTGonincover page. at the
the spaces 1 /11
Write in dark blue or black pen on both sides of the paper.
You mayall
Answer use a soft pencil
questions. for diagrams,
Write graphs
your answers in or
therough
writing 2 /11
working.
papers provided.
Do not use staples, paper clips, highlighters, glue or
3 /8
correction fluid.
For numerical answers, all working should be shown
clearly.
Section A 4 /10
Answer all questions.
At the end of the paper, fasten all your answer to. Sub-total /40
Section B
Answer two questions on separate answer paper. Section B
INFORMATION FOR CANDIDATES Q
At the
The end of of
number themarks
examination,
is givenfasten all your [work
in brackets ] at securely
together.
Q
The number of marks is given in brackets [ ] at the end of
each question or part question. Sub-total /40

Total /80

This question paper consists of 16 printed pages


2

Section A

Answer all the questions in this section in the spaces provided.

1 Aluminium can exists as two isotopes, 26Al and 27


Al.
(a) (i) With the aid of the Data Booklet, write down the quantity of the following
sub-atomic particles of the isotope 26Al and 27
Al.

26 27
Al Al
Protons:

Neutrons:

Electrons:

(ii) Define the term isotopes.

(iii) Given that the relative atomic mass of Al is 26.8. Calculate the relative
abundance of both isotopes.

[5]

8872/JC2 Prelim/YJC/P2/2010
3

(b) Aluminium reacts with chloride to form aluminium chloride, AlCl3.


In the vapour state, the apparent molecular mass of aluminium chloride is
267.

In the solid state, aluminium chloride is a non-conductor of electricity. When


dissolve in water, aluminium chloride is found to be able to conduct
electricity.

(i) Draw a dot-and-cross diagram of aluminium chloride in the vapour state.

(ii) Explain in terms of structure and bonding, why solid aluminium chloride
does not conduct electricity.

(iii) Explain, with the aid of an equation, why aluminium chloride is able to
conduct electricity in water.

[6]

[Total: 11]

8872/JC2 Prelim/YJC/P2/2010
4

2 (a) After consuming food the pH in mouth can drop from pH 6.8 to a pH of about
4.8 as the sugar is broken down to lactic acid. In time, the hydrogen
carbonate ions (HCO3) in saliva restore the pH to its original value.

(i) Calculate the H+ concentration in the mouth at pH 6.8.

(ii) Write an ionic equation to show how hydrogen carbonate ions restore
the pH to its original value.

[2]

(b) Ammonia is manufactured in the Haber process according to the equation:

N2 (g) + 3 H2 (g) 2 NH3 (g) H = 92.4 kJ mol1

(i) State the typical conditions used in the Haber process and explain how
the yield is affected using Le Chateliers Principle.

Temperature:
Reasons:

Pressure:
Reasons:

[4]
8872/JC2 Prelim/YJC/P2/2010
5

(c) (i) In the space provided below, sketch a graph of the melting point of
oxides of elements from sodium to sulphur.

(ii) Explain as fully as you can why the melting point varies in the way
shown.

[5]
[Total: 11]

8872/JC2 Prelim/YJC/P2/2010
6

3 (a) Compound A is a sweet smelling compound with molecular formula C9H10O2.


Heating of A with dilute sulfuric acid produces B, C7H6O2 and C which gives
out steamy white fumes when reacted with an anhydrous PCl5 to give
compound D. Compound B and C reacts with Na, but only B reacts with
NaOH.

Suggest a possible structure for each of the compounds A to D.

A B

C D

[4]

8872/JC2 Prelim/YJC/P2/2010
7

(b) The reaction scheme below represents reactions of organic compounds.

OH I OH
II

Br Br
III
IV
dil H2SO4, KMnO4
heat

(i) What are the reagents and conditions used in Steps I, II and III?

Step Reagents Conditions


I

II

III

(ii) Give the structural formula of product formed in step IV.

[4]
[Total: 8]

8872/JC2 Prelim/YJC/P2/2010
8

4 Our bodies get energy by the digestion and metabolism of food. A combustion
reaction occurs in our bodies when glucose, C6H12O6 is burned in the presence of
oxygen, carbon dioxide, water, and heat is produced. The energy released in the
combustion of food molecules is converted into heat energy (to maintain our
constant body temperature), mechanical energy (to move our muscles), and
electrical energy (for nerve transmission). The total amount of energy released by
the digestion and metabolism of food is called its calorie content.

The amount of energy in food can be measured by burning the food in a


calorimeter. The temperature change produced as the water heated, the specific
heat capacity of water (4.184 J/gK) or 1 calorie/gK ), and the mass of the water
being heated are used to determine the amount of calories in the food.

A nutritional Calorie (Cal) is equivalent to one energy kilocalorie (kcal) or


1000 calories (cal).

A student wanted to determine the amount of calories in corn puffs by performing


a calorimetric experiment as shown:

50 cm3 of water was added to the calorimeter and 0.35 g of food corn puffs was
combusted. A temperature rise of 20 C was recorded.

8872/JC2 Prelim/YJC/P2/2010
9

(a) (i) Calculate the calories absorbed by the water.

(ii) Calculate the heat energy released by burning the corn puff.

(iii) Calculate the calories per gram of corn puff.

(iv) Convert calories per gram of food to nutritional Calories/ gram of food
(Cal/g).

(v) According to the nutritional information of the package of corn puff, how
does the nutritional Calories (Cal) compare with the students
experimental data?

8872/JC2 Prelim/YJC/P2/2010
10

(vi) List one possible source of error in this experiment and suggest how
to overcome the error.

[7]

(b) The structure of glucose is


OH OH O
HO
H
OH OH
(i) Identify the functional groups present in glucose.

(ii) The presence of glucose can be determined by simple chemical test


based on the functional groups present. Give a simple chemical
test and observation expected.

Test:

Observations:

[3]
[Total: 10]

8872/JC2 Prelim/YJC/P2/2010
11

Section B

Answer two of the three questions in this section on separate paper.

5 (a) Hydrogen peroxide decomposes according to the equation below:

2H2O2 (aq) 2H2O (l) + O2 (g)

Hydrogen peroxide can also be oxidised to oxygen by potassium


manganate(VII).

(i) What is the oxidation state of oxygen in hydrogen peroxide?

(ii) What volume of acidified potassium manganate(VII) of concentration


0.0200 mol dm3 is decolourised by 50 cm3 of hydrogen peroxide of
concentration 0.0100 mol dm3?

(iii) What volume of oxygen is evolved at room temperature and pressure in


this titration?

[5]

(b) The sketch below shows the first five ionisation energy for phosphorus.
-1
Ionisation energy / kJmol

0 1 2 3 4 5 6 7
number of electrons removed

(i) Write the electronic configuration of phosphorus in the ground state.

(ii) Write an equation to represent the second ionisation of phosphorus.

8872/JC2 Prelim/YJC/P2/2010
12

(iii) On the sketch above, draw an X to indicate a value for the


6th ionisation energy of phosphorus.

(iv) Explain briefly why the fourth ionisation energy of phosphorus is higher
than expected compared to the third ionisation energy.

[5]

(c) (i) Draw dot and cross diagrams of NO and ClO2 and state the shape of
ClO2 drawn.

(ii) The boiling points of chlorine dioxide and chlorine are 11 oC and 34 oC
respectively. Explain the difference in their boiling points.

[6]

(d) The melting points of magnesium chloride and silicon tetrachloride are as
follows:

compound melting point / oC


MgCl2 714
SiCl4 70

(i) Briefly relate these melting points to the structure and bonding in each
of these chlorides.

(ii) The lattice energy of sodium oxide is 2000 kJ mol-1.


Explain how you would expect the magnitude of the lattice energy of
sodium sulphide, Na2S, to compare with that of sodium oxide, Na2O.

[4]

[Total: 20]

8872/JC2 Prelim/YJC/P2/2010
13

6 Hydrogen sulfide is produced when thioacetamide undergoes acidic hydrolysis.


The reaction is as shown:

S O

CH3 C NH2 + 2H2O + H+ CH3 C OH + NH4+ + H2S

(a) A series of experiments were carried out to investigate the kinetics of the
reaction. The following results were obtained.

Experiment Initial concentration of reactant Initial rate of


formation of
Thioacetamide, H+, H2S
mol dm3 mol dm3 mol dm3 s1
1 0.240 0.480 8.0
2 0.240 0.120 2.0
3 0.360 0.240 9.0
4 0.120 0.120 0.5
5 0.240 0.060 1.0
6 0.014 1.350 a

Use the data given in the table above to find the

(i) rate equation.

(ii) rate constant, stating its units.

(iii) value of initial rate, a for experiment 6.


[6]

(b) Using collision theory, explain the effect of adding water on the rate and rate
constant of the reaction in (a).

[3]

(c) With the aid of a suitable diagram, explain why an increase in temperature
causes an increase in rate of formation of H2S.

[3]
8872/JC2 Prelim/YJC/P2/2010
14

(d) One use of hydrogen sulfide is to produce sulfur and sulfur dioxide. The
latter dissolves in water readily to form an acidic solution.

(i) Write an equation for sulfur dioxide in water.


(ii) Given that the concentration of H+ ion is 3.98 x 10-3 mol dm-3, calculate
the pH of the acidic solution.

[2]
(e) One of the side products of this reaction is ethanoic acid. When equal
amounts of sodium ethanoate and ethanoic acid are mixed, an acidic
buffer solution is obtained.
(i) Explain what is meant by the term buffer solution.
(ii) With the aid of equations, describe how the acidic buffer solution works
as a buffer.
[3]
(f) Ethanoic acid is a weak acid. Sodium hydroxide can be titrated with aqueous
ethanoic acid or with aqueous hydrochloric acid. Which indicators from the
table below could you use for each of the two titrations. Explain your
answers.
Indicator pH range of colour change
Methyl orange 3.2 4.4
Bromophenol blue 2.8 4.6
Phenolphthalein 8.2 10.0
[3]
[Total: 20]

8872/JC2 Prelim/YJC/P2/2010
15

7 This question concerns the preparation of an organic compound A using the


apparatus illustrated below.

A mixture of concentrated H2SO4 and CH3CH(CH2CH3)CH(OH)CH3 is heated in


the flask to around 170 C. A compound B, which is the major product of the
reaction, was formed and B reacted with bromine in the second test tube to form
C.
(a) Give the structural formula of the compound B.

[1]

(b) Give the structural formula of compound C and give its IUPAC name.
[2]

(c) Suggest a reason why aqueous sodium hydroxide was used in this
procedure.
[1]
(d) The reaction mixture in the flask developed a light brown colour, which
becomes black on heating to about 170 C. Suggest a possible identity for
the black colouration.
[1]

8872/JC2 Prelim/YJC/P2/2010
16

(e) State the type of reaction involved in the formation of C from B.

[1]

(f) By structure, compound B is characterised by one functional group.

(i) Identify the functional group.

(ii) A herbicide, mecoprop has the following structure

OHC OC(CH3)2COOH

Cl CH=CHCH3

Other than the functional group identified in B; your answer in f(i),


identify 2 other functional groups in the structure of mecoprop.

(iii) State the type of stereoisomerism displayed by the structure of


mecoprop. Draw the displayed formulae of the isomers. Label each
isomer clearly.
(iv) Describe two tests which can be performed on mecoprop separately to
prove the presence of these two functional groups identified in f(ii).

You should include the reagents and conditions, observations and


balanced equation for each test.

(v) Draw the structure of the product formed when mecoprop reacts with
I PCl5
II CH3CH2OH

For appreciable yield, II is carried out under a special condition. Briefly


discuss this procedure.

[You may disregard the minor side-reaction involving other functional


groups in mecoprop.]
[14]
[Total: 20]

8872/JC2 Prelim/YJC/P2/2010
2010 H1 Preliminary Examination

Paper 1: MCQ

1 C 11 B 21 A

2 D 12 A 22 A

3 B 13 A 23 B

4 B 14 D 24 B

5 C 15 B 25 C

6 A 16 D 26 D

7 B 17 C 27 B

8 D 18 A 28 C

9 A 19 C 29 D

10 C 20 C 30 B
2

Paper 2

Section A

Answer all the questions in this section in the spaces provided.

1 Aluminium can exists as two isotopes, 26Al and 27


Al.
(a) (i) With the aid of the Data Booklet, write down the quantity of the following
sub-atomic particles of the isotope 26Al and 27
Al.

26 27
Al Al
Protons:
13 13

Neutrons:
15 14

Electrons:
13 13

(ii) Define the term isotopes.

Isotopes are element with the same atomic number (number of protons)
but different mass number (due to different number of neutrons).

(iii) Given that the relative atomic mass of Al is 26.8. Calculate the relative
abundance of both isotopes.

26 27
Let the relative abundance of Al be x, the relative abundance of Al
be (100-x)

26x + 27(100-x) / 100 = 26.8

x = 20%

26 27
The relative abundance of Al is 20%, the relative abundance of Al is
80%

[5]

8872/YJC/P2/2010
3

(b) Aluminium reacts with chloride to form aluminium chloride, AlCl3. In the
vapour state, the apparent molecular mass of aluminium chloride is 267.

In the solid state, aluminium chloride is a non-conductor of electricity. When


dissolve in water, aluminium chloride is found to be able to conduct
electricity.

(i) Draw a dot-and-cross diagram of aluminium chloride in the vapour state.

(ii) Explain in terms of structure and bonding, why solid aluminium chloride
does not conduct electricity.

Simple covalent molecule/simple molecular structure with covalent


bonds, no free electrons or ions to conduct electricity.

(iii) Explain, with the aid of an equation, why aluminium chloride is able to
conduct electricity in water.

AlCl3 dissolves in water and undergoes partial hydrolysis to give a


weakly acidic solution (pH = 4 which contains ions to act as charge
carriers to conduct electricity. (1m)

AlCl3 (s) + aq [Al (H2O)6]3+ (aq) + 3Cl (aq)

[Al(H2O)6]3+ (aq) [Al(H2O)5(OH)]2+ (aq) + H+ (aq)

[6]

[Total: 11]

8872/YJC/P2/2010
4

2 (a) After consuming food the pH in mouth can drop from pH 6.8 to a pH of about
4.8 as the sugar is broken down to lactic acid. In time, the hydrogen
carbonate ions (HCO3-) in saliva restore the pH to its original value.

(i) Calculate the H+ concentration in the mouth at pH 6.8


[H+] = 10-6.8 =1.58 x 10-7 mol dm-3

(ii) Write an ionic equation to show how hydrogen carbonate ions restore
the pH to its original value.
HCO3- + H+ H2O + CO2

[2]
(b) Ammonia is manufactured in the Haber process according to the equation:
N2 (g) + 3 H2 (g) 2 NH3 (g) (H = -92.4 kJ mol1)

(i) State the typical conditions used in the Haber process and explain how
the yield is affected using Le Chateliers Principle.

Temperature: 500C

Reasons: Le Chaterliers principle predicts that low temperature gives


higher yield as the forward reaction is exothermic, low temperature will
cause equilibrium shift right to produce more heat.

Pressure: 200 atm

Reasons: Le Chaterliers principle predicts that high pressure

gives higher yield as the equilibrium shift right to reduce no of

moles of gaseous molecules/particles.

[4]

8872/YJC/P2/2010
5

(c) (i) In the space provided below, sketch a graph of the melting point of
oxides of elements from sodium to sulphur.

(ii) Explain as fully as you can why the melting point varies in the way
shown.
Oxides of Na, Mg and Al are ionic, hence they have high melting
and boiling points. Increasing melting points of the oxides are
due to increasing lattice energies of the compounds.
SiO2 has a giant molecular structure. Numerous strong SiO
covalent bonds must be broken before melting occurs. Hence it
has high melting and boiling points as well.
Oxides of P and S have simple molecular structures with weak
VDW interactions between the discreet molecules which need to
be overcome before melting occurs. Hence they have low melting
points.

[5]
[Total: 11]

8872/YJC/P2/2010
6

3 (a) Compound A is a sweet smelling compound with molecular formula C9H10O2.


Heating of A with dilute sulfuric acid produces B, C7H6O2 and C which gives
out steamy white fumes when reacted with a anhydrous PCl5 to give
compound D. Compound B and C reacts with Na, but only B reacts with
NaOH.

Suggest a possible structure for each of the compounds A to D.

A B

Ethyl benzoate Benzoic acid


O O

C CH2CH3 C H
O O

C D

Ethanol Chloroethane
CH3CH2OH CH3CH2Cl

[4]

8872/YJC/P2/2010
7

(b) The reaction scheme below represents reactions of organic compounds.


OH I OH
II

Br Br
III
IV
dil H2SO4, KMnO4
heat

(i) What are the reagents and conditions used in Steps I, II and III?

Step Reagents Conditions


I Br2 UV

II Conc H2SO4 170C

III Ethanolic KOH heat

(ii) Give the structural formulae of product formed in step IV.

Structure of butanedioic acid or carbon dioxide

[4]
[Total: 8]

8872/YJC/P2/2010
8

4 Our bodies get energy by the digestion and metabolism of food. A combustion
reaction occurs in our bodies when glucose, C6H12O6 is burned in the presence of
oxygen, carbon dioxide, water, and heat is produced. The energy released in the
combustion of food molecules is converted into heat energy (to maintain our
constant body temperature), mechanical energy (to move our muscles), and
electrical energy (for nerve transmission). The total amount of energy released by
the digestion and metabolism of food is called its calorie content.

The amount of energy in food can be measured by burning the food in a


calorimeter. The temperature change produced as the water heated, the specific
heat of water (4.184 J/gK) or 1 calorie/gK ), and the mass of the water being
heated are used to determine the amount of calories in the food.

A nutritional Calorie (Cal) is equivalent to one energy kilocalorie (kcal) or


1000 calories (cal).

A student wanted to determine the amount of calories in corn puffs by performing


a calorimetric experiment as shown:

50 cm3 of water was added to the calorimeter and 0.35 g of food corn puffs
combusted. A temperature rise of 20 C was recorded.

8872/YJC/P2/2010
9

(a) (i) Calculate the calories absorbed by the water.


calories absorbed by water = mcT
= 50 x 1 x 20
=1000 cal

(ii) Calculate the heat released by burning the corn puff.


heat released = 1000 x 4.184 = 4184 J

(iii) Calculate the calories per gram of corn puff.


calories per gram of food = 1000 / 0.35
= 2857 cal

(iv) Convert calories per gram of food to nutritional Calories/ gram of food
(Cal/g).

1 nutritional Calories = 1000 calories


2857 calories = 2.86 nutritional Calories/g

(v) According to the nutritional information of the package of corn


puff, how does the nutritional Calories (Cal) compare with the
students experimental data?

The nutritional information on the packaging will be higher than

the experimental one as in the experiment, heat is loss during

burning of food, resulting in less heat being absorb by the water

and lower cal calculated.

(vi) List one possible source of error in this experiment and


suggest how to overcome the error.
Heat loss. Insulate the calorimeter/lag the calorimeter

[7]

8872/YJC/P2/2010
10

(b) The structure of glucose is


OH OH O
HO
H
OH OH
(i) Identify the functional groups present in glucose.
alcohol and aldehyde

(ii) The presence of glucose can be determined by simple chemical test


based on the functional groups present. Give a simple chemical test
and observation expected.

Test: Tollens reagent, warm

Observations: silver mirror

Other tests includes 2,4-DNPH, Na, Fehlings solution, KMnO4


[3]
[Total: 10]

8872/YJC/P2/2010
11

Section B

Answer two of the three questions in this section on separate paper.

5 (a) Hydrogen peroxide decomposes according to the equation below:

2H2O2(aq) 2H2O(l) + O2(g)

Hydrogen peroxide can also be oxidised to oxygen by potassium


manganate(VII).

(i) What is the oxidation state of oxygen in hydrogen peroxide?


-1
(ii) What volume of acidified potassium manganate(VII) of concentration
0.0200 mol dm-3 is decolourised by 50 cm3 of hydrogen peroxide of
concentration 0.0100 mol dm-3?

Number of mol of H2O2 = 0.01 x 50 x 10-3 = 5 x 10-4 mol

Number of mol of MnO4- = 5 x 10-4 x (2/5) = 2 x 10-4 mol

Volume of MnO4- = 2 x 10-4 0.02 = 0.01 dm3 = 10 cm3

(iii) What volume of oxygen is evolved at room temperature and


pressure in this titration?
Number of mol of O2 = Number of mol of H2O2 = 5 x 10-4 mol

Volume of O2 at r.t.p. = 5 x 10-4 x 24 = 0.0120 dm3 or 12 cm3 [5]

8872/YJC/P2/2010
12

(b) The sketch below shows the first five ionisation energy for phosphorus.

-1
Ionisation energy / kJmol

0 1 2 3 4 5 6 7
number of electrons removed

(i) Write the electronic configuration of phosphorus in the ground state.

1s2 2s2 2p6 3s2 3p3

(ii) Write an equation to represent the second ionisation of phosphorus.

P+ (g) P2+ (g) + e-

(iii) On the sketch above, draw an X to indicate a value for the 6th
ionisation energy of phosphorus.

X
-1
Ionisation energy / kJmol

0 1 2 3 4 5 6 7
number of electrons removed

8872/YJC/P2/2010
13

(iv) Explain briefly why the fourth ionisation energy of phosphorus is


higher than expected compared to the third ionisation energy.

The third e removed is a 3p e and the fourth e removed is a 3s e.

Hence the fourth e, being at a lower energy level, requires more


energy for removal (slight increase in IE to be associated with
subshell).
[5]

(c) (i) Draw dot and cross diagrams of NO and ClO2 and state the shape
of ClO2 drawn.
xx xx xx
x x x x
N x
x O x x O x Cl x O xx

ClO2: bent/v-shape

(ii) The boiling points of chlorine dioxide and chlorine are 11oC and
34oC respectively. Explain the difference.
Cl2 has instantaneous dipole induced dipole forces of attraction
between molecules.

ClO2 has permanent dipole permanent dipole forces of attraction


between molecules.

Hence greater energy is required to overcome the stronger


permanent dipole permanent dipole forces of attraction in ClO2
and hence its higher melting point.
[6]

8872/YJC/P2/2010
14

(d) The melting points of magnesium chloride and silicon tetrachloride are as
follows:

compound melting point / oC


MgCl2 714
SiCl4 70

(i) Briefly relate these melting points to the structure and bonding in
each of these oxides.
MgCl2 has a giant ionic structure.
A lot of energy is required to overcome the strong electrostatic
forces of attraction between the oppositely charged ions.

SiCl4 has a simple molecular structure.


It has discrete molecules held together by weak van der Waals
forces. Less energy is required to break these bonds.
Hence, SO3 has a very low melting point.

(ii) The lattice energy of sodium oxide is 2000 kJ mol-1.


Explain how you would expect the numerical magnitude of the lattice
energy of sodium sulphide, Na2S, to compare with that of sodium
oxide, Na2O.

q+ q-
Lattice energy
r+ + r-

Ionic radius of S2- is greater than that of O2-


Therefore, lattice energy of Na2S is smaller than that of Na2O
or lattice energy of Na2O is greater than that of Na2S.
[4]

[Total: 20]

8872/YJC/P2/2010
15

6 Hydrogen sulfide is produced when thioacetamide undergoes acidic hydrolysis.


The reaction is as shown:

S O

CH3 C NH2 + 2H2O + H+ CH3 C OH + NH4+ + H2S

(a) A series of experiments were carried out to investigate the kinetics of the
reaction. The following results were obtained.

Experiment Initial concentration of reactant Initial rate of


formation of
Thioacetamide, H+, H2S
mol dm-3 mol dm-3 mol dm-3 s-1
1 0.240 0.480 8.0
2 0.240 0.120 2.0
3 0.360 0.240 9.0
4 0.120 0.120 0.5
5 0.240 0.060 1.0
6 0.014 1.350 a

Use the data given in the table above to find the

(i) rate equation


Compare expt 1 and 2, [H+] decrease by 4, rate decrease by 4,
order w.r.t H+ is 1.

Compare expt 2 and 4, [Thioacetamide] halved, rate decrease by 4,


order w.r.t Thioacetamide is 2.

Rate = k[Thioacetamide]2 [H+]

(ii) rate constant, stating its units

using expt 1, k = 8/(0.240)2 x (0.480)


= 289.4 mol-2 dm6 s-1

(iii) value of initial rate, a for experiment 6.


a = 0.0766 mol dm-3 s-1 [6]

8872/YJC/P2/2010
16

(b) Using collision theory, explain the effect of adding water on the rate and
rate constant of the reaction in (a).
Adding water lowers the concentration both reactants (less no of
moles of particles per unit volume). No of effective collision decrease,
rate decreases. Since there are no change to temperature, rate
constant remains unchanged/constant.
[3]

(c) With the aid of a suitable diagram, explain why an increase in


temperature causes an increase in rate of formation of H2S.
Blotzmann
Increase temperature results increase in particles with energy greater
than or equal to activation energy. Thus no of effective collision
increased rate of reaction increased.
[3]

(d) One use of hydrogen sulfide is to produce sulfur and sulfur dioxide. The
latter dissolves in water readily to form an acidic solution.

(i) Write an equation for sulfur dioxide in water.


SO2 + H2O H2SO3
(ii) Given that the concentration of H+ ion is 3.98 x 10-3 mol dm-3,
calculate the pH of the acidic solution.
pH = - log 10 3.98 x 10-3 = 2.4
[2]
(e) One of the side products of this reaction is ethanoic acid. When equal
amounts of sodium ethanoate and ethanoic acid are mixed, an acidic
buffer solution is obtained.
(i) Explain what is meant by the term buffer solution.
A solution that maintains fairly constant pH when small amount of acid
or base is added to it.
(ii) With the aid of equations, describe how the acidic buffer solution works
as a buffer.
CH3COOH + OH- CH3COO- + H2O
CH3COO- + H+ CH3COOH

8872/YJC/P2/2010
17

[3]
(f) Ethanoic acid is a weak acid. Sodium hydroxide can be titrated with
aqueous ethanoic acid or with aqueous hydrochloride acid. Which
indicators from the table below could you use for each of the two
titrations. Explain your answers.
Indicator pH range of colour change
Methyl orange 3.2 - 4.4
Bromophenol blue 2.8 4.6
Phenolphthalein 8.2 10.0

Any indicator for sodium hydroxide and hydrochloric acid and


Phenolphthalein for sodium hydroxide and ethanoic acid. The pH
range at end point of both indicators lies within the working range of
the indicators selected. [3]
[Total: 20]

8872/YJC/P2/2010
18

7 This question concerns the preparation of an organic compound A using


the apparatus illustrated below.

A mixture of concentrated H2SO4 and CH3CH(CH2CH3)CH(OH)CH3 heated


in the flask to about 170 C. A compound B, which is the major product of
the reaction, was formed and B reacted with bromine in the second test
tube to form C.

(a) Give the structural formula of the compound B.


H

H3C C C CH3
CH2CH3 [1]

(b) Give the structural formula of compound C and give its IUPAC name.
Br Br
H3C C C CH3
C2H5 H

2,3-dibromo-3-methylpentane
[2]

(c) Suggest a reason why aqueous sodium hydroxide was used in this
procedure.
[1]
To remove traces of conc H2SO4
8872/YJC/P2/2010
19

(d) The reaction mixture in the flask developed a light brown colour, which
becomes black on heating to about 170 C. Suggest a possible
identity for the black colouration.
Carbon
[1]
(e) State the type of reaction involved in the formation of C from B.

Addition
[1]

(f) By structure, compound B is characterised by one functional group.

(i) Identify the functional group.


Alkene

(ii) A herbicide, mecoprop has the following structure

OHC OC(CH3)2COOH

Cl CH=CHCH3

Other than the functional group identified in B-your answer in f(i),


identify 2 other functional groups in the structure of mecoprop.
Aldehyde/Benzaldehyde and carboxylic acid

(iii) State the type of stereoisomerism displayed by the structure of


mecoprop. Draw the displayed formulae of the isomers. Label
each isomer clearly.
Cis-trans isomerism / Geometric isomerism

OHC OC(CH3)2COOH
CH3
Cl C C
H H Cis-isomer

8872/YJC/P2/2010
20

OHC OC(CH3)2COOH
H
Cl C C
H CH3
Trans-isomer

(iv) Describe two tests which can be performed on mecoprop


separately to prove the presence of these two functional groups
identified in f(ii).

You should include the reagents and conditions, observations


andbalanced equation for each test.
Test 1: Aldehyde : 2,4-DNPH. Orange ppt
Test 2: Acid: Na2CO3, CO2 effervescence

(v) Draw the structure of the product formed when mecoprop reacts
with
I PCl5
II CH3CH2OH

For appreciable yield, II is carried out under a special condition. Briefly


discuss this procedure.
[You may disregard the minor side-reaction involving other functional
groups in mecoprop.]
Structures
I II
OHC OC(CH3)2COCl OHC OC(CH3)2COOCH2CH3

Cl CH=CHCH3 Cl CH=CHCH3

A small amount of conc H2SO4 is to be added as a catalyst for this


esterification process between an alcohol and the acid in mecoprop.
[14]
[Total: 20]

8872/YJC/P2/2010

Вам также может понравиться